Vous êtes sur la page 1sur 135

PAGE 1 of 135 CONSTITUTIONAL LAW -EXECUTIVE DEPARTMENT PART 2 –SESSION 12

EN BANC
WHEREAS, one of the Good Government reforms of the Arroyo administration is rationalizing
ANAK MINDANAO PARTY-LIST GROUP, as G.R. No. 166052 the bureaucracy by consolidating related functions into one department;
represented by Rep. Mujiv S. Hataman, Present:
andMAMALO DESCENDANTS WHEREAS, under law and jurisprudence, the President of the Philippines has broad powers to
ORGANIZATION, INC., as represented by its PUNO, C.J., reorganize the offices under her supervision and control;
Chairman Romy Pardi, QUISUMBING,
Petitioners, YNARES-SANTIAGO, NOW[,] THEREFORE[,] I, Gloria Macapagal-Arroyo, by the powers vested in me as President
SANDOVAL-GUTIERREZ, of the Republic of the Philippines, do hereby order:
CARPIO,
AUSTRIA-MARTINEZ, SECTION 1. The Department of Agrarian Reform is hereby transformed into the Department
- versus - CORONA, of Land Reform. It shall be responsible for all land reform in the country, including agrarian
CARPIO MORALES, reform, urban land reform, and ancestral domain reform.
AZCUNA,
TINGA, SECTION 2. The PCUP is hereby placed under the supervision and control of the
CHICO-NAZARIO, Department of Land Reform. The Chairman of the PCUP shall be ex-officio Undersecretary of
THE EXECUTIVE GARCIA, the Department of Land Reform for Urban Land Reform.
SECRETARY, THE HON. EDUARDO R. VELASCO, JR.,
ERMITA, and THE SECRETARY OF NACHURA, and SECTION 3. The NCIP is hereby placed under the supervision and control of the Department of
AGRARIAN/LAND REFORM, THE HON. RENE REYES, JJ. Land Reform. The Chairman of the NCIP shall be ex-officio Undersecretary of the Department
C. VILLA, Promulgated: of Land Reform for Ancestral Domain Reform.
Respondents.
August 29, 2007 SECTION 4. The PCUP and the NCIP shall have access to the services provided by the
x----------------------------------------------------------------------------------------x Departments Finance, Management and Administrative Office; Policy, Planning and Legal
Affairs Office, Field Operations and Support Services Office, and all other offices of the
Department of Land Reform.
DECISION
SECTION 5. All previous issuances that conflict with this Executive Order are hereby repealed
or modified accordingly.
CARPIO MORALES, J.:
SECTION 6. This Executive Order takes effect immediately. (Emphasis and underscoring
Petitioners Anak Mindanao Party-List Group (AMIN) and Mamalo Descendants Organization, Inc. (MDOI) assail supplied)
the constitutionality of Executive Order (E.O.) Nos. 364 and 379, both issued in 2004, via the present Petition for
Certiorari and Prohibition with prayer for injunctive relief. E.O. No. 379, which amended E.O. No. 364 a month later or on October 26, 2004, reads:

E.O. No. 364, which President Gloria Macapagal-Arroyo issued on September 27, 2004, reads: EXECUTIVE ORDER NO. 379

EXECUTIVE ORDER NO. 364 AMENDING EXECUTIVE ORDER NO. 364 ENTITLED TRANSFORMING THE DEPARTMENT
OF AGRARIAN REFORM INTO THE DEPARTMENT OF LAND REFORM
TRANSFORMING THE DEPARTMENT OF AGRARIAN REFORM INTO THE DEPARTMENT
OF LAND REFORM WHEREAS, Republic Act No. 8371 created the National Commission on Indigenous Peoples;

WHEREAS, one of the five reform packages of the Arroyo administration is Social Justice and WHEREAS, pursuant to the Administrative Code of 1987, the President has the continuing
Basic [N]eeds; authority to reorganize the administrative structure of the National Government.

WHEREAS, one of the five anti-poverty measures for social justice is asset reform; NOW, THEREFORE, I, GLORIA MACAPAGAL-ARROYO, President of the Republic of
the Philippines, by virtue of the powers vested in me by the Constitution and existing laws, do
WHEREAS, asset reforms covers [sic] agrarian reform, urban land reform, and ancestral domain hereby order:
reform;
Section 1. Amending Section 3 of Executive Order No. 364. Section 3 of Executive Order No.
WHEREAS, urban land reform is a concern of the Presidential Commission [for] the Urban Poor 364, dated September 27, 2004 shall now read as follows:
(PCUP) and ancestral domain reform is a concern of the National Commission on Indigenous
Peoples (NCIP); Section 3. The National Commission on Indigenous Peoples (NCIP) shall be
an attached agency of the Department of Land Reform.
WHEREAS, another of the five reform packages of the Arroyo administration is Anti-Corruption
and Good Government; Section 2. Compensation. The Chairperson shall suffer no diminution in rank and salary.
PAGE 2 of 135 CONSTITUTIONAL LAW -EXECUTIVE DEPARTMENT PART 2 –SESSION 12
as to assure that concrete adverseness which sharpens the presentation of issues upon which the court depends
Section 3. Repealing Clause. All executive issuances, rules and regulations or parts thereof for illumination of difficult constitutional questions. [10]
which are inconsistent with this Executive Order are hereby revoked, amended or modified
accordingly. It has been held that a party who assails the constitutionality of a statute must have a direct and personal interest. It
must show not only that the law or any governmental act is invalid, but also that it sustained or is in immediate
Section 4. Effectivity. This Executive Order shall take effect immediately. (Emphasis and danger of sustaining some direct injury as a result of its enforcement, and not merely that it suffers thereby in
underscoring in the original) some indefinite way. It must show that it has been or is about to be denied some right or privilege to which it is
lawfully entitled or that it is about to be subjected to some burdens or penalties by reason of the statute or act
complained of.[11]
Petitioners contend that the two presidential issuances are unconstitutional for violating:
For a concerned party to be allowed to raise a constitutional question, it must show that (1) it has personally
- THE CONSTITUTIONAL PRINCIPLES OF SEPARATION OF POWERS AND OF THE suffered some actual or threatened injury as a result of the allegedly illegal conduct of the government, (2) the
RULE OF LAW[;] injury is fairly traceable to the challenged action, and (3) the injury is likely to be redressed by a favorable action. [12]
- THE CONSTITUTIONAL SCHEME AND POLICIES FOR AGRARIAN REFORM, URBAN
LAND REFORM, INDIGENOUS PEOPLES RIGHTS AND ANCESTRAL DOMAIN[; AND] An examination of MDOIs nebulous claims of negative impact and probable setbacks [13] shows that they are too
- THE CONSTITUTIONAL RIGHT OF THE PEOPLE AND THEIR ORGANIZATIONS TO abstract to be considered judicially cognizable. And the line of causation it proffers between the challenged action
EFFECTIVE AND REASONABLE PARTICIPATION IN DECISION-MAKING, INCLUDING and alleged injury is too attenuated.
THROUGH ADEQUATE CONSULTATION[.][1]
Vague propositions that the implementation of the assailed orders will work injustice and violate the rights
of its members cannot clothe MDOI with the requisite standing. Neither would its status as a peoples organization
By Resolution of December 6, 2005, this Court gave due course to the Petition and required the submission of vest it with the legal standing to assail the validity of the executive orders. [14]
memoranda, with which petitioners and respondents complied on March 24, 2006and April 11, 2006, respectively.
La Bugal-Blaan Tribal Association, Inc. v. Ramos,[15] which MDOI cites in support of its claim to legal
The issue on the transformation of the Department of Agrarian Reform (DAR) into the Department of Land Reform standing, is inapplicable as it is not similarly situated with the therein petitioners who alleged personal and
(DLR) became moot and academic, however, the department having reverted to its former name by virtue of E.O. substantial injury resulting from the mining activities permitted by the assailed statute. And so is Cruz v. Secretary
No. 456[2] which was issued on August 23, 2005. of Environment and Natural Resources,[16] for the indigenous peoples leaders and organizations were not the
petitioners therein, who necessarily had to satisfy the locus standi requirement, but were intervenors who sought
The Court is thus left with the sole issue of the legality of placing the Presidential Commission[3] for the and were allowed to be impleaded, not to assail but to defend the constitutionality of the statute.
Urban Poor (PCUP) under the supervision and control of the DAR, and the National Commission on Indigenous
Peoples (NCIP) under the DAR as an attached agency. Moreover, MDOI raises no issue of transcendental importance to justify a relaxation of the rule on legal
standing. To be accorded standing on the ground of transcendental importance, Senate of the Philippines v.
Before inquiring into the validity of the reorganization, petitioners locus standi or legal standing, inter Ermita[17] requires that the following elements must be established: (1) the public character of the funds or other
alia,[4] becomes a preliminary question. assets involved in the case, (2) the presence of a clear case of disregard of a constitutional or statutory prohibition
by the public respondent agency or instrumentality of government, and (3) the lack of any other party with a more
The Office of the Solicitor General (OSG), on behalf of respondents, concedes that AMIN[5] has the requisite legal direct and specific interest in raising the questions being raised. The presence of these elements MDOI failed to
standing to file this suit as member[6] of Congress. establish, much less allege.

Petitioners find it impermissible for the Executive to intrude into the domain of the Legislature. They posit that an Francisco, Jr. v. Fernando[18] more specifically declares that the transcendental importance of the issues raised
act of the Executive which injures the institution of Congress causes a derivative but nonetheless substantial injury, must relate to the merits of the petition.
which can be questioned by a member of Congress. [7] They add that to the extent that the powers of Congress
are impaired, so is the power of each member thereof, since his office confers a right to participate in the exercise This Court, not being a venue for the ventilation of generalized grievances, must thus deny adjudication of the
of the powers of that institution.[8] matters raised by MDOI.

Indeed, a member of the House of Representatives has standing to maintain inviolate the prerogatives, Now, on AMINs position. AMIN charges the Executive Department with transgression of the principle of separation
powers and privileges vested by the Constitution in his office. [9] of powers.

The OSG questions, however, the standing of MDOI, a registered peoples organization Under the principle of separation of powers, Congress, the President, and the Judiciary may not encroach
of Teduray and Lambangian tribesfolk of (North) Upi and South Upi in the province of Maguindanao. on fields allocated to each of them. The legislature is generally limited to the enactment of laws, the executive to
As co-petitioner, MDOI alleges that it is concerned with the negative impact of NCIPs becoming an attached the enforcement of laws, and the judiciary to their interpretation and application to cases and controversies. The
agency of the DAR on the processing of ancestral domain claims. It fears that transferring the NCIP to the DAR principle presupposes mutual respect by and between the executive, legislative and judicial departments of the
would affect the processing of ancestral domain claims filed by its members. government and calls for them to be left alone to discharge their duties as they see fit. [19]

Locus standi or legal standing has been defined as a personal and substantial interest in a case such that the AMIN contends that since the DAR, PCUP and NCIP were created by statutes, [20] they can only be transformed,
party has sustained or will sustain direct injury as a result of the governmental act that is being challenged. The merged or attached by statutes, not by mere executive orders.
gist of the question of standing is whether a party alleges such personal stake in the outcome of the controversy
PAGE 3 of 135 CONSTITUTIONAL LAW -EXECUTIVE DEPARTMENT PART 2 –SESSION 12
While AMIN concedes that the executive power is vested in the President [21] who, as Chief Executive, holds the executive department are concerned, the power of control may justify the President to deactivate the functions of
power of control of all the executive departments, bureaus, and offices, [22] it posits that this broad power of control a particular office. Or a law may expressly grant the President the broad authority to carry out reorganization
including the power to reorganize is qualified and limited, for it cannot be exercised in a manner contrary to law, measures.[29] The Administrative Code of 1987 is one such law: [30]
citing the constitutional duty[23] of the President to ensure that the laws, including those creating the agencies, be
faithfully executed. SEC. 30. Functions of Agencies under the Office of the President. Agencies under the Office of
the President shall continue to operate and function in accordance with their respective charters
AMIN cites the naming of the PCUP as a presidential commission to be clearly an extension of the President, and or laws creating them, except as otherwise provided in this Code or by law.
the creation of the NCIP as an independent agency under the Office of the President. [24] It thus argues that since SEC. 31. Continuing Authority of the President to Reorganize his Office. The President, subject
the legislature had seen fit to create these agencies at separate times and with distinct mandates, the President to the policy in the Executive Office and in order to achieve simplicity, economy and
should respect that legislative disposition. efficiency, shall have continuing authority to reorganize the administrative structure of the Office
of the President. For this purpose, he may take any of the following actions:
In fine, AMIN contends that any reorganization of these administrative agencies should be the subject of
a statute. (1) Restructure the internal organization of the Office of the President Proper, including the
immediate Offices, the Presidential Special Assistants/Advisers System and the Common Staff
AMINs position fails to impress. Support System, by abolishing, consolidating, or merging units thereof or transferring functions
from one unit to another;
The Constitution confers, by express provision, the power of control over executive departments, bureaus (2) Transfer any function under the Office of the President to any other Department or Agency
and offices in the President alone. And it lays down a limitation on the legislative power. as well as transfer functions to the Office of the President from other Departments and Agencies;
and
The line that delineates the Legislative and Executive power is not indistinct. Legislative power
is the authority, under the Constitution, to make laws, and to alter and repeal them. The (3) Transfer any agency under the Office of the President to any other department or agency as
Constitution, as the will of the people in their original, sovereign and unlimited capacity, has well as transfer agencies to the Office of the President from other departments or
vested this power in the Congress of the Philippines. The grant of legislative power to Congress agencies.[31] (Italics in the original; emphasis and underscoring supplied)
is broad, general and comprehensive. The legislative body possesses plenary power for all
purposes of civil government. Any power, deemed to be legislative by usage and tradition, is
necessarily possessed by Congress, unless the Constitution has lodged it elsewhere.In In carrying out the laws into practical operation, the President is best equipped to assess whether an executive
fine, except as limited by the Constitution, either expressly or impliedly, legislative power agency ought to continue operating in accordance with its charter or the law creating it. This is not to say that the
embraces all subjects and extends to matters of general concern or common interest. legislature is incapable of making a similar assessment and appropriate action within its plenary power. The
Administrative Code of 1987 merely underscores the need to provide the President with suitable solutions to
While Congress is vested with the power to enact laws, the President executes the laws. The situations on hand to meet the exigencies of the service that may call for the exercise of the power of control.
executive power is vested in the President. It is generally defined as the power to enforce and
administer the laws. It is the power of carrying the laws into practical operation and enforcing x x x The law grants the President this power in recognition of the recurring need of every
their due observance. President to reorganize his office to achieve simplicity, economy and efficiency. The Office of
the President is the nerve center of the Executive Branch. To remain effective and efficient, the
As head of the Executive Department, the President is the Chief Executive. He represents the Office of the President must be capable of being shaped and reshaped by the President in the
government as a whole and sees to it that all laws are enforced by the officials and employees manner he deems fit to carry out his directives and policies. After all, the Office of the President
of his department. He has control over the executive department, bureaus and offices. This is the command post of the President. This is the rationale behind the Presidents continuing
means that he has the authority to assume directly the functions of the executive department, authority to reorganize the administrative structure of the Office of the President. [32]
bureau and office, or interfere with the discretion of its officials. Corollary to the power of control,
the President also has the duty of supervising and enforcement of laws for the maintenance of
general peace and public order. Thus, he is granted administrative power over bureaus and The Office of the President consists of the Office of the President proper and the agencies under it. [33] It is not
offices under his control to enable him to discharge his duties effectively.[25] (Italics omitted, disputed that PCUP and NCIP were formed as agencies under the Office of the President. [34] The Agencies under
underscoring supplied) the Office of the President refer to those offices placed under the chairmanship of the President, those under the
supervision and control of the President, those under the administrative supervision of the Office of the President,
those attached to the Office for policy and program coordination, and those that are not placed by law or order
The Constitutions express grant of the power of control in the President justifies an executive action to carry out creating them under any special department.[35]
reorganization measures under a broad authority of law.[26]
As thus provided by law, the President may transfer any agency under the Office of the President to any other
In enacting a statute, the legislature is presumed to have deliberated with full knowledge of all existing laws and department or agency, subject to the policy in the Executive Office and in order to achieve simplicity, economy
jurisprudence on the subject.[27] It is thus reasonable to conclude that in passing a statute which places an agency and efficiency. Gauged against these guidelines,[36] the challenged executive orders may not be said to have been
under the Office of the President, it was in accordance with existing laws and jurisprudence on the Presidents issued with grave abuse of discretion or in violation of the rule of law.
power to reorganize.
The references in E.O. 364 to asset reform as an anti-poverty measure for social justice and to rationalization of
In establishing an executive department, bureau or office, the legislature necessarily ordains an executive agencys the bureaucracy in furtherance of good government[37] encapsulate a portion of the existing policy in the Executive
position in the scheme of administrative structure. Such determination is primary,[28] but subject to the Presidents Office. As averred by the OSG, the President saw it fit to streamline the agencies so as not to hinder the delivery
continuing authority to reorganize the administrative structure. As far as bureaus, agencies or offices in the of crucial social reforms.[38]
PAGE 4 of 135 CONSTITUTIONAL LAW -EXECUTIVE DEPARTMENT PART 2 –SESSION 12
weight.[49] And so must reliance on sub-headings,[50] or the lack thereof, to support a strained deduction be given
The consolidation of functions in E.O. 364 aims to attain the objectives of simplicity, economy and efficiency as the weight of helium.
gathered from the provision granting PCUP and NCIP access to the range of services provided by the DARs
technical offices and support systems.[39] Secondary aids may be consulted to remove, not to create doubt.[51] AMINs thesis unsettles, more than
settles the order of things in construing the Constitution. Its interpretation fails to clearly establish that the so-called
The characterization of the NCIP as an independent agency under the Office of the President does not remove ordering or arrangement of provisions in the Constitution was consciously adopted to imply a signification in terms
said body from the Presidents control and supervision with respect to its performance of administrative of government hierarchy from where a constitutional mandate can per se be derived or asserted. It fails to
functions. So it has been opined: demonstrate that the ordering or layout was not simply a matter of style in constitutional drafting but one of intention
in government structuring. With its inherent ambiguity, the proposed interpretation cannot be made a basis for
That Congress did not intend to place the NCIP under the control of the President in all declaring a law or governmental act unconstitutional.
instances is evident in the IPRA itself, which provides that the decisions of the NCIP in the
exercise of its quasi-judicial functions shall be appealable to the Court of Appeals, like those of A law has in its favor the presumption of constitutionality. For it to be nullified, it must be shown that there is a
the National Labor Relations Commission (NLRC) and the Securities and Exchange clear and unequivocal breach of the Constitution. The ground for nullity must be clear and beyond reasonable
Commission (SEC). Nevertheless, the NCIP, although independent to a certain degree, was doubt.[52] Any reasonable doubt should, following the universal rule of legal hermeneutics, be resolved in favor of
placed by Congress under the office of the President and, as such, is still subject to the the constitutionality of a law.[53]
Presidents power of control and supervision granted under Section 17, Article VII of the Ople v. Torres[54] on which AMIN relies is unavailing. In that case, an administrative order involved a system of
Constitution with respect to its performance of administrative functions[.][40] (Underscoring identification that required a delicate adjustment of various contending state policies properly lodged in the
supplied) legislative arena. It was declared unconstitutional for dealing with a subject that should be covered by law and for
violating the right to privacy.

In transferring the NCIP to the DAR as an attached agency, the President effectively tempered the exercise of In the present case, AMIN glaringly failed to show how the reorganization by executive fiat would hamper the
presidential authority and considerably recognized that degree of independence. exercise of citizens rights and privileges. It rested on the ambiguous conclusion that the reorganization jeopardizes
economic, social and cultural rights. It intimated, without expounding, that the agendum behind the issuances is
The Administrative Code of 1987 categorizes administrative relationships into (1) supervision and control, (2) to weaken the indigenous peoples rights in favor of the mining industry. And it raised concerns about the possible
administrative supervision, and (3) attachment. [41] With respect to the third category, it has been held that an retrogression in DARs performance as the added workload may impede the implementation of the comprehensive
attached agency has a larger measure of independence from the Department to which it is attached than one agrarian reform program.
which is under departmental supervision and control or administrative supervision. This is borne out by the lateral AMIN has not shown, however, that by placing the NCIP as an attached agency of the DAR, the President altered
relationship between the Department and the attached agency. The attachment is merely for policy and program the nature and dynamics of the jurisdiction and adjudicatory functions of the NCIP concerning all claims and
coordination.[42]Indeed, the essential autonomous character of a board is not negated by its attachment to a disputes involving rights of indigenous cultural communities and
commission.[43]
indigenous peoples. Nor has it been shown, nay alleged, that the reorganization was made in bad faith.[55]
AMIN argues, however, that there is an anachronism of sorts because there can be no policy and program
coordination between conceptually different areas of reform. It claims that the new framework subsuming agrarian As for the other arguments raised by AMIN which pertain to the wisdom or soundness of the executive decision,
reform, urban land reform and ancestral domain reform is fundamentally incoherent in view of the widely different the Court finds it unnecessary to pass upon them. The raging debate on the most fitting framework in the delivery
contexts.[44] And it posits that it is a substantive transformation or reorientation that runs contrary to the of social services is endless in the political arena. It is not the business of this Court to join in the fray. Courts have
constitutional scheme and policies. no judicial power to review cases involving political questions and, as a rule, will desist from taking cognizance of
speculative or hypothetical cases, advisory opinions and cases that have become moot. [56]
AMIN goes on to proffer the concept of ordering the law[45] which, so it alleges, can be said of the Constitutions
distinct treatment of these three areas, as reflected in separate provisions in different parts of the Constitution.[46] It Finally, a word on the last ground proffered for declaring the unconstitutionality of the assailed issuances ─ that
argues that the Constitution did not intend an over-arching concept of agrarian reform to encompass the two other they violate Section 16, Article XIII of the Constitution[57] on the peoples right to participate in decision-making
areas, and that how the law is ordered in a certain way should not be undermined by mere executive orders in the through adequate consultation mechanisms.
guise of administrative efficiency.
The framers of the Constitution recognized that the consultation mechanisms were already operating
The Court is not persuaded. without the States action by law, such that the role of the State would be mere facilitation, not necessarily creation
of these consultation mechanisms. The State provides the support, but eventually it is the people, properly
organized in their associations, who can assert the right and pursue the objective. Penalty for failure on the part
of the government to consult could only be reflected in the ballot box and would not nullify government action. [58]
The interplay of various areas of reform in the promotion of social justice is not something implausible or
unlikely.[47] Their interlocking nature cuts across labels and works against a rigid pigeonholing of executive tasks
among the members of the Presidents official family. Notably, the Constitution inhibited from identifying and
compartmentalizing the composition of the Cabinet.In vesting executive power in one person rather than in a plural WHEREFORE, the petition is DISMISSED. Executive Order Nos. 364 and 379 issued on September 27,
executive, the evident intention was to invest the power holder with energy. [48] 2004 and October 26, 2004, respectively, are declared not unconstitutional.

AMIN takes premium on the severed treatment of these reform areas in marked provisions of the Constitution. It SO ORDERED.
is a precept, however, that inferences drawn from title, chapter or section headings are entitled to very little
PAGE 5 of 135 CONSTITUTIONAL LAW -EXECUTIVE DEPARTMENT PART 2 –SESSION 12
On July 24, 2000, the Secretary of Health issued Department Memorandum No. 136, Series of 2000, ordering the
FIRST DIVISION Undersecretary, Assistant Secretaries, Bureau or Service Directors and Program Managers of the Department of
Health to direct all employees under their respective offices to accomplish and submit the Personal Information
Sheet due to the approval of the Department of Health Rationalization and Streamlining Plan.
MALARIA EMPLOYEES AND G.R. No. 160093 On July 28, 2000, the Secretary of Health again issued Department Circular No. 221, Series of 2000, stating that
WORKERS ASSOCIATION OF the Department will start implementing the Rationalization and Streamlining Plan by a process of selection,
THE PHILIPPINES, INC. (MEWAP), placement or matching of personnel to the approved organizational chart and the list of the approved plantilla
represented by its National President, items.[3] The Secretary also issued Administrative Order (A.O.) No. 94, Series of 2000, which set the implementing
DR. RAMON A. SULLA, and MEWAP guidelines for the restructuring process on personnel selection and placement, retirement and/or voluntary
DOH Central Office Chapter President, resignation. A.O. No. 94 outlined the general guidelines for the selection and placement of employees adopting
DR. GRACELA FIDELA MINA-RAMOS, the procedures and standards set forth in R.A. No. 6656[4] or the Rules on Governmental Reorganization, Civil
and PRISCILLA CARILLO, and Service Rules and Regulations, Sections 76 to 78 of the GAA for the Year 2000, and Section 42 of E.O. No. 292.
HERMINIO JAVIER,
Petitioners, On August 29, 2000, the Secretary of Health issued Department Memorandum No. 157, Series of 2000, viz.:

Present: Pursuant to the Notice of Organization, Staffing and Compensation Action (NOSCA) approved
PUNO, C.J., Chairperson, by the DBM on 8 July 2000 and Memorandum Circular No. 62 issued by the Presidential
- versus - SANDOVAL-GUTIERREZ, Committee on Effective Governance (PCEG) on 17 July 2000, Implementing E.O. 102 dated 24
CORONA, May 1999, the following approved Placement List of DOH Personnel is hereby disseminated for
AZCUNA, and your information and guidance.
GARCIA, JJ.
All personnel are hereby directed to report to their new assignments on or before 2 October
THE HONORABLE EXECUTIVE 2000 pending processing of new appointments, required clearances and other pertinent
SECRETARY ALBERTO ROMULO, documents.
(substituting the former Executive
Secretary Renato de Villa), THE All Heads of Office/Unit in the Department of Health are hereby directed to facilitate the
HONORABLE SECRETARY OF Promulgated: implementation of E.O. 102, to include[,] among others, the transfer or movement of personnel,
HEALTH MANUEL DAYRIT properties, records and documents to appropriate office/unit and device other necessary means
and THE HONORABLE SECRETARY to minimize disruption of office functions and delivery of health services.
OF BUDGET AND MANAGEMENT
EMILIA T. BONCODIN, Appeals, oversights, issues and concerns of personnel related to this Placement List shall be
Respondents. July 31, 2007 made in writing using the Appeals Form (available at the Administrative Service) addressed to
x--------------------------------------------------x the Appeals Committee chaired by Dr. Gerardo Bayugo. All Appeals Forms shall be submitted
to the Re-Engineering Secretariat xxx not later than 18 September 2000. [5]
DECISION Petitioner Malaria Employees and Workers Association of the Philippines, Inc. (MEWAP) is a union of affected
employees in the Malaria Control Service of the Department of Health. MEWAP filed a complaint, docketed as
PUNO, C.J.: Civil Case No. 00-98793, with the Regional Trial Court of Manila seeking to nullify Department Memorandum No.
157, the NOSCA and the Placement List of Department of Health Personnel and other issuances implementing
E.O. No. 102.
At bar is a Petition for Review on Certiorari of the Decision of the Court of Appeals in CA-G.R. SP No. 65475 dated
September 12, 2003 which upheld the validity of Executive Order (E.O.) No. 102, [1] the law Redirecting the On May 2, 2001, while the civil case was pending at the Regional Trial Court of Manila, Branch 22, petitioners filed
Functions and Operations of the Department of Health. Then President Joseph E. Estrada issued E.O. No. 102 with this Court a petition for certiorari under Rule 65 of the Rules of Court. Petitioners sought to nullify E.O. No.
on May 24, 1999 pursuant to Section 20, Chapter 7, Title I, Book III of E.O. No. 292, otherwise known as the 102 for being issued with grave abuse of discretion amounting to lack or excess of jurisdiction as it allegedly
Administrative Code of 1987, and Sections 78 and 80 of Republic Act (R.A.) No. 8522, also known as the General violates certain provisions of E.O. No. 292 and R.A. No. 8522. The petition was referred to the Court of Appeals
Appropriations Act (GAA) of 1998. E.O. No. 102 provided for structural changes and redirected the functions and which dismissed the same in its assailed Decision. Hence, this appeal where petitioners ask for a re-examination
operations of the Department of Health. of the pertinent pronouncements of this Court that uphold the authority of the President to reorganize a department,
bureau or office in the executive department. Petitioners raise the following issues, viz.:
On October 19, 1999, the President issued E.O. No. 165 Directing the Formulation of an Institutional Strengthening 1. WHETHER SECTIONS 78 AND 80 OF THE GENERAL PROVISION OF REPUBLIC ACT
and Streamlining Program for the Executive Branch which created the Presidential Committee on Executive NO. 8522, OTHERWISE KNOWN AS THE GENERAL APPROPRIATION[S] ACT OF 1998[,]
Governance (PCEG) composed of the Executive Secretary as chair and the Secretary of the Department of Budget EMPOWER FORMER PRESIDENT JOSEPH E. ESTRADA TO REORGANIZE
and Management (DBM) as co-chair. STRUCTURALLY AND FUNCTIONALLY THE DEPARTMENT OF HEALTH.

The DBM, on July 8, 2000, issued the Notice of Organization, Staffing and Compensation Action (NOSCA). On July 2. WHETHER SECTION 20, CHAPTER I, TITLE I, BOOK III OF THE ADMINISTRATIVE CODE
17, 2000, the PCEG likewise issued Memorandum Circular (M.C.) No. 62, entitled Implementing Executive Order OF 1987 PROVIDES LEGAL BASIS IN REORGANIZING THE DEPARTMENT OF HEALTH.
No. 102, Series of 1999 Redirecting the Functions and Operations of the Department of Health.[2] M.C. No. 62
directed the rationalization and streamlining of the said Department.
PAGE 6 of 135 CONSTITUTIONAL LAW -EXECUTIVE DEPARTMENT PART 2 –SESSION 12
(A) WHETHER PRESIDENTIAL DECREE NO. 1416, AS AMENDED BY government, which includes the power to group, consolidate bureaus and agencies, to
PRESIDENTIAL DECREE NO. 1772, HAS BEEN REPEALED. abolish offices, to transfer functions, to create and classify functions, services and
activities and to standardize salaries and materials. The validity of these two decrees [is]
3. WHETHER THE PRESIDENT HAS AUTHORITY UNDER SECTION 17, ARTICLE VIII OF unquestionable. The 1987 Constitution clearly provides that all laws, decrees, executive orders,
THE CONSTITUTION TO EFFECT A REORGANIZATION OF A DEPARTMENT UNDER THE proclamations, letters of instructions and other executive issuances not inconsistent with this
EXECUTIVE BRANCH. Constitution shall remain operative until amended, repealed or revoked. So far, there is yet no
law amending or repealing said decrees.[12]
4. WHETHER THERE HAS BEEN ABUSE OF DISCRETION AMOUNTING TO LACK OR
EXCESS OF JURISDICTION ON THE PART OF FORMER PRESIDENT JOSEPH E. The pertinent provisions of Presidential Decree No. 1416, as amended by Presidential Decree No. 1772,
ESTRADA IN ISSUING EXECUTIVE ORDER NO. 102, REDIRECTING THE FUNCTIONS AND clearly support the Presidents continuing power to reorganize the executive branch, viz.:
OPERATIONS OF THE DEPARTMENT OF HEALTH. 1. The President of the Philippines shall have continuing authority to reorganize the
National Government. In exercising this authority, the President shall be guided by generally
5. WHETHER EXECUTIVE ORDER NO. 102 IS NULL AND VOID.[6] acceptable principles of good government and responsive national development, including but
not limited to the following guidelines for a more efficient, effective, economical and
development-oriented governmental framework:
We deny the petition. xxx
b) Abolish departments, offices, agencies or functions which may not be necessary, or
The President has the authority to carry out a reorganization of the Department of Health under the Constitution create those which are necessary, for the efficient conduct of government functions, services
and statutory laws. This authority is an adjunct of his power of control under Article VII, Sections 1 and 17 of the and activities;
1987 Constitution, viz.: c) Transfer functions, appropriations, equipment, properties, records and personnel from
Section 1. The executive power shall be vested in the President of the Philippines. one department, bureau, office, agency or instrumentality to another;
d) Create, classify, combine, split, and abolish positions;
Section 17. The President shall have control of all the executive departments, bureaus and e) Standardize salaries, materials, and equipment;
offices. He shall ensure that the laws be faithfully executed. f) Create, abolish, group, consolidate, merge, or integrate entities, agencies,
instrumentalities, and units of the National Government, as well as expand, amend, change, or
In Canonizado v. Aguirre,[7] we held that reorganization involves the reduction of personnel, consolidation of otherwise modify their powers, functions, and authorities, including, with respect to government-
offices, or abolition thereof by reason of economy or redundancy of functions. It alters the existing structure of owned or controlled corporations, their corporate life, capitalization, and other relevant aspects
government offices or units therein, including the lines of control, authority and responsibility between of their charters;
them.[8] While the power to abolish an office is generally lodged with the legislature, the authority of the President g) Take such other related actions as may be necessary to carry out the purposes and
to reorganize the executive branch, which may include such abolition, is permissible under our present laws, viz.: objectives of this Decree.
The general rule has always been that the power to abolish a public office is lodged with the Petitioners argue that the residual powers of the President under Section 20, Title I, Book III of E.O. No. 292 refer
legislature. This proceeds from the legal precept that the power to create includes the power to only to the Office of the President and not to the departments, bureaus or offices within the executive branch. They
destroy. A public office is either created by the Constitution, by statute, or by authority of law. invoke Section 31, Chapter 10, Title III, Book III of the same law, viz.:
Thus, except where the office was created by the Constitution itself, it may be abolished by the
same legislature that brought it into existence. Section 31. Continuing Authority of the President to Reorganize his Office. The President, subject to the
The exception, however, is that as far as bureaus, agencies or offices in the executive policy in the Executive Office and in order to achieve simplicity, economy and efficiency, shall
department are concerned, the Presidents power of control may justify him to inactivate the have continuing authority to reorganize the administrative structure of the Office of the
functions of a particular office, or certain laws may grant him the broad authority to carry out President. x x x
reorganization measures.[9]
The interpretation of petitioners is illogically restrictive and lacks legal basis. The residual powers granted to the
The Presidents power to reorganize the executive branch is also an exercise of his residual powers under Section President under Section 20, Title I, Book III are too broad to be construed as having a sole application to the
20, Title I, Book III of E.O. No. 292 which grants the President broad organization powers to implement Office of the President. As correctly stated by respondents, there is nothing in E.O. No. 292 which provides that
reorganization measures, viz.: the continuing authority should apply only to the Office of the President. [13] If such was the intent of the law, the
SEC. 20. Residual Powers. Unless Congress provides otherwise, the President shall exercise such same should have been expressly stated. To adopt the argument of petitioners would result to two conflicting
other powers and functions vested in the President which are provided for under the provisions in one statute. It is a basic canon of statutory construction that in interpreting a statute, care should be
laws and which are not specifically enumerated above, or which are not delegated by the taken that every part thereof be given effect, on the theory that it was enacted as an integrated measure and not
President in accordance with law.[10] as a hodge-podge of conflicting provisions. The rule is that a construction that would render a provision inoperative
should be avoided; instead, apparently inconsistent provisions should be reconciled whenever possible as parts
of a coordinated and harmonious whole.[14]
We explained the nature of the Presidents residual powers under this section in the case of Larin v. Executive
Secretary, [11] viz.: In fact, as pointed out by respondents, the Presidents power to reorganize the executive department even finds
further basis under Sections 78 and 80 of R.A. No. 8522, viz.:[15]
This provision speaks of such other powers vested in the President under the
law. What law then gives him the power to reorganize? It is Presidential Decree No. 1772 Section 78. Organizational Changes Unless otherwise provided by law or directed by the President of
which amended Presidential Decree No. 1416. These decrees expressly grant the the Philippines, no organizational unit or changes in key positions in any department or agency
President of the Philippines the continuing authority to reorganize the national
PAGE 7 of 135 CONSTITUTIONAL LAW -EXECUTIVE DEPARTMENT PART 2 –SESSION 12
shall be authorized in their respective organizational structure and funded from appropriations
provided by this Act.
Section 80. Scaling Down and Phase-out of Activities of Agencies within the Executive Branch The heads
of departments, bureaus, offices and agencies are hereby directed to identify their respective
activities which are no longer essential in the delivery of public services and which may be
scaled down, phased-out or abolished subject to Civil Service rules and regulations. Said
activities shall be reported to the Office of the President through the Department of Budget and
Management and to the Chairman, Committee on Appropriations of the House of
Representatives and the Chairman, Committee on Finance of the Senate. Actual scaling down,
phase-out or abolition of the activities shall be effected pursuant to Circulars or Orders issued
for the purpose by the Office of the President.

Petitioners contend that Section 78 refers only to changes in organizational units or key positions in any
department or agency, while Section 80 refers merely to scaling down and phasing out of activities within the
executive department. They argue that neither section authorizes reorganization. Thus, the realignment of the
appropriations to implement the reorganization of the Department of Health under E.O. No. 102 is illegal.
Again, petitioners construction of the law is unduly restrictive. This Court has consistently held
in Larin[16] and Buklod ng Kawanihang EIIB v. Zamora[17] that the corresponding pertinent provisions in the GAA
in these subject cases authorize the President to effect organizational changes in the department or agency
concerned.

Be that as it may, the President must exercise good faith in carrying out the reorganization of any branch or agency
of the executive department. Reorganization is effected in good faith if it is for the purpose of economy or to make
bureaucracy more efficient.[18] R.A. No. 6656[19] provides for the circumstances which may be considered as
evidence of bad faith in the removal of civil service employees made as a result of reorganization, to wit: (a) where
there is a significant increase in the number of positions in the new staffing pattern of the department or agency
concerned; (b) where an office is abolished and another performing substantially the same functions is created;
(c) where incumbents are replaced by those less qualified in terms of status of appointment, performance and
merit; (d) where there is a classification of offices in the department or agency concerned and the reclassified
offices perform substantially the same functions as the original offices; and (e) where the removal violates the
order of separation.

We agree with the ruling of the Court of Appeals that the President did not commit bad faith in the questioned
reorganization, viz.:

In this particular case, there is no showing that the reorganization undertaking in the
[Department of Health] had violated this requirement, nor [are] there adequate allegations to
that effect. It is only alleged that the petitioners were directly affected by the reorganization
ordered under E.O. [No.] 102. Absent is any showing that bad faith attended the actual
implementation of the said presidential issuance.

IN VIEW WHEREOF, the petition is DENIED. The assailed Decision of the Court of Appeals in CA-G.R. SP No.
65475 dated September 12, 2003 is AFFIRMED.
Costs against petitioners.

SO ORDERED.
PAGE 8 of 135 CONSTITUTIONAL LAW -EXECUTIVE DEPARTMENT PART 2 –SESSION 12
G.R. No. 140423 July 14, 2006 XXX IN HOLDING THAT THE RESOLUTIONS OF THE DEPARTMENT OF JUSTICE ASSAILED IN CA
JOSE LUIS ANGEL B. OROSA, petitioner, G.R. SP NO. 53190 ARE NOT REVIEWABLE UNDER RULE 65 (sic) OF THE 1997 RULES OF CIVIL
vs. PROCEDURE SINCE THESE RESOLUTIONS WERE ISSUED BY THE SECRETARY OF JUSTICE IN
ALBERTO C. ROA, respondent. THE EXERCISE OF HIS POWER OF CONTROL AND SUPERVISION OVER PROSECUTORS.
DECISION IV
GARCIA, J.: XXX IN NOT RESOLVING THE PETITION IN CA G.R. SP NO. 53190 ON THE MERITS.
Assailed and sought to be set aside in this petition for review is the Resolution1 dated July 8, 1999 of the Court of V
Appeals (CA) in CA-G.R. SP No. 53190, dismissing the petition for review under Rule 43 of the 1997 Rules of Civil XXX IN NOT REVERSING THE ASSAILED RESOLUTION OF THE DEPARTMENT OF JUSTICE IN CA
Procedure thereat filed by the herein petitioner from an adverse resolution of the Secretary of Justice. G.R. SP NO. 53190 ON THE FOLLOWING GROUNDS:
The petition is casts against the following factual backdrop: a. RESPONDENT'S APPEAL FROM THE RESOLUTION OF THE DEPARTMENT OF
On November 27, 1996, petitioner, a dentist by profession, filed with the Pasig City Prosecution Office a complaint- JUSTICE, THROUGH THE CHIEF STATE PROSECUTOR, DATED JANUARY 22, 1998, WAS
affidavit charging respondent Alberto C. Roa, likewise a dentist, with the crime of libel. The complaint, docketed FATALLY DEFECTIVE.
in said office as I.S. No. 96-5442, stemmed from an article entitled "Truth vs. Rumors: Questions against Dr. b. RESPONDENT'S ARTICLE WAS DEFAMATORY.
Orosa" written by respondent and published in the March-April 1996 issue of the Dental Trading Post, a bi-monthly c. MALICE ATTENDED THE PUBLICATION OF RESPONDENT'S ARTICLE.
publication of the Dental Exchange Co., Inc. In gist, the article delved into the possibility of a father, who happened d. RESPONDENT'S ARTICLE WAS NOT PROTECTED BY THE MANTLE OF PRIVILEGED
to be an examiner in a licensure examination for dentistry where his sons were examinees, manipulating the MATTER.
examinations or the results thereof to enable his children to top the same. As the Court sees it, the petition commends for its consideration the issue of whether or not a petition for review
In his complaint-affidavit, petitioner alleged that the article in question is defamatory as it besmirched his honor under Rule 43 of the 1997 Rules of Civil Procedure is a proper mode of appeal from a resolution of the Secretary
and reputation as a dentist and as the topnotcher in the dental board examinations held in May 1994. of Justice directing the prosecutor to withdraw an information in a criminal case.
Respondent denied the accusation, claiming that the article constitutes a "fair and accurate report on a matter of It is petitioner's thesis that Rule 43 was intended to apply to all quasi-judicial agencies exercising quasi-judicial
both public and social concern." He averred that the article in question was not written with malice but with a functions. Upon this premise, petitioner submits that resolutions of the DOJ in the exercise of its quasi-judicial
sincere desire to contribute to the improvement of the integrity of professional examinations. functions are properly appealable to the CA via a petition for review under Rule 43, adding that the quasi-judicial
After preliminary investigation, Pasig City Prosecutor Noel Paz issued a Resolution, dismissing petitioner's bodies enumerated under said Rule are not exclusive.
complaint in this wise: Petitioner's above posture, while valid to a point, will not carry the day for him.
The publication being a bona fide communication on matters of public concern, and made without malice, Rule 43 governs all appeals from the Court of Tax Appeals and quasi-judicial bodies to the CA. Section 1 thereof
we find the respondent entitled to the protection of the rule on privileged matters under Article 354 of the provides:
Revised Penal Code. Section 1. Scope.― This Rule shall apply to appeals from judgments or final orders of the Court of Tax
Petitioner appealed to the Department of Justice (DOJ). Acting on the appeal, Chief State Prosecutor Jovencito Appeals, and from awards, judgments, final orders or resolutions of or authorized by any quasi-judicial
Zuño issued a Resolution (Zuño Resolution), setting aside the findings of the City Prosecutor and directing the agency in the exercise of its quasi-judicial functions. Among these agencies are the Civil Service
latter to file an Information for libel against respondent. Accordingly, in the Regional Trial Court (RTC) of Pasig Commission, Central Board of Assessment Appeals, Securities and Exchange Commission, Office of the
City, an Information for libel was filed against respondent, thereat docketed as Criminal Case No. 114517. President, Land Registration Authority, Social Security Commission, Civil Aeronautics Board, Bureau of
Adversely affected, respondent appealed to the Secretary of Justice. On October 28, 1998, then Justice Secretary Patents, Trademarks and Technology Transfer, National Electrification Administration, Energy
Serafin Cuevas reversed the Zuño Resolution and directed the City Prosecutor of Pasig to withdraw the Regulatory Board, National Telecommunications Commission, Department of Agrarian Reform under
Information earlier filed with the RTC. In compliance therewith, a "Motion to Withdraw Information" was accordingly Republic Act No. 6657, Government Service and Insurance System, Employees' Compensation
filed in court by the Pasig City Prosecution Office. Commission, Agricultural Inventions Board, Insurance Commission, Philippine Atomic Energy
Petitioner seasonably moved for a reconsideration but his motion was denied by the Secretary of Justice in his Commission, Board of Investments, Construction Industry Arbitration Commission, and voluntary
Resolution of May 12, 1999. arbitrators authorized by law.
Therefrom, petitioner went to the CA on a petition for review under Rule 432 of the 1997 Rules of Civil Procedure, As may be noted, the DOJ is not among the agencies expressly enumerated under Section 1 of Rule 43, albeit
docketed as CA-G.R. No. SP No. 53190. any suggestion that it does not perform quasi-judicial functions may have to be rejected. However, its absence
As stated at the outset hereof, the CA, in the herein assailed Resolution dated July 8, 1999, dismissed petitioner's from the list of agencies mentioned thereunder does not, by this fact alone, already imply its exclusion from the
petition for review. Partly says the CA in its dismissal Resolution: coverage of said Rule. This is because said Section 1 uses the phrase "among these agencies," thereby implying
The Pasig City Prosecution Office and the Department of Justice are not among the quasi-judicial that the enumeration made is not exclusive of the agencies therein listed.
agencies included in Section 1 of Rule 43 whose final orders or resolutions are subject to review by the There is compelling reason to believe, however, that the exclusion of the DOJ from the list is deliberate, being in
Court of Appeals. consonance with the constitutional power of control4 lodged in the President over executive departments, bureaus
The Supreme Court in its Resolution En Banc dated April 8, 1997, approving the 1997 Rules of Civil and offices. This power of control, which even Congress cannot limit, let alone withdraw, means the power of the
Procedure in Bar Matter No. 803, did not include final orders or resolutions issued by these agencies as Chief Executive to review, alter, modify, nullify, or set aside what a subordinate, e.g., members of the Cabinet and
appealable under Rule 43. The Court of Appeals is therefore not at liberty to supply the omissions in the heads of line agencies, had done in the performance of their duties and to substitute the judgment of the former
Rule, that would constitute an encroachment on the rule making power of the Supreme Court.3 for that of the latter.5
With his motion for reconsideration having been denied by the CA in its subsequent Resolution of October 14, Being thus under the control of the President, the Secretary of Justice, or, to be precise, his decision is subject to
1999, petitioner is now with this Court on his submission that the appellate court erred: review of the former. In fine, recourse from the decision of the Secretary of Justice should be to the President,
I instead of the CA, under the established principle of exhaustion of administrative remedies. The thrust of the rule
XXX IN HOLDING THAT THE RESOLUTIONS OF THE DEPARTMENT OF JUSTICE ARE NOT on exhaustion of administrative remedies is that if an appeal or remedy obtains or is available within the
REVIEWABLE BY IT UNDER RULE 43 OF THE 1997 RULES OF CIVIL PROCEDURE. administrative machinery, this should be resorted to before resort can be made to the courts.6 Immediate recourse
II to the court would be premature and precipitate; 7 subject to defined exception, a case is susceptible of dismissal
XXX IN FINDING THE PETITION IN CA G.R. SP NO. 53190 [WAS] PREMATURELY FILED. for lack of cause of action should a party fail to exhaust administrative remedies. 8 Notably, Section 1, supra, of
III Rule 43 includes the Office of the President in the agencies named therein, thereby accentuating the fact that
PAGE 9 of 135 CONSTITUTIONAL LAW -EXECUTIVE DEPARTMENT PART 2 –SESSION 12
appeals from rulings of department heads must first be taken to and resolved by that office before any appellate
recourse may be resorted to.
Given the above perspective, the question of whether or not a preliminary investigation is a quasi-judicial
proceeding, as petitioner posits, or whether or not the Secretary of Justice performs quasi-judicial functions when
he reviews the findings of a state or city prosecutor is of little moment. The Court wishes, however, to draw attention
to what it said in Santos v. Go9 where the Court, citing Bautista v. Court of Appeals, 10 stated:
[t]he prosecutor in a preliminary investigation does not determine the guilt or innocence of the accused.
He does not exercise adjudication nor rule-making functions. Preliminary investigation is merely
inquisitorial, and is often the only means of discovering the persons who may be reasonably charged
with a crime and to enable the fiscal [prosecutor] to prepare his complaint or information. It is not a trial
of the case on the merits and has no purpose except that of determining whether a crime has been
committed and whether there is probable cause to believe that the accused is guilty thereof. While the
fiscal [prosecutor] makes that determination, he cannot be said to be acting as a quasi-court, for it is the
courts, ultimately that pass judgment on the accused, not the fiscal [prosecutor]. (Words in bracket ours)
While now perhaps anti-climactic to delve into, the ensuing holdings of the appellate court are worth quoting:
The petition is premature. The Information charging respondent with the crime of libel, docketed as
Criminal Case No. 114517, is now with Branch 155 of the Regional Trial Court in Pasig City. Thus
understood, the said trial court has now the control of the case. The remedy of petitioner is to reiterate
the reasons or grounds alleged in his present petition by way of an appropriate opposition to the Pasig
City Prosecution Office's "Motion to Withdraw Information" dated November 5, 1998, filed in compliance
with the assailed directive of the Secretary of Justice. Having control of the case, the trial court can look
into the claim of petitioner. This will enable the trial court to rule on the matter first without the precipitate
intervention of this Court. In other words, this is a prerequisite to the elevation of the case to this Court.11
In view of the foregoing disquisition, the Court deems it unnecessary to address the other issues raised in the
petition.
WHEREFORE, the instant petition is DENIED and the assailed resolution of the Court of Appeals is AFFIRMED.
SO ORDERED.
PAGE 10 of 135 CONSTITUTIONAL LAW -EXECUTIVE DEPARTMENT PART 2 –SESSION 12
EN BANC no matter what may be the tides of time. It cannot be simply made to sway and accommodate the call of situations
and much more tailor itself to the whims and caprices of government and the people who run it. [4]

For consideration before the Court are two consolidated cases[5] both of which essentially assail the validity and
LOUIS BAROK C. BIRAOGO, G.R. No. 192935 constitutionality of Executive Order No. 1, dated July 30, 2010, entitled Creating the Philippine Truth Commission
Petitioner, of 2010.

- versus -
The first case is G.R. No. 192935, a special civil action for prohibition instituted by petitioner Louis
THE PHILIPPINE TRUTH COMMISSION OF 2010, Biraogo (Biraogo) in his capacity as a citizen and taxpayer. Biraogo assails Executive Order No. 1 for being
Respondent. violative of the legislative power of Congress under Section 1, Article VI of the Constitution [6] as it usurps the
x-----------------------x constitutional authority of the legislature to create a public office and to appropriate funds therefor. [7]
REP. EDCEL C. LAGMAN,
REP. RODOLFO B. ALBANO, JR., REP. SIMEON A. G.R. No. 193036 The second case, G.R. No. 193036, is a special civil action for certiorari and prohibition filed by petitioners Edcel
DATUMANONG, and REP. ORLANDO B. FUA, SR., C. Lagman, Rodolfo B. Albano Jr., Simeon A. Datumanong, and Orlando B. Fua, Sr. (petitioners-legislators) as
Petitioners, Present: incumbent members of the House of Representatives.

CORONA, C.J., The genesis of the foregoing cases can be traced to the events prior to the historic May 2010 elections, when then
CARPIO, Senator Benigno Simeon Aquino III declared his staunch condemnation of graft and corruption with his
- versus - CARPIO MORALES, slogan, Kung walang corrupt, walang mahirap. The Filipino people, convinced of his sincerity and of his ability to
VELASCO, JR., carry out this noble objective, catapulted the good senator to the presidency.
NACHURA,
LEONARDO-DE CASTRO, To transform his campaign slogan into reality, President Aquino found a need for a special body to
BRION, investigate reported cases of graft and corruption allegedly committed during the previous administration.
PERALTA,
BERSAMIN, Thus, at the dawn of his administration, the President on July 30, 2010, signed Executive Order No. 1
DEL CASTILLO, establishing the Philippine Truth Commission of 2010 (Truth Commission). Pertinent provisions of said executive
ABAD, order read:
EXECUTIVE SECRETARY PAQUITO N. OCHOA, JR. VILLARAMA, JR., EXECUTIVE ORDER NO. 1
and DEPARTMENT OF BUDGET AND PEREZ,
MANAGEMENT SECRETARY FLORENCIO B. ABAD, MENDOZA, and CREATING THE PHILIPPINE TRUTH COMMISSION OF 2010
Respondents. SERENO, JJ.
WHEREAS, Article XI, Section 1 of the 1987 Constitution of the Philippines solemnly enshrines
Promulgated: the principle that a public office is a public trust and mandates that public officers and employees,
who are servants of the people, must at all times be accountable to the latter, serve them with
December 7, 2010 utmost responsibility, integrity, loyalty and efficiency, act with patriotism and justice, and lead
modest lives;
x -------------------------------------------------------------------------------------- x
WHEREAS, corruption is among the most despicable acts of defiance of this principle and
notorious violation of this mandate;

DECISION WHEREAS, corruption is an evil and scourge which seriously affects the political, economic,
and social life of a nation; in a very special way it inflicts untold misfortune and misery on the
MENDOZA, J.: poor, the marginalized and underprivileged sector of society;

When the judiciary mediates to allocate constitutional boundaries, it does not assert WHEREAS, corruption in the Philippines has reached very alarming levels, and undermined the
any superiority over the other departments; it does not in reality nullify or invalidate an act of the peoples trust and confidence in the Government and its institutions;
legislature, but only asserts the solemn and sacred obligation assigned to it by the Constitution
to determine conflicting claims of authority under the Constitution and to establish for the parties WHEREAS, there is an urgent call for the determination of the truth regarding certain reports of
in an actual controversy the rights which that instrument secures and guarantees to them. large scale graft and corruption in the government and to put a closure to them by the filing of
the appropriate cases against those involved, if warranted, and to deter others from committing
--- Justice Jose P. Laurel[1] the evil, restore the peoples faith and confidence in the Government and in their public servants;
The role of the Constitution cannot be overlooked. It is through the Constitution that the fundamental powers of
government are established, limited and defined, and by which these powers are distributed among the several WHEREAS, the Presidents battlecry during his campaign for the Presidency in the last
departments.[2] The Constitution is the basic and paramount law to which all other laws must conform and to which elections kung walang corrupt, walang mahirap expresses a solemn pledge that if elected, he
all persons, including the highest officials of the land, must defer.[3] Constitutional doctrines must remain steadfast would end corruption and the evil it breeds;
PAGE 11 of 135 CONSTITUTIONAL LAW -EXECUTIVE DEPARTMENT PART 2 –SESSION 12
g) Turn over from time to time, for expeditious prosecution, to the appropriate prosecutorial
WHEREAS, there is a need for a separate body dedicated solely to investigating and finding out authorities, by means of a special or interim report and recommendation, all evidence on
the truth concerning the reported cases of graft and corruption during the previous corruption of public officers and employees and their private sector co-principals, accomplices
administration, and which will recommend the prosecution of the offenders and secure justice or accessories, if any, when in the course of its investigation the Commission finds that there is
for all; reasonable ground to believe that they are liable for graft and corruption under pertinent
WHEREAS, Book III, Chapter 10, Section 31 of Executive Order No. 292, otherwise known as applicable laws;
the Revised Administrative Code of the Philippines, gives the President the continuing authority
to reorganize the Office of the President. h) Call upon any government investigative or prosecutorial agency such as the Department
of Justice or any of the agencies under it, and the Presidential Anti-Graft Commission, for such
NOW, THEREFORE, I, BENIGNO SIMEON AQUINO III, President of the Republic of the assistance and cooperation as it may require in the discharge of its functions and duties;
Philippines, by virtue of the powers vested in me by law, do hereby order:
i) Engage or contract the services of resource persons, professionals and other personnel
SECTION 1. Creation of a Commission. There is hereby created the PHILIPPINE TRUTH determined by it as necessary to carry out its mandate;
COMMISSION, hereinafter referred to as the COMMISSION, which shall primarily seek and find
the truth on, and toward this end, investigate reports of graft and corruption of such scale and j) Promulgate its rules and regulations or rules of procedure it deems necessary to effectively
magnitude that shock and offend the moral and ethical sensibilities of the people, committed by and efficiently carry out the objectives of this Executive Order and to ensure the orderly conduct
public officers and employees, their co-principals, accomplices and accessories from the private of its investigations, proceedings and hearings, including the presentation of evidence;
sector, if any, during the previous administration; and thereafter recommend the appropriate
action or measure to be taken thereon to ensure that the full measure of justice shall be served k) Exercise such other acts incident to or are appropriate and necessary in connection with
without fear or favor. the objectives and purposes of this Order.
The Commission shall be composed of a Chairman and four (4) members who will act as an SECTION 3. Staffing Requirements. x x x.
independent collegial body.
SECTION 4. Detail of Employees. x x x.
SECTION 2. Powers and Functions. The Commission, which shall have all the powers of an SECTION 5. Engagement of Experts. x x x
investigative body under Section 37, Chapter 9, Book I of the Administrative Code of 1987, is
primarily tasked to conduct a thorough fact-finding investigation of reported cases of graft and SECTION 6. Conduct of Proceedings. x x x.
corruption referred to in Section 1, involving third level public officers and higher, their co- SECTION 7. Right to Counsel of Witnesses/Resource Persons. x x x.
principals, accomplices and accessories from the private sector, if any, during the previous SECTION 8. Protection of Witnesses/Resource Persons. x x x.
administration and thereafter submit its finding and recommendations to the President, SECTION 9. Refusal to Obey Subpoena, Take Oath or Give Testimony. Any government official
Congress and the Ombudsman. or personnel who, without lawful excuse, fails to appear upon subpoena issued by the
In particular, it shall: Commission or who, appearing before the Commission refuses to take oath or affirmation, give
testimony or produce documents for inspection, when required, shall be subject to administrative
a) Identify and determine the reported cases of such graft and corruption which it will disciplinary action. Any private person who does the same may be dealt with in accordance with
investigate; law.
SECTION 10. Duty to Extend Assistance to the Commission. x x x.
b) Collect, receive, review and evaluate evidence related to or regarding the cases of large SECTION 11. Budget for the Commission. The Office of the President shall provide the
scale corruption which it has chosen to investigate, and to this end require any agency, official necessary funds for the Commission to ensure that it can exercise its powers, execute its
or employee of the Executive Branch, including government-owned or controlled corporations, functions, and perform its duties and responsibilities as effectively, efficiently, and expeditiously
to produce documents, books, records and other papers; as possible.
SECTION 12. Office. x x x.
c) Upon proper request or representation, obtain information and documents from the Senate
and the House of Representatives records of investigations conducted by committees thereof SECTION 13. Furniture/Equipment. x x x.
relating to matters or subjects being investigated by the Commission;
SECTION 14. Term of the Commission. The Commission shall accomplish its mission on or
d) Upon proper request and representation, obtain information from the courts, including the before December 31, 2012.
Sandiganbayan and the Office of the Court Administrator, information or documents in respect
to corruption cases filed with the Sandiganbayan or the regular courts, as the case may be; SECTION 15. Publication of Final Report. x x x.

e) Invite or subpoena witnesses and take their testimonies and for that purpose, administer SECTION 16. Transfer of Records and Facilities of the Commission. x x x.
oaths or affirmations as the case may be;
SECTION 17. Special Provision Concerning Mandate. If and when in the judgment of the
f) Recommend, in cases where there is a need to utilize any person as a state witness to President there is a need to expand the mandate of the Commission as defined in Section 1
ensure that the ends of justice be fully served, that such person who qualifies as a state witness hereof to include the investigation of cases and instances of graft and corruption during the prior
under the Revised Rules of Court of the Philippines be admitted for that purpose; administrations, such mandate may be so extended accordingly by way of a supplemental
Executive Order.
PAGE 12 of 135 CONSTITUTIONAL LAW -EXECUTIVE DEPARTMENT PART 2 –SESSION 12
The order ruled out reconciliation. It translated the Draconian code spelled out by
SECTION 18. Separability Clause. If any provision of this Order is declared unconstitutional, the Aquino in his inaugural speech: To those who talk about reconciliation, if they mean that they
same shall not affect the validity and effectivity of the other provisions hereof. would like us to simply forget about the wrongs that they have committed in the past, we have
this to say: There can be no reconciliation without justice. When we allow crimes to go
SECTION 19. Effectivity. This Executive Order shall take effect immediately. unpunished, we give consent to their occurring over and over again.

DONE in the City of Manila, Philippines, this 30th day of July 2010. The Thrusts of the Petitions

(SGD.) BENIGNO S. AQUINO III Barely a month after the issuance of Executive Order No. 1, the petitioners asked the Court to declare it
unconstitutional and to enjoin the PTC from performing its functions. A perusal of the arguments of the petitioners
By the President: in both cases shows that they are essentially the same. The petitioners-legislators summarized them in the
following manner:
(SGD.) PAQUITO N. OCHOA, JR.
Executive Secretary (a) E.O. No. 1 violates the separation of powers as it arrogates the power of the
Congress to create a public office and appropriate funds for its operation.
Nature of the Truth Commission
(b) The provision of Book III, Chapter 10, Section 31 of the Administrative Code of 1987
As can be gleaned from the above-quoted provisions, the Philippine Truth Commission (PTC) is a cannot legitimize E.O. No. 1 because the delegated authority of the President to structurally
mere ad hoc body formed under the Office of the President with the primary task to investigate reports of graft and reorganize the Office of the President to achieve economy, simplicity and efficiency does not
corruption committed by third-level public officers and employees, their co-principals, accomplices and include the power to create an entirely new public office which was hitherto inexistent like the
accessories during the previous administration, and thereafter to submit its finding and recommendations to the Truth Commission.
President, Congress and the Ombudsman. Though it has been described as an independent collegial body, it is
essentially an entity within the Office of the President Proper and subject to his control. Doubtless, it constitutes a (c) E.O. No. 1 illegally amended the Constitution and pertinent statutes when it vested
public office, as an ad hoc body is one.[8] the Truth Commission with quasi-judicial powers duplicating, if not superseding, those of the
Office of the Ombudsman created under the 1987 Constitution and the Department of Justice
To accomplish its task, the PTC shall have all the powers of an investigative body under Section 37, created under the Administrative Code of 1987.
Chapter 9, Book I of the Administrative Code of 1987. It is not, however, a quasi-judicial body as it cannot
adjudicate, arbitrate, resolve, settle, or render awards in disputes between contending parties. All it can do is (d) E.O. No. 1 violates the equal protection clause as it selectively targets for
gather, collect and assess evidence of graft and corruption and make recommendations. It may have subpoena investigation and prosecution officials and personnel of the previous administration as if
powers but it has no power to cite people in contempt, much less order their arrest. Although it is a fact-finding corruption is their peculiar species even as it excludes those of the other administrations, past
body, it cannot determine from such facts if probable cause exists as to warrant the filing of an information in our and present, who may be indictable.
courts of law. Needless to state, it cannot impose criminal, civil or administrative penalties or sanctions.
The PTC is different from the truth commissions in other countries which have been created as official, (e) The creation of the Philippine Truth Commission of 2010 violates the consistent and
transitory and non-judicial fact-finding bodies to establish the facts and context of serious violations of human general international practice of four decades wherein States constitute truth commissions to
rights or of international humanitarian law in a countrys past. [9] They are usually established by states emerging exclusively investigate human rights violations, which customary practice forms part of the
from periods of internal unrest, civil strife or authoritarianism to serve as mechanisms for transitional justice. generally accepted principles of international law which the Philippines is mandated to adhere
to pursuant to the Declaration of Principles enshrined in the Constitution.
Truth commissions have been described as bodies that share the following characteristics: (1) they
examine only past events; (2) they investigate patterns of abuse committed over a period of time, as opposed to (f) The creation of the Truth Commission is an exercise in futility, an adventure in
a particular event; (3) they are temporary bodies that finish their work with the submission of a report containing partisan hostility, a launching pad for trial/conviction by publicity and a mere populist propaganda
conclusions and recommendations; and (4) they are officially sanctioned, authorized or empowered by the to mistakenly impress the people that widespread poverty will altogether vanish if corruption is
State.[10] Commissions members are usually empowered to conduct research, support victims, and propose policy eliminated without even addressing the other major causes of poverty.
recommendations to prevent recurrence of crimes. Through their investigations, the commissions may aim to
discover and learn more about past abuses, or formally acknowledge them. They may aim to prepare the way for (g) The mere fact that previous commissions were not constitutionally challenged is of
prosecutions and recommend institutional reforms. [11] no moment because neither laches nor estoppel can bar an eventual question on the
constitutionality and validity of an executive issuance or even a statute. [13]
Thus, their main goals range from retribution to reconciliation. The Nuremburg and Tokyo war crime
tribunals are examples of a retributory or vindicatory body set up to try and punish those responsible for crimes
against humanity. A form of a reconciliatory tribunal is the Truth and Reconciliation Commission of South Africa, In their Consolidated Comment,[14] the respondents, through the Office of the Solicitor
the principal function of which was to heal the wounds of past violence and to prevent future conflict by providing General (OSG), essentially questioned the legal standing of petitioners and defended the assailed executive order
a cathartic experience for victims. with the following arguments:

The PTC is a far cry from South Africas model. The latter placed more emphasis on reconciliation than 1] E.O. No. 1 does not arrogate the powers of Congress to create a public office because
on judicial retribution, while the marching order of the PTC is the identification and punishment of perpetrators. As the Presidents executive power and power of control necessarily include the inherent power to
one writer[12] puts it: conduct investigations to ensure that laws are faithfully executed and that, in any event, the
PAGE 13 of 135 CONSTITUTIONAL LAW -EXECUTIVE DEPARTMENT PART 2 –SESSION 12
Constitution, Revised Administrative Code of 1987 (E.O. No. 292), [15]Presidential Decree (P.D.) claiming to be the subject of the commissions investigations, petitioners will not sustain injury in its creation or as
No. 1416[16] (as amended by P.D. No. 1772), R.A. No. 9970,[17] and settled jurisprudence that a result of its proceedings.[20]
authorize the President to create or form such bodies.
The Court disagrees with the OSG in questioning the legal standing of the petitioners-legislators to
2] E.O. No. 1 does not usurp the power of Congress to appropriate funds because there assail Executive Order No. 1. Evidently, their petition primarily invokes usurpation of the power of the Congress
is no appropriation but a mere allocation of funds already appropriated by Congress. as a body to which they belong as members. This certainly justifies their resolve to take the cudgels for Congress
as an institution and present the complaints on the usurpation of their power and rights as members of the
3] The Truth Commission does not duplicate or supersede the functions of the Office of legislature before the Court. As held in Philippine Constitution Association v. Enriquez,[21]
the Ombudsman (Ombudsman) and the Department of Justice (DOJ), because it is a fact-
finding body and not a quasi-judicial body and its functions do not duplicate, supplant or erode To the extent the powers of Congress are impaired, so is the power of each member
the latters jurisdiction. thereof, since his office confers a right to participate in the exercise of the powers of that
institution.
4] The Truth Commission does not violate the equal protection clause because it was An act of the Executive which injures the institution of Congress causes a derivative
validly created for laudable purposes. but nonetheless substantial injury, which can be questioned by a member of Congress. In such
a case, any member of Congress can have a resort to the courts.

The OSG then points to the continued existence and validity of other executive orders and presidential Indeed, legislators have a legal standing to see to it that the prerogative, powers and privileges vested
issuances creating similar bodies to justify the creation of the PTC such as Presidential Complaint and Action by the Constitution in their office remain inviolate. Thus, they are allowed to question the validity of any official
Commission (PCAC) by President Ramon B. Magsaysay, Presidential Committee on Administrative Performance action which, to their mind, infringes on their prerogatives as legislators. [22]
Efficiency (PCAPE) by President Carlos P. Garcia and Presidential Agency on Reform and Government
Operations (PARGO) by President Ferdinand E. Marcos.[18] With regard to Biraogo, the OSG argues that, as a taxpayer, he has no standing to question the creation
From the petitions, pleadings, transcripts, and memoranda, the following are the principal issues to be of the PTC and the budget for its operations. [23] It emphasizes that the funds to be used for the creation and
resolved: operation of the commission are to be taken from those funds already appropriated by Congress. Thus, the
allocation and disbursement of funds for the commission will not entail congressional action but will simply be an
1. Whether or not the petitioners have the legal standing to file their exercise of the Presidents power over contingent funds.
respective petitions and question Executive Order No. 1;
As correctly pointed out by the OSG, Biraogo has not shown that he sustained, or is in danger of
2. Whether or not Executive Order No. 1 violates the principle of sustaining, any personal and direct injury attributable to the implementation of Executive Order No. 1. Nowhere in
separation of powers by usurping the powers of Congress to create and to appropriate funds for his petition is an assertion of a clear right that may justify his clamor for the Court to exercise judicial power and
public offices, agencies and commissions; to wield the axe over presidential issuances in defense of the Constitution. The case of David v.
3. Whether or not Executive Order No. 1 supplants the powers of the Ombudsman and Arroyo[24] explained the deep-seated rules on locus standi. Thus:
the DOJ;
Locus standi is defined as a right of appearance in a court of justice on a given
4. Whether or not Executive Order No. 1 violates the equal protection clause; and question. In private suits, standing is governed by the real-parties-in interest rule as contained
in Section 2, Rule 3 of the 1997 Rules of Civil Procedure, as amended. It provides that every
5. Whether or not petitioners are entitled to injunctive relief. action must be prosecuted or defended in the name of the real party in
interest. Accordingly, the real-party-in interest is the party who stands to be benefited or injured
Essential requisites for judicial review by the judgment in the suit or the party entitled to the avails of the suit. Succinctly put, the
plaintiffs standing is based on his own right to the relief sought.
Before proceeding to resolve the issue of the constitutionality of Executive Order No. 1, the Court needs to
ascertain whether the requisites for a valid exercise of its power of judicial review are present. The difficulty of determining locus standi arises in public suits. Here, the plaintiff who
asserts a public right in assailing an allegedly illegal official action, does so as a representative
Like almost all powers conferred by the Constitution, the power of judicial review is subject to limitations, to wit: of the general public. He may be a person who is affected no differently from any other
(1) there must be an actual case or controversy calling for the exercise of judicial power; (2) the person challenging person. He could be suing as a stranger, or in the category of a citizen, or taxpayer. In either
the act must have the standing to question the validity of the subject act or issuance; otherwise stated, he must case, he has to adequately show that he is entitled to seek judicial protection. In other words,
have a personal and substantial interest in the case such that he has sustained, or will sustain, direct injury as a he has to make out a sufficient interest in the vindication of the public order and the securing of
result of its enforcement; (3) the question of constitutionality must be raised at the earliest opportunity; and (4) the relief as a citizen or taxpayer.
issue of constitutionality must be the very lis mota of the case.[19]
Case law in most jurisdictions now allows both citizen and taxpayer standing in public
Among all these limitations, only the legal standing of the petitioners has been put at issue. actions. The distinction was first laid down in Beauchamp v. Silk, where it was held that the
plaintiff in a taxpayers suit is in a different category from the plaintiff in a citizens suit. In the
Legal Standing of the Petitioners former, the plaintiff is affected by the expenditure of public funds, while in the latter, he is but the
mere instrument of the public concern. As held by the New York Supreme Court in People ex
The OSG attacks the legal personality of the petitioners-legislators to file their petition for failure to rel Case v. Collins: In matter of mere public right, howeverthe people are the real partiesIt is at
demonstrate their personal stake in the outcome of the case. It argues that the petitioners have not shown that least the right, if not the duty, of every citizen to interfere and see that a public offence be properly
they have sustained or are in danger of sustaining any personal injury attributable to the creation of the PTC. Not pursued and punished, and that a public grievance be remedied. With respect to taxpayers
PAGE 14 of 135 CONSTITUTIONAL LAW -EXECUTIVE DEPARTMENT PART 2 –SESSION 12
suits, Terr v. Jordan held that the right of a citizen and a taxpayer to maintain an action in courts
to restrain the unlawful use of public funds to his injury cannot be denied. Similarly, in G.R. No. 193036, petitioners-legislators argue that the creation of a public office lies within
the province of Congress and not with the executive branch of government. They maintain that the delegated
However, to prevent just about any person from seeking judicial interference in any authority of the President to reorganize under Section 31 of the Revised Administrative Code: 1) does not permit
official policy or act with which he disagreed with, and thus hinders the activities of governmental the President to create a public office, much less a truth commission; 2) is limited to the reorganization of the
agencies engaged in public service, the United State Supreme Court laid down the more administrative structure of the Office of the President; 3) is limited to the restructuring of the internal organs of the
stringent direct injury test in Ex Parte Levitt, later reaffirmed in Tileston v. Ullman. The same Office of the President Proper, transfer of functions and transfer of agencies; and 4) only to achieve simplicity,
Court ruled that for a private individual to invoke the judicial power to determine the validity of economy and efficiency.[36] Such continuing authority of the President to reorganize his office is limited, and by
an executive or legislative action, he must show that he has sustained a direct injury as a issuing Executive Order No. 1, the President overstepped the limits of this delegated authority.
result of that action, and it is not sufficient that he has a general interest common to all
members of the public. The OSG counters that there is nothing exclusively legislative about the creation by the President of a
fact-finding body such as a truth commission. Pointing to numerous offices created by past presidents, it argues
This Court adopted the direct injury test in our jurisdiction. In People v. Vera, it held that the authority of the President to create public offices within the Office of the President Proper has long been
that the person who impugns the validity of a statute must have a personal and substantial recognized.[37] According to the OSG, the Executive, just like the other two branches of government, possesses
interest in the case such that he has sustained, or will sustain direct injury as a the inherent authority to create fact-finding committees to assist it in the performance of its constitutionally
result. The Vera doctrine was upheld in a litany of cases, such as, Custodio v. President of the mandated functions and in the exercise of its administrative functions. [38] This power, as the OSG explains it, is
Senate, Manila Race Horse Trainers Association v. De la Fuente, Pascual v. Secretary of Public but an adjunct of the plenary powers wielded by the President under Section 1 and his power of control under
Works and Anti-Chinese League of the Philippines v. Felix. [Emphases included. Citations Section 17, both of Article VII of the Constitution.[39]
omitted]
It contends that the President is necessarily vested with the power to conduct fact-finding investigations,
pursuant to his duty to ensure that all laws are enforced by public officials and employees of his department and
Notwithstanding, the Court leans on the doctrine that the rule on standing is a matter of procedure, hence, in the exercise of his authority to assume directly the functions of the executive department, bureau and office, or
can be relaxed for nontraditional plaintiffs like ordinary citizens, taxpayers, and legislators when the public interest interfere with the discretion of his officials.[40]The power of the President to investigate is not limited to the exercise
so requires, such as when the matter is of transcendental importance, of overreaching significance to society, of his power of control over his subordinates in the executive branch, but extends further in the exercise of his
or of paramount public interest.[25] other powers, such as his power to discipline subordinates, [41] his power for rule making, adjudication and licensing
purposes[42] and in order to be informed on matters which he is entitled to know. [43]
Thus, in Coconut Oil Refiners Association, Inc. v. Torres,[26] the Court held that in cases of paramount
importance where serious constitutional questions are involved, the standing requirements may be relaxed and a The OSG also cites the recent case of Banda v. Ermita,[44] where it was held that the President has the
suit may be allowed to prosper even where there is no direct injury to the party claiming the right of judicial power to reorganize the offices and agencies in the executive department in line with his constitutionally granted
review. In the first Emergency Powers Cases,[27]ordinary citizens and taxpayers were allowed to question the power of control and by virtue of a valid delegation of the legislative power to reorganize executive offices under
constitutionality of several executive orders although they had only an indirect and general interest shared in existing statutes.
common with the public.
Thus, the OSG concludes that the power of control necessarily includes the power to create offices. For
The OSG claims that the determinants of transcendental importance[28] laid down in CREBA v. ERC and the OSG, the President may create the PTC in order to, among others, put a closure to the reported large scale
[29]
Meralco are non-existent in this case. The Court, however, finds reason in Biraogos assertion that the petition graft and corruption in the government.[45]
covers matters of transcendental importance to justify the exercise of jurisdiction by the Court. There
are constitutional issues in the petition which deserve the attention of this Court in view of their seriousness, The question, therefore, before the Court is this: Does the creation of the PTC fall within the ambit of the
novelty and weight as precedents. Where the issues are of transcendental and paramount importance not only to power to reorganize as expressed in Section 31 of the Revised Administrative Code? Section 31 contemplates
the public but also to the Bench and the Bar, they should be resolved for the guidance of all.[30] Undoubtedly, the reorganization as limited by the following functional and structural lines: (1) restructuring the internal organization
Filipino people are more than interested to know the status of the Presidents first effort to bring about a promised of the Office of the President Proper by abolishing, consolidating or merging units thereof or transferring functions
change to the country. The Court takes cognizance of the petition not due to overwhelming political undertones from one unit to another; (2) transferring any function under the Office of the President to any other
that clothe the issue in the eyes of the public, but because the Court stands firm in its oath to perform its Department/Agency or vice versa; or (3) transferring any agency under the Office of the President to any other
constitutional duty to settle legal controversies with overreaching significance to society. Department/Agency or vice versa. Clearly, the provision refers to reduction of personnel, consolidation of offices,
or abolition thereof by reason of economy or redundancy of functions. These point to situations where a body or
Power of the President to Create the Truth Commission an office is already existent but a modification or alteration thereof has to be effected. The creation of an office is
nowhere mentioned, much less envisioned in said provision. Accordingly, the answer to the question is in the
In his memorandum in G.R. No. 192935, Biraogo asserts that the Truth Commission is a public office and negative.
not merely an adjunct body of the Office of the President. [31] Thus, in order that the President may create a public
office he must be empowered by the Constitution, a statute or an authorization vested in him by law. According to To say that the PTC is borne out of a restructuring of the Office of the President under Section 31 is a
petitioner, such power cannot be presumed[32] since there is no provision in the Constitution or any specific law misplaced supposition, even in the plainest meaning attributable to the term restructure an alteration of an existing
that authorizes the President to create a truth commission. [33] He adds that Section 31 of the Administrative Code structure. Evidently, the PTC was not part of the structure of the Office of the President prior to the enactment of
of 1987, granting the President the continuing authority to reorganize his office, cannot serve as basis for the Executive Order No. 1. As held in Buklod ng Kawaning EIIB v. Hon. Executive Secretary, [46]
creation of a truth commission considering the aforesaid provision merely uses verbs such as reorganize, transfer,
consolidate, merge, and abolish.[34] Insofar as it vests in the President the plenary power to reorganize the Office
of the President to the extent of creating a public office, Section 31 is inconsistent with the principle of separation But of course, the list of legal basis authorizing the President to reorganize any
of powers enshrined in the Constitution and must be deemed repealed upon the effectivity thereof. [35] department or agency in the executive branch does not have to end here. We must not lose
PAGE 15 of 135 CONSTITUTIONAL LAW -EXECUTIVE DEPARTMENT PART 2 –SESSION 12
sight of the very source of the power that which constitutes an express grant of power. Under
Section 31, Book III of Executive Order No. 292 (otherwise known as the Administrative Code ASSOCIATE JUSTICE CARPIO: That is why, that P.D. 1416 was issued. Now would you agree
of 1987), "the President, subject to the policy in the Executive Office and in order to achieve with me that P.D. 1416 should not be
simplicity, economy and efficiency, shall have the continuing authority to reorganize the considered effective anymore upon the
administrative structure of the Office of the President." For this purpose, he may transfer the promulgation, adoption, ratification of the 1987
functions of other Departments or Agencies to the Office of the President. In Canonizado v. Constitution.
Aguirre [323 SCRA 312 (2000)], we ruled that reorganization "involves the reduction of
personnel, consolidation of offices, or abolition thereof by reason of economy or redundancy of SOLICITOR GENERAL CADIZ: Not the whole of P.D. [No.] 1416, Your Honor.
functions." It takes place when there is an alteration of the existing structure of
government offices or units therein, including the lines of control, authority and ASSOCIATE JUSTICE CARPIO: The power of the President to reorganize the entire National
responsibility between them. The EIIB is a bureau attached to the Department of Finance. It Government is deemed repealed, at least, upon
falls under the Office of the President. Hence, it is subject to the Presidents continuing authority the adoption of the 1987 Constitution, correct.
to reorganize. [Emphasis Supplied]
SOLICITOR GENERAL CADIZ: Yes, Your Honor.[50]

In the same vein, the creation of the PTC is not justified by the Presidents power of control. Control is
essentially the power to alter or modify or nullify or set aside what a subordinate officer had done in the While the power to create a truth commission cannot pass muster on the basis of P.D. No. 1416 as amended by
performance of his duties and to substitute the judgment of the former with that of the latter. [47] Clearly, the power P.D. No. 1772, the creation of the PTC finds justification under Section 17, Article VII of the Constitution, imposing
of control is entirely different from the power to create public offices. The former is inherent in the Executive, while upon the President the duty to ensure that the laws are faithfully executed. Section 17 reads:
the latter finds basis from either a valid delegation from Congress, or his inherent duty to faithfully execute the
laws. Section 17. The President shall have control of all the executive departments, bureaus,
and offices. He shall ensure that the laws be faithfully executed. (Emphasis supplied).
The question is this, is there a valid delegation of power from Congress, empowering the President to
create a public office? As correctly pointed out by the respondents, the allocation of power in the three principal branches of
government is a grant of all powers inherent in them. The Presidents power to conduct investigations to aid him in
According to the OSG, the power to create a truth commission pursuant to the above provision finds ensuring the faithful execution of laws in this case, fundamental laws on public accountability and transparency is
statutory basis under P.D. 1416, as amended by P.D. No. 1772. [48] The said law granted the President the inherent in the Presidents powers as the Chief Executive. That the authority of the President to conduct
continuing authority to reorganize the national government, including the power to group, consolidate bureaus and investigations and to create bodies to execute this power is not explicitly mentioned in the Constitution or in statutes
agencies, to abolish offices, to transfer functions, to create and classify functions, services and activities, transfer does not mean that he is bereft of such authority. [51] As explained in the landmark case of Marcos v. Manglapus:[52]
appropriations, and to standardize salaries and materials. This decree, in relation to Section 20, Title I, Book III of
E.O. 292 has been invoked in several cases such as Larin v. Executive Secretary.[49] x x x. The 1987 Constitution, however, brought back the presidential system of
government and restored the separation of legislative, executive and judicial powers by their
The Court, however, declines to recognize P.D. No. 1416 as a justification for the President to create a actual distribution among three distinct branches of government with provision for checks and
public office. Said decree is already stale, anachronistic and inoperable. P.D. No. 1416 was a delegation to then balances.
President Marcos of the authority to reorganize the administrative structure of the national government including
the power to create offices and transfer appropriations pursuant to one of the purposes of the decree, embodied It would not be accurate, however, to state that "executive power" is the power to
in its last Whereas clause: enforce the laws, for the President is head of state as well as head of government and whatever
powers inhere in such positions pertain to the office unless the Constitution itself withholds
WHEREAS, the transition towards the parliamentary form of government will it. Furthermore, the Constitution itself provides that the execution of the laws is only one of the
necessitate flexibility in the organization of the national government. powers of the President. It also grants the President other powers that do not involve the
execution of any provision of law, e.g., his power over the country's foreign relations.

Clearly, as it was only for the purpose of providing manageability and resiliency during the interim, P.D.
No. 1416, as amended by P.D. No. 1772, became functus oficio upon the convening of the First Congress, as On these premises, we hold the view that although the 1987 Constitution imposes
expressly provided in Section 6, Article XVIII of the 1987 Constitution. In fact, even the Solicitor General agrees limitations on the exercise of specific powers of the President, it maintains intact what is
with this view. Thus: traditionally considered as within the scope of "executive power." Corollarily, the powers of the
President cannot be said to be limited only to the specific powers enumerated in the
Constitution. In other words, executive power is more than the sum of specific powers so
ASSOCIATE JUSTICE CARPIO: Because P.D. 1416 was enacted was the last whereas clause enumerated.
of P.D. 1416 says it was enacted to prepare the
transition from presidential to parliamentary. It has been advanced that whatever power inherent in the government that is neither
Now, in a parliamentary form of government, legislative nor judicial has to be executive. x x x.
the legislative and executive powers are fused,
correct?
Indeed, the Executive is given much leeway in ensuring that our laws are faithfully executed. As stated above, the
SOLICITOR GENERAL CADIZ: Yes, Your Honor. powers of the President are not limited to those specific powers under the Constitution. [53] One of the recognized
PAGE 16 of 135 CONSTITUTIONAL LAW -EXECUTIVE DEPARTMENT PART 2 –SESSION 12
powers of the President granted pursuant to this constitutionally-mandated duty is the power to create ad or resolving a controversy involved in the facts inquired into by application of the law to the facts
hoc committees. This flows from the obvious need to ascertain facts and determine if laws have been faithfully established by the inquiry.
executed. Thus, in Department of Health v. Camposano,[54] the authority of the President to issue Administrative
Order No. 298, creating an investigative committee to look into the administrative charges filed against the The legal meaning of "investigate" is essentially the same: "(t)o follow up step by step
employees of the Department of Health for the anomalous purchase of medicines was upheld. In said case, it was by patient inquiry or observation. To trace or track; to search into; to examine and inquire into
ruled: with care and accuracy; to find out by careful inquisition; examination; the taking of evidence; a
legal inquiry;" "to inquire; to make an investigation," "investigation" being in turn described as
The Chief Executives power to create the Ad hoc Investigating Committee cannot be "(a)n administrative function, the exercise of which ordinarily does not require a hearing. 2 Am
doubted. Having been constitutionally granted full control of the Executive Department, to which J2d Adm L Sec. 257; x x an inquiry, judicial or otherwise, for the discovery and collection of facts
respondents belong, the President has the obligation to ensure that all executive officials and concerning a certain matter or matters."
employees faithfully comply with the law. With AO 298 as mandate, the legality of the
investigation is sustained.Such validity is not affected by the fact that the investigating team and "Adjudicate," commonly or popularly understood, means to adjudge, arbitrate, judge,
the PCAGC had the same composition, or that the former used the offices and facilities of the decide, determine, resolve, rule on, settle. The dictionary defines the term as "to settle finally
latter in conducting the inquiry. [Emphasis supplied] (the rights and duties of the parties to a court case) on the merits of issues raised: x x to pass
judgment on: settle judicially: x x act as judge." And "adjudge" means "to decide or rule upon as
It should be stressed that the purpose of allowing ad hoc investigating bodies to exist is to allow an inquiry a judge or with judicial or quasi-judicial powers: x x to award or grant judicially in a case of
into matters which the President is entitled to know so that he can be properly advised and guided in the controversy x x."
performance of his duties relative to the execution and enforcement of the laws of the land. And if history is to be In the legal sense, "adjudicate" means: "To settle in the exercise of judicial authority.
revisited, this was also the objective of the investigative bodies created in the past like the PCAC, PCAPE, To determine finally. Synonymous with adjudge in its strictest sense;" and "adjudge" means: "To
PARGO, the Feliciano Commission, the Melo Commission and the Zenarosa Commission. There being no pass on judicially, to decide, settle or decree, or to sentence or condemn. x x. Implies a judicial
changes in the government structure, the Court is not inclined to declare such executive power as non-existent determination of a fact, and the entry of a judgment." [Italics included. Citations Omitted]
just because the direction of the political winds have changed.
Fact-finding is not adjudication and it cannot be likened to the judicial function of a court of justice, or
On the charge that Executive Order No. 1 transgresses the power of Congress to appropriate funds for even a quasi-judicial agency or office. The function of receiving evidence and ascertaining therefrom the facts of
the operation of a public office, suffice it to say that there will be no appropriation but only an allotment or allocations a controversy is not a judicial function. To be considered as such, the act of receiving evidence and arriving at
of existing funds already appropriated. Accordingly, there is no usurpation on the part of the Executive of the power factual conclusions in a controversy must be accompanied by the authority of applying the law to the factual
of Congress to appropriate funds. Further, there is no need to specify the amount to be earmarked for the operation conclusions to the end that the controversy may be decided or resolved authoritatively, finally and definitively,
of the commission because, in the words of the Solicitor General, whatever funds the Congress has provided for subject to appeals or modes of review as may be provided by law.[60] Even respondents themselves admit that the
the Office of the President will be the very source of the funds for the commission. [55] Moreover, since the amount commission is bereft of any quasi-judicial power.[61]
that would be allocated to the PTC shall be subject to existing auditing rules and regulations, there is no impropriety
in the funding. Contrary to petitioners apprehension, the PTC will not supplant the Ombudsman or the DOJ or erode their
respective powers. If at all, the investigative function of the commission will complement those of the two offices. As
Power of the Truth Commission to Investigate pointed out by the Solicitor General, the recommendation to prosecute is but a consequence of the overall task of
the commission to conduct a fact-finding investigation.[62] The actual prosecution of suspected offenders, much
The Presidents power to conduct investigations to ensure that laws are faithfully executed is well less adjudication on the merits of the charges against them, [63] is certainly not a function given to the
recognized. It flows from the faithful-execution clause of the Constitution under Article VII, Section 17 thereof.[56] As commission.The phrase, when in the course of its investigation, under Section 2(g), highlights this fact and gives
the Chief Executive, the president represents the government as a whole and sees to it that all laws are enforced credence to a contrary interpretation from that of the petitioners. The function of determining probable cause for
by the officials and employees of his department. He has the authority to directly assume the functions of the the filing of the appropriate complaints before the courts remains to be with the DOJ and the Ombudsman. [64]
executive department.[57]
At any rate, the Ombudsmans power to investigate under R.A. No. 6770 is not exclusive but is shared with other
Invoking this authority, the President constituted the PTC to primarily investigate reports of graft and corruption similarly authorized government agencies. Thus, in the case of Ombudsman v. Galicia,[65] it was written:
and to recommend the appropriate action. As previously stated, no quasi-judicial powers have been vested in the
said body as it cannot adjudicate rights of persons who come before it. It has been said that Quasi-judicial powers This power of investigation granted to the Ombudsman by the 1987 Constitution and The
involve the power to hear and determine questions of fact to which the legislative policy is to apply and to decide Ombudsman Act is not exclusive but is shared with other similarly authorized government
in accordance with the standards laid down by law itself in enforcing and administering the same law. [58] In simpler agencies such as the PCGG and judges of municipal trial courts and municipal circuit trial
terms, judicial discretion is involved in the exercise of these quasi-judicial power, such that it is exclusively vested courts. The power to conduct preliminary investigation on charges against public employees
in the judiciary and must be clearly authorized by the legislature in the case of administrative agencies. and officials is likewise concurrently shared with the Department of Justice. Despite the passage
of the Local Government Code in 1991, the Ombudsman retains concurrent jurisdiction with the
The distinction between the power to investigate and the power to adjudicate was delineated by the Court Office of the President and the local Sanggunians to investigate complaints against local elective
in Cario v. Commission on Human Rights.[59] Thus: officials. [Emphasis supplied].

"Investigate," commonly understood, means to examine, explore, inquire or delve or


probe into, research on, study. The dictionary definition of "investigate" is "to observe or study Also, Executive Order No. 1 cannot contravene the power of the Ombudsman to investigate criminal cases under
closely: inquire into systematically: "to search or inquire into: x x to subject to an official probe x Section 15 (1) of R.A. No. 6770, which states:
x: to conduct an official inquiry." The purpose of investigation, of course, is to discover, to find
out, to learn, obtain information. Nowhere included or intimated is the notion of settling, deciding
PAGE 17 of 135 CONSTITUTIONAL LAW -EXECUTIVE DEPARTMENT PART 2 –SESSION 12
(1) Investigate and prosecute on its own or on complaint by any person, any act or and corruption solely during the said administration.[71] Assuming arguendo that the commission would confine its
omission of any public officer or employee, office or agency, when such act or omission appears proceedings to officials of the previous administration, the petitioners argue that no offense is committed against
to be illegal, unjust, improper or inefficient. It has primary jurisdiction over cases cognizable the equal protection clause for the segregation of the transactions of public officers during the previous
by the Sandiganbayan and, in the exercise of its primary jurisdiction, it may take over, at any administration as possible subjects of investigation is a valid classification based on substantial distinctions and is
stage, from any investigatory agency of government, the investigation of such germane to the evils which the Executive Order seeks to correct. [72] To distinguish the Arroyo administration from
cases. [Emphases supplied] past administrations, it recited the following:

First. E.O. No. 1 was issued in view of widespread reports of large scale graft and
The act of investigation by the Ombudsman as enunciated above contemplates the conduct of a corruption in the previous administration which have eroded public confidence in public
preliminary investigation or the determination of the existence of probable cause. This is categorically out of the institutions. There is, therefore, an urgent call for the determination of the truth regarding certain
PTCs sphere of functions. Its power to investigate is limited to obtaining facts so that it can advise and guide the reports of large scale graft and corruption in the government and to put a closure to them by the
President in the performance of his duties relative to the execution and enforcement of the laws of the land. In this filing of the appropriate cases against those involved, if warranted, and to deter others from
regard, the PTC commits no act of usurpation of the Ombudsmans primordial duties. committing the evil, restore the peoples faith and confidence in the Government and in their
public servants.
The same holds true with respect to the DOJ. Its authority under Section 3 (2), Chapter 1, Title III, Book IV in the
Revised Administrative Code is by no means exclusive and, thus, can be shared with a body likewise tasked to Second. The segregation of the preceding administration as the object of fact-finding
investigate the commission of crimes. is warranted by the reality that unlike with administrations long gone, the current administration
will most likely bear the immediate consequence of the policies of the previous administration.
Finally, nowhere in Executive Order No. 1 can it be inferred that the findings of the PTC are to be accorded
conclusiveness. Much like its predecessors, the Davide Commission, the Feliciano Commission and the Zenarosa Third. The classification of the previous administration as a separate class for
Commission, its findings would, at best, be recommendatory in nature. And being so, the Ombudsman and the investigation lies in the reality that the evidence of possible criminal activity, the evidence that
DOJ have a wider degree of latitude to decide whether or not to reject the recommendation. These offices, could lead to recovery of public monies illegally dissipated, the policy lessons to be learned to
therefore, are not deprived of their mandated duties but will instead be aided by the reports of the PTC for possible ensure that anti-corruption laws are faithfully executed, are more easily established in the
indictments for violations of graft laws. regime that immediately precede the current administration.

Violation of the Equal Protection Clause Fourth. Many administrations subject the transactions of their predecessors to
investigations to provide closure to issues that are pivotal to national life or even as a routine
Although the purpose of the Truth Commission falls within the investigative power of the President, the measure of due diligence and good housekeeping by a nascent administration like the
Court finds difficulty in upholding the constitutionality of Executive Order No. 1 in view of its apparent transgression Presidential Commission on Good Government (PCGG), created by the late President Corazon
of the equal protection clause enshrined in Section 1, Article III (Bill of Rights) of the 1987 Constitution. Section 1 C. Aquino under Executive Order No. 1 to pursue the recovery of ill-gotten wealth of her
reads: predecessor former President Ferdinand Marcos and his cronies, and
the Saguisag Commission created by former President Joseph Estrada under Administrative
Section 1. No person shall be deprived of life, liberty, or property without due process Order No, 53, to form an ad-hoc and independent citizens committee to investigate all the facts
of law, nor shall any person be denied the equal protection of the laws. and circumstances surrounding Philippine Centennial projects of his predecessor, former
President Fidel V. Ramos.[73] [Emphases supplied]
The petitioners assail Executive Order No. 1 because it is violative of this constitutional safeguard. They
contend that it does not apply equally to all members of the same class such that the intent of singling out the Concept of the Equal Protection Clause
previous administration as its sole object makes the PTC an adventure in partisan hostility.[66] Thus, in order to be
accorded with validity, the commission must also cover reports of graft and corruption in virtually all administrations One of the basic principles on which this government was founded is that of the equality of right which is embodied
previous to that of former President Arroyo.[67] in Section 1, Article III of the 1987 Constitution. The equal protection of the laws is embraced in the concept of
due process, as every unfair discrimination offends the requirements of justice and fair play. It has been embodied
The petitioners argue that the search for truth behind the reported cases of graft and corruption must in a separate clause, however, to provide for a more specific guaranty against any form of undue favoritism or
encompass acts committed not only during the administration of former President Arroyo but also during prior hostility from the government. Arbitrariness in general may be challenged on the basis of the due process clause.
administrations where the same magnitude of controversies and anomalies [68] were reported to have been But if the particular act assailed partakes of an unwarranted partiality or prejudice, the sharper weapon to cut it
committed against the Filipino people. They assail the classification formulated by the respondents as it does not down is the equal protection clause.[74]
fall under the recognized exceptions because first, there is no substantial distinction between the group of officials
targeted for investigation by Executive Order No. 1 and other groups or persons who abused their public office for According to a long line of decisions, equal protection simply requires that all persons or things similarly
personal gain; and second, the selective classification is not germane to the purpose of Executive Order No. 1 to situated should be treated alike, both as to rights conferred and responsibilities imposed. [75] It requires public
end corruption.[69] In order to attain constitutional permission, the petitioners advocate that the commission should bodies and institutions to treat similarly situated individuals in a similar manner.[76] The purpose of the equal
deal with graft and grafters prior and subsequent to the Arroyo administration with the strong arm of the law with protection clause is to secure every person within a states jurisdiction against intentional and arbitrary
equal force.[70] discrimination, whether occasioned by the express terms of a statue or by its improper execution through the
states duly constituted authorities.[77] In other words, the concept of equal justice under the law requires the state
Position of respondents to govern impartially, and it may not draw distinctions between individuals solely on differences that are irrelevant
to a legitimate governmental objective. [78]
According to respondents, while Executive Order No. 1 identifies the previous administration as the initial
subject of the investigation, following Section 17 thereof, the PTC will not confine itself to cases of large scale graft
PAGE 18 of 135 CONSTITUTIONAL LAW -EXECUTIVE DEPARTMENT PART 2 –SESSION 12
The equal protection clause is aimed at all official state actions, not just those of the legislature. [79] Its WHEREAS, there is a need for a separate body dedicated solely to investigating and finding out
inhibitions cover all the departments of the government including the political and executive departments, and the truth concerning the reported cases of graft and corruption during the previous
extend to all actions of a state denying equal protection of the laws, through whatever agency or whatever guise administration, and which will recommend the prosecution of the offenders and secure justice
is taken. [80] for all;

It, however, does not require the universal application of the laws to all persons or things without SECTION 1. Creation of a Commission. There is hereby created the PHILIPPINE TRUTH
distinction. What it simply requires is equality among equals as determined according to a valid classification. COMMISSION, hereinafter referred to as the COMMISSION, which shall primarily seek and find
Indeed, the equal protection clause permits classification. Such classification, however, to be valid must pass the the truth on, and toward this end, investigate reports of graft and corruption of such scale and
test of reasonableness. The test has four requisites: (1) The classification rests on substantial distinctions; (2) It magnitude that shock and offend the moral and ethical sensibilities of the people, committed by
is germane to the purpose of the law; (3) It is not limited to existing conditions only; and public officers and employees, their co-principals, accomplices and accessories from the private
(4) It applies equally to all members of the same class. [81] Superficial differences do not make for a valid sector, if any, during the previous administration; and thereafter recommend the appropriate
classification.[82] action or measure to be taken thereon to ensure that the full measure of justice shall be served
without fear or favor.
For a classification to meet the requirements of constitutionality, it must include or embrace all persons
who naturally belong to the class.[83] The classification will be regarded as invalid if all the members of the class SECTION 2. Powers and Functions. The Commission, which shall have all the powers of an
are not similarly treated, both as to rights conferred and obligations imposed. It is not necessary that the investigative body under Section 37, Chapter 9, Book I of the Administrative Code of 1987, is
classification be made with absolute symmetry, in the sense that the members of the class should possess the primarily tasked to conduct a thorough fact-finding investigation of reported cases of graft and
same characteristics in equal degree. Substantial similarity will suffice; and as long as this is achieved, all those corruption referred to in Section 1, involving third level public officers and higher, their co-
covered by the classification are to be treated equally. The mere fact that an individual belonging to a class differs principals, accomplices and accessories from the private sector, if any, during the previous
from the other members, as long as that class is substantially distinguishable from all others, does not justify the administration and thereafter submit its finding and recommendations to the President,
non-application of the law to him.[84] Congress and the Ombudsman. [Emphases supplied]

The classification must not be based on existing circumstances only, or so constituted as to preclude In this regard, it must be borne in mind that the Arroyo administration is but just a member of a class, that
addition to the number included in the class. It must be of such a nature as to embrace all those who may thereafter is, a class of past administrations. It is not a class of its own. Not to include past administrations similarly situated
be in similar circumstances and conditions. It must not leave out or underinclude those that should otherwise fall constitutes arbitrariness which the equal protection clause cannot sanction. Such discriminating differentiation
into a certain classification. As elucidated in Victoriano v. Elizalde Rope Workers' Union[85] and reiterated in a long clearly reverberates to label the commission as a vehicle for vindictiveness and selective retribution.
line of cases,[86]
The guaranty of equal protection of the laws is not a guaranty of equality in the Though the OSG enumerates several differences between the Arroyo administration and other past
application of the laws upon all citizens of the state. It is not, therefore, a requirement, in order administrations, these distinctions are not substantial enough to merit the restriction of the investigation to the
to avoid the constitutional prohibition against inequality, that every man, woman and child should previous administration only. The reports of widespread corruption in the Arroyo administration cannot be taken
be affected alike by a statute. Equality of operation of statutes does not mean indiscriminate as basis for distinguishing said administration from earlier administrations which were also blemished by similar
operation on persons merely as such, but on persons according to the circumstances widespread reports of impropriety. They are not inherent in, and do not inure solely to, the Arroyo administration.
surrounding them. It guarantees equality, not identity of rights. The Constitution does not require As Justice Isagani Cruz put it, Superficial differences do not make for a valid classification.[88]
that things which are different in fact be treated in law as though they were the same. The equal
protection clause does not forbid discrimination as to things that are different. It does not prohibit
legislation which is limited either in the object to which it is directed or by the territory within The public needs to be enlightened why Executive Order No. 1 chooses to limit the scope of the intended
which it is to operate. investigation to the previous administration only. The OSG ventures to opine that to include other past
administrations, at this point, may unnecessarily overburden the commission and lead it to lose its
The equal protection of the laws clause of the Constitution allows classification. Classification in effectiveness.[89] The reason given is specious. It is without doubt irrelevant to the legitimate and noble objective
law, as in the other departments of knowledge or practice, is the grouping of things in speculation of the PTC to stamp out or end corruption and the evil it breeds. [90]
or practice because they agree with one another in certain particulars. A law is not invalid
because of simple inequality. The very idea of classification is that of inequality, so that it goes The probability that there would be difficulty in unearthing evidence or that the earlier reports involving
without saying that the mere fact of inequality in no manner determines the matter of the earlier administrations were already inquired into is beside the point. Obviously, deceased presidents and
constitutionality. All that is required of a valid classification is that it be reasonable, which means cases which have already prescribed can no longer be the subjects of inquiry by the PTC. Neither is the PTC
that the classification should be based on substantial distinctions which make for real expected to conduct simultaneous investigations of previous administrations, given the bodys limited time and
differences, that it must be germane to the purpose of the law; that it must not be limited to resources. The law does not require the impossible (Lex non cogit ad impossibilia).[91]
existing conditions only; and that it must apply equally to each member of the class. This Court
has held that the standard is satisfied if the classification or distinction is based on a reasonable Given the foregoing physical and legal impossibility, the Court logically recognizes the unfeasibility of
foundation or rational basis and is not palpably arbitrary. [Citations omitted] investigating almost a centurys worth of graft cases. However, the fact remains that Executive Order No. 1 suffers
from arbitrary classification. The PTC, to be true to its mandate of searching for the truth, must not exclude the
Applying these precepts to this case, Executive Order No. 1 should be struck down as violative of the other past administrations. The PTC must, at least, have the authority to investigate all past
equal protection clause. The clear mandate of the envisioned truth commission is to investigate and find out the administrations. While reasonable prioritization is permitted, it should not be arbitrary lest it be struck down for
truth concerning the reported cases of graft and corruption during the previous administration[87] only. The intent being unconstitutional. In the often quoted language of Yick Wo v. Hopkins,[92]
to single out the previous administration is plain, patent and manifest. Mention of it has been made in at least three
portions of the questioned executive order. Specifically, these are:
PAGE 19 of 135 CONSTITUTIONAL LAW -EXECUTIVE DEPARTMENT PART 2 –SESSION 12
Though the law itself be fair on its face and impartial in appearance, yet, if applied and administrations, it does not guarantee that they would be covered in the future. Such expanded mandate of the
administered by public authority with an evil eye and an unequal hand, so as practically to make commission will still depend on the whim and caprice of the President. If he would decide not to include them, the
unjust and illegal discriminations between persons in similar circumstances, material to their section would then be meaningless. This will only fortify the fears of the petitioners that the Executive Order No. 1
rights, the denial of equal justice is still within the prohibition of the was crafted to tailor-fit the prosecution of officials and personalities of the Arroyo administration.[105]
constitution. [Emphasis supplied]

It could be argued that considering that the PTC is an ad hoc body, its scope is limited. The Court,
however, is of the considered view that although its focus is restricted, the constitutional guarantee of equal The Court tried to seek guidance from the pronouncement in the case of Virata v. Sandiganbayan,[106] that
protection under the laws should not in any way be circumvented. The Constitution is the fundamental and the PCGG Charter (composed of Executive Orders Nos. 1, 2 and 14) does not violate the equal protection clause.
paramount law of the nation to which all other laws must conform and in accordance with which all private rights The decision, however, was devoid of any discussion on how such conclusory statement was arrived at, the
determined and all public authority administered.[93] Laws that do not conform to the Constitution should be stricken principal issue in said case being only the sufficiency of a cause of action.
down for being unconstitutional.[94] While the thrust of the PTC is specific, that is, for investigation of acts of graft
and corruption, Executive Order No. 1, to survive, must be read together with the provisions of the Constitution. To A final word
exclude the earlier administrations in the guise of substantial distinctions would only confirm the petitioners lament
that the subject executive order is only an adventure in partisan hostility. In the case of US v. Cyprian,[95] it was The issue that seems to take center stage at present is - whether or not the Supreme Court, in the
written: A rather limited number of such classifications have routinely been held or assumed to be arbitrary; those exercise of its constitutionally mandated power of Judicial Review with respect to recent initiatives of the legislature
include: race, national origin, gender, political activity or membership in a political party, union activity or and the executive department, is exercising undue interference. Is the Highest Tribunal, which is expected to be
membership in a labor union, or more generally the exercise of first amendment rights. the protector of the Constitution, itself guilty of violating fundamental tenets like the doctrine of separation of
powers? Time and again, this issue has been addressed by the Court, but it seems that the present political
To reiterate, in order for a classification to meet the requirements of constitutionality, it must include or situation calls for it to once again explain the legal basis of its action lest it continually be accused of being a
embrace all persons who naturally belong to the class. [96] Such a classification must not be based on existing hindrance to the nations thrust to progress.
circumstances only, or so constituted as to preclude additions to the number included within a class, but must be
of such a nature as to embrace all those who may thereafter be in similar circumstances and The Philippine Supreme Court, according to Article VIII, Section 1 of the 1987 Constitution, is vested with
conditions. Furthermore, all who are in situations and circumstances which are relative to the discriminatory Judicial Power that includes the duty of the courts of justice to settle actual controversies involving rights which
legislation and which are indistinguishable from those of the members of the class must be brought under the are legally demandable and enforceable, and to determine whether or not there has been a grave of abuse of
influence of the law and treated by it in the same way as are the members of the class. [97] discretion amounting to lack or excess of jurisdiction on the part of any branch or instrumentality of the government.

The Court is not unaware that mere underinclusiveness is not fatal to the validity of a law under the equal Furthermore, in Section 4(2) thereof, it is vested with the power of judicial review which is the power to
protection clause.[98] Legislation is not unconstitutional merely because it is not all-embracing and does not include declare a treaty, international or executive agreement, law, presidential decree, proclamation, order, instruction,
all the evils within its reach.[99] It has been written that a regulation challenged under the equal protection clause ordinance, or regulation unconstitutional. This power also includes the duty to rule on the constitutionality of the
is not devoid of a rational predicate simply because it happens to be incomplete. [100] In several instances, the application, or operation of presidential decrees, proclamations, orders, instructions, ordinances, and other
underinclusiveness was not considered a valid reason to strike down a law or regulation where the purpose can regulations. These provisions, however, have been fertile grounds of conflict between the Supreme Court, on one
be attained in future legislations or regulations. These cases refer to the step by step process. [101] With regard to hand, and the two co-equal bodies of government, on the other. Many times the Court has been accused of
equal protection claims, a legislature does not run the risk of losing the entire remedial scheme simply because it asserting superiority over the other departments.
fails, through inadvertence or otherwise, to cover every evil that might conceivably have been attacked. [102]
To answer this accusation, the words of Justice Laurel would be a good source of enlightenment, to wit:
In Executive Order No. 1, however, there is no inadvertence. That the previous administration was picked And when the judiciary mediates to allocate constitutional boundaries, it does not assert any superiority over the
out was deliberate and intentional as can be gleaned from the fact that it was underscored at least three times in other departments; it does not in reality nullify or invalidate an act of the legislature, but only asserts the solemn
the assailed executive order. It must be noted that Executive Order No. 1 does not even mention any particular and sacred obligation assigned to it by the Constitution to determine conflicting claims of authority under the
act, event or report to be focused on unlike the investigative commissions created in the past. The equal protection Constitution and to establish for the parties in an actual controversy the rights which that instrument secures and
clause is violated by purposeful and intentional discrimination.[103] guarantees to them.[107]

To disprove petitioners contention that there is deliberate discrimination, the OSG clarifies that the Thus, the Court, in exercising its power of judicial review, is not imposing its own will upon a co-equal
commission does not only confine itself to cases of large scale graft and corruption committed during the previous body but rather simply making sure that any act of government is done in consonance with the authorities and
administration.[104] The OSG points to Section 17 of Executive Order No. 1, which provides: rights allocated to it by the Constitution. And, if after said review, the Court finds no constitutional violations of any
sort, then, it has no more authority of proscribing the actions under review. Otherwise, the Court will not be deterred
to pronounce said act as void and unconstitutional.
SECTION 17. Special Provision Concerning Mandate. If and when in the judgment of the
President there is a need to expand the mandate of the Commission as defined in Section 1 It cannot be denied that most government actions are inspired with noble intentions, all geared towards
hereof to include the investigation of cases and instances of graft and corruption during the prior the betterment of the nation and its people. But then again, it is important to remember this ethical principle: The
administrations, such mandate may be so extended accordingly by way of a supplemental end does not justify the means. No matter how noble and worthy of admiration the purpose of an act, but if the
Executive Order. means to be employed in accomplishing it is simply irreconcilable with constitutional parameters, then it cannot
still be allowed.[108] The Court cannot just turn a blind eye and simply let it pass. It will continue to uphold the
Constitution and its enshrined principles.
The Court is not convinced. Although Section 17 allows the President the discretion to expand the scope
of investigations of the PTC so as to include the acts of graft and corruption committed in other past
PAGE 20 of 135 CONSTITUTIONAL LAW -EXECUTIVE DEPARTMENT PART 2 –SESSION 12
The Constitution must ever remain supreme. All must bow to the mandate of this law.
Expediency must not be allowed to sap its strength nor greed for power debase its rectitude.[109]

Lest it be misunderstood, this is not the death knell for a truth commission as nobly envisioned by the
present administration. Perhaps a revision of the executive issuance so as to include the earlier past
administrations would allow it to pass the test of reasonableness and not be an affront to the
Constitution. Of all the branches of the government, it is the judiciary which is the most interested in knowing the
truth and so it will not allow itself to be a hindrance or obstacle to its attainment. It must, however, be emphasized
that the search for the truth must be within constitutional bounds for ours is still a government of laws and not of
men.[110]

WHEREFORE, the petitions are GRANTED. Executive Order No. 1 is hereby


declared UNCONSTITUTIONAL insofar as it is violative of the equal protection clause of the Constitution.

As also prayed for, the respondents are hereby ordered to cease and desist from carrying out the
provisions of Executive Order No. 1.

SO ORDERED.
PAGE 21 of 135 CONSTITUTIONAL LAW -EXECUTIVE DEPARTMENT PART 2 –SESSION 12
FIRST DIVISION Respondents, employees of the DENR Region XII who are members of the employees association,
COURAGE, represented by their Acting President, Baguindanai A. Karim, filed with the Regional Trial Court of
Cotabato, a petition for nullity of orders with prayer for preliminary injunction.
On December 8, 1999, the trial court issued a temporary restraining order enjoining petitioner from
[G.R. No. 149724. August 19, 2003] implementing the assailed Memorandum. The dispositive portion of the Order reads:

WHEREFORE, defendants DENR Secretary Antonio H. Cerilles and Regional Executive Director Israel C. Gaddi
are hereby ordered to cease and desist from doing the act complained of, namely, to stop the transfer of DENR
DEPARTMENT OF ENVIRONMENT AND NATURAL RESOURCES, represented herein by its [Region] 12 offices from Cotabato City to Korandal (Marbel), South Cotabato.
Secretary, HEHERSON T. ALVAREZ, petitioner, vs. DENR REGION 12 EMPLOYEES, represented
by BAGUIDALI KARIM, Acting President of COURAGE (DENR Region 12 Chapter), respondents.
xxx xxx xxx.

DECISION SO ORDERED.[5]
YNARES-SANTIAGO, J.:
Petitioner filed a Motion for Reconsideration with Motion to Dismiss, raising the following grounds:
This is a petition for review assailing the Resolutions dated May 31, 2000[1] of the Court of Appeals which I.
dismissed the petition for certiorari in CA-G.R. SP No. 58896, and its Resolution dated August 20, 2001[2], which
denied the motion for reconsideration.
The power to transfer the Regional Office of the Department of Environment and Natural Resources (DENR) is
The facts are as follows: executive in nature.
On November 15, 1999, Regional Executive Director of the Department of Environment and Natural
Resources for Region XII, Israel C. Gaddi, issued a Memorandum[3] directing the immediate transfer of the DENR II.
XII Regional Offices from Cotabato City to Koronadal (formerly Marbel), South Cotabato. The Memorandum was
issued pursuant to DENR Administrative Order No. 99-14, issued by then DENR Secretary Antonio H. Cerilles, The decision to transfer the Regional Office is based on Executive Order No. 429, which reorganized Region XII.
which reads in part:
III.
Subject: Providing for the Redefinition of Functions and Realignment of Administrative Units in the
Regional and Field Offices: The validity of EO 429 has been affirmed by the Honorable Supreme Court in the Case of Chiongbian vs. Orbos
(1995) 245 SCRA 255.
Pursuant to Executive Order No. 192, dated June 10, 1987 and as an interim administrative arrangement to
improve the efficiency and effectiveness of the Department of Environment and Natural Resources (DENR) in IV.
delivering its services pending approval of the government-wide reorganization by Congress, the following
redefinition of functions and realignment of administrative units in the regional and field offices are hereby
promulgated: Since the power to reorganize the Administrative Regions is Executive in Nature citing Chiongbian, the Honorable
Court has no jurisdiction to entertain this petition. [6]
Section 1. Realignment of Administrative Units:
On January 14, 2000, the trial court rendered judgment, the dispositive portion of which reads:
The DENR hereby adopts a policy to establish at least one Community Environment and Natural Resources
Office (CENRO) or Administrative Unit per Congressional District except in the Autonomous Region of Muslim CONSEQUENTLY, order is hereby issued ordering the respondents herein to cease and desist from enforcing
Mindanao (ARMM) and the National Capital Region (NCR). The Regional Executive Directors (REDs) are their Memorandum Order dated November 15, 1999 relative to the transfer of the DENR Regional Offices from
hereby authorized to realign/relocate existing CENROs and implement this policy in accordance with the Region 12 to Region 11 at Koronadal, South Cotabato for being bereft of legal basis and issued with grave
attached distribution list per region which forms part of this Order. Likewise, the following realignment and abuse of discretion amounting to lack or excess of jurisdiction on their part, and they are further ordered to return
administrative arrangements are hereby adopted: back the seat of the DENR Regional Offices 12 to Cotabato City.

xxxxxxxxx SO ORDERED.[7]

1.6. The supervision of the Provinces of South Cotabato and Sarangani shall be transferred from Region XI to Petitioners motion for reconsideration was denied in an Order dated April 10, 2000. A petition
XII.[4] for certiorari under Rule 65 was filed before the Court of Appeals, docketed as CA-G.R. SP No. 58896. The petition
was dismissed outright for: (1) failure to submit a written explanation why personal service was not done on the
adverse party; (2) failure to attach affidavit of service; (3) failure to indicate the material dates when copies of the
orders of the lower court were received; (4) failure to attach certified true copy of the order denying petitioners
PAGE 22 of 135 CONSTITUTIONAL LAW -EXECUTIVE DEPARTMENT PART 2 –SESSION 12
motion for reconsideration; (5) for improper verification, the same being based on petitioners knowledge and belief, reglementary period to appeal, nevertheless, the departure from the general rule that the extraordinary writ of
and (6) wrong remedy of certiorari under Rule 65 to substitute a lost appeal.[8] certiorari cannot be a substitute for the lost remedy of appeal is justified because the execution of the assailed
decision would amount to an oppressive exercise of judicial authority. [13]
The motion for reconsideration was denied in a resolution dated August 20, 2001. [9] Hence, this petition
based on the following assignment of errors: Petitioner maintains that the assailed DAO-99-14 and the implementing memorandum were valid and that
the trial court should have taken judicial notice of Republic Act No. 6734, otherwise known as An Organic Act for
I the Autonomous Region in Muslim Mindanao, and its implementing Executive Order 429,[14] as the legal bases for
the issuance of the assailed DAO-99-14. Moreover, the validity of R.A. No. 6734 and E.O. 429 were upheld in the
RULES OF PROCEDURE CAN NOT BE USED TO DEFEAT THE ENDS OF SUBSTANTIAL JUSTICE case of Chiongbian v. Orbos.[15] Thus, the respondents cannot, by means of an injunction, force the DENR XII
Regional Offices to remain in Cotabato City, as the exercise of the authority to transfer the same is executive in
II nature.
It is apropos to reiterate the elementary doctrine of qualified political agency, thus:
THE DECISION OF THE LOWER COURT DATED 14 JANUARY 2000 WHICH WAS AFFIRMED IN THE
QUESTIONED RESOLUTIONS OF THE COURT OF APPEALS DATED 31 MAY 2000 AND 20 AUGUST 2001 Under this doctrine, which recognizes the establishment of a single executive, all executive and administrative
IS PATENTLY ILLEGAL AND SHOULD BE NULLIFIED, CONSIDERING THAT: organizations are adjuncts of the Executive Department, the heads of the various executive departments are
assistants and agents of the Chief Executive, and, except in cases where the Chief Executive is required by the
A. RESPONDENTS HAVE NO CAUSE OF ACTION AGAINST PETITIONER AS THEY HAVE Constitution or law to act in person or the exigencies of the situation demand that he act personally, the
NO RIGHT TO CAUSE THE DENR REGION 12 OFFICE TO REMAIN IN multifarious executive and administrative functions of the Chief Executive are performed by and through the
COTABATO CITY. executive departments, and the acts of the Secretaries of such departments, performed and promulgated in the
regular course of business, are, unless disapproved or reprobated by the Chief Executive, presumptively the
B. THE STATE DID NOT GIVE ITS CONSENT TO BE SUED. acts of the Chief Executive.[16]

C. THE DECISION OF THE LOWER COURT DATED 14 JANUARY 2000 IS CONTRARY TO This doctrine is corollary to the control power of the President as provided for under Article VII, Section 17
THE RULE OF PRESUMPTION OF REGULARITY IN THE PERFORMANCE OF of the 1987 Constitution, which reads:
OFFICIAL FUNCTIONS.
Sec. 17. The President shall have control of all the executive departments, bureaus, and offices. He shall ensure
D. IN ANY EVENT, THE DECISION OF THE LOWER COURT DATED 14 JANUARY 2000 IS that the laws be faithfully executed.
CONTRARY TO THE LETTER AND INTENT OF EXECUTIVE ORDER NO. 429
AND REPUBLIC ACT NO. 6734. However, as head of the Executive Department, the President cannot be expected to exercise his control
(and supervisory) powers personally all the time. He may delegate some of his powers to the Cabinet members
E. THE DETERMINATION OF THE PROPRIETY AND PRACTICALITY OF THE TRANSFER except when he is required by the Constitution to act in person or the exigencies of the situation demand that he
OF REGIONAL OFFICES IS INHERENTLY EXECUTIVE, AND THEREFORE, NON- acts personally.[17]
JUSTICIABLE.[10] In Buklod ng Kawaning EIIB v. Zamora,[18] this Court upheld the continuing authority of the President to carry
out the reorganization in any branch or agency of the executive department. Such authority includes the creation,
In essence, petitioner argues that the trial court erred in enjoining it from causing the transfer of the DENR alteration or abolition of public offices.[19] The Chief Executives authority to reorganize the National Government
XII Regional Offices, considering that it was done pursuant to DENR Administrative Order 99-14. finds basis in Book III, Section 20 of E.O. No. 292, otherwise known as the Administrative Code of 1987, viz:

The issues to be resolved in this petition are: (1) Whether DAO-99-14 and the Memorandum implementing
Section 20. Residual Powers. Unless Congress provides otherwise, the President shall exercise such other
the same were valid; and (2) Whether the DENR Secretary has the authority to reorganize the DENR.
powers and functions vested in the President which are provided for under the laws and which are not
Prefatorily, petitioner prays for a liberal application of procedural rules considering the greater interest of specifically enumerated above or which are not delegated by the President in accordance with law.
justice.
Further, in Larin v. Executive Secretary,[20] this Court had occasion to rule:
This Court is fully aware that procedural rules are not to be simply disregarded for these prescribed
procedures ensure an orderly and speedy administration of justice. However, it is equally true that litigation is not
merely a game of technicalities. Time and again, courts have been guided by the principle that the rules of This provision speaks of such other powers vested in the President under the law. What law then gives him the
procedure are not to be applied in a very rigid and technical manner, as rules of procedure are used only to help power to reorganize? It is Presidential Decree No. 1772 which amended Presidential Decree No. 1416. These
secure and not to override substantial justice.[11] Thus, if the application of the Rules would tend to frustrate rather decrees expressly grant the President of the Philippines the continuing authority to reorganize the national
than promote justice, it is always within the power of this Court to suspend the rules, or except a particular case government, which includes the power to group, consolidate bureaus and agencies, to abolish offices, to transfer
from its operation.[12] functions, to create and classify functions, services and activities and to standardize salaries and materials. The
validity of these two decrees is unquestionable. The 1987 Constitution clearly provides that all laws, decrees,
Despite the presence of procedural flaws, we find it necessary to address the issues because of the demands executive orders, proclamations, letters of instructions and other executive issuances not inconsistent with this
of public interest, including the need for stability in the public service and the serious implications this case may Constitution shall remain operative until amended, repealed or revoked. So far, there is yet no law amending or
cause on the effective administration of the executive department. Although no appeal was made within the repealing said decrees.
PAGE 23 of 135 CONSTITUTIONAL LAW -EXECUTIVE DEPARTMENT PART 2 –SESSION 12
Applying the doctrine of qualified political agency, the power of the President to reorganize the National The Municipality of Koronadal (Marinduque) in South Cotabato shall serve as the regional center.
Government may validly be delegated to his cabinet members exercising control over a particular executive
department. Thus, in DOTC Secretary v. Mabalot,[21] we held that the President through his duly constituted In Chiongbian v. Orbos, this Court stressed the rule that the power of the President to reorganize the
political agent and alter ego, the DOTC Secretary may legally and validly decree the reorganization of the administrative regions carries with it the power to determine the regional centers. In identifying the regional centers,
Department, particularly the establishment of DOTC-CAR as the LTFRB Regional Office at the Cordillera the President purposely intended the effective delivery of the field services of government agencies.[23] The same
Administrative Region, with the concomitant transfer and performance of public functions and responsibilities intention can be gleaned from the preamble of the assailed DAO-99-14 which the DENR sought to achieve, that
appurtenant to a regional office of the LTFRB. is, to improve the efficiency and effectiveness of the DENR in delivering its services.
Similarly, in the case at bar, the DENR Secretary can validly reorganize the DENR by ordering the transfer It may be true that the transfer of the offices may not be timely considering that: (1) there are no buildings
of the DENR XII Regional Offices from Cotabato City to Koronadal, South Cotabato. The exercise of this authority yet to house the regional offices in Koronadal, (2) the transfer falls on the month of Ramadan, (3) the children of
by the DENR Secretary, as an alter ego, is presumed to be the acts of the President for the latter had not expressly the affected employees are already enrolled in schools in Cotabato City, (4) the Regional Development Council
repudiated the same. was not consulted, and (5) the Sangguniang Panglungsond, through a resolution, requested the DENR Secretary
The trial court should have taken judicial notice of R.A. No. 6734, as implemented by E.O. No. 429, as legal to reconsider the orders. However, these concern issues addressed to the wisdom of the transfer rather than to
basis of the Presidents power to reorganize the executive department, specifically those administrative regions its legality. It is basic in our form of government that the judiciary cannot inquire into the wisdom or expediency of
which did not vote for their inclusion in the ARMM. It is axiomatic that a court has the mandate to apply relevant the acts of the executive or the legislative department,[24] for each department is supreme and independent of the
statutes and jurisprudence in determining whether the allegations in a complaint establish a cause of action. While others, and each is devoid of authority not only to encroach upon the powers or field of action assigned to any of
it focuses on the complaint, a court clearly cannot disregard decisions material to the proper appreciation of the the other department, but also to inquire into or pass upon the advisability or wisdom of the acts performed,
questions before it.[22] In resolving the motion to dismiss, the trial court should have taken cognizance of the official measures taken or decisions made by the other departments. [25]
acts of the legislative, executive, and judicial departments because they are proper subjects of mandatory judicial The Supreme Court should not be thought of as having been tasked with the awesome responsibility of
notice as provided by Section 1 of Rule 129 of the Rules of Court, to wit: overseeing the entire bureaucracy. Unless there is a clear showing of constitutional infirmity or grave abuse of
discretion amounting to lack or excess of jurisdiction, the Courts exercise of the judicial power, pervasive and
A court shall take judicial notice, without the introduction of evidence, of the existence and territorial extent of limitless it may seem to be, still must succumb to the paramount doctrine of separation of powers. [26] After a careful
states, their political history, forms of government and symbols of nationality, the law of nations, the admiralty review of the records of the case, we find that this jurisprudential element of abuse of discretion has not been
and maritime courts of the world and their seals, the political constitution and history of the Philippines, the shown to exist.
official acts of the legislative, executive and judicial departments of the Philippines, the laws of nature, the
measure of time, and the geographical divisions. (Emphasis supplied) WHEREFORE, in view of the foregoing, the petition for review is GRANTED. The resolutions of the Court of
Appeals in CA-G.R. SP No. 58896 dated May 31, 2000 and August 20, 2001, as well as the decision dated January
14, 2000 of the Regional Trial Court of Cotabato City, Branch 15, in Civil Case No 389, are REVERSED and SET
Article XIX, Section 13 of R.A. No. 6734 provides: ASIDE. The permanent injunction, which enjoined the petitioner from enforcing the Memorandum Order of the
DENR XII Regional Executive Director, is LIFTED.
SECTION 13. The creation of the Autonomous Region in Muslim Mindanao shall take effect when approved by a
majority of the votes cast by the constituent units provided in paragraph (2) of Sec. 1 of Article II of this Act in a SO ORDERED.
plebiscite which shall be held not earlier than ninety (90) days or later than one hundred twenty (120) days after
the approval of this Act: Provided, That only the provinces and cities voting favorably in such plebiscite shall be
included in the Autonomous Region in Muslim Mindanao. The provinces and cities which in the plebiscite do not
vote for inclusion in the Autonomous Region shall remain in the existing administrative regions: Provided,
however, That the President may, by administrative determination, merge the existing regions.

Pursuant to the authority granted by the aforequoted provision, then President Corazon C. Aquino issued on
October 12, 1990 E.O. 429, Providing for the Reorganization of the Administrative Regions in Mindanao. Section
4 thereof provides:

SECTION 4. REGION XII, to be known as CENTRAL MINDANAO, shall include the following provinces and
cities:

Provinces
Sultan Kudarat
Cotabato
South Cotabato

Cities
Cotabato
General Santos
PAGE 24 of 135 CONSTITUTIONAL LAW -EXECUTIVE DEPARTMENT PART 2 –SESSION 12
Republic of the Philippines construct, operate, and maintain toll facilities in the North and South Luzon Toll Expressways for a period of 30
SUPREME COURT years starting 1 May1977.
Manila
TRB and PNCC later entered into a Toll Operation Agreement,3 which prescribed the operating conditions of the
right granted to PNCC under P.D. 1113.
FIRST DIVISION

P.D. 1113 was amended by P.D. 1894,4 which granted PNCC the right, privilege, and authority to construct,
G.R. No. 181293 February 23, 2015
maintain, and operate the North Luzon, South Luzon and Metro Manila Expressways, together with the toll
facilities appurtenant thereto. The term of 30 years provided under P. D. 1113 starting from 1 May 1977
ANA THERESIA "RISA" HONTIVEROS-BARAQUEL, DANIEL L. EDRALIN, VICTOR M. GONZALES, SR., remained the same for the North and the South Luzon Expressways, while the franchise granted for the Metro
JOSE APOLLO R. ADO, RENE D. SORIANO, ALLIANCE OF PROGRESSIVE LABOR, BUKLURAN NG Manila Expressway (MME) provided a term of 30 years commencing from the date of completion of the project.
MANGGAGAWANG PILIPINO, LAHING PILIPINO MULTIPURPOSE TRANSPORT SERVICE
COOPERATIVE, PNCC SKYWAY CORPORATION EMPLOYEES UNION (PSCEU), and PNCC TRAFFIC
On 22 September 1993, PNCC entered into an agreement5 with PT Citra Lamtoro Gung Persada (CITRA), a
MANAGEMENT & SECURITY DEPARTMENT WORKERS ORGANIZATION (PTMSDWO), Petitioners,
limited liability company organized and established under the laws of the Republic of Indonesia, whereby the
vs.
latter committed to provide PNCC with a pre-feasibility study on the proposed MME project. The agreement was
TOLL REGULATORY BOARD, THE SECRETARY OF THE DEPARTMENT OF TRANSPORTATION AND
supplemented6 on 14 February 1994 with a related undertaking on the part of CITRA. CITRA was to provide a
COMMUNICATIONS (DOTC), PNCC SKYWAY CORPORATION, PHILIPPINE NATIONAL CONSTRUCTION
preliminary feasibility study on the Metro Manila Skyways (MMS) project, a system of elevated roadway
CORPORATION, SKYWAY O & M CORPORATION, and CITRA METRO MANILA TOLLWAYS
networks passing through the heart of the Metropolitan Manila area. In order to accelerate the actual
CORP.,Respondents.
implementation of both the MME and the MMS projects, PNCC and CITRA entered into a second
agreement.7 Through that agreement, CITRA committed to finance and undertake the preparation, updating, and
DECISION revalidation of previous studies on the construction, operation, and maintenance of the projects.

SERENO, CJ: As a result of the feasibility and related studies, PNCC and CITRA submitted, through the TRB, a Joint
Investment Proposal (JIP) to the Republic of the Philippines. 8 The JIP embodied the implementation schedule
This is an original petition for certiorari and prohibition under Rule 65 of the Rules of Court, with a prayer for the for the financing, design and construction of the MMS in three stages: the South Metro Manila Skyway, the North
issuance of a writ of preliminary injunction and/or temporary restraining order, seeking the annulment of the Metro Manila Skyway, and the Central Metro Manila Skyway. 9
following:
The TRB reviewed, evaluated and approved the JIP, particularly as it related to Stage 1, Phases 1 and 2; and
1. The Amendment to the Supplemental Toll Operation Agreement executed on 18 July 2007 between Stage 2, Phase 1 of the South Metro Manila Skyway.
the Republic of the Philippines, the Philippine National Construction Corporation, and Citra Metro
Manila Tollways Corporation; On 30 August 1995, PNCC and CITRA entered into a Business and Joint Venture Agreement10 and created the
Citra Metro Manila Tollways Corporation (CMMTC). CMMTC was a joint venture corporation organized under
2. The Memorandum dated 20 July 2007 of the Secretary of Transportation and Communications, Philippine laws to serve as a channel through which CITRA shall participate in the construction and development
approving the Amendment to the Supplemental Toll Operation Agreement; of the project.

3. The Memorandum of Agreement executed on 21 December 2007 between the Philippine National On 27 November 1995, the Republic of the Philippines - through the TRB - as Grantor, CMMTC as Investor, and
Construction Corporation, PNCC Skyway Corporation, and Citra Metro Manila Tollways Corporation; PNCC as Operator executed a Supplemental Toll Operation Agreement (STOA)11 covering Stage 1, Phases 1
and and 2; and Stage 2, Phase 1 of the South Metro Manila Skyway. Under the STOA, the design and construction
of the project roads became the primary and exclusive privilege and responsibility of CMMTC. The operation and
maintenance of the project roads became the primary and exclusive privilege and responsibility of the PNCC
4. The Toll Operation Certificate issued by the Toll Regulatory Board on 28 December 2007 in favor of
Skyway Corporation (PSC), a wholly owned subsidiary of PNCC, which undertook and performed the latter's
Skyway O & M Corporation. obligations under the STOA.

The annulment of the above is sought for being unconstitutional, contrary to law, and grossly disadvantageous
CMMTC completed the design and construction of Stage 1 of the South Metro Manila Skyway, which was
to the government. Petitioners also seek to prohibit Skyway O & M Corporation from assuming operations and
operated and maintained by PSC.12
maintenance responsibilities over the Skyway toll facilities. ANTECEDENT FACTS

On 18 July 2007, the Republic of the Philippines, through the TRB, CMMTC, and PNCC executed the assailed
The Toll Regulatory Board (TRB) was created on 31 March 1977 by Presidential Decree No. (P.D.) 11121 in Amendment to the Supplemental Toll Operation Agreement (ASTOA).13 The ASTOA incorporated the
order to supervise and regulate, on behalf of the government, the collection of toll fees and the operation of toll
amendments, revisions, and modifications necessary to cover the design and construction of Stage 2 of the
facilities by the private sector.
South Metro Manila Skyway. Also under the ASTOA, Skyway 0 & M Corporation (SOMCO) replaced PSC in
performing the operations and maintenance of Stage 1 of the South Metro Manila Skyway.
On the same date, P.D. 11132 was issued granting to the Construction and Development Corporation of the
Philippines (now Philippine National Construction Corporation or PNCC) the right, privilege, and authority to
PAGE 25 of 135 CONSTITUTIONAL LAW -EXECUTIVE DEPARTMENT PART 2 –SESSION 12
Pursuant to the authority granted to him under Executive Order No. (E.O.) 49714 dated 24 January 2006, with prejudice. They pointed out that petitioners PSCEU and PTMSDWO had acted in bad faith by filing the
Department of Transportation and Communications (DOTC) Secretary Leandro Mendoza approved the ASTOA complaint before the RTC, despite the pendency of a labor case over which the Secretary of Labor and
through the challenged Memorandum dated 20 July 2007.15 Employment had assumed jurisdiction. Defendant CMMTC joined PNCC and PSC in moving for a partial
reconsideration of the RTC Order.34
On 21 December 2007, PNCC, PSC, and CMMTC entered into the assailed Memorandum of Agreement
(MOA)16providing for the successful and seamless assumption by SOMCO of the operations and maintenance The RTC denied the Motions for Partial Reconsideration in an Order dated 13 June 2008. 35
of Stage 1 of the South Metro Manila Skyway. Under the MOA, PSC received the amount of ₱320 million which
was used for the settlement of its liabilities arising from the consequent retrenchment or separation of its affected Before this Court, SOMCO,36 PSC,37 PNCC,38 CMMTC,39 and TRB40 filed their respective Comments on the
employees. Petition.

The TRB issued the challenged Toll Operation Certificate (TOC)17 to SOM CO on 28 December 2007, THE PARTIES' POSITIONS
authorizing the latter to operate and maintain Stage 1 of the South Metro Manila Skyway effective 10:00 p.m. on
31December2007.
Petitioners argue that the franchise for toll operations was exclusively vested by P.D. 1113 in PNCC, which
exercised the powers under its franchise through PSC in accordance with the STOA. By agreeing to the
Meanwhile, on 28 December 2007, petitioner PNCC Traffic Management and Security Department Workers arrangement whereby SOMCO would replace PSC in the toll operations and management, PNCC seriously
Organization (PTMSDWO) filed a Notice of Strike against PSC on the ground of unfair labor practice, specifically breached the terms and conditions of its undertaking under the franchise and effectively abdicated its rights and
union busting.18 The Secretary of Labor and Employment19 assumed jurisdiction over the dispute in an Order privileges in favor of SOMCO.
dated 31 December 2007 and set the initial hearing of the case on 2 January 2008.20
Furthermore, the TOC granted to SOMCO was highly irregular and contrary to law, because 1) it did not indicate
On 3 January 2008, petitioners PTMSDWO and PNCC Skyway Corporation Employees Union (PSCEU) filed the conditions that shall be imposed on SOMCO as provided under P.D. 1112;41 2) none of the requirements on
before the Regional Trial Court of Parañaque City, Branch 258 (RTC), a complaint against respondents TRB, public bidding, negotiations, or even publication was complied with before the issuance of the TOC to SOMCO;
PNCC, PSC, CMMTC, and SOMCO. The complaint was for injunction and prohibition with a prayer for a writ of 3) applying the stricter "grandfather rule," SOMCO does not qualify as a facility operator as defined under R.A.
preliminary injunction and/or a temporary restraining order, and sought to prohibit the implementation of the AS 6957,42 as amended by R.A. 7718;43 and 4) there were no public notices and hearings conducted wherein all
TOA and the MOA, as well as the assumption of the toll operations by SOMCO. 21 Petitioners PSCEU and legitimate issues and concerns about the transfer of the toll operations would have been properly ventilated.
PTMSDWO also sought the subsequent nullification of the ASTOA and the MOA for being contrary to law and
for being grossly disadvantageous to the government. 22 They later filed an Amended Complaint23 dated 8
January 2008, additionally praying that PSC be allowed to continue the toll operations. With the exception of Petitioners also claim that the approval by the DOTC Secretary of the AS TOA could not take the place of the
TRB, all defendants therein filed their Opposition. presidential approval required under P.D. 111344 and P.D. 189445 concerning the franchise granted to PNCC.

On 23 January 2008, the RTC issued an Order24 denying the prayer for the issuance of a temporary restraining Finally, petitioners claim that the assumption of the toll operations by SOM CO was grossly disadvantageous to
order and/or writ of preliminary injunction. According to the RTC, petitioners were seeking to enjoin a national the government, because 1) for a measly capital investment of ₱2.5 million, SOMCO stands to earn ₱400 million
government infrastructure project. Under Republic Act No. (R.A.) 8975,25 lower courts are prohibited from in gross revenues based on official and historical records; 2) with its measly capital, SOMCO would not be able
issuing a temporary restraining order or preliminary injunction against the government - or any person or entity to cover the direct overhead for personal services in the amount of ₱226 million as borne out by Commission on
acting under the government's direction - to restrain the execution, implementation, or operation of any such Audit reports; 3) the net revenue from toll operations would go to private shareholders of SOMCO, whereas all
contract or project. Furthermore, the RTC ruled that it could no longer issue a temporary restraining order or earnings of PSC when it was still in charge of the toll operations went to PNCC - the mother company whose
preliminary injunction, considering that the act sought to be restrained had already been consummated. 26 The earnings, as an "acquired-asset corporation," formed part of the public treasury; 4) the new arrangement would
AS TOA, the MOA, and the assumption of the toll operations by SOMCO took effect at 10:00 p.m. on 31 result in the poor delivery of toll services by SOMCO, which had no proven track record; 5) PSC received only
December 2007, while petitioners PSCEU and PTMSDWO sought to prohibit their implementation only on 3 ₱320 million as settlement for the transfer of toll operations to SOMCO.
January 2008.
All respondents counter that petitioners do not have the requisite legal standing to file the petition. According to
In view of its denial of the ancillary prayer, the RTC required defendants to file their respective Answers to the respondents, petitioner Hontiveros-Baraquel filed the instant petition as a legislator in her capacity as party-list
Amended Complaint.27 representative of Akbayan. As such, she was only allowed to sue to question the validity of any official action
when it infringed on her prerogative as a legislator.46 Presently, she has cited no such prerogative, power, or
privilege that is adversely affected by the assailed acts. 47
On 28 January 2008, petitioners PSCEU and PTMSDWO filed a Notice of Dismissal with Urgent Ex-Parte
Motion for the Issuance of Order Confirming the Dismissal,28 considering that no Answers had yet been filed. On
the basis thereof, the R TC dismissed the case without prejudice on 29 January 2008. 29 While suing as citizens, the individual petitioners have not shown any personal or substantial interest in the case
indicating that they sustained or will sustain direct injury as a result of the implementation of the assailed
acts.48 The maintenance of the suit by petitioners as taxpayers has no merit either because the assailed acts do
On 4 February 2008, petitioners filed the instant Petition30 before this Court. On 13 February 2008, we required not involve the disbursement of public funds.49 Finally, the bringing of the suit by petitioners as people's
respondents to comment on the same. 31 organizations does not automatically confer legal standing, especially since petitioner-organizations do not even
allege that they represent their members,50 nor do they cite any particular constitutional provision that has been
Meanwhile, defendants PNCC32 and PSC33 filed their respective Motions for Partial Reconsideration of the Order violated or disregarded by the assailed acts.51 In fact, the suit raises only issues of contract law, and none of the
of the R TC dismissing the case without prejudice. Both argued that the RTC should have dismissed the case petitioners is a party or is privy to the assailed agreements and issuances.52
PAGE 26 of 135 CONSTITUTIONAL LAW -EXECUTIVE DEPARTMENT PART 2 –SESSION 12
Respondents also argue that petitioners violate the hierarchy of courts. In particular, it is alleged that while lower ISSUES
courts are prohibited from issuing temporary restraining orders or preliminary injunctions against national
government projects under R.A. 8975, the law does not preclude them from assuming jurisdiction over The instant case shall be resolved on the basis of the following issues:
complaints that seek the nullification of a national government project as ultimate relief.53
Procedural:
As a final procedural challenge to the petition, respondents aver that petitioners are guilty of forum shopping.
When petitioners filed the instant petition, the case before the R TC seeking similar reliefs was still pending, as
respondents PNCC, PSC and CMMTC had moved for the partial reconsideration of the RTC's Order of dismissal I. Whether petitioners have standing;
within the reglementary period.54 Furthermore, the instant case and the one before the RTC were filed while
petitioners' labor grievances seeking similar reliefs were also being heard before the Department of Labor and II. Whether petitioners are guilty of forum-shopping;
Employment.55
Substantive:
On the merits of the arguments in the petition, respondents argue that nothing in the ASTOA, the approval
thereof by the DOTC Secretary, the MOA, or the TOC was violative of the Constitution. It is argued that the III. Whether the TRB has the power to grant authority to operate a toll facility;
authority to operate a public utility can be granted by administrative agencies when authorized by law. 56 Under
P.D. 1112, the TRB is empowered to grant authority and enter into contracts for the construction, operation, and
maintenance of a toll facility,57 such as the ASTOA in this case. Also, the ASTOA was an amendment, not to the IV. Whether the TOC issued to SOMCO was valid;
legislative franchise of PNCC, but to the STOA previously executed between the Republic of the Philippines
through the TRB, PNCC, and CMMTC.58 In fact, PNCC's franchise was never sold, transferred, or otherwise V. Whether the approval of the ASTOA by the DOTC Secretary was valid; and
assigned to SOMCO59 in the same way that PSC's previous assumption of the operation and maintenance of the
South Metro Manila Skyway did not amount to a sale, transfer or assignment of PNCC's franchise. 60
VI. Whether the assumption of toll operations by SOMCO is disadvantageous to the government.

There can be no valid objection to the approval of the ASTOA by the DOTC Secretary, because he was
OUR RULING
authorized by the President to do so by virtue of E.O. 497.61 Also, the phrase "subject to the approval of the
President of the Philippines" in P.D. 1112 and 1113 does not in any way mean that the presidential approval
must be obtained prior to the execution of a contract, or that the approval be made personally by the I
President.62 The presidential approval may be obtained under the doctrine of qualified political agency. 63
Not all petitioners have personality to sue.
Respondents argue that there is no merit in the claim that the TOC granted to SOMCO was highly irregular and
contrary to law. First, the TOC clearly states that the toll operation and maintenance by SOMCO shall be Standing is a constitutional law concept allowing suits to be brought not necessarily by parties personally injured
regulated by the Republic of the Philippines in accordance with P.D. 1112, the STOA, the toll operations and by the operation of a law or official action, but by concerned citizens, taxpayers, or voters who sue in the public
maintenance rules and regulations, and lawful orders, instructions, and conditions that may be imposed from interest.75 Determining the standing of concerned citizens, taxpayers, or voters requires a partial consideration of
time to time.64Second, there is no need to comply with the public bidding and negotiation requirements, because the substantive merit of the constitutional question,76 or at least a preliminary estimate thereof.77
the South Metro Manila Skyway is an ongoing project, not a new one.65 Furthermore, the STOA, which was the
basis for the ASTOA, was concluded way before the effectivity of R.A. 918466 in 2003.67
In this case, petitioners raise the power of Congress to grant franchises as a constitutional question. They allege
that the execution of the ASTOA and the MOA, the approval of the AS TOA by the DOTC Secretary and the
Third, SOMCO is a Filipino corporation with substantial 72% Filipino ownership. 68 Fourth, the law requires prior issuance of the TOC infringed on the constitutional power of Congress, which has the sole authority to grant
notice and hearing only in an administrative body's exercise of quasi-judicial functions.69 In this case, the transfer franchises for the operation of public utilities. This Court has had a few occasions to rule that a franchise from
of the toll operations and maintenance to SOM CO was a contractual arrangement entered into in accordance Congress is not required before each and every public utility may operate. 78 Unless there is a law that
with law.70 specifically requires a franchise for the operation of a public utility, particular agencies in the executive branch
may issue authorizations and licenses for the operation of certain classes of public utilities. 79 In the instant case,
Finally, the assumption of the toll operation and maintenance by SOMCO is not disadvantageous to the there is no law that states that a legislative franchise is necessary for the operation of toll facilities.
government. Petitioners belittle the ₱2.5 million capitalization of SOMCO, considering that PSC's capitalization
at the time it was incorporated was merely ₱500,000. 71 In PAL v. Civil Aeronautics Board,80 this Court enunciated:

Respondents claim that under the ASTOA, PNCC shall get a direct share in the toll revenues without any Congress has granted certain administrative agencies the power to grant licenses for, or to authorize the
corollary obligation, unlike the arrangement in the STOA whereby PNCC's 10% share in the toll revenues was operation of certain public utilities. With the growing complexity of modem life, the multiplication of the subjects
intended primarily for the toll operation and maintenance by PSC. 72 of governmental regulation, and the increased difficulty of administering the laws, there is a constantly growing
tendency towards the delegation of greater powers by the legislature, and towards the approval of the practice
Finally, respondents assert that there is no reason to fear that the assumption by SOMCO would result in poor by the courts. It is generally recognized that a franchise may be derived indirectly from the state through a duly
delivery of toll services. CITRA and the other shareholders of SOMCO are entities with experience and proven designated agency, and to this extent, the power to grant franchises has frequently been delegated, even to
track record in toll operations.73 Also, SOM CO hired or absorbed more than 300 PSC employees, 74 who brought agencies other than those of a legislative nature. In pursuance of this, it has been held that privileges conferred
with them their work expertise and experience.
PAGE 27 of 135 CONSTITUTIONAL LAW -EXECUTIVE DEPARTMENT PART 2 –SESSION 12
by grant by local authorities as agents for the state constitute as much a legislative franchise as though the grant material facts and circumstances and raise basically the same issues either pending in or already resolved by
had been made by an act of the Legislature.81 some other court or administrative agency. 93 What is pivotal in determining whether forum shopping exists is the
vexation caused to the courts and litigants and the possibility of conflicting decisions being rendered by different
It is thus clear that Congress does not have the sole authority to grant franchises for the operation of public courts and/or administrative agencies upon the same issues. 94
utilities. Considering the foregoing, we find that the petition raises no issue of constitutional import. More
particularly, no legislative prerogative, power, or privilege has been impaired. Hence, legislators have no The elements of forum shopping are as follows: a) identity of parties or at least such parties that represent the
standing to file the instant petition, for they are only allowed to sue to question the validity of any official action same interests in both actions; b) identity of rights asserted and the relief prayed for, the relief founded on the
when it infringes on their prerogatives as members of Congress. 82 Standing is accorded to them only if there is same facts; and c) identity of the two preceding particulars, such that any judgment rendered in one action will
an unmistakable showing that the challenged official act affects or impairs their rights and prerogatives as amount to res judicata in the other.95 Respondents argue that petitioners PSCEU and PTMSDWO committed
legislators.83 forum shopping by filing the complaint for injunction and prohibition before the RTC during the pendency of
NCMB-NCR-NS-12-188-07 entitled In Re: Labor Dispute at PNCC Skyway Corporation. It was a case they also
In line with our ruling in Kilosbayan, Inc. v. Morato, 84 the rule concerning a real party in interest - which is filed, over which the Secretary of Labor and Employment has assumed jurisdiction.
applicable to private litigation – rather than the liberal rule on standing, should be applied to petitioners.
The case involves a Notice of Strike filed against PSC on the ground of unfair labor practice. While the specific
A real party in interest is one who stands to be benefited or injured by the judgment in the suit, or the party act in question is not specified, the prohibited acts constituting unfair labor practice96 essentially relate to
entitled to the avails of the suit.85 One's interest must be personal and not one based on a desire to vindicate the violations concerning the workers' right to self-organization.97 When compared with the complaint filed with the
constitutional right of some third and unrelated party. 86 The purposes of the rule are to prevent the prosecution RTC for injunction and prohibition seeking to prohibit the implementation of the ASTOA and the MOA, as well as
of actions by persons without any right or title to or interest in the case; to require that the actual party entitled to the assumption of the toll operations by SOM CO for being unconstitutional, contrary to law and
legal relief be the one to prosecute the action; to avoid a multiplicity of suits; and to discourage litigation and disadvantageous to the government, it is easily discernible that there is no identity of rights asserted and relief
keep it within certain bounds, pursuant to sound public policy.87 prayed for. These cases are distinct and dissimilar in their nature and character.

At bottom, what is being questioned in the petition is the relinquishment by PSC of the toll operations in favor of For the sake of argument, let us assume that, in order to hurt the unions, PSC feigned a cessation of business
SOMCO, effectively leading to the cessation of the former' s business. In this case, we find that among that led to the retrenchment and separation of all employees. That is an unfair labor practice. In that complaint,
petitioners, the only real parties in interest are the labor unions PSCEU and PTMSDWO. the unions cannot be expected to ask for, or the Secretary of Labor and Employment to grant, the annulment of
the ASTOA and the MOA and the continuation of toll operations by PSC. The Secretary would only focus on the
legality of the retrenchment and separation, and on the presence or absence of bad faith in PSC's cessation of
PSCEU and PTMSDWO filed the petition not as a representative suit on behalf of their members who are rank- business. On the other hand, the complaint before the RTC would require it to focus on the legality of the
and-file employees of PSC, but as people's organizations "invested with a public duty to defend the rule of ASTOA, the MOA and the transfer of toll operations. Ultimately, even if the Secretary of Labor and Employment
law."88PSCEU and PTMSDWO cite Kilosbayan v. Ermita89 as authority to support their standing to file the instant makes a finding of unfair labor practice, this determination would not amount to res judicata as regards the case
suit. before the RTC.

It is well to point out that the Court, in Ermita, accorded standing to people's organizations to file the suit, We also reject the claim of respondents that petitioners PSCEU and PTMSDWO committed forum shopping by
because the matter involved therein was the qualification of a person to be appointed as a member of this Court filing the instant petition before this Court while the motion for partial reconsideration of the RTC's Order of
-"an issue of utmost and far-reaching constitutional importance."90 As discussed, the instant petition raises no dismissal without prejudice was still pending. Section 1, Rule 17 of the Rules of Court states:
genuine constitutional issues.
SECTION 1. Dismissal upon notice by plaintiff. - A complaint may be dismissed by the plaintiff by filing a notice
Nevertheless, for a different reason, we accord standing to PSCEU and PTMSDWO to file the instant suit. With of dismissal at any time before service of the answer or of a motion for summary judgment. Upon such notice
the transfer of toll operations to SOMCO and the resulting cessation of PSC's business comes the retrenchment being filed, the court shall issue an order confirming the dismissal. Unless otherwise stated in the notice, the
and separation of all its employees. The existence of petitioner labor unions would terminate with the dissolution dismissal is without prejudice, except that a notice operates as an adjudication upon the merits when filed by a
of its employer and the separation of its members. This is why the petition also prays that this Court issue an plaintiff who has once dismissed in a competent court an action based on or including the same claim.
order "that would smoothly preserve the toll operations services of respondent PNCC and/or respondent PSC
under its legislative franchise."91
In this case, petitioners PSCEU and PTMSDWO had filed a notice of dismissal of the complaint before the RTC
on 28 January 2008, before respondents filed their Answers. The following day, the RTC issued an order
We have recognized that the right of self-preservation is inherent in every labor union or any organization for confirming the dismissal. Under the above-cited rule, this confirmation is the only qualification imposed on the
that matter.92 Thus, PSCEU and PTMSDWO, as real parties in interest, have the personality to question the right of a party to dismiss the action before the adverse party files an answer. 98 In this case, the dismissal of the
assumption of the toll operations by SOMCO. action therefore became effective upon that confirmation by the RTC despite the subsequent filing of the
motions for partial reconsideration.
II
Thus, when the instant petition was filed on 4 February 2008, the complaint before the RTC was no longer
PSCEU and PTMSDWO are not guilty of forum-shopping. pending. The complaint was dismissed without prejudice by virtue of the notice of dismissal filed by petitioners
PSCEU and PTMSDWO. Consequently, there was not even any need for petitioners to mention the prior filing
Forum shopping refers to the act of availing of several remedies in different courts and/or administrative and dismissal of the complaint in the certificate of non-forum shopping in the instant petition,99 but they did so
agencies, either simultaneously or successively, when these remedies are substantially founded on the same anyway.100
PAGE 28 of 135 CONSTITUTIONAL LAW -EXECUTIVE DEPARTMENT PART 2 –SESSION 12
Parenthetically, in their motions for partial reconsideration, respondents PNCC and PSC insisted that the (4) That in time of war, rebellion, public peril, emergency, calamity, disaster or disturbance of peace
dismissal should have been with prejudice, because petitioners allegedly acted in bad faith in filing the notice of and order, the President of the Philippines may cause the total or partial closing of the toll facility or
dismissal, were guilty of forum shopping, and did not notify respondents of their intention to file a notice of order to take over thereof by the Government without prejudice to the payment of just compensation.
dismissal. With regard to the first and the third allegation, petitioners may ask for dismissal at any time before
the filing of the answer as a matter of right, even if the notice cites "the most ridiculous of grounds for (5) That no guarantee, Certificate of Indebtedness, collateral, securities, or bonds shall be issued by
dismissal."101 As to the second, we have already ruled that there was no forum shopping as regards the any government agency or government-owned or controlled corporation on any financing program of
successive filings of the labor case and the complaint before the RTC. the toll operator in connection with his undertaking under the Toll Operation Certificate.

II (6) The Toll Operation Certificate may be amended, modified or revoked whenever the public interest
so requires.
TRB has the power to grant authority to operate a toll facility.
(a) The Board shall promulgate rules and regulations governing the procedures for the grant of
This matter has already been settled by the Court in Francisco, Jr. v. TRB, 102 which ruled thus: Toll Certificates. The rights and privileges of a grantee under a Toll Operation Certificate shall
be defined by the Board.
It is abundantly clear that Sections 3 (a) and (e) of P.D. 1112 in relation to Section 4 of P.D. 1894 have invested
the TRB with sufficient power to grant a qualified person or entity with authority to construct, maintain, and (b) To issue rules and regulations to carry out the purposes of this Decree.
operate a toll facility and to issue the corresponding toll operating permit or TOC.
SECTION 4. The Toll Regulatory Board is hereby given jurisdiction and supervision over the GRANTEE with
Sections 3 (a) and (e) of P.D. 1112 and Section 4 of P.D. 1894 amply provide the power to grant authority to respect to the Expressways, the toll facilities necessarily appurtenant thereto and, subject to the provisions of
operate toll facilities: Section 8 and 9 hereof, the toll that the GRANTEE will charge the users thereof.

Section 3. Powers and Duties of the Board. - The Board shall have in addition to its general powers of By explicit provision of law, the TRB was given the power to grant administrative franchise for toll facility
administration the following powers and duties: projects.103(Emphases supplied)

(a) Subject to the approval of the President of the Philippines, to enter into contracts in behalf of the Republic of We cannot abide by the contention of petitioners that the franchise for toll operations was exclusively vested in
the Philippines with persons, natural or juridical, for the construction, operation and maintenance of toll facilities PNCC, which effectively breached its franchise when it transferred the toll operations to SOMCO. First, there is
such as but not limited to national highways, roads, bridges, and public thoroughfares. Said contract shall be nothing in P.D. 1113 or P.D. 1894 that states that the franchise granted to PNCC is to the exclusion of all others.
open to citizens of the Philippines and/or to corporations or associations qualified under the Constitution and
authorized by law to engage in toll operations; Second, if we were to go by the theory of petitioners, it is only the operation and maintenance of the toll facilities
that is vested with PNCC. This interpretation is contrary to the wording of P.D. 1113 and P.D. 1894 g ranting
xxxx PNCC the right, privilege and authority to construct, operate and maintain the North Luzon, South Luzon and
Metro Manila Expressways and their toll facilities.
(e) To grant authority to operate a toll facility and to issue therefore the necessary "Toll Operation Certificate"
subject to such conditions as shall be imposed by the Board including inter alia the following: It appears that petitioners have confused the franchise granted under P.D. 1113 and P.D. 1894 with particular
provisions in the STOA. To clarify, the operation and maintenance of the project roads were the primary and
(1) That the Operator shall desist from collecting toll upon the expiration of the Toll Operation exclusive privilege and responsibility of PNCC through PSC under the STOA. On the other hand, the design and
Certificate. construction of the project roads were the primary and exclusive privilege and responsibility of CMMTC.
However, with the execution of the AS TOA, the parties agreed that SOM CO shall replace PSC in undertaking
the operations and maintenance of the project roads. Thus, the "exclusivity clause" was a matter of agreement
(2) That the entire facility operated as a toll system including all operation and maintenance equipment between the parties, which amended it in a later contract; it was not a matter provided under the law.
directly related thereto shall be turned over to the government immediately upon the expiration of the
Toll Operation Certificate.
Third, aside from having been granted the power to grant administrative franchises for toll facility projects, TRB
is also empowered to modify, amend, and impose additional conditions on the franchise of PNCC in an
(3) That the toll operator shall not lease, transfer, grant the usufruct of, sell or assign the rights or appropriate contract, particularly when public interest calls for it. This is provided under Section 3 of P.D. 1113
privileges acquired under the Toll Operation Certificate to any person, firm, company, corporation or and Section 6 of P.D. 1894, to wit:
other commercial or legal entity, nor merge with any other company or corporation organized for the
same purpose, without the prior approval of the President of the Philippines. In the event of any valid
transfer of the Toll Operation Certificate, the Transferee shall be subject to all the conditions, terms, SECTION 3. This franchise is granted subject to such conditions as may be imposed by the [Toll Regulatory]
restrictions and limitations of this Decree as fully and completely and to the same extent as if the Toll Board in an appropriate contract to be executed for this purpose, and with the understanding and upon the
Operation Certificate has been granted to the same person, firm, company, corporation or other condition that it shall be subject to amendment, alteration or repeal when public interest so requires.
commercial or legal entity.
xxx
PAGE 29 of 135 CONSTITUTIONAL LAW -EXECUTIVE DEPARTMENT PART 2 –SESSION 12
SECTION 6. This franchise is granted subject to such conditions, consistent with the provisions of this Decree, right of delectus personae. Thus, the subject tollway projects were undertaken by companies, which are the
as may be imposed by the Toll Regulatory Board in the Toll Operation Agreement and such other modifications product of the joint ventures between PNCC and its chosen partners.107
or amendments that may be made thereto, and with the understanding and upon the condition that it shall be
subject to amendment or alteration when public interest so dictates. Under the STOA in this case, PNCC partnered with CMMTC in Stages 1 and 2 of the South Metro Manila
Skyway. The STOA gave birth to PSC, which was put in charge of the operation and maintenance of the project
Section 6 of P.D. 1894 specifically mentions the Toll Operation Agreement. The STOA was one such roads. The ASTOA had to be executed for Stage 2 to accommodate changes and modifications in the original
modification or amendment of the franchise of PNCC. So was the ASTOA, which further modified the franchise. design. The ASTOA then brought forth the incorporation of SOMCO to replace PSC in the operations and
PNCC cannot be said to have breached its franchise when it transferred the toll operations to SOMCO. PNCC maintenance of Stage 1 of the South Metro Manila Skyway. Clearly, no public bidding was necessary because
remained the franchise holder for the construction, operation, and maintenance of the project roads; it only opted PNCC, the franchisee, merely exercised its management prerogative when it decided to undertake the
to partner with investors in the exercise of its franchise leading to the organization of companies such as PSC construction, operation, and maintenance of the project roads through companies which are products of joint
and SOMCO. ventures with chosen partners.

Again, considering that PNCC was granted the right, privilege, and authority to construct, operate, and maintain Petitioners also insist that SOMCO is not qualified to operate a toll facility, because it does not meet the
the North Luzon, South Luzon, and Metro Manila Expressways and their toll facilities, we have not heard nationality requirement for a corporation when scrutinized under the "grandfather rule." Other than advancing
petitioners decrying the "breach" by PNCC of its franchise when it agreed to make CMMTC responsible for the this argument, however, petitioners have not shown how SOMCO fails to meet the nationality requirement for a
design and construction of the project roads under the STOA. public utility operator. Petitioners only aver in their petition that 40% of SOMCO is owned by CMMTC, a foreign
company, while the rest is owned by the following: a) Toll Road Operation and Maintenance Venture
IV Corporation (TROMVC), almost 40% of which is owned by a Singaporean company; b) Asset values Holding
Company, Inc. (AHCI), of which almost 40% is Dutch-owned; and c) Metro Strategic Infrastructure Holdings, Inc.
(MSIHI), 40% of which is owned by Metro Pacific Corporation, whose ownership or nationality was not
The TOC issued to SOMCO was not irregular. specified.108

Petitioners argue that the conditions provided under Section 3(e) of P.D. 1112104 were not imposed on SOMCO, Section 11, Article XII of the Constitution provides that "[n]o franchise, certificate, or any other form of
because these do not appear on the face of the TOC. Petitioners are mistaken. authorization for the operation of a public utility shall be granted except to citizens of the Philippines or to
corporations or associations organized under the laws of the Philippines at least sixty per centum of whose
The TOC, as a grant of authority from the government, is subject to the latter's control insofar as the grant capital is owned by such citizens x x x." Clearly, under the Constitution, a corporation at least 60% of whose
affects or concerns the public.105 Like all other franchises or licenses issued by the government, the TOC is capital is owned by Filipinos is of Philippine nationality. Considering this constitutional provision, petitioners'
issued subject to terms, conditions, and limitations under existing laws and agreements. This rule especially silence on the ownership of the remaining 60% of the corporations cited is very telling.
holds true in this instance since the TRB has the power to issue "the necessary 'Toll Operation Certificate'
subject to such conditions as shall be imposed by the Board including inter alia" those specified under Section In order to rebut petitioners' allegations, respondents readily present matrices showing the itemization of
3(e) of P.D. 1112. Thus, impliedly written into every TOC are the conditions prescribed therein. percentage ownerships of the subscribed capital stock of SOMCO, as well as that of TROMVC, AHCI, and
MSIHI. Respondents attempt to show that all these corporations are of Philippine nationality, with 60% of their
In any case, part of the TOC issued to SOMCO reads: capital stock owned by Filipino citizens. We need not reproduce the itemization here. Suffice it to say that in their
Consolidated Reply,109petitioners did not refute the unanimous claim of respondents. It is axiomatic that one who
Pursuant to Section 3(e) of Presidential Decree No. 1112 or the Toll Operation Decree, Skyway O & M alleges a fact has the burden of proving it. On this matter, we find that petitioners have failed to prove their
Corporation is hereby given authority to operate and maintain Stage 1 of the South Metro Manila Skyway allegation that SOMCO is not qualified to operate a toll facility for failure to meet the nationality requirement
effective as of 10:00 p.m. of 31 December 2007. under the Constitution.

This authorization is issued upon the clear understanding that the operation and maintenance of Stage 1 of the Finally, no public notices and hearings were necessary prior to the issuance of the TOC to SOMCO. For the
South Metro Manila Skyway as a toll facility and the collection of toll fees shall be closely supervised and same reason that a public bidding is not necessary, PNCC cannot be required to call for public hearings
regulated by the Grantor, by and through the Board of Directors, in accordance with the terms and conditions set concerning matters within its prerogative. At any rate, we have studied P.D. 1112 and the Implementing Rules
forth in the STOA, as amended, the rules and regulations duly promulgated by the Grantor for toll road and Regulations Authorizing the Establishment of Toll Facilities and found no provision requiring the issuance of
operations and maintenance, as well as the lawful orders, instructions and conditions which the Grantor, through public notices and the conduct of public hearings prior to the issuance of a TOC.
the TRB, may impose from time to time in view of the public nature of the facility.
V
As regards the allegation that none of the requirements for public bidding was observed before the TOC was
issued to SOMCO, this matter was also squarely answered by the Court in Francisco, Jr. v. TRB,106 to wit: Approval of the AS TOA by the DOTC Secretary was approval by the President.

Where, in the instant case, a franchisee undertakes the tollway projects of construction, rehabilitation and The doctrine of qualified political agency declares that, save in matters on which the Constitution or the
expansion of the tollways under its franchise, there is no need for a public bidding. In pursuing the projects with circumstances require the President to act personally, executive and administrative functions are exercised
the vast resource requirements, the franchisee can partner with other investors, which it may choose in the through executive departments headed by cabinet secretaries, whose acts are presumptively the acts of the
exercise of its management prerogatives. In this case, no public bidding is required upon the franchisee in President unless disapproved by the latter.110 As explained in Villena v. Executive Secretary,111 this doctrine is
choosing its partners as such process was done in the exercise of management prerogatives and in pursuit of its rooted in the Constitution:
PAGE 30 of 135 CONSTITUTIONAL LAW -EXECUTIVE DEPARTMENT PART 2 –SESSION 12
x x x With reference to the Executive Department of the government, there is one purpose which is crystal-clear The argument of petitioners is founded on the assumption that PNCC in some way leased, transferred, granted
and is readily visible without the projection of judicial searchlight, and that is, the establishment of a single, not the usufruct of, sold, or assigned to SOMCO its franchise or the rights or privileges PNCC had acquired by it.
plural, Executive. The first section of Article VII of the Constitution, dealing with the Executive Department, Here lies the error in petitioners' stand. First, as discussed above, the power to grant franchises or issue
begins with the enunciation of the principle that "The executive power shall be vested in a President of the authorizations for the operation of a public utility is not exclusively exercised by Congress. Second, except
Philippines." This means that the President of the Philippines is the Executive of the Government of the where the situation falls within that special class that demands the exclusive and personal exercise by the
Philippines, and no other. The heads of the executive departments occupy political positions and hold office in President of constitutionally vested power,117 the President acts through alter egos whose acts are as if the
an advisory capacity, and, in the language of Thomas Jefferson, "should be of the President's bosom Chief Executive's own.
confidence," and, in the language of Attorney-General Cushing, "are subject to the direction of the President."
Without minimizing the importance of the heads of the various departments, their personality is in reality but the Third, no lease, transfer, grant of usufruct, sale, or assignment of franchise by PNCC or its merger with another
projection of that of the President. Stated otherwise, and as forcibly characterized by Chief Justice Taft of the company ever took place.
Supreme Court of the United States, "each head of a department is, and must be, the President's alter ego in the
matters of that department where the President is required by law to exercise authority." Secretaries of
departments, of course, exercise certain powers under the law but the law cannot impair or in any way affect the The creation of the TRB and the grant of franchise to PNCC were made in the light of the recognition on the part
constitutional power of control and direction of the President. As a matter of executive policy, they may be of the government that the private sector had to be involved as an alternative source of financing for the
granted departmental autonomy as to certain matters but this is by mere concession of the executive, in the pursuance of national infrastructure projects. As the franchise holder for the construction, maintenance and
absence of valid legislation in the particular field. If the President, then, is the authority in the Executive operation of infrastructure toll facilities, PNCC was equipped with the right and privilege, but not necessarily the
Department, he assumes the corresponding responsibility. The head of a department is a man of his confidence; means, to undertake the project. This is where joint ventures with private investors become necessary.
he controls and directs his acts; he appoints him and can remove him at pleasure; he is the executive, not any of
his secretaries.112 x x x (Citations omitted) A joint venture is an association of companies jointly undertaking a commercial endeavor, with all of them
contributing assets and sharing risks, profits, and losses. 118 It is hardly distinguishable from a partnership
Applying the doctrine of qualified political agency, we have ruled that the Secretary of Environment and Natural considering that their elements are similar and, thus, generally governed by the law on partnership. 119
Resources can validly order the transfer of a regional office by virtue of the power of the President to reorganize
the national government.113 In Constantino v. Cuisia,114 the Court upheld the authority of the Secretary of In joint ventures with investor companies, PNCC contributes the franchise it possesses, while the partner
Finance to execute debt-relief contracts. The authority emanates from the power of the President to contract contributes the financing - both necessary for the construction, maintenance, and operation of the toll facilities.
foreign loans under Section 20, Article VII of the Constitution. In Angeles v. Gaite, 115 the Court ruled that there PNCC did not thereby lease, transfer, grant the usufruct of, sell, or assign its franchise or other rights or
can be no issue with regard to the President's act of limiting his power to review decisions and orders of the privileges. This remains true even though the partnership acquires a distinct and separate personality from that
Secretary of Justice, especially since the decision or order was issued by the secretary, the President's "own of the joint venturers or leads to the formation of a new company that is the product of such joint venture, such
alter ego."116 as PSC and SOMCO in this case.

There can be no question that the act of the secretary is the act of the President, unless repudiated by the latter. Hence, when we say that the approval by the DOTC Secretary in this case was approval by the President, it was
In this case, approval of the ASTOA by the DOTC Secretary had the same effect as approval by the President. not in connection with the franchise of PNCC, as required under Section 8 of P.D. 1113 and Section 13 of P.D.
The same would be true even without the issuance of E.O. 497, in which the President, on 24 January 2006, 1894. Rather, the approval was in connection with the powers of the TRB to enter into contracts on behalf of the
specifically delegated to the DOTC Secretary the authority to approve contracts entered into by the TRB. government as provided under Section 3(a) of P.D. 1112, which states:

Petitioners are unimpressed. They cite Section 8 of P.D. 1113 and Section 13 of P.D. 1894 as follows: SECTION 3. Powers and Duties of the Board. - The Board shall have in addition to its general powers of
administration the following powers and duties:
SECTION 8. The GRANTEE shall not lease, transfer, grant the usufruct of, sell or assign this franchise nor the
rights or privileges acquired hereby, to any person, firm, company, corporation or other commercial or legal (a) Subject to the approval of the President of the Philippines, to enter into contracts in behalf of the Republic of
entity, nor merge with any other company or corporation without the prior approval of the President of the the Philippines with persons, natural or juridical, for the construction, operation and maintenance of toll facilities
Philippines. In the event that this franchise is sold, transferred or assigned, the transferee shall be subject to all such as but not limited to national highways, roads, bridges, and public thoroughfares. Said contract shall be
the conditions, terms, restrictions and limitations of this Decree as fully and completely and to the same extents open to citizens of the Philippines and/or to corporations or associations qualified under the Constitution and
as if the franchise has been granted to the same person, firm, company, corporation or other commercial or legal authorized by law to engage in toll operations; (Emphasis supplied)
entity. (Emphasis supplied)
VI
SECTION 13. The GRANTEE shall not lease, transfer, grant the usufruct of, sell or assign this franchise nor the
rights or privileges required hereby, to any person, firm, company, corporation or other legal entity, nor merge Petitioners have not shown that the transfer of toll operations to SOM CO was grossly disadvantageous to the
with any other company or corporation without the prior approval of the President of the Philippines. In the event government.
that this franchise is sold, transferred or assigned, the transferee shall be subject to all the conditions, terms,
restrictions and limitations of this Decree as fully and completely and to the same extent as if the franchise has
been granted to the said person, firm, company, corporation or other legal entity. (Emphasis supplied) In support of their contention that the transfer of toll operations from PSC to SOMCO was grossly
Petitioners insist that based on the above provisions, it is the President who should give personal approval disadvantageous to the government, petitioners belittle the initial capital investment, private ownership, and
considering that the power to grant franchises was exclusively vested in Congress. Hence, to allow the DOTC track record of SOMCO.
Secretary to exercise the power of approval would supposedly dilute that legislative prerogative.
PAGE 31 of 135 CONSTITUTIONAL LAW -EXECUTIVE DEPARTMENT PART 2 –SESSION 12
When one uses the term "grossly disadvantageous to the government," the allegations in support thereof must It was therefore error on the part of petitioners to come directly before this Court for the sole reason that the
reflect the meaning accorded to the phrase. "Gross" means glaring, reprehensible, culpable, flagrant, and lower courts will not be able to grant the prayer for the issuance of a writ of preliminary injunction and/or
shocking.120 It requires that the mere allegation shows that the disadvantage on the part of the government is temporary restraining order to enjoin the assumption of toll operations by SOMCO. The error even takes on a
unmistakable, obvious, and certain. whole new meaning, because SOMCO assumed responsibility for the operations and maintenance of the South
Metro Manila Skyway at 10:00 p.m. on 31 December 2007. On the other hand, the complaint before the RTC
In this case, we find that the allegations of petitioners are nothing more than speculations, apprehensions, and seeking to enjoin the assumption by SOMCO was filed only on 3 January 2008, while the instant petition was
suppositions.1âwphi1 They speculate that with its "measly" capital investment, SOMCO would not be able to filed on 4 February 2008.
cover the overhead expenses for personal services alone. They fear that the revenue from toll operations would
go to "private pockets" in exchange for a small settlement amount to be given to PSC. Given that SOMCO has As we held in Aznar Brothers Realty, Inc. v. CA,125 injunction does not lie when the act sought to be enjoined
no proven track record, petitioners deduce that its assumption of the toll operations would lead to poor delivery has already become a fait accompli or an accomplished or consummated act.
of toll services to the public.
Parties must observe the hierarchy of courts before seeking relief from this Court. Observance thereof minimizes
The aim in the establishment of toll facilities is to draw from private resources the financing of government the imposition on the already limited time of this Court and prevents delay, intended or otherwise, in the
infrastructure projects. Naturally, these private investors would want to receive reasonable return on their adjudication of cases.126 We do not appreciate the litigants' practice of directly seeking recourse before this
investments. Thus, the collection of toll fees for the use of public improvements has been authorized, subject to Court, relying on the gravitas of a personality yet making serious claims without the proof to support them.
supervision and regulation by the national government. 121 As regards the ₱320 million settlement given to PSC,
the amount was to be used principally for the payment of its liabilities of PSC arising from the retrenchment of its WHEREFORE, the petition is DISMISSED. The prayer for the issuance of a writ of preliminary injunction and/or
employees. We note that under the MOA, the residual assets of PSC shall still be offered for sale to CMMTC, temporary restraining order is DENIED.
subject to valuation.122 Thus, it would be inaccurate to say that PSC would receive only ₱320 million for the
entire arrangement.
SO ORDERED.
It is quite understandable that SOMCO does not yet have a proven track record in toll operations, considering
that it was only the ASTOA and the MOA that gave birth to it. We are not prepared to rule that this lack of track
record would result in poor delivery of toll services, especially because most of the former employees of PSC
have been rehired by SOMCO, an allegation of respondents that was never refuted by petitioners. Neither are
we prepared to take the amount of SOMCO's initial capital investment against it, as it is considerably higher than
₱500,000, the authorized capital stock of PSC as of 2002.123

A FINAL NOTE

R.A. 8975 prohibits lower courts from issuing any temporary restraining order, preliminary injunction, or
preliminary mandatory injunction against the government - or any of its subdivisions, officials or any person or
entity, whether public or private, acting under the government's direction - to restrain, prohibit or compel acts
related to the implementation and completion of government infrastructure projects.

The rationale for the law is easily discernible. Injunctions and restraining orders tend to derail the expeditious
and efficient implementation and completion of government infrastructure projects; increase construction,
maintenance and repair costs; and delay the enjoyment of the social and economic benefits therefrom. Thus,
unless the matter is of extreme urgency involving a constitutional issue, judges of lower courts who shall issue
injunctive writs or restraining orders in violation of the law shall be administratively liable.

The law is clear that what is prohibited is merely the issuance of provisional orders enjoining the implementation
of a national government project. R.A. 8975 does not bar lower courts from assuming jurisdiction over
complaints that seek the nullification or implementation of a national government infrastructure project as
ultimate relief.124

There is no question that the ultimate prayer in the instant case is the nullification of a national government
project considering that the ASTOA involved the design and construction of Stage 2 of the South Metro Manila
Skyway, as well as the operation and maintenance of Stage 1 thereof. The prayer is grounded on the contract's
alleged unconstitutionality, violation of the law, and gross disadvantage to the government. Such principal action
and relief were within the jurisdiction of the RTC, which acted correctly when it ordered respondents to file their
respective answers to the complaint, even while it denied the prayer for the issuance of a writ of preliminary
injunction and/or temporary restraining order in observance of R.A. 8975.
PAGE 32 of 135 CONSTITUTIONAL LAW -EXECUTIVE DEPARTMENT PART 2 –SESSION 12
THIRD DIVISION respondent herein] from incumbent Schools Division Superintendent of Quezon City to Vocational
Schools Superintendent of the Marikina Institute of Science and Technology."[4]
[G.R. No. 119903. August 15, 2000]
On December 21, 1994, the Court of Appeals issued another resolution setting the hearing of the petition
HON. RICARDO T. GLORIA, in his capacity as SECRETARY, AND DIRECTOR NILO L. ROSAS in for the issuance of a writ of preliminary injunction and enjoining the petitioners from implementing the
his capacity as REGIONAL DIRECTOR, DEPARTMENT OF EDUCATION, CULTURE AND reassignment of the private respondent.
SPORTS, petitioners, vs. HON. COURT OF APPEALS AND DR. BIENVENIDO A.
ICASIANO, respondents. On March 28, 1995, it issued its assailed decision; holding as follows:

DECISION "WHEREFORE, for lack of a period or any indication that it is only temporary, the reassignment
of the petitioner from Schools Division Superintendent, Division of City Schools, Quezon City,
PURISIMA, J.: to Vocational Schools Superintendent of the Marikina Institute of Science and Technology
pursuant to the Memorandum of Secretary Ricardo T. Gloria to the President of the Philippines
dated 10 October 1994, is hereby declared to be violative of petitioners right to security of tenure,
This is a petition for review on certiorari under Rule 45 of the Rules of Court brought by Secretary and and the respondents are hereby prohibited from implementing the same.
the Director for the National Capital Region of the Department of Education, Culture and Sports (DECS),
to question the decision[1] of the Court of Appeals in CA-G.R. SP No. 35505.
SO ORDERED."[5]
The Court of Appeals found the facts as follows:
Petitioners are now before the Court seeking relief from the decision of the appellate court, contending
that:
"On June 29, 1989, petitioner [private respondent herein] was appointed Schools Division
Superintendent, Division of City Schools, Quezon City, by the then President Corazon C.
Aquino. I

On October 10, 1994, respondent Secretary Gloria recommended to the President of the RESPONDENT COURT OF APPEALS HAS ALLOWED ITSELF TO BE INSTRUMENTAL IN PRIVATE
Philippines that the petitioner be reassigned as Superintendent of the MIST [Marikina Institute RESPONDENTS CIRCUMVENTION OF THE PRESIDENTIAL IMMUNITY FROM SUIT BY GIVING
of Science and Technology], to fill up the vacuum created by the retirement of its DUE COURSE AND GRANTING RELIEFS PRAYED FOR IN A SUIT PURPORTEDLY FILED AGAINST
Superintendent, Mr. Bannaoag F. Lauro, on June 17, 1994. PETITIONERS BUT ACTUALLY QUESTIONING AN ACT OF THE PRESIDENT.

On October 12, 1994, the President approved the recommendation of Secretary Gloria. II

On October 13, 1994, a copy of the recommendation for petitioners reassignment, as approved RESPONDENT COURT OF APPEALS HAS DECIDED A QUESTION OF SUBSTANCE IN A WAY NOT
by the President, was transmitted by Secretary Gloria to Director Rosas for implementation. IN ACCORD WITH LAW OR APPLICABLE DECISIONS OF THE SUPREME COURT[6]

On October 14, 1994, Director Rosas, informed the petitioner of his reassignment, effective The pivotal issue for resolution here is whether the reassignment of private respondent from School
October 17, 1994. Division Superintendent of Quezon City to Vocational School Superintendent of MIST is violative of his
security of tenure? Petitioners maintain that there is no violation of security of tenure involved. Private
respondent maintains otherwise.
Petitioner requested respondent Secretary Gloria to reconsider the reassignment, but the latter
denied the request. The petitioner prepared a letter dated October 18, 1994 to the President of
the Philippines, asking for a reconsideration of his reassignment, and furnished a copy of the In taking favorable action on private respondents petition for prohibition, the Court of Appeals
same to the DECS. However, he subsequently changed his mind and refrained from filing the ratiocinated:
letter with the Office of President.
"Notwithstanding the protestations of counsel for the respondents, the reassignment of the
On October 19, 1994, the petitioner filed the instant petition."[2] petitioner to MIST appears to be indefinite. No period is fixed. No objective or purpose, from
which the temporariness of the assignment may be inferred, is set. In fact, the recommendation
of respondent Secretary Gloria to the President that the position of superintendent of MIST will
On October 26, 1994, the Court of Appeals denied private respondents prayer for the issuance of a best fit his (petitioners) qualifications and experience. (Exh. C-2) implies that the proposed
Temporary Restraining Order (TRO).[3] reassignment will be indefinite."[7]

On November 22, 1994, it set aside its earlier resolution denying the prayer for the issuance of a TRO; Petitioners theorize that the present petition for prohibition is improper because the same attacks an act
and thereafter, restrained the petitioners "from implementing the re-assignment of the petitioner [private of the President, in violation of the doctrine of presidential immunity from suit.
PAGE 33 of 135 CONSTITUTIONAL LAW -EXECUTIVE DEPARTMENT PART 2 –SESSION 12
Petitioners contention is untenable for the simple reason that the petition is directed against petitioners SO ORDERED.
and not against the President. The questioned acts are those of petitioners and not of the President.
Furthermore, presidential decisions may be questioned before the courts where there is grave abuse of
discretion or that the President acted without or in excess of jurisdiction. [8]

Petitioners submission that the petition of private respondent with the Court of Appeals is improper for
failing to show that petitioners constituted themselves into a "court" conducting a "proceeding" and for
failing to show that any of the petitioners acted beyond their jurisdiction in the exercise of their judicial or
ministerial functions, is barren of merit. Private respondent has clearly averred that the petitioners acted
with grave abuse of discretion amounting to lack of jurisdiction and/or excess of jurisdiction in reassigning
the private respondent in a way that infringed upon his security of tenure. And petitioners themselves
admitted that their questioned act constituted a ministerial duty, such that they could be subject to charges
of insubordination if they did not comply with the presidential order. What is more, where an administrative
department acts with grave abuse of discretion, which is equivalent to a capricious and whimsical
exercise of judgment, or where the power is exercised in an arbitrary or despotic manner, there is a
justification for the courts to set aside the administrative determination thus reached. [9]

Petitioners contend that the doctrine enunciated in Bentain vs. Court of Appeals[10] -- that "a reassignment
that is indefinite and results in a reduction in rank, status and salary, is in effect, a constructive removal
from the service" -- does not apply in the present case for the reassignment in question was merely
temporary, lasting only until the appointment of a new Vocational School Superintendent of MIST.

After a careful study, the Court upholds the finding of the respondent court that the reassignment of
petitioner to MIST "appears to be indefinite". The same can be inferred from the Memorandum[11] of
Secretary Gloria for President Fidel V. Ramos to the effect that the reassignment of private respondent
will "best fit his qualifications and experience" being "an expert in vocational and technical education." It
can thus be gleaned that subject reassignment is more than temporary as the private respondent has
been described as fit for the (reassigned) job, being an expert in the field. Besides, there is nothing in the
said Memorandum to show that the reassignment of private respondent is temporary or would only last
until a permanent replacement is found as no period is specified or fixed; which fact evinces an intention
on the part of petitioners to reassign private respondent with no definite period or duration. Such feature
of the reassignment in question is definitely violative of the security of tenure of the private respondent.
As held in Bentain:

"Security of tenure is a fundamental and constitutionally guaranteed feature of our civil service.
The mantle of its protection extends not only to employees removed without cause but also to
cases of unconsented transfers which are tantamount to illegal removals (Department of
Education, Culture and Sports vs. Court of Appeals, 183 SCRA 555; Ibanez vs. COMELEC, 19
SCRA 1002; Brillantes vs. Guevarra, 27 SCRA 138).

While a temporary transfer or assignment of personnel is permissible even without the


employees prior consent, it cannot be done when the transfer is a preliminary step toward his
removal, or is a scheme to lure him away from his permanent position, or designed to indirectly
terminate his service, or force his resignation. Such a transfer would in effect circumvent the
provision which safeguards the tenure of office of those who are in the Civil Service (Sta. Maria
vs. Lopez, 31 SCRA 651; Garcia vs. Lejano, 109 Phil. 116)."[12]

Having found the reassignment of private respondent to the MIST to be violative of his security of tenure,
the order for his reassignment to the MIST cannot be countenanced.

WHEREFORE, the petition is hereby DENIED, and the Decision of the Court of Appeals in CA-G.R. SP
No. 35505 AFFIRMED. No pronouncement as to costs.
PAGE 34 of 135 CONSTITUTIONAL LAW -EXECUTIVE DEPARTMENT PART 2 –SESSION 12
EN BANC The said circular likewise provided for its immediate effectivity without need of publication:

5.0 EFFECTIVITY

[G.R. No. 125350. December 3, 2002] This Circular shall take effect immediately.

Acting on the DBM directive, the Mandaue City Auditor issued notices of disallowance to herein petitioners,
namely, Honorable RTC Judges Mercedes G. Dadole, Ulric R. Caete, Agustin R. Vestil, Honorable MTC Judges
HON. RTC JUDGES MERCEDES G. DADOLE (Executive Judge, Branch 28), ULRIC R. CAETE (Presiding Temistocles M. Boholst, Vicente C. Fanilag and Wilfredo A. Dagatan, in excess of the amount authorized by LBC
Judge, Branch 25), AGUSTINE R. VESTIL (Presiding Judge, Branch 56), HON. MTC JUDGES 55. Beginning October, 1994, the additional monthly allowances of the petitioner judges were reduced to P1,000
TEMISTOCLES M. BOHOLST (Presiding Judge, Branch 1), VICENTE C. FANILAG (Judge each. They were also asked to reimburse the amount they received in excess of P1,000 from April to September,
Designate, Branch 2), and WILFREDO A. DAGATAN (Presiding Judge, Branch 3), all of Mandaue 1994.
City, petitioners, vs. COMMISSION ON AUDIT, respondent.
The petitioner judges filed with the Office of the City Auditor a protest against the notices of disallowance.
But the City Auditor treated the protest as a motion for reconsideration and indorsed the same to the COA Regional
DECISION Office No. 7. In turn, the COA Regional Office referred the motion to the head office with a recommendation that
CORONA, J.: the same be denied.
On September 21, 1995, respondent COA rendered a decision denying petitioners motion for
Before us is a petition for certiorari under Rule 64 to annul the decision[1] and resolution[2], dated September reconsideration. The COA held that:
21, 1995 and May 28, 1996, respectively, of the respondent Commission on Audit (COA) affirming the notices of
the Mandaue City Auditor which diminished the monthly additional allowances received by the petitioner judges The issue to be resolved in the instant appeal is whether or not the City Ordinance of Mandaue which provides a
of the Regional Trial Court (RTC) and Municipal Trial Court (MTC) stationed in Mandaue City. higher rate of allowances to the appellant judges may prevail over that fixed by the DBM under Local Budget
The undisputed facts are as follows: Circular No. 55 dated March 15, 1994.

In 1986, the RTC and MTC judges of Mandaue City started receiving monthly allowances of P1,260 each xxx xxx xxx
through the yearly appropriation ordinance enacted by the Sangguniang Panlungsod of the said city. In 1991,
Mandaue City increased the amount to P1,500 for each judge.
Applying the foregoing doctrine, appropriation ordinance of local government units is subject to the organizational,
On March 15, 1994, the Department of Budget and Management (DBM) issued the disputed Local Budget budgetary and compensation policies of budgetary authorities (COA 5th Ind., dated March 17, 1994 re: Province
Circular No. 55 (LBC 55) which provided that: of Antique; COA letter dated May 17, 1994 re: Request of Hon. Renato Leviste, Cong. 1st Dist. Oriental Mindoro).
In this regard, attention is invited to Administrative Order No. 42 issued on March 3, 1993 by the President of the
xxx xxx xxx Philippines clarifying the role of DBM in the compensation and classification of local government positions under
RA No. 7160 vis-avis the provisions of RA No. 6758 in view of the abolition of the JCLGPA. Section 1 of said
Administrative Order provides that:
2.3.2. In the light of the authority granted to the local government units under the Local Government Code to
provide for additional allowances and other benefits to national government officials and employees assigned in
their locality, such additional allowances in the form of honorarium at rates not exceeding P1,000.00 in provinces Section 1. The Department of Budget and Management as the lead administrator of RA No. 6758 shall, through
and cities and P700.00 in municipalities may be granted subject to the following conditions: its Compensation and Position Classification Bureau, continue to have the following responsibilities in connection
with the implementation of the Local Government Code of 1991:
a) That the grant is not mandatory on the part of the LGUs;
a) Provide guidelines on the classification of local government positions and on the specific
rates of pay therefore;
b) That all contractual and statutory obligations of the LGU including the implementation of R.A. 6758 shall have
been fully provided in the budget;
b) Provide criteria and guidelines for the grant of all allowances and additional forms of
compensation to local government employees; xxx. (underscoring supplied)
c) That the budgetary requirements/limitations under Section 324 and 325 of R.A. 7160 should be satisfied and/or
complied with; and
To operationalize the aforecited presidential directive, DBM issued LBC No. 55, dated March 15, 1994, whose
effectivity clause provides that:
d) That the LGU has fully implemented the devolution of functions/personnel in accordance with R.A.
7160.[3] (italics supplied)
xxx xxx xxx
xxx xxx xxx
5.0 EFFECTIVITY
PAGE 35 of 135 CONSTITUTIONAL LAW -EXECUTIVE DEPARTMENT PART 2 –SESSION 12
This Circular shall take effect immediately. They maintain that said circular is not supported by any law and therefore goes beyond the supervisory powers of
the President. They further allege that said circular is void for lack of publication.
It is a well-settled rule that implementing rules and regulations promulgated by administrative or executive officer On the other hand, the yearly appropriation ordinance providing for additional allowances to judges is allowed
in accordance with, and as authorized by law, has the force and effect of law or partake the nature of a statute by Section 458, par. (a)(1)[xi], of RA 7160, otherwise known as the Local Government Code of 1991, which
(Victorias Milling Co., Inc., vs. Social Security Commission, 114 Phil. 555, cited in Agpalos Statutory Construction, provides that:
2nd Ed. P. 16; Justice Cruzs Phil. Political Law, 1984 Ed., p. 103; Espanol vs. Phil Veterans Administration, 137
SCRA 314; Antique Sawmills Inc. vs. Tayco, 17 SCRA 316).
Sec. 458. Powers, Duties, Functions and Compensation. (a) The sangguniang panlungsod, as the legislative body
of the city, shall enact ordinances, approve resolutions and appropriate funds for the general welfare of the city
xxx xxx xxx and its inhabitants pursuant to Section 16 of this Code and in the proper exercise of the corporate powers of the
city as provided for under Section 22 of this Code, and shall:
There being no statutory basis to grant additional allowance to judges in excess of P1,000.00 chargeable against
the local government units where they are stationed, this Commission finds no substantial grounds or cogent (1) Approve ordinances and pass resolutions necessary for an efficient and effective city government, and in this
reason to disturb the decision of the City Auditor, Mandaue City, disallowing in audit the allowances in question. connection, shall:
Accordingly, the above-captioned appeal of the MTC and RTC Judges of Mandaue City, insofar as the same is
not covered by Circular Letter No. 91-7, is hereby dismissed for lack of merit.
xxx xxx xxx
xxx xxx xxx[4]
(xi) When the finances of the city government allow, provide for additional allowances and other benefits to judges,
prosecutors, public elementary and high school teachers, and other national government officials stationed in or
On November 27, 1995, Executive Judge Mercedes Gozo-Dadole, for and in behalf of the petitioner judges, assigned to the city; (italics supplied)
filed a motion for reconsideration of the decision of the COA. In a resolution dated May 28, 1996, the COA denied
the motion.
Instead of filing a comment on behalf of respondent COA, the Solicitor General filed a manifestation
Hence, this petition for certiorari by the petitioner judges, submitting the following questions for resolution: supporting the position of the petitioner judges. The Solicitor General argues that (1) DBM only enjoys the power
to review and determine whether the disbursements of funds were made in accordance with the ordinance passed
I by a local government unit while (2) the COA has no more than auditorial visitation powers over local government
units pursuant to Section 348 of RA 7160 which provides for the power to inspect at any time the financial accounts
HAS THE CITY OF MANDAUE STATUTORY AND CONSTITUTIONAL BASIS TO PROVIDE ADDITIONAL of local government units.
ALLOWANCES AND OTHER BENEFITS TO JUDGES STATIONED IN AND ASSIGNED TO THE CITY?
Moreover, the Solicitor General opines that the DBM and the respondent are only authorized under RA 7160
to promulgate a Budget Operations Manual for local government units, to improve and systematize methods,
II techniques and procedures employed in budget preparation, authorization, execution and accountability pursuant
to Section 354 of RA 7160. The Solicitor General points out that LBC 55 was not exercised under any of the
CAN AN ADMINISTRATIVE CIRCULAR OR GUIDELINE SUCH AS LOCAL BUDGET CIRCULAR NO. 55 aforementioned provisions.
RENDER INOPERATIVE THE POWER OF THE LEGISLATIVE BODY OF A CITY BY SETTING A LIMIT TO THE
EXTENT OF THE EXERCISE OF SUCH POWER? Respondent COA, on the other hand, insists that the constitutional and statutory authority of a city
government to provide allowances to judges stationed therein is not absolute. Congress may set limitations on the
exercise of autonomy. It is for the President, through the DBM, to check whether these legislative limitations are
III being followed by the local government units.

HAS THE COMMISSION ON AUDIT CORRECTLY INTERPRETED LOCAL BUDGET CIRCULAR NO. 55 TO One such law imposing a limitation on a local government units autonomy is Section 458, par. (a) (1) [xi], of
INCLUDE MEMBERS OF THE JUDICIARY IN FIXING THE CEILING OF ADDITIONAL ALLOWANCES AND RA 7160, which authorizes the disbursement of additional allowances and other benefits to judges subject to the
BENEFITS TO BE PROVIDED TO JUDGES STATIONED IN AND ASSIGNED TO MANDAUE CITY BY THE condition that the finances of the city government should allow the same. Thus, DBM is merely enforcing the
CITY GOVERNMENT AT P1,000.00 PER MONTH NOTWITHSTANDING THAT THEY HAVE BEEN RECEIVING condition of the law when it sets a uniform maximum amount for the additional allowances that a city government
ALLOWANCES OF P1,500.00 MONTHLY FOR THE PAST FIVE YEARS? can release to judges stationed therein.
Assuming arguendo that LBC 55 is void, respondent COA maintains that the provisions of the yearly
IV approved ordinance granting additional allowances to judges are still prohibited by the appropriation laws passed
by Congress every year. COA argues that Mandaue City gets the funds for the said additional allowances of judges
IS LOCAL BUDGET CIRCULAR NO. 55 DATED MARCH 15, 1994 ISSUED BY THE DEPARTMENT OF BUDGET from the Internal Revenue Allotment (IRA). But the General Appropriations Acts of 1994 and 1995 do not mention
AND MANAGEMENT VALID AND ENFORCEABLE CONSIDERING THAT IT WAS NOT DULY PUBLISHED IN the disbursement of additional allowances to judges as one of the allowable uses of the IRA. Hence, the provisions
ACCODANCE WITH LAW?[5] of said ordinance granting additional allowances, taken from the IRA, to herein petitioner judges are void for being
contrary to law.
Petitioner judges argue that LBC 55 is void for infringing on the local autonomy of Mandaue City by dictating To resolve the instant petition, there are two issues that we must address: (1) whether LBC 55 of the DBM
a uniform amount that a local government unit can disburse as additional allowances to judges stationed therein. is void for going beyond the supervisory powers of the President and for not having been published and (2) whether
PAGE 36 of 135 CONSTITUTIONAL LAW -EXECUTIVE DEPARTMENT PART 2 –SESSION 12
the yearly appropriation ordinance enacted by the City of Mandaue that provides for additional allowances to local government units. Hence, the President or any of his or her alter egos cannot interfere in local affairs as long
judges contravenes the annual appropriation laws enacted by Congress. as the concerned local government unit acts within the parameters of the law and the Constitution. Any directive
therefore by the President or any of his or her alter egos seeking to alter the wisdom of a law-conforming judgment
We rule in favor of the petitioner judges. on local affairs of a local government unit is a patent nullity because it violates the principle of local autonomy and
On the first issue, we declare LBC 55 to be null and void. separation of powers of the executive and legislative departments in governing municipal corporations.

We recognize that, although our Constitution[6] guarantees autonomy to local government units, the exercise Does LBC 55 go beyond the law it seeks to implement? Yes.
of local autonomy remains subject to the power of control by Congress and the power of supervision by the LBC 55 provides that the additional monthly allowances to be given by a local government unit should not
President. Section 4 of Article X of the 1987 Philippine Constitution provides that: exceed P1,000 in provinces and cities and P700 in municipalities. Section 458, par. (a)(1)(xi), of RA 7160, the law
that supposedly serves as the legal basis of LBC 55, allows the grant of additional allowances to judges when the
Sec. 4. The President of the Philippines shall exercise general supervision over local governments. x x x finances of the city government allow. The said provision does not authorize setting a definite maximum limit to
the additional allowances granted to judges. Thus, we need not belabor the point that the finances of a city
In Pimentel vs. Aguirre[7], we defined the supervisory power of the President and distinguished it from the government may allow the grant of additional allowances higher than P1,000 if the revenues of the said city
power of control exercised by Congress. Thus: government exceed its annual expenditures. Thus, to illustrate, a city government with locally generated annual
revenues of P40 million and expenditures of P35 million can afford to grant additional allowances of more
than P1,000 each to, say, ten judges inasmuch as the finances of the city can afford it.
This provision (Section 4 of Article X of the 1987 Philippine Constitution) has been interpreted to exclude the power
of control. In Mondano v. Silvosa,[i][5] the Court contrasted the President's power of supervision over local Setting a uniform amount for the grant of additional allowances is an inappropriate way of enforcing the
government officials with that of his power of control over executive officials of the national government. It was criterion found in Section 458, par. (a)(1)(xi), of RA 7160. The DBM over-stepped its power of supervision over
emphasized that the two terms -- supervision and control -- differed in meaning and extent. The Court distinguished local government units by imposing a prohibition that did not correspond with the law it sought to implement. In
them as follows: other words, the prohibitory nature of the circular had no legal basis.
Furthermore, LBC 55 is void on account of its lack of publication, in violation of our ruling in Taada vs.
"x x x In administrative law, supervision means overseeing or the power or authority of an officer to see that Tuvera[8] where we held that:
subordinate officers perform their duties. If the latter fail or neglect to fulfill them, the former may take such action
or step as prescribed by law to make them perform their duties. Control, on the other hand, means the power of
an officer to alter or modify or nullify or set aside what a subordinate officer ha[s] done in the performance of his xxx. Administrative rules and regulations must also be published if their purpose is to enforce or implement existing
duties and to substitute the judgment of the former for that of the latter." [ii][6] law pursuant to a valid delegation.

In Taule v. Santos,[iii][7] we further stated that the Chief Executive wielded no more authority than that of checking Interpretative regulations and those merely internal in nature, that is, regulating only the personnel of an
whether local governments or their officials were performing their duties as provided by the fundamental law and administrative agency and the public, need not be published. Neither is publication required of the so-called letters
by statutes. He cannot interfere with local governments, so long as they act within the scope of their of instruction issued by administrative superiors concerning the rules or guidelines to be followed by their
authority. "Supervisory power, when contrasted with control, is the power of mere oversight over an inferior body; subordinates in the performance of their duties.
it does not include any restraining authority over such body,"[iv][8] we said.
Respondent COA claims that publication is not required for LBC 55 inasmuch as it is merely an interpretative
In a more recent case, Drilon v. Lim,[v][9] the difference between control and supervision was further regulation applicable to the personnel of an LGU. We disagree. In De Jesus vs. Commission on Audit[9] where we
delineated. Officers in control lay down the rules in the performance or accomplishment of an act. If these rules dealt with the same issue, this Court declared void, for lack of publication, a DBM circular that disallowed payment
are not followed, they may, in their discretion, order the act undone or redone by their subordinates or even decide of allowances and other additional compensation to government officials and employees. In refuting respondent
to do it themselves. On the other hand, supervision does not cover such authority. Supervising officials merely COAs argument that said circular was merely an internal regulation, we ruled that:
see to it that the rules are followed, but they themselves do not lay down such rules, nor do they have the discretion
to modify or replace them. If the rules are not observed, they may order the work done or redone, but only to On the need for publication of subject DBM-CCC No. 10, we rule in the affirmative. Following the doctrine
conform to such rules. They may not prescribe their own manner of execution of the act. They have no discretion enunciated in Taada v. Tuvera, publication in the Official Gazette or in a newspaper of general circulation in the
on this matter except to see to it that the rules are followed. Philippines is required since DBM-CCC No. 10 is in the nature of an administrative circular the purpose of
which is to enforce or implement an existing law. Stated differently, to be effective and enforceable, DBM-
Under our present system of government, executive power is vested in the President. [vi][10] The members of the CCC No. 10 must go through the requisite publication in the Official Gazette or in a newspaper of general
Cabinet and other executive officials are merely alter egos. As such, they are subject to the power of control of circulation in the Philippines.
the President, at whose will and behest they can be removed from office; or their actions and decisions changed,
suspended or reversed.[vii][11] In contrast, the heads of political subdivisions are elected by the people. Their In the present case under scrutiny, it is decisively clear that DBM-CCC No. 10, which completely disallows payment
sovereign powers emanate from the electorate, to whom they are directly accountable. By constitutional fiat, they of allowances and other additional compensation to government officials and employees, starting November 1,
are subject to the Presidents supervision only, not control, so long as their acts are exercised within the sphere of 1989, is not a mere interpretative or internal regulation. It is something more than that. And why not, when it tends
their legitimate powers. By the same token, the President may not withhold or alter any authority or power given to deprive government workers of their allowance and additional compensation sorely needed to keep body and
them by the Constitution and the law. soul together. At the very least, before the said circular under attack may be permitted to substantially
reduce their income, the government officials and employees concerned should be apprised and alerted
Clearly then, the President can only interfere in the affairs and activities of a local government unit if he or by the publication of subject circular in the Official Gazette or in a newspaper of general circulation in the
she finds that the latter has acted contrary to law. This is the scope of the Presidents supervisory powers over Philippines to the end that they be given amplest opportunity to voice out whatever opposition they may
PAGE 37 of 135 CONSTITUTIONAL LAW -EXECUTIVE DEPARTMENT PART 2 –SESSION 12
have, and to ventilate their stance on the matter. This approach is more in keeping with democratic independent component cities, and municipalities within the Metropolitan Manila Area in accordance with the
precepts and rudiments of fairness and transparency. (emphasis supplied) immediately succeeding Section.

In Philippine International Trading Corporation vs. Commission on Audit [10], we again declared the same Section 327. Review of Appropriation Ordinances of Component Cities and Municipalities.- The sangguninang
circular as void, for lack of publication, despite the fact that it was re-issued and then submitted for publication. panlalawigan shall review the ordinance authorizing annual or supplemental appropriations of component cities
Emphasizing the importance of publication to the effectivity of a regulation, we therein held that: and municipalities in the same manner and within the same period prescribed for the review of other ordinances.

It has come to our knowledge that DBM-CCC No. 10 has been re-issued in its entirety and submitted for publication If within ninety (90) days from receipt of copies of such ordinance, the sangguniang panlalawigan takes
in the Official Gazette per letter to the National Printing Office dated March 9, 1999. Would the subsequent no action thereon, the same shall be deemed to have been reviewed in accordance with law and shall
publication thereof cure the defect and retroact to the time that the above-mentioned items were disallowed in continue to be in full force and effect. (emphasis supplied)
audit?
Within 90 days from receipt of the copies of the appropriation ordinance, the DBM should have taken positive
The answer is in the negative, precisely for the reason that publication is required as a condition precedent to the action. Otherwise, such ordinance was deemed to have been properly reviewed and deemed to have taken effect.
effectivity of a law to inform the public of the contents of the law or rules and regulations before their rights and Inasmuch as, in the instant case, the DBM did not follow the appropriate procedure for reviewing the subject
interests are affected by the same. From the time the COA disallowed the expenses in audit up to the filing of ordinance of Mandaue City and allowed the 90-day period to lapse, it can no longer question the legality of the
herein petition the subject circular remained in legal limbo due to its non-publication. As was stated in Taada v. provisions in the said ordinance granting additional allowances to judges stationed in the said city.
Tuvera, prior publication of laws before they become effective cannot be dispensed with, for the reason that it
would deny the public knowledge of the laws that are supposed to govern it. [11] WHEREFORE, the petition is hereby GRANTED, and the assailed decision and resolution, dated September
21, 1995 and May 28, 1996, respectively, of the Commission on Audit are hereby set aside.
We now resolve the second issue of whether the yearly appropriation ordinance enacted by Mandaue City No costs.
providing for fixed allowances for judges contravenes any law and should therefore be struck down as null and
void. SO ORDERED

According to respondent COA, even if LBC 55 were void, the ordinances enacted by Mandaue City granting
additional allowances to the petitioner judges would still (be) bereft of legal basis for want of a lawful source of
funds considering that the IRA cannot be used for such purposes. Respondent COA showed that Mandaue Citys
funds consisted of locally generated revenues and the IRA. From 1989 to 1995, Mandaue Citys yearly
expenditures exceeded its locally generated revenues, thus resulting in a deficit. During all those years, it was the
IRA that enabled Mandaue City to incur a surplus. Respondent avers that Mandaue City used its IRA to pay for
said additional allowances and this violated paragraph 2 of the Special Provisions, page 1060, of RA 7845 (The
General Appropriations Act of 1995)[12] and paragraph 3 of the Special Provision, page 1225, of RA 7663 (The
General Appropriations Act of 1994)[13] which specifically identified the objects of expenditure of the IRA. Nowhere
in said provisions of the two budgetary laws does it say that the IRA can be used for additional allowances of
judges. Respondent COA thus argues that the provisions in the ordinance providing for such disbursement are
against the law, considering that the grant of the subject allowances is not within the specified use allowed by the
aforesaid yearly appropriations acts.
We disagree.
Respondent COA failed to prove that Mandaue City used the IRA to spend for the additional allowances of
the judges. There was no evidence submitted by COA showing the breakdown of the expenses of the city
government and the funds used for said expenses. All the COA presented were the amounts expended, the locally
generated revenues, the deficit, the surplus and the IRA received each year. Aside from these items, no data or
figures were presented to show that Mandaue City deducted the subject allowances from the IRA. In other words,
just because Mandaue Citys locally generated revenues were not enough to cover its expenditures, this did not
mean that the additional allowances of petitioner judges were taken from the IRA and not from the citys own
revenues.
Moreover, the DBM neither conducted a formal review nor ordered a disapproval of Mandaue Citys
appropriation ordinances, in accordance with the procedure outlined by Sections 326 and 327 of RA 7160 which
provide that:

Section 326. Review of Appropriation Ordinances of Provinces, Highly Urbanized Cities, Independent Component
Cities, and Municipalities within the Metropolitan Manila Area. The Department of Budget and Management shall
review ordinances authorizing the annual or supplemental appropriations of provinces, highly-urbanized cities,
PAGE 38 of 135 CONSTITUTIONAL LAW -EXECUTIVE DEPARTMENT PART 2 –SESSION 12
allowed by the Constitution and by law to prevent and suppress all incidents of lawless violence in the named
places.

Three days later or on November 27, President Arroyo also issued Administrative Order 273 (AO
EN BANC 273)[2] transferring supervision of the Autonomous Region of Muslim Mindanao (ARMM) from the Office of the
President to the Department of Interior and Local Government (DILG). But, due to issues raised over the
terminology used in AO 273, the President issued Administrative Order 273-A (AO 273-A) amending the former,
DATU ZALDY UY AMPATUAN, G.R. No. 190259 by delegating instead of transferring supervision of the ARMM to the DILG. [3]
ANSARUDDIN ADIONG, REGIE
SAHALI-GENERALE Claiming that the Presidents issuances encroached on the ARMMs autonomy, petitioners Datu Zaldy Uy
Petitioners, Present: Ampatuan, Ansaruddin Adiong, and Regie Sahali-Generale, all ARMM officials,[4] filed this petition for prohibition
CORONA, C.J., under Rule 65. They alleged that the proclamation and the orders empowered the DILG Secretary to take over
CARPIO, ARMMs operations and seize the regional governments powers, in violation of the principle of local autonomy
CARPIO MORALES, under Republic Act 9054 (also known as the Expanded ARMM Act) and the Constitution. The President gave the
VELASCO, JR., DILG Secretary the power to exercise, not merely administrative supervision, but control over the ARMM since
NACHURA, the latter could suspend ARMM officials and replace them.[5]
LEONARDO-DE CASTRO,
- versus - BRION, Petitioner ARMM officials claimed that the President had no factual basis for declaring a state of emergency,
PERALTA, especially in the Province of Sultan Kudarat and the City of Cotabato, where no critical violent incidents occurred.
BERSAMIN, The deployment of troops and the taking over of the ARMM constitutes an invalid exercise of the Presidents
DEL CASTILLO, emergency powers.[6] Petitioners asked that Proclamation 1946 as well as AOs 273 and 273-A be declared
ABAD, unconstitutional and that respondents DILG Secretary, the AFP, and the PNP be enjoined from implementing
VILLARAMA, JR., them.
PEREZ,
MENDOZA, and In its comment for the respondents,[7] the Office of the Solicitor General (OSG) insisted that the President
SERENO, JJ. issued Proclamation 1946, not to deprive the ARMM of its autonomy, but to restore peace and order in subject
HON. RONALDO PUNO, in his capacity places.[8] She issued the proclamation pursuant to her calling out power[9] as Commander-in-Chief under the first
as Secretary of the Department of Interior sentence of Section 18, Article VII of the Constitution. The determination of the need to exercise this power rests
and Local Government and alter-ego of solely on her wisdom.[10] She must use her judgment based on intelligence reports and such best information as
President Gloria Macapagal-Arroyo, are available to her to call out the armed forces to suppress and prevent lawless violence wherever and whenever
and anyone acting in his stead and on these reared their ugly heads.
behalf of the President of the Philippines, On the other hand, the President merely delegated through AOs 273 and 273-A her supervisory powers
ARMED FORCES OF THE PHILIPPINES over the ARMM to the DILG Secretary who was her alter ego any way. These orders did not authorize a take over
(AFP), or any of their units operating in of the ARMM. They did not give him blanket authority to suspend or replace ARMM officials. [11] The delegation
the Autonomous Region in Muslim was necessary to facilitate the investigation of the mass killings. [12] Further, the assailed proclamation and
Mindanao (ARMM), and PHILIPPINE administrative orders did not provide for the exercise of emergency powers.[13]
NATIONAL POLICE, or any of their Promulgated:
units operating in ARMM, Although normalcy has in the meantime returned to the places subject of this petition, it might be relevant to rule
Respondents. June 7, 2011 on the issues raised in this petition since some acts done pursuant to Proclamation 1946 and AOs 273 and 273-
A could impact on the administrative and criminal cases that the government subsequently filed against those
x ---------------------------------------------------------------------------------------- x believed affected by such proclamation and orders.

The Issues Presented

The issues presented in this case are:

1. Whether or not Proclamation 1946 and AOs 273 and 273-A violate the principle of local autonomy
DECISION
under Section 16, Article X of the Constitution, and Section 1, Article V of the Expanded ARMM Organic Act;
ABAD, J.:
2. Whether or not President Arroyo invalidly exercised emergency powers when she called out the AFP
and the PNP to prevent and suppress all incidents of lawless violence in Maguindanao, Sultan Kudarat, and
Cotabato City; and
On November 24, 2009, the day after the gruesome massacre of 57 men and women, including some news
reporters, then President Gloria Macapagal-Arroyo issued Proclamation 1946,[1]placing the Provinces of
3. Whether or not the President had factual bases for her actions.
Maguindanao and Sultan Kudarat and the City of Cotabato under a state of emergency. She directed the Armed
Forces of the Philippines (AFP) and the Philippine National Police (PNP) to undertake such measures as may be The Rulings of the Court
PAGE 39 of 135 CONSTITUTIONAL LAW -EXECUTIVE DEPARTMENT PART 2 –SESSION 12
We dismiss the petition. the President might decide that there is a need to call out the armed forces may be of a
nature not constituting technical proof.
One. The claim of petitioners that the subject proclamation and administrative orders violate the principle
of local autonomy is anchored on the allegation that, through them, the President authorized the DILG Secretary On the other hand, the President, as Commander-in-Chief has a vast intelligence
to take over the operations of the ARMM and assume direct governmental powers over the region. network to gather information, some of which may be classified as highly confidential or
affecting the security of the state. In the exercise of the power to call, on-the-spot
But, in the first place, the DILG Secretary did not take over control of the powers of the ARMM. After law decisions may be imperatively necessary in emergency situations to avert great loss of
enforcement agents took respondent Governor of ARMM into custody for alleged complicity in the Maguindanao human lives and mass destruction of property. Indeed, the decision to call out the military
massacre, the ARMM Vice-Governor, petitioner Ansaruddin Adiong, assumed the vacated post on December 10, to prevent or suppress lawless violence must be done swiftly and decisively if it were to
2009 pursuant to the rule on succession found in Article VII, Section 12, [14] of RA 9054. In turn, Acting Governor have any effect at all. x x x.[20]
Adiong named the then Speaker of the ARMM Regional Assembly, petitioner Sahali-Generale, Acting ARMM Vice-
Governor.[15] In short, the DILG Secretary did not take over the administration or operations of the ARMM. Here, petitioners failed to show that the declaration of a state of emergency in the Provinces of Maguindanao,
Sultan Kudarat and Cotabato City, as well as the Presidents exercise of the calling out power had no factual basis.
Two. Petitioners contend that the President unlawfully exercised emergency powers when she ordered They simply alleged that, since not all areas under the ARMM were placed under a state of emergency, it follows
the deployment of AFP and PNP personnel in the places mentioned in the proclamation. [16] But such deployment that the take over of the entire ARMM by the DILG Secretary had no basis too. [21]
is not by itself an exercise of emergency powers as understood under Section 23 (2), Article VI of the Constitution,
which provides: But, apart from the fact that there was no such take over to begin with, the OSG also clearly explained
the factual bases for the Presidents decision to call out the armed forces, as follows:

SECTION 23. x x x (2) In times of war or other national emergency, the Congress The Ampatuan and Mangudadatu clans are prominent families engaged in the
may, by law, authorize the President, for a limited period and subject to such restrictions political control of Maguindanao. It is also a known fact that both families have an arsenal
as it may prescribe, to exercise powers necessary and proper to carry out a declared of armed followers who hold elective positions in various parts of the ARMM and the rest
national policy. Unless sooner withdrawn by resolution of the Congress, such powers of Mindanao.
shall cease upon the next adjournment thereof.
Considering the fact that the principal victims of the brutal bloodshed are
The President did not proclaim a national emergency, only a state of emergency in the three places members of the Mangudadatu family and the main perpetrators of the brutal killings are
mentioned. And she did not act pursuant to any law enacted by Congress that authorized her to exercise members and followers of the Ampatuan family, both the military and police had to
extraordinary powers. The calling out of the armed forces to prevent or suppress lawless violence in such places prepare for and prevent reported retaliatory actions from the Mangudadatu clan and
is a power that the Constitution directly vests in the President. She did not need a congressional authority to additional offensive measures from the Ampatuan clan.
exercise the same.
xxxx
Three. The Presidents call on the armed forces to prevent or suppress lawless violence springs from the
power vested in her under Section 18, Article VII of the Constitution, which provides. [17] The Ampatuan forces are estimated to be approximately two thousand four
hundred (2,400) persons, equipped with about two thousand (2,000) firearms, about four
hundred (400) of which have been accounted for. x x x
SECTION 18. The President shall be the Commander-in-Chief of all armed forces
of the Philippines and whenever it becomes necessary, he may call out such armed
As for the Mangudadatus, they have an estimated one thousand eight hundred
forces to prevent or suppress lawless violence, invasion or rebellion. x x x
(1,800) personnel, with about two hundred (200) firearms. x x x
While it is true that the Court may inquire into the factual bases for the Presidents exercise of the above Apart from their own personal forces, both clans have Special Civilian Auxiliary
power,[18] it would generally defer to her judgment on the matter. As the Court acknowledged in Integrated Bar of
Army (SCAA) personnel who support them: about five hundred (500) for the Ampatuans
the Philippines v. Hon. Zamora,[19] it is clearly to the President that the Constitution entrusts the determination of
and three hundred (300) for the Mangudadatus.
the need for calling out the armed forces to prevent and suppress lawless violence. Unless it is shown that such
determination was attended by grave abuse of discretion, the Court will accord respect to the Presidents judgment. What could be worse than the armed clash of two warring clans and their armed
Thus, the Court said:
supporters, especially in light of intelligence reports on the potential involvement of rebel
armed groups (RAGs).
If the petitioner fails, by way of proof, to support the assertion that the President
acted without factual basis, then this Court cannot undertake an independent
One RAG was reported to have planned an attack on the forces of Datu Andal
investigation beyond the pleadings. The factual necessity of calling out the armed forces
Ampatuan, Sr. to show support and sympathy for the victims. The said attack shall
is not easily quantifiable and cannot be objectively established since matters considered
worsen the age-old territorial dispute between the said RAG and the Ampatuan family.
for satisfying the same is a combination of several factors which are not always
accessible to the courts. Besides the absence of textual standards that the court may
xxxx
use to judge necessity, information necessary to arrive at such judgment might also
prove unmanageable for the courts. Certain pertinent information might be difficult to
On the other hand, RAG faction which is based in Sultan Kudarat was reported
verify, or wholly unavailable to the courts. In many instances, the evidence upon which
to have received three million pesos (P3,000,000.00) from Datu Andal Ampatuan, Sr. for
the procurement of ammunition. The said faction is a force to reckon with because the
PAGE 40 of 135 CONSTITUTIONAL LAW -EXECUTIVE DEPARTMENT PART 2 –SESSION 12
group is well capable of launching a series of violent activities to divert the attention of
the people and the authorities away from the multiple murder case. x x x

In addition, two other factions of a RAG are likely to support the Mangudadatu
family. The Cotabato-based faction has the strength of about five hundred (500) persons
and three hundred seventy-two (372) firearms while the Sultan Kudarat-based faction has
the strength of about four hundred (400) persons and three hundred (300) firearms and
was reported to be moving towards Maguindanao to support the Mangudadatu clan in its
armed fight against the Ampatuans.[22]

In other words, the imminence of violence and anarchy at the time the President issued Proclamation 1946 was
too grave to ignore and she had to act to prevent further bloodshed and hostilities in the places
mentioned. Progress reports also indicated that there was movement in these places of both high-powered
firearms and armed men sympathetic to the two clans. [23] Thus, to pacify the peoples fears and stabilize the
situation, the President had to take preventive action. She called out the armed forces to control the proliferation
of loose firearms and dismantle the armed groups that continuously threatened the peace and security in the
affected places.

Notably, the present administration of President Benigno Aquino III has not withdrawn the declaration of a state
of emergency under Proclamation 1946. It has been reported[24] that the declaration would not be lifted soon
because there is still a need to disband private armies and confiscate loose firearms. Apparently, the presence of
troops in those places is still necessary to ease fear and tension among the citizenry and prevent and suppress
any violence that may still erupt, despite the passage of more than a year from the time of the Maguindanao
massacre.

Since petitioners are not able to demonstrate that the proclamation of state of emergency in the subject
places and the calling out of the armed forces to prevent or suppress lawless violence there have clearly no factual
bases, the Court must respect the Presidents actions.

WHEREFORE, the petition is DISMISSED for lack of merit.

SO ORDERED.
PAGE 41 of 135 CONSTITUTIONAL LAW -EXECUTIVE DEPARTMENT PART 2 –SESSION 12
EN BANC In quelling or suppressing the rebellion, the authorities may only resort to warrantless arrests of persons
suspected of rebellion, as provided under Section 5, Rule 113 of the Rules of Court, if the circumstances so
warrant.The warrantless arrest feared by petitioners is, thus, not based on the declaration of a state of rebellion.
Moreover, petitioners contention in G.R. No. 147780 (Lacson Petition), 147781 (Defensor-Santiago Petition),
[G.R. No. 147780. May 10, 2001] and 147799 (Lumbao Petition) that they are under imminent danger of being arrested without warrant do not justify
their resort to the extraordinary remedies of mandamus and prohibition, since an individual subjected to
warrantless arrest is not without adequate remedies in the ordinary course of law. Such an individual may ask for
a preliminary investigation under Rule 112 of the Rules of court, where he may adduce evidence in his defense,
PANFILO LACSON, MICHAEL RAY B. AQUINO and CESAR O. MANCAO, petitioners, vs. SECRETARY or he may submit himself to inquest proceedings to determine whether or not he should remain under custody and
HERNANDO PEREZ, P/DIRECTOR LEANDRO MENDOZA, and P/SR. SUPT. REYNALDO correspondingly be charged in court. Further, a person subject of a warrantless arrest must be delivered to the
BERROYA, respondents. proper judicial authorities within the periods provided in Article 125 of the Revised Penal Code, otherwise the
arresting officer could be held liable for delay in the delivery of detained persons. Should the detention be without
legal ground, the person arrested can charge the arresting officer with arbitrary detention. All this is without
RESOLUTION prejudice to his filing an action for damages against the arresting officer under Article 32 of the Civil Code. Verily,
MELO, J.: petitioners have a surfeit of other remedies which they can avail themselves of, thereby making the prayer for
prohibition and mandamus improper at this time (Sections 2 and 3, Rule 65, Rules of Court).

On May 1, 2001, President Macapagal-Arroyo, faced by an angry and violent mob armed with explosives, Aside from the foregoing reasons, several considerations likewise inevitably call for the dismissal of the
firearms, bladed weapons, clubs, stones and other deadly weapons assaulting and attempting to break into petitions at bar.
Malacaang, issued Proclamation No. 38 declaring that there was a state of rebellion in the National Capital
Region. She likewise issued General Order No. 1 directing the Armed Forces of the Philippines and the Philippine
National Police to suppress the rebellion in the National Capital Region. Warrantless arrests of several alleged
G.R. No. 147780
leaders and promoters of the rebellion were thereafter effected.
Aggrieved by the warrantless arrests, and the declaration of a state of rebellion, which allegedly gave a
semblance of legality to the arrests, the following four related petitions were filed before the Court- In connection with their alleged impending warrantless arrest, petitioners Lacson, Aquino, and Mancao pray
that the appropriate court before whom the informations against petitioners are filed be directed to desist from
(1) G.R. No. 147780 for prohibition, injunction, mandamus, and habeas corpus (with an urgent application
arraigning and proceeding with the trial of the case, until the instant petition is finally resolved. This relief is clearly
for the issuance of temporary restraining order and/or writ of preliminary injunction) filed by Panfilo M. Lacson,
premature considering that as of this date, no complaints or charges have been filed against any of the petitioners
Michael Ray B. Aquino, and Cezar O. Mancao; (2) G.R. No. 147781 for mandamus and/or review of the factual
for any crime. And in the event that the same are later filed, this court cannot enjoin criminal prosecution conducted
basis for the suspension of the privilege of the writ of habeas corpus, with prayer for a temporary restraining order
in accordance with the Rules of Court, for by that time any arrest would have been in pursuance of a duly issued
filed by Miriam Defensor-Santiago; (3) G.R. No. 147799 for prohibition and injunction with prayer for a writ of
warrant.
preliminary injunction and/or restraining order filed by Rolando A. Lumbao; and (4) G.R. No. 147810 for certiorari
and prohibition filed by the political party Laban ng Demokratikong Pilipino. As regards petitioners prayer that the hold departure orders issued against them be declared null and void ab
initio, it is to be noted that petitioners are not directly assailing the validity of the subject hold departure orders in
All the foregoing petitions assail the declaration of a state of rebellion by President Gloria Macapagal-Arroyo their petition. The are not even expressing intention to leave the country in the near future. The prayer to set aside
and the warrantless arrests allegedly effected by virtue thereof, as having no basis both in fact an in
the same must be made in proper proceedings initiated for that purpose.
law.Significantly, on May 6, 2001, President Macapagal-Arroyo ordered the lifting of the declaration of a state of
rebellion in Metro Manila. Accordingly, the instant petitions have been rendered moot and academic. As to Anent petitioners allegations ex abundante ad cautelam in support of their application for the issuance of a
petitioners claim that the proclamation of a state of rebellion is being used by the authorities to justify warrantless writ of habeas corpus, it is manifest that the writ is not called for since its purpose is to relieve petitioners from
arrests, the Secretary of Justice denies that it has issued a particular order to arrest specific persons in connection unlawful restraint (Ngaya-an v. Balweg, 200 SCRA 149 [1991]), a matter which remains speculative up to this very
with the rebellion. He states that what is extant are general instructions to law enforcement officers and military day.
agencies to implement Proclamation No. 38. Indeed, as stated in respondents Joint Comments:

[I]t is already the declared intention of the Justice Department and police authorities to obtain regular G.R. No. 147781
warrants of arrests from the courts for all acts committed prior to and until May 1, 2001 which means
that preliminary investigators will henceforth be conducted.
The petition herein is denominated by petitioner Defensor-Santiago as one for mandamus. It is basic in
(Comment, G.R. No. 147780, p. 28; G.R. No. 147781, p. matters relating to petitions for mandamus that the legal right of the petitioner to the performance of a particular
18; G.R. No. 147799, p. 16; G.R. No. 147810, p. 24) act which is sought to be compelled must be clear and complete. Mandamus will not issue the right to relief is
clear at the time of the award (Palileo v. Ruiz Castro, 85 Phil. 272). Up to the present time, petitioner Defensor-
With this declaration, petitioners apprehensions as to warrantless arrests should be laid to rest. Santiago has not shown that she is in imminent danger of being arrested without a warrant. In point of fact, the
authorities have categorically stated that petitioner will not be arrested without a warrant.
PAGE 42 of 135 CONSTITUTIONAL LAW -EXECUTIVE DEPARTMENT PART 2 –SESSION 12
G.R. No. 147799
original jurisdiction of the Court to cases affecting ambassadors, other public ministers and consuls, and over
petitions for certiorari, prohibition, mandamus, quo warranto, and habeas corpus.

Petitioner Lumbao, leader of the Peoples Movement against Poverty (PMAP), for his part, argues that the WHEREFORE, premises considered, the petitions are hereby DISMISSED. However, in G.R. No. 147780,
declaration of a state of rebellion is violative of the doctrine of separation of powers, being an encroachment on 147781, and 147799, respondents, consistent and congruent with their undertaking earlier adverted to, together
the domain of the judiciary which has the constitutional prerogative to determine or interpret what took place on with their agents, representatives, and all persons acting for and in their behalf, are hereby enjoined from arresting
May 1, 2001, and that the declaration of a state of rebellion cannot be an exception to the general rule on the petitioners therein without the required judicial warrant for all acts committed in relation to or in connection with
allocation of the governmental powers. the May 1, 2001 siege of Malacaang.

We disagree. To be sure, section 18, Article VII of the Constitution expressly provides that [t]he President SO ORDERED.
shall be the Commander-in-Chief of all armed forces of the Philippines and whenever it becomes necessary, he
may call out such armed forces to prevent or suppress lawless violence, invasion or rebellion thus, we held in
Integrated Bar of the Philippines v. Hon. Zamora, (G.R. No. 141284, August 15, 2000):

xxx The factual necessity of calling out the armed forces is not easily quantifiable and cannot be objectively
established since matters considered for satisfying the same is a combination of several factors which are not
always accessible to the courts. Besides the absence of testual standards that the court may use to judge
necessity, information necessary to arrive at such judgment might also prove unmanageable for the
courts. Certain pertinent information necessary to arrive at such judgment might also prove unmanageable for
the courts. Certain pertinent information might be difficult to verify, or wholly unavailable to the courts. In many
instances, the evidence upon which the President might decide that there is a need to call out the armed forces
may be of a nature not constituting technical proof.

On the other hand, the President as Commander-in-Chief has a vast intelligence network to gather information,
some of which may be classified as highly confidential or affecting the security of the state. In the exercise of the
power to call, on-the-spot decisions may be imperatively necessary in emergency situations to avert great loss of
human lives and mass destruction of property. xxx

(at pp. 22-23)

The Court, in a proper case, may look into the sufficiency of the factual basis of the exercise of this
power. However, this is no longer feasible at this time, Proclamation No. 38 having been lifted.

G.R. No. 147810

Petitioner Laban ng Demoktratikong Pilipino is not a real party-in-interest. The rule requires that a party must
show a personal stake in the outcome of the case or an injury to himself that can be redressed by a favorable
decision so as to warrant an invocation of the courts jurisdiction and to justify the exercise of the courts remedial
powers in his behalf (KMU Labor Center v. Garcia, Jr., 239 SCRA 386 [1994]). Here, petitioner has not
demonstrated any injury to itself which would justify resort to the Court. Petitioner is a juridical person not subject
to arrest. Thus, it cannot claim to be threatened by a warrantless arrest. Nor is it alleged that its leaders, members,
and supporters are being threatened with warrantless arrest and detention for the crime of rebellion. Every action
must be brought in the name of the party whose legal right has been invaded or infringed, or whose legal right is
under imminent threat of invasion or infringement.
At best, the instant petition may be considered as an action for declaratory relief, petitioner claiming that its
right to freedom of expression and freedom of assembly is affected by the declaration of a state of rebellion and
that said proclamation is invalid for being contrary to the Constitution.
However, to consider the petition as one for declaratory relief affords little comfort to petitioner, this Court not
having jurisdiction in the first instance over such a petition. Section 5[1], Article VIII of the Constitution limits the
PAGE 43 of 135 CONSTITUTIONAL LAW -EXECUTIVE DEPARTMENT PART 2 –SESSION 12
EN BANC Criminal incidents in Metro Manila have been perpetrated not only by ordinary criminals but also by organized
syndicates whose members include active and former police/military personnel whose training, skill, discipline
and firepower prove well-above the present capability of the local police alone to handle. The deployment of a
joint PNP NCRPO-Philippine Marines in the conduct of police visibility patrol in urban areas will reduce the
incidence of crimes specially those perpetrated by active or former police/military personnel.
[G.R. No. 141284. August 15, 2000]

4. MISSION:

INTEGRATED BAR OF THE PHILIPPINES, petitioner, vs. HON. RONALDO B. ZAMORA, GEN. PANFILO M. The PNP NCRPO will organize a provisional Task Force to conduct joint NCRPO-PM visibility patrols to keep
LACSON, GEN. EDGAR B. AGLIPAY, and GEN. ANGELO REYES, respondents. Metro Manila streets crime-free, through a sustained street patrolling to minimize or eradicate all forms of high-
profile crimes especially those perpetrated by organized crime syndicates whose members include those that
are well-trained, disciplined and well-armed active or former PNP/Military personnel.
DECISION
KAPUNAN, J.: 5. CONCEPT IN JOINT VISIBILITY PATROL OPERATIONS:

At bar is a special civil action for certiorari and prohibition with prayer for issuance of a temporary restraining a. The visibility patrols shall be conducted jointly by the NCRPO [National Capital Regional Police Office] and the
order seeking to nullify on constitutional grounds the order of President Joseph Ejercito Estrada commanding the Philippine Marines to curb criminality in Metro Manila and to preserve the internal security of the state against
deployment of the Philippine Marines (the Marines) to join the Philippine National Police (the PNP) in visibility insurgents and other serious threat to national security, although the primary responsibility over Internal Security
patrols around the metropolis. Operations still rests upon the AFP.

In view of the alarming increase in violent crimes in Metro Manila, like robberies, kidnappings and
carnappings, the President, in a verbal directive, ordered the PNP and the Marines to conduct joint visibility patrols b. The principle of integration of efforts shall be applied to eradicate all forms of high-profile crimes perpetrated
for the purpose of crime prevention and suppression. The Secretary of National Defense, the Chief of Staff of the by organized crime syndicates operating in Metro Manila. This concept requires the military and police to work
Armed Forces of the Philippines (the AFP), the Chief of the PNP and the Secretary of the Interior and Local cohesively and unify efforts to ensure a focused, effective and holistic approach in addressing crime prevention.
Government were tasked to execute and implement the said order. In compliance with the presidential mandate, Along this line, the role of the military and police aside from neutralizing crime syndicates is to bring a
the PNP Chief, through Police Chief Superintendent Edgar B. Aglipay, formulated Letter of Instruction wholesome atmosphere wherein delivery of basic services to the people and development is achieved. Hand-in-
02/2000[1] (the LOI) which detailed the manner by which the joint visibility patrols, called Task hand with this joint NCRPO-Philippine Marines visibility patrols, local Police Units are responsible for the
Force Tulungan, would be conducted.[2] Task Force Tulungan was placed under the leadership of the Police Chief maintenance of peace and order in their locality.
of Metro Manila.
c. To ensure the effective implementation of this project, a provisional Task Force TULUNGAN shall be
Subsequently, the President confirmed his previous directive on the deployment of the Marines in a organized to provide the mechanism, structure, and procedures for the integrated planning, coordinating,
Memorandum, dated 24 January 2000, addressed to the Chief of Staff of the AFP and the PNP Chief. [3] In the monitoring and assessing the security situation.
Memorandum, the President expressed his desire to improve the peace and order situation in Metro Manila
through a more effective crime prevention program including increased police patrols. [4]The President further
stated that to heighten police visibility in the metropolis, augmentation from the AFP is necessary. [5] Invoking his xxx.[8]
powers as Commander-in-Chief under Section 18, Article VII of the Constitution, the President directed the AFP The selected areas of deployment under the LOI are: Monumento Circle, North Edsa (SM City), Araneta
Chief of Staff and PNP Chief to coordinate with each other for the proper deployment and utilization of the Marines Shopping Center, Greenhills, SM Megamall, Makati Commercial Center, LRT/MRT Stations and the NAIA and
to assist the PNP in preventing or suppressing criminal or lawless violence. [6] Finally, the President declared that Domestic Airport.[9]
the services of the Marines in the anti-crime campaign are merely temporary in nature and for a reasonable period
only, until such time when the situation shall have improved.[7] On 17 January 2000, the Integrated Bar of the Philippines (the IBP) filed the instant petition to annul LOI
02/2000 and to declare the deployment of the Philippine Marines, null and void and unconstitutional, arguing that:
The LOI explains the concept of the PNP-Philippine Marines joint visibility patrols as follows:
I
xxx
THE DEPLOYMENT OF THE PHILIPPINE MARINES IN METRO MANILA IS VIOLATIVE OF THE
2. PURPOSE: CONSTITUTION, IN THAT:

The Joint Implementing Police Visibility Patrols between the PNP NCRPO and the Philippine Marines A) NO EMERGENCY SITUATION OBTAINS IN METRO MANILA AS WOULD JUSTIFY, EVEN ONLY
partnership in the conduct of visibility patrols in Metro Manila for the suppression of crime prevention and other REMOTELY, THE DEPLOYMENT OF SOLDIERS FOR LAW ENFORCEMENT WORK; HENCE, SAID
serious threats to national security. DEPLOYMENT IS IN DEROGATION OF ARTICLE II, SECTION 3 OF THE CONSTITUTION;

3. SITUATION: B) SAID DEPLOYMENT CONSTITUTES AN INSIDIOUS INCURSION BY THE MILITARY IN A CIVILIAN


FUNCTION OF GOVERNMENT (LAW ENFORCEMENT) IN DEROGATION OF ARTICLE XVI, SECTION 5 (4),
OF THE CONSTITUTION;
PAGE 44 of 135 CONSTITUTIONAL LAW -EXECUTIVE DEPARTMENT PART 2 –SESSION 12
C) SAID DEPLOYMENT CREATES A DANGEROUS TENDENCY TO RELY ON THE MILITARY TO PERFORM challenged.[13] The term interest means a material interest, an interest in issue affected by the decree, as
THE CIVILIAN FUNCTIONS OF THE GOVERNMENT. distinguished from mere interest in the question involved, or a mere incidental interest. [14]The gist of the question
of standing is whether a party alleges such personal stake in the outcome of the controversy as to assure that
II concrete adverseness which sharpens the presentation of issues upon which the court depends for illumination of
difficult constitutional questions.[15]
IN MILITARIZING LAW ENFORCEMENT IN METRO MANILA, THE ADMINISTRATION IS UNWITTINGLY In the case at bar, the IBP primarily anchors its standing on its alleged responsibility to uphold the rule of law
MAKING THE MILITARY MORE POWERFUL THAN WHAT IT SHOULD REALLY BE UNDER THE and the Constitution. Apart from this declaration, however, the IBP asserts no other basis in support of its locus
CONSTITUTION.[10] standi. The mere invocation by the IBP of its duty to preserve the rule of law and nothing more, while undoubtedly
true, is not sufficient to clothe it with standing in this case. This is too general an interest which is shared by other
Asserting itself as the official organization of Filipino lawyers tasked with the bounden duty to uphold the rule groups and the whole citizenry. Based on the standards above-stated, the IBP has failed to present a specific and
of law and the Constitution, the IBP questions the validity of the deployment and utilization of the Marines to assist substantial interest in the resolution of the case. Its fundamental purpose which, under Section 2, Rule 139-A of
the PNP in law enforcement. the Rules of Court, is to elevate the standards of the law profession and to improve the administration of justice is
alien to, and cannot be affected by the deployment of the Marines. It should also be noted that the interest of the
Without granting due course to the petition, the Court in a Resolution, [11] dated 25 January 2000, required National President of the IBP who signed the petition, is his alone, absent a formal board resolution authorizing
the Solicitor General to file his Comment on the petition. On 8 February 2000, the Solicitor General submitted his him to file the present action. To be sure, members of the BAR, those in the judiciary included, have varying
Comment. opinions on the issue. Moreover, the IBP, assuming that it has duly authorized the National President to file the
petition, has not shown any specific injury which it has suffered or may suffer by virtue of the questioned
The Solicitor General vigorously defends the constitutionality of the act of the President in deploying the governmental act. Indeed, none of its members, whom the IBP purportedly represents, has sustained any form of
Marines, contending, among others, that petitioner has no legal standing; that the question of deployment of the injury as a result of the operation of the joint visibility patrols. Neither is it alleged that any of its members has been
Marines is not proper for judicial scrutiny since the same involves a political question; that the organization and arrested or that their civil liberties have been violated by the deployment of the Marines. What the IBP projects as
conduct of police visibility patrols, which feature the team-up of one police officer and one Philippine Marine soldier, injurious is the supposed militarization of law enforcement which might threaten Philippine democratic institutions
does not violate the civilian supremacy clause in the Constitution. and may cause more harm than good in the long run. Not only is the presumed injury not personal in character, it
is likewise too vague, highly speculative and uncertain to satisfy the requirement of standing. Since petitioner has
The issues raised in the present petition are: (1) Whether or not petitioner has legal standing; (2) Whether or
not successfully established a direct and personal injury as a consequence of the questioned act, it does not
not the Presidents factual determination of the necessity of calling the armed forces is subject to judicial review;
possess the personality to assail the validity of the deployment of the Marines. This Court, however, does not
and, (3) Whether or not the calling of the armed forces to assist the PNP in joint visibility patrols violates the
categorically rule that the IBP has absolutely no standing to raise constitutional issues now or in the future. The
constitutional provisions on civilian supremacy over the military and the civilian character of the PNP.
IBP must, by way of allegations and proof, satisfy this Court that it has sufficient stake to obtain judicial resolution
The petition has no merit. of the controversy.

First, petitioner failed to sufficiently show that it is in possession of the requisites of standing to raise the Having stated the foregoing, it must be emphasized that this Court has the discretion to take cognizance of
issues in the petition. Second, the President did not commit grave abuse of discretion amounting to lack or excess a suit which does not satisfy the requirement of legal standing when paramount interest is involved.[16] In not a few
of jurisdiction nor did he commit a violation of the civilian supremacy clause of the Constitution. cases, the Court has adopted a liberal attitude on the locus standi of a petitioner where the petitioner is able to
craft an issue of transcendental significance to the people. [17] Thus, when the issues raised are of paramount
The power of judicial review is set forth in Section 1, Article VIII of the Constitution, to wit: importance to the public, the Court may brush aside technicalities of procedure. [18] In this case, a reading of the
petition shows that the IBP has advanced constitutional issues which deserve the attention of this Court in view of
Section 1. The judicial power shall be vested in one Supreme Court and in such lower courts as may be their seriousness, novelty and weight as precedents. Moreover, because peace and order are under constant
established by law. threat and lawless violence occurs in increasing tempo, undoubtedly aggravated by the Mindanao insurgency
problem, the legal controversy raised in the petition almost certainly will not go away. It will stare us in the face
again. It, therefore, behooves the Court to relax the rules on standing and to resolve the issue now, rather than
Judicial power includes the duty of the courts of justice to settle actual controversies involving rights which are
later.
legally demandable and enforceable, and to determine whether or not there has been grave abuse of discretion
amounting to lack or excess of jurisdiction on the part of any branch or instrumentality of the Government.

The President did not commit grave abuse of discretion in calling out the Marines.
When questions of constitutional significance are raised, the Court can exercise its power of judicial review
only if the following requisites are complied with, namely: (1) the existence of an actual and appropriate case; (2)
a personal and substantial interest of the party raising the constitutional question; (3) the exercise of judicial review
is pleaded at the earliest opportunity; and (4) the constitutional question is the lis mota of the case.[12] In the case at bar, the bone of contention concerns the factual determination of the President of the necessity
of calling the armed forces, particularly the Marines, to aid the PNP in visibility patrols. In this regard, the IBP
admits that the deployment of the military personnel falls under the Commander-in-Chief powers of the President
as stated in Section 18, Article VII of the Constitution, specifically, the power to call out the armed forces to prevent
The IBP has not sufficiently complied with the requisites of standing in this case. or suppress lawless violence, invasion or rebellion. What the IBP questions, however, is the basis for the calling
of the Marines under the aforestated provision. According to the IBP, no emergency exists that would justify the
need for the calling of the military to assist the police force. It contends that no lawless violence, invasion or
Legal standing or locus standi has been defined as a personal and substantial interest in the case such that rebellion exist to warrant the calling of the Marines. Thus, the IBP prays that this Court review the sufficiency of
the party has sustained or will sustain direct injury as a result of the governmental act that is being the factual basis for said troop [Marine] deployment. [19]
PAGE 45 of 135 CONSTITUTIONAL LAW -EXECUTIVE DEPARTMENT PART 2 –SESSION 12
The Solicitor General, on the other hand, contends that the issue pertaining to the necessity of calling the The 1987 Constitution expands the concept of judicial review by providing that (T)he Judicial power shall be
armed forces is not proper for judicial scrutiny since it involves a political question and the resolution of factual vested in one Supreme Court and in such lower courts as may be established by law. Judicial power includes the
issues which are beyond the review powers of this Court. duty of the courts of justice to settle actual controversies involving rights which are legally demandable and
enforceable, and to determine whether or not there has been a grave abuse of discretion amounting to lack or
As framed by the parties, the underlying issues are the scope of presidential powers and limits, and the excess of jurisdiction on the part of any branch or instrumentality of the Government. [25] Under this definition, the
extent of judicial review. But, while this Court gives considerable weight to the parties formulation of the issues, Court cannot agree with the Solicitor General that the issue involved is a political question beyond the jurisdiction
the resolution of the controversy may warrant a creative approach that goes beyond the narrow confines of the of this Court to review. When the grant of power is qualified, conditional or subject to limitations, the issue of
issues raised. Thus, while the parties are in agreement that the power exercised by the President is the power to whether the prescribed qualifications or conditions have been met or the limitations respected, is justiciable - the
call out the armed forces, the Court is of the view that the power involved may be no more than the maintenance problem being one of legality or validity, not its wisdom. [26] Moreover, the jurisdiction to delimit constitutional
of peace and order and promotion of the general welfare.[20]For one, the realities on the ground do not show that boundaries has been given to this Court.[27] When political questions are involved, the Constitution limits the
there exist a state of warfare, widespread civil unrest or anarchy. Secondly, the full brunt of the military is not determination as to whether or not there has been a grave abuse of discretion amounting to lack or excess of
brought upon the citizenry, a point discussed in the latter part of this decision. In the words of the late Justice Irene jurisdiction on the part of the official whose action is being questioned. [28]
Cortes in Marcos v. Manglapus:
By grave abuse of discretion is meant simply capricious or whimsical exercise of judgment that is patent and
More particularly, this case calls for the exercise of the Presidents powers as protector of the gross as to amount to an evasion of positive duty or a virtual refusal to perform a duty enjoined by law, or to act
peace. [Rossiter, The American Presidency]. The power of the President to keep the peace is not limited merely at all in contemplation of law, as where the power is exercised in an arbitrary and despotic manner by reason of
to exercising the commander-in-chief powers in times of emergency or to leading the State against external and passion or hostility.[29] Under this definition, a court is without power to directly decide matters over which full
internal threats to its existence. The President is not only clothed with extraordinary powers in times of discretionary authority has been delegated. But while this Court has no power to substitute its judgment for that of
emergency, but is also tasked with attending to the day-to-day problems of maintaining peace and order and Congress or of the President, it may look into the question of whether such exercise has been made in grave
ensuring domestic tranquility in times when no foreign foe appears on the horizon. Wide discretion, within the abuse of discretion.[30] A showing that plenary power is granted either department of government, may not be an
bounds of law, in fulfilling presidential duties in times of peace is not in any way diminished by the relative want obstacle to judicial inquiry, for the improvident exercise or abuse thereof may give rise to justiciable controversy. [31]
of an emergency specified in the commander-in-chief provision. For in making the President commander-in-chief When the President calls the armed forces to prevent or suppress lawless violence, invasion or rebellion, he
the enumeration of powers that follow cannot be said to exclude the Presidents exercising as Commander-in- necessarily exercises a discretionary power solely vested in his wisdom. This is clear from the intent of the framers
Chief powers short of the calling of the armed forces, or suspending the privilege of the writ of habeas corpus or and from the text of the Constitution itself. The Court, thus, cannot be called upon to overrule the Presidents
declaring martial law, in order to keep the peace, and maintain public order and security. wisdom or substitute its own. However, this does not prevent an examination of whether such power was exercised
within permissible constitutional limits or whether it was exercised in a manner constituting grave abuse of
xxx[21] discretion. In view of the constitutional intent to give the President full discretionary power to determine the
necessity of calling out the armed forces, it is incumbent upon the petitioner to show that the Presidents decision
Nonetheless, even if it is conceded that the power involved is the Presidents power to call out the armed is totally bereft of factual basis.The present petition fails to discharge such heavy burden as there is no evidence
forces to prevent or suppress lawless violence, invasion or rebellion, the resolution of the controversy will reach a to support the assertion that there exist no justification for calling out the armed forces. There is, likewise, no
similar result. evidence to support the proposition that grave abuse was committed because the power to call was exercised in
We now address the Solicitor Generals argument that the issue involved is not susceptible to review by the such a manner as to violate the constitutional provision on civilian supremacy over the military. In the performance
judiciary because it involves a political question, and thus, not justiciable. of this Courts duty of purposeful hesitation[32] before declaring an act of another branch as unconstitutional, only
where such grave abuse of discretion is clearly shown shall the Court interfere with the Presidents judgment. To
As a general proposition, a controversy is justiciable if it refers to a matter which is appropriate for court doubt is to sustain.
review.[22] It pertains to issues which are inherently susceptible of being decided on grounds recognized by law.
Nevertheless, the Court does not automatically assume jurisdiction over actual constitutional cases brought before There is a clear textual commitment under the Constitution to bestow on the President full discretionary
it even in instances that are ripe for resolution. One class of cases wherein the Court hesitates to rule on are power to call out the armed forces and to determine the necessity for the exercise of such power.Section 18,
political questions. The reason is that political questions are concerned with issues dependent upon the wisdom, Article VII of the Constitution, which embodies the powers of the President as Commander-in-Chief, provides in
not the legality, of a particular act or measure being assailed. Moreover, the political question being a function of part:
the separation of powers, the courts will not normally interfere with the workings of another co-equal branch unless
the case shows a clear need for the courts to step in to uphold the law and the Constitution. The President shall be the Commander-in-Chief of all armed forces of the Philippines and whenever it becomes
necessary, he may call out such armed forces to prevent or suppress lawless violence, invasion or rebellion. In
As Taada v. Cuenco[23] puts it, political questions refer to those questions which, under the Constitution, are case of invasion or rebellion, when the public safety requires it, he may, for a period not exceeding sixty days,
to be decided by the people in their sovereign capacity, or in regard to which full discretionary authority has been suspend the privilege of the writ of habeas corpus, or place the Philippines or any part thereof under martial law.
delegated to the legislative or executive branch of government. Thus, if an issue is clearly identified by the text of
the Constitution as matters for discretionary action by a particular branch of government or to the people
themselves then it is held to be a political question. In the classic formulation of Justice Brennan in Baker v. xxx
Carr,[24] [p]rominent on the surface of any case held to involve a political question is found a textually demonstrable The full discretionary power of the President to determine the factual basis for the exercise of the calling out
constitutional commitment of the issue to a coordinate political department; or a lack of judicially discoverable and power is also implied and further reinforced in the rest of Section 18, Article VII which reads, thus:
manageable standards for resolving it; or the impossibility of deciding without an initial policy determination of a
kind clearly for nonjudicial discretion; or the impossibility of a courts undertaking independent resolution without xxx
expressing lack of the respect due coordinate branches of government; or an unusual need for unquestioning
adherence to a political decision already made; or the potentiality of embarassment from multifarious Within forty-eight hours from the proclamation of martial law or the suspension of the privilege of the writ
pronouncements by various departments on the one question. of habeas corpus, the President shall submit a report in person or in writing to the Congress. The Congress,
PAGE 46 of 135 CONSTITUTIONAL LAW -EXECUTIVE DEPARTMENT PART 2 –SESSION 12
voting jointly, by a vote of at least a majority of all its Members in regular or special session, may revoke such instead of imposing martial law or suspending the writ of habeas corpus, he must necessarily have to call the
proclamation or suspension, which revocation shall not be set aside by the President.Upon the initiative of the Armed Forces of the Philippines as their Commander-in-Chief. Is that the idea?
President, the Congress may, in the same manner, extend such proclamation or suspension for a period to be
determined by the Congress, if the invasion or rebellion shall persist and public safety requires it. MR. REGALADO. That does not require any concurrence by the legislature nor is it subject to judicial review. [34]

The Congress, if not in session, shall within twenty-four hours following such proclamation or suspension, The reason for the difference in the treatment of the aforementioned powers highlights the intent to grant the
convene in accordance with its rules without need of a call. President the widest leeway and broadest discretion in using the power to call out because it is considered as the
lesser and more benign power compared to the power to suspend the privilege of the writ of habeas corpus and
The Supreme Court may review, in an appropriate proceeding filed by any citizen, the sufficiency of the factual the power to impose martial law, both of which involve the curtailment and suppression of certain basic civil rights
basis of the proclamation of martial law or the suspension of the privilege of the writ or the extension thereof, and individual freedoms, and thus necessitating safeguards by Congress and review by this Court.
and must promulgate its decision thereon within thirty days from its filing.
Moreover, under Section 18, Article VII of the Constitution, in the exercise of the power to suspend the
privilege of the writ of habeas corpus or to impose martial law, two conditions must concur: (1) there must be an
A state of martial law does not suspend the operation of the Constitution, nor supplant the functioning of the civil actual invasion or rebellion and, (2) public safety must require it. These conditions are not required in the case of
courts or legislative assemblies, nor authorize the conferment of jurisdiction on military courts and agencies over the power to call out the armed forces. The only criterion is that whenever it becomes necessary, the President
civilians where civil courts are able to function, nor automatically suspend the privilege of the writ. may call the armed forces to prevent or suppress lawless violence, invasion or rebellion." The implication is that
the President is given full discretion and wide latitude in the exercise of the power to call as compared to the two
The suspension of the privilege of the writ shall apply only to persons judicially charged for rebellion or offenses other powers.
inherent in or directly connected with invasion.
If the petitioner fails, by way of proof, to support the assertion that the President acted without factual basis,
then this Court cannot undertake an independent investigation beyond the pleadings. The factual necessity of
During the suspension of the privilege of the writ, any person thus arrested or detained shall be judicially calling out the armed forces is not easily quantifiable and cannot be objectively established since matters
charged within three days, otherwise he shall be released. considered for satisfying the same is a combination of several factors which are not always accessible to the
courts. Besides the absence of textual standards that the court may use to judge necessity, information necessary
Under the foregoing provisions, Congress may revoke such proclamation or suspension and the Court may to arrive at such judgment might also prove unmanageable for the courts. Certain pertinent information might be
review the sufficiency of the factual basis thereof. However, there is no such equivalent provision dealing with the difficult to verify, or wholly unavailable to the courts. In many instances, the evidence upon which the President
revocation or review of the Presidents action to call out the armed forces. The distinction places the calling out might decide that there is a need to call out the armed forces may be of a nature not constituting technical proof.
power in a different category from the power to declare martial law and the power to suspend the privilege of the
writ of habeas corpus, otherwise, the framers of the Constitution would have simply lumped together the three On the other hand, the President as Commander-in-Chief has a vast intelligence network to gather
powers and provided for their revocation and review without any qualification. Expressio unius est exclusio information, some of which may be classified as highly confidential or affecting the security of the state. In the
alterius. Where the terms are expressly limited to certain matters, it may not, by interpretation or construction, be exercise of the power to call, on-the-spot decisions may be imperatively necessary in emergency situations to
extended to other matters.[33]That the intent of the Constitution is exactly what its letter says, i.e., that the power avert great loss of human lives and mass destruction of property. Indeed, the decision to call out the military to
to call is fully discretionary to the President, is extant in the deliberation of the Constitutional Commission, to wit: prevent or suppress lawless violence must be done swiftly and decisively if it were to have any effect at all. Such
a scenario is not farfetched when we consider the present situation in Mindanao, where the insurgency problem
could spill over the other parts of the country. The determination of the necessity for the calling out power if
FR. BERNAS. It will not make any difference. I may add that there is a graduated power of the President as subjected to unfettered judicial scrutiny could be a veritable prescription for disaster, as such power may be unduly
Commander-in-Chief. First, he can call out such Armed Forces as may be necessary to suppress lawless straitjacketed by an injunction or a temporary restraining order every time it is exercised.
violence; then he can suspend the privilege of the writ of habeas corpus, then he can impose martial law. This is
a graduated sequence. Thus, it is the unclouded intent of the Constitution to vest upon the President, as Commander-in-Chief of the
Armed Forces, full discretion to call forth the military when in his judgment it is necessary to do so in order to
When he judges that it is necessary to impose martial law or suspend the privilege of the writ of habeas corpus, prevent or suppress lawless violence, invasion or rebellion. Unless the petitioner can show that the exercise of
his judgment is subject to review. We are making it subject to review by the Supreme Court and subject to such discretion was gravely abused, the Presidents exercise of judgment deserves to be accorded respect from
concurrence by the National Assembly. But when he exercises this lesser power of calling on the Armed Forces, this Court.
when he says it is necessary, it is my opinion that his judgment cannot be reviewed by anybody. The President has already determined the necessity and factual basis for calling the armed forces. In his
Memorandum, he categorically asserted that, [V]iolent crimes like bank/store robberies, holdups, kidnappings and
xxx carnappings continue to occur in Metro Manila...[35] We do not doubt the veracity of the Presidents assessment of
the situation, especially in the light of present developments. The Court takes judicial notice of the recent bombings
FR. BERNAS. Let me just add that when we only have imminent danger, the matter can be handled by the first perpetrated by lawless elements in the shopping malls, public utilities, and other public places. These are among
sentence: The President may call out such armed forces to prevent or suppress lawless violence, invasion or the areas of deployment described in the LOI 2000. Considering all these facts, we hold that the President has
rebellion. So we feel that that is sufficient for handling imminent danger. sufficient factual basis to call for military aid in law enforcement and in the exercise of this constitutional power.

MR. DE LOS REYES. So actually, if a President feels that there is imminent danger, the matter can be handled
by the First Sentence: The President....may call out such Armed Forces to prevent or suppress lawless violence, The deployment of the Marines does not violate the civilian supremacy clause nor does it infringe the
invasion or rebellion. So we feel that that is sufficient for handling imminent danger, of invasion or rebellion, civilian character of the police force.
PAGE 47 of 135 CONSTITUTIONAL LAW -EXECUTIVE DEPARTMENT PART 2 –SESSION 12
Prescinding from its argument that no emergency situation exists to justify the calling of the Marines, the IBP 13. Sanitary inspections;[54]
asserts that by the deployment of the Marines, the civilian task of law enforcement is militarized in violation of
Section 3, Article II[36] of the Constitution. 14. Conduct of census work;[55]

We disagree. The deployment of the Marines does not constitute a breach of the civilian supremacy 15. Administration of the Civil Aeronautics Board;[56]
clause. The calling of the Marines in this case constitutes permissible use of military assets for civilian law 16. Assistance in installation of weather forecasting devices; [57]
enforcement. The participation of the Marines in the conduct of joint visibility patrols is appropriately
circumscribed. The limited participation of the Marines is evident in the provisions of the LOI itself, which sufficiently 17. Peace and order policy formulation in local government units. [58]
provides the metes and bounds of the Marines authority. It is noteworthy that the local police forces are the ones
in charge of the visibility patrols at all times, the real authority belonging to the PNP. In fact, the Metro Manila This unquestionably constitutes a gloss on executive power resulting from a systematic, unbroken, executive
Police Chief is the overall leader of the PNP-Philippine Marines joint visibility patrols.[37] Under the LOI, the police practice, long pursued to the knowledge of Congress and, yet, never before questioned. [59]What we have here is
forces are tasked to brief or orient the soldiers on police patrol procedures. [38] It is their responsibility to direct and mutual support and cooperation between the military and civilian authorities, not derogation of civilian supremacy.
manage the deployment of the Marines.[39] It is, likewise, their duty to provide the necessary equipment to the
In the United States, where a long tradition of suspicion and hostility towards the use of military force for
Marines and render logistical support to these soldiers. [40] In view of the foregoing, it cannot be properly argued
domestic purposes has persisted,[60] and whose Constitution, unlike ours, does not expressly provide for the power
that military authority is supreme over civilian authority. Moreover, the deployment of the Marines to assist the
to call, the use of military personnel by civilian law enforcement officers is allowed under circumstances similar to
PNP does not unmake the civilian character of the police force. Neither does it amount to an insidious incursion
those surrounding the present deployment of the Philippine Marines.Under the Posse Comitatus Act[61] of the US,
of the military in the task of law enforcement in violation of Section 5(4), Article XVI of the Constitution. [41]
the use of the military in civilian law enforcement is generally prohibited, except in certain allowable
In this regard, it is not correct to say that General Angelo Reyes, Chief of Staff of the AFP, by his alleged circumstances. A provision of the Act states:
involvement in civilian law enforcement, has been virtually appointed to a civilian post in derogation of the
aforecited provision. The real authority in these operations, as stated in the LOI, is lodged with the head of a 1385. Use of Army and Air Force as posse comitatus
civilian institution, the PNP, and not with the military. Such being the case, it does not matter whether the AFP
Chief actually participates in the Task Force Tulungan since he does not exercise any authority or control over the
Whoever, except in cases and under circumstances expressly authorized by the Constitution or Act of Congress,
same. Since none of the Marines was incorporated or enlisted as members of the PNP, there can be no
willfully uses any part of the Army or the Air Force as posse comitatus or otherwise to execute the laws shall be
appointment to civilian position to speak of. Hence, the deployment of the Marines in the joint visibility patrols does
fined not more than $10,000 or imprisoned not more than two years, or both. [62]
not destroy the civilian character of the PNP.
Considering the above circumstances, the Marines render nothing more than assistance required in To determine whether there is a violation of the Posse Comitatus Act in the use of military personnel, the US
conducting the patrols. As such, there can be no insidious incursion of the military in civilian affairs nor can there courts[63] apply the following standards, to wit:
be a violation of the civilian supremacy clause in the Constitution.
It is worth mentioning that military assistance to civilian authorities in various forms persists in Philippine Were Army or Air Force personnel used by the civilian law enforcement officers at Wounded Knee in such a
jurisdiction. The Philippine experience reveals that it is not averse to requesting the assistance of the military in manner that the military personnel subjected the citizens to the exercise of military power which was regulatory,
the implementation and execution of certain traditionally civil functions. As correctly pointed out by the Solicitor proscriptive, or compulsory[64] George Washington Law Review, pp. 404-433 (1986), which discusses the four
General, some of the multifarious activities wherein military aid has been rendered, exemplifying the activities that divergent standards for assessing acceptable involvement of military personnel in civil law
bring both the civilian and the military together in a relationship of cooperation, are: enforcement. Seelikewise HONORED IN THE BREECH: PRESIDENTIAL AUTHORITY TO EXECUTE THE
LAWS WITH MILITARY FORCE, 83 Yale Law Journal, pp. 130-152, 1973. 64 in nature, either presently or
1. Elections;[42] prospectively?
2. Administration of the Philippine National Red Cross; [43]
xxx
3. Relief and rescue operations during calamities and disasters; [44]
4. Amateur sports promotion and development;[45] When this concept is transplanted into the present legal context, we take it to mean that military involvement,
even when not expressly authorized by the Constitution or a statute, does not violate the Posse Comitatus Act
5. Development of the culture and the arts;[46] unless it actually regulates, forbids or compels some conduct on the part of those claiming relief. A mere threat
of some future injury would be insufficient. (emphasis supplied)
6. Conservation of natural resources;[47]
7. Implementation of the agrarian reform program;[48] Even if the Court were to apply the above rigid standards to the present case to determine whether there is
permissible use of the military in civilian law enforcement, the conclusion is inevitable that no violation of the civilian
8. Enforcement of customs laws;[49] supremacy clause in the Constitution is committed. On this point, the Court agrees with the observation of the
9. Composite civilian-military law enforcement activities;[50] Solicitor General:

10. Conduct of licensure examinations;[51] 3. The designation of tasks in Annex A[65] does not constitute the exercise of regulatory, proscriptive, or
11. Conduct of nationwide tests for elementary and high school students;[52] compulsory military power. First, the soldiers do not control or direct the operation. This is evident from
Nos. 6,[66] 8(k)[67] and 9(a)[68] of Annex A. These soldiers, second, also have no power to prohibit or
12. Anti-drug enforcement activities;[53] condemn. In No. 9(d)[69] of Annex A, all arrested persons are brought to the nearest police stations for
proper disposition. And last, these soldiers apply no coercive force. The materials or equipment issued to
PAGE 48 of 135 CONSTITUTIONAL LAW -EXECUTIVE DEPARTMENT PART 2 –SESSION 12
them, as shown in No. 8(c)[70] of Annex A, are all low impact and defensive in character. The conclusion is
that there being no exercise of regulatory, proscriptive or compulsory military power, the deployment of a
handful of Philippine Marines constitutes no impermissible use of military power for civilian law
enforcement.[71]

It appears that the present petition is anchored on fear that once the armed forces are deployed, the military
will gain ascendancy, and thus place in peril our cherished liberties. Such apprehensions, however, are
unfounded. The power to call the armed forces is just that - calling out the armed forces. Unless, petitioner IBP
can show, which it has not, that in the deployment of the Marines, the President has violated the fundamental law,
exceeded his authority or jeopardized the civil liberties of the people, this Court is not inclined to overrule the
Presidents determination of the factual basis for the calling of the Marines to prevent or suppress lawless violence.
One last point. Since the institution of the joint visibility patrol in January, 2000, not a single citizen has
complained that his political or civil rights have been violated as a result of the deployment of the Marines. It was
precisely to safeguard peace, tranquility and the civil liberties of the people that the joint visibility patrol was
conceived. Freedom and democracy will be in full bloom only when people feel secure in their homes and in the
streets, not when the shadows of violence and anarchy constantly lurk in their midst.
WHEREFORE, premises considered, the petition is hereby DISMISSED.
SO ORDERED.
PAGE 49 of 135 CONSTITUTIONAL LAW -EXECUTIVE DEPARTMENT PART 2 –SESSION 12
Republic of the Philippines 1) The Provincial Government shall source the funds and logistics needed for the activation of the CEF;
SUPREME COURT
Manila 2) The Provincial Government shall identify the Local Government Units which shall participate in the
operations and to propose them for the approval of the parties to this agreement;
EN BANC
3) The Provincial Government shall ensure that there will be no unilateral action(s) by the CEF without
G.R. No. 187298 July 03, 2012 the knowledge and approval by both parties.

JAMAR M. KULAYAN, TEMOGEN S. TULAWIE, HJI. MOH. YUSOP ISMI, JULHAJAN AWADI, and SPO1 Responsibilities of AFP/PNP/ TF ICRC (Task Force ICRC):
SATTAL H. JADJULI, Petitioners,
vs. 1) The AFP/PNP shall remain the authority as prescribed by law in military operations and law
GOV. ABDUSAKUR M. TAN, in his capacity as Governor of Sulu; GEN. JUANCHO SABAN, COL. enforcement;
EUGENIO CLEMEN PN, P/SUPT. JULASIRIM KASIM and P/SUPT. BIENVENIDO G. LATAG, in their
capacity as officers of the Phil. Marines and Phil. National Police, respectively, Respondents.
2) The AFP/PNP shall ensure the orderly deployment of the CEF in the performance of their assigned
task(s);
DECISION
3) The AFP/PNP shall ensure the safe movements of the CEF in identified areas of operation(s);
SERENO, J.:
4) The AFP/PNP shall provide the necessary support and/or assistance as called for in the course of
On 15 January 2009, three members from the International Committee of the Red Cross (ICRC) were kidnapped operation(s)/movements of the CEF.8
in the vicinity of the Provincial Capitol in Patikul, Sulu. 1 Andres Notter, a Swiss national and head of the ICRC in
Zamboanga City, Eugenio Vagni, an Italian national and ICRC delegate, and Marie Jean Lacaba, a Filipino
engineer, were purportedly inspecting a water and sanitation project for the Sulu Provincial Jail when inspecting Meanwhile, Ronaldo Puno, then Secretary of the Department of Interior and Local Government, announced to
a water and sanitation project for the Sulu Provincial Jail when they were seized by three armed men who were the media that government troops had cornered some one hundred and twenty (120) Abu Sayyaf members
later confirmed to be members of the Abu Sayyaf Group (ASG). 2 The leader of the alleged kidnappers was along with the three (3) hostages.9 However, the ASG made
identified as Raden Abu, a former guard at the Sulu Provincial Jail. News reports linked Abu to Albader Parad,
one of the known leaders of the Abu Sayyaf. contact with the authorities and demanded that the military pull its troops back from the jungle area. 10 The
government troops yielded and went back to their barracks; the Philippine Marines withdrew to their camp, while
On 21 January 2009, a task force was created by the ICRC and the Philippine National Police (PNP), which then police and civilian forces pulled back from the terrorists’ stronghold by ten (10) to fifteen (15) kilometers.
organized a parallel local group known as the Local Crisis Committee.3 The local group, later renamed Sulu Threatening that one of the hostages will be beheaded, the ASG further demanded the evacuation of the military
Crisis Management Committee, convened under the leadership of respondent Abdusakur Mahail Tan, the camps and bases in the different barangays in Jolo. 11 The authorities were given no later than 2:00 o’clock in the
Provincial Governor of Sulu. Its armed forces component was headed by respondents General Juancho Saban, afternoon of 31 March 2009 to comply. 12
and his deputy, Colonel Eugenio Clemen. The PNP component was headed by respondent Police
Superintendent Bienvenido G. Latag, the Police Deputy Director for Operations of the Autonomous Region of On 31 March 2009, Governor Tan issued Proclamation No. 1, Series of 2009 (Proclamation 1-09), declaring a
Muslim Mindanao (ARMM).4 state of emergency in the province of Sulu. 13 It cited the kidnapping incident as a ground for the said declaration,
describing it as a terrorist act pursuant to the Human Security
Governor Tan organized the Civilian Emergency Force (CEF), a group of armed male civilians coming from
different municipalities, who were redeployed to surrounding areas of Patikul. 5 The organization of the CEF was Act (R.A. 9372). It also invoked Section 465 of the Local Government Code of 1991 (R.A. 7160), which bestows
embodied in a "Memorandum of Understanding"6 entered into on the Provincial Governor the power to carry out emergency measures during man-made and natural disasters
and calamities, and to call upon the appropriate national law enforcement agencies to suppress disorder and
between three parties: the provincial government of Sulu, represented by Governor Tan; the Armed Forces of lawless violence.
the Philippines, represented by Gen. Saban; and the Philippine National Police, represented by P/SUPT. Latag.
The Whereas clauses of the Memorandum alluded to the extraordinary situation in Sulu, and the willingness of In the same Proclamation, respondent Tan called upon the PNP and the CEF to set up checkpoints and
civilian supporters of the municipal mayors to offer their services in order that "the early and safe rescue of the chokepoints, conduct general search and seizures including arrests, and other actions necessary to ensure
hostages may be achieved."7 public safety. The pertinent portion of the proclamation states:

This Memorandum, which was labeled ‘secret’ on its all pages, also outlined the responsibilities of each of the NOW, THEREFORE, BY VIRTUE OF THE POWERS VESTED IN ME BY LAW, I, ABDUSAKUR MAHAIL TAN,
party signatories, as follows: GOVERNOR OF THE PROVINCE OF SULU, DO HEREBY DECLARE A STATE OF EMERGENCY IN THE
PROVINCE OF SULU, AND CALL ON THE PHILIPPINE NATIONAL POLICE WITH THE ASSISTANCE OF
Responsibilities of the Provincial Government: THE ARMED FORCES OF THE PHILIPPINES AND THE CIVILIAN EMERGENCY FORCE TO IMPLEMENT
THE FOLLOWING:
PAGE 50 of 135 CONSTITUTIONAL LAW -EXECUTIVE DEPARTMENT PART 2 –SESSION 12
1. The setting-up of checkpoints and chokepoints in the province; Supreme Court under Rule 65.21 This is the only procedural defense raised by respondent Tan. Respondents
Gen. Juancho Saban, Col. Eugenio Clemen, P/SUPT. Julasirim Kasim, and P/SUPT. Bienvenido Latag did not
2. The imposition of curfew for the entire province subject to such Guidelines as may be issued by file their respective Comments.1âwphi1
proper authorities;
On the substantive issues, respondents deny that Proclamation 1-09 was issued ultra vires, as Governor Tan
3. The conduct of General Search and Seizure including arrests in the pursuit of the kidnappers and allegedly acted pursuant to Sections 16 and 465 of the Local Government Code, which empowers the Provincial
their supporters; and Governor to carry out emergency measures during calamities and disasters, and to call upon the appropriate
national law enforcement agencies to suppress disorder, riot, lawless violence, rebellion or
sedition.22 Furthermore, the Sangguniang Panlalawigan of Sulu authorized the declaration of a state of
4. To conduct such other actions or police operations as may be necessary to ensure public safety. emergency as evidenced by Resolution No. 4, Series of 2009 issued on 31 March 2009 during its regular
session.23
DONE AT THE PROVINCIAL CAPITOL, PROVINCE OF SULU THIS
The threshold issue in the present case is whether or not Section 465, in relation to Section 16, of the Local
31STDAY OF MARCH 2009. Sgd. Abdusakur M. Tan Governor.14 Government Code authorizes the respondent governor to declare a state of emergency, and exercise the
powers enumerated under Proclamation 1-09, specifically the conduct of general searches and seizures.
On 1 April 2009, SPO1 Sattal Jadjuli was instructed by his superior to report to respondent P/SUPT. Julasirim Subsumed herein is the secondary question of whether or not the provincial governor is similarly clothed with
Kasim.15 Upon arriving at the police station, he was booked, and interviewed about his relationship to Musin, authority to convene the CEF under the said provisions.
Jaiton, and Julamin, who were all his deceased relatives. Upon admitting that he was indeed related to the
three, he was detained. After a few hours, former Punong Barangay Juljahan Awadi, Hadji Hadjirul Bambra, We grant the petition.
Abdugajir Hadjirul, as well as PO2 Marcial Hajan, SPO3 Muhilmi Ismula, Punong Barangay Alano Mohammad
and jeepney driver Abduhadi Sabdani, were also arrested.16 The affidavit17 of the apprehending officer alleged I. Transcendental public Importance warrants a relaxation of the Doctrine of Hierarchy of Courts
that they were suspected ASG supporters and were being arrested under Proclamation 1-09. The following day,
2 April 2009, the hostage Mary Jane Lacaba was released by the ASG.
We first dispose of respondents’ invocation of the doctrine of hierarchy of courts which allegedly prevents judicial
review by this Court in the present case, citing for this specific purpose, Montes v. Court of Appeals and Purok
On 4 April 2009, the office of Governor Tan distributed to civic organizations, copies of the "Guidelines for the Bagong Silang Association, Inc. v. Yuipco.24 Simply put, the
Implementation of Proclamation No. 1, Series of 2009 Declaring a State of Emergency in the Province of
Sulu."18These Guidelines suspended all Permits to Carry
doctrine provides that where the issuance of an extraordinary writ is also within the competence of the CA or the
RTC, it is in either of these courts and not in the Supreme Court, that the specific action for the issuance of such
Firearms Outside of Residence (PTCFORs) issued by the Chief of the PNP, and allowed civilians to seek writ must be sought unless special and important laws are clearly and specifically set forth in the petition. The
exemption from the gun ban only by applying to the Office of the Governor and obtaining the appropriate reason for this is that this Court is a court of last resort and must so remain if it is to perform the functions
identification cards. The said guidelines also allowed general searches and seizures in designated checkpoints assigned to it by the Constitution and immemorial tradition. It cannot be burdened with deciding cases in the first
and chokepoints. instance.25

On 16 April 2009, Jamar M. Kulayan, Temogen S. Tulawie, Hadji Mohammad Yusop Ismi, Ahajan Awadi, and The said rule, however, is not without exception. In Chavez v. PEA-Amari,26 the Court stated:
SPO1 Sattal H. Jadjuli, residents of Patikul, Sulu, filed the present Petition for Certiorari and
Prohibition,19 claiming that Proclamation 1-09 was issued with grave abuse of discretion amounting to lack or
excess of jurisdiction, as it threatened fundamental freedoms guaranteed under Article III of the 1987 PEA and AMARI claim petitioner ignored the judicial hierarchy by seeking relief directly from the Court. The
Constitution. principle of hierarchy of courts applies generally to cases involving factual questions. As it is not a trier of facts,
the Court cannot entertain cases involving factual issues. The instant case, however, raises constitutional
questions of transcendental importance to the public. The Court can resolve this case without determining any
Petitioners contend that Proclamation No. 1 and its Implementing Guidelines were issued ultra vires, and thus factual issue related to the case. Also, the instant case is a petition for mandamus which falls under the original
null and void, for violating Sections 1 and 18, Article VII of the Constitution, which grants the President sole jurisdiction of the Court under Section 5, Article VIII of the Constitution. We resolve to exercise primary
authority to exercise emergency powers and calling-out powers as the chief executive of the Republic and jurisdiction over the instant case.27
commander-in-chief of the armed forces.20 Additionally, petitioners claim that the Provincial Governor is not
authorized by any law to create civilian armed forces under his command, nor regulate and limit the issuances of
PTCFORs to his own private army. The instant case stems from a petition for certiorari and prohibition, over which the Supreme Court possesses
original jurisdiction.28 More crucially, this case involves acts of a public official which pertain to restrictive
custody, and is thus impressed with transcendental public importance that would warrant the relaxation of the
In his Comment, Governor Tan contended that petitioners violated the doctrine on hierarchy of courts when they general rule. The Court would be remiss in its constitutional duties were it to dismiss the present petition solely
filed the instant petition directly in the court of last resort, even if both the Court of Appeals (CA) and the due to claims of judicial hierarchy.
Regional Trial Courts (RTC) possessed concurrent jurisdiction with the
In David v. Macapagal-Arroyo,29 the Court highlighted the transcendental public importance involved in cases
that concern restrictive custody, because judicial review in these cases serves as "a manifestation of the crucial
defense of civilians ‘in police power’ cases due to the diminution of their basic liberties under the guise of a state
PAGE 51 of 135 CONSTITUTIONAL LAW -EXECUTIVE DEPARTMENT PART 2 –SESSION 12
of emergency."30 Otherwise, the importance of the high tribunal as the court of last resort would be put to naught, Corollarily, it is only the President, as Executive, who is authorized to exercise emergency powers as provided
considering the nature of "emergency" cases, wherein the proclamations and issuances are inherently short- under Section 23, Article VI, of the Constitution, as well as what became known as the calling-out powers under
lived. In finally disposing of the claim that the issue had become moot and academic, the Court also cited Section 7, Article VII thereof.
transcendental public importance as an exception, stating:
ii. The exceptional character of Commander-in-Chief powers dictate that they are exercised by one president
Sa kabila ng pagiging akademiko na lamang ng mga isyu tungkol sa mahigpit na pangangalaga (restrictive
custody) at pagmonitor ng galaw (monitoring of movements) ng nagpepetisyon, dedesisyunan namin ito (a) dahil Springing from the well-entrenched constitutional precept of One President is the notion that there are certain
sa nangingibabaw na interes ng madla na nakapaloob dito, acts which, by their very nature, may only be performed by the president as the Head of the State. One of these
acts or prerogatives is the bundle of Commander-in-Chief powers to which the "calling-out" powers constitutes a
(b) dahil sa posibilidad na maaaring maulit ang pangyayari at (c) dahil kailangang maturuan ang kapulisan portion. The President’s Emergency Powers, on the other hand, is balanced only by the legislative act of
tungkol dito. Congress, as embodied in the second paragraph of Section 23, Article 6 of the Constitution:

The moot and academic principle is not a magical formula that can automatically dissuade the courts in resolving Article 6, Sec 23(2). In times of war or other national emergency, the Congress may, by law, authorize the
a case. Courts will decide cases, otherwise moot and academic, if: first, there is a grave violation of the President, for a limited period and subject to such restrictions as it may prescribe, to exercise powers necessary
Constitution; second, the exceptional character of the situation and the paramount public interest is involved; and proper to carry out a declared national policy. Unless sooner withdrawn by resolution of the Congress, such
third, when [the] constitutional issue raised requires formulation of controlling principles to guide the bench, the powers shall cease upon the next adjournment thereof.35
bar, and the public; and fourth, the case is capable of repetition yet evading review.
Article 7, Sec 18. The President shall be the Commander-in-Chief of all armed forces of the Philippines and
…There is no question that the issues being raised affect the public interest, involving as they do the people’s whenever it becomes necessary, he may call out such armed forces to prevent or suppress lawless violence,
basic rights to freedom of expression, of assembly and of the press. Moreover, the invasion or rebellion. In case of invasion or rebellion, when the public safety requires it, he may, for a period not
exceeding sixty days, suspend the privilege of the writ of habeas corpus or place the Philippines or any part
Court has the duty to formulate guiding and controlling constitutional precepts, doctrines or rules. It has the thereof under martial law. Within forty-eight hours from the proclamation of martial law or the suspension of the
symbolic function of educating the bench and the bar, and in the present petitions, the military and the police, on privilege of the writ of habeas corpus, the President shall submit a report in person or in writing to the Congress.
the extent of the protection given by constitutional guarantees. And lastly, respondents contested actions are The Congress, voting jointly, by a vote of at least a majority of all its Members in regular or special session, may
capable of repetition. Certainly, the petitions are subject to judicial review. revoke such proclamation or suspension, which revocation shall not be set aside by the President. Upon the
initiative of the President, the Congress may, in the same manner, extend such proclamation or suspension for a
period to be determined by the Congress, if the invasion or rebellion shall persist and public safety requires it.
Evidently, the triple reasons We advanced at the start of Our ruling are justified under the foregoing exceptions.
Every bad, unusual incident where police officers figure in generates public interest and people watch what will
be done or not done to them. Lack of disciplinary steps taken against them erode public confidence in the police The Congress, if not in session, shall, within twenty-four hours following such proclamation or suspension,
institution. As petitioners themselves assert, the restrictive custody of policemen under investigation is an convene in accordance with its rules without need of a call.36
existing practice, hence, the issue is bound to crop up every now and then. The matter is capable of repetition or
susceptible of recurrence. It better be resolved now for the education and guidance of all The power to declare a state of martial law is subject to the Supreme Court’s authority to review the factual basis
concerned.31 (Emphasis supplied) thereof. 37 By constitutional fiat, the calling-out powers, which is of lesser gravity than the power to declare
martial law, is bestowed upon the President alone. As noted in Villena, "(t)here are certain constitutional powers
Hence, the instant petition is given due course, impressed as it is with transcendental public importance. and prerogatives of the Chief Executive of the Nation which must be exercised by him in person and no amount
of approval or ratification will validate the exercise of any of those powers by any other person. Such, for
instance, is his power to suspend the writ of habeas corpus and proclaim martial law x x x. 38
II. Only the President is vested with calling-out powers, as the commander-in-chief of the Republic
Indeed, while the President is still a civilian, Article II, Section 339 of the Constitution mandates that civilian
i. One executive, one commander-in-chief authority is, at all times, supreme over the military, making the civilian president the nation’s supreme military
leader. The net effect of Article II, Section 3, when read with Article VII,
As early as Villena v. Secretary of Interior,32 it has already been established that there is one repository of
executive powers, and that is the President of the Republic. This means that when Section 1, Article VII of the Section 18, is that a civilian President is the ceremonial, legal and administrative head of the armed forces. The
Constitution speaks of executive power, it is granted to the President and no one else. 33 As emphasized by Constitution does not require that the President must be possessed of military training and talents, but as
Justice Jose P. Laurel, in his ponencia in Villena: Commander-in-Chief, he has the power to direct military operations and to determine military strategy. Normally,
he would be expected to delegate the actual command of the armed forces to military experts; but the ultimate
With reference to the Executive Department of the government, there is one purpose which is crystal-clear and power is his.40 As Commander-in-Chief, he is authorized to direct the movements of the naval and military forces
is readily visible without the projection of judicial searchlight, and that is the establishment of a single, not plural, placed by law at his command, and to employ them in the manner he may deem most effectual. 41
Executive. The first section of Article VII of the Constitution, dealing with the Executive Department, begins with
the enunciation of the principle that "The executive power shall be vested in a President of the Philippines." This In the case of Integrated Bar of the Philippines v. Zamora, 42 the Court had occasion to rule that the calling-out
means that the President of the Philippines is the Executive of the Government of the Philippines, and no powers belong solely to the President as commander-in-chief:
other.34
PAGE 52 of 135 CONSTITUTIONAL LAW -EXECUTIVE DEPARTMENT PART 2 –SESSION 12
When the President calls the armed forces to prevent or suppress lawless violence, invasion or rebellion, he co-equal branches of government. The declaration of martial law, the suspension of the writ of habeas corpus,
necessarily exercises a discretionary power solely vested in his wisdom. This is clear from the intent of the and the exercise of the pardoning power, notwithstanding the judicial determination of guilt of the accused, all
framers and from the text of the Constitution itself. The Court, thus, cannot be called upon to overrule the fall within this special class that demands the exclusive exercise by the President of the constitutionally vested
President’s wisdom or substitute its own. However, this does not prevent an examination of whether such power power. The list is by no means exclusive, but there must be a showing that the executive power in question is of
was exercised within permissible constitutional limits or whether it was exercised in a manner constituting grave similar gravitas and exceptional import.47
abuse of discretion. In view of the constitutional intent to give the President full discretionary power to determine
the necessity of calling out the armed forces, it is incumbent upon the petitioner to show that the President’s In addition to being the commander-in-chief of the armed forces, the President also acts as the leader of the
decision is totally bereft of factual basis. country’s police forces, under the mandate of Section 17, Article VII of the Constitution, which provides that,
"The President shall have control of all the executive departments, bureaus, and offices. He shall ensure that the
There is a clear textual commitment under the Constitution to bestow on the President full discretionary power to laws be faithfully executed." During the deliberations of the Constitutional Commission on the framing of this
call out the armed forces and to determine the necessity for the exercise of such power.43(Emphasis supplied) provision, Fr. Bernas defended the retention of the word "control," employing the same rationale of singularity of
the office of the president, as the only Executive under the presidential form of government. 48
Under the foregoing provisions, Congress may revoke such proclamation or suspension and the Court may
review the sufficiency of the factual basis thereof. However, there is no such equivalent provision dealing with Regarding the country’s police force, Section 6, Article XVI of the Constitution states that: "The State shall
the revocation or review of the President’s action to call out the armed forces. The distinction places the calling establish and maintain one police force, which shall be national in scope and civilian in character, to be
out power in a different category from the power to declare martial law and the power to suspend the privilege of administered and controlled by a national police commission. The authority of local executives over the police
the writ of habeas corpus, otherwise, the framers of the Constitution would have simply lumped together the units in their jurisdiction shall be provided by law."49
three powers and provided for their revocation and review without any qualification.44
A local chief executive, such as the provincial governor, exercises operational supervision over the police, 50 and
That the power to call upon the armed forces is discretionary on the president is clear from the deliberation of may exercise control only in day-to-day operations, viz:
the Constitutional Commission:
Mr. Natividad: By experience, it is not advisable to provide either in our Constitution or by law full control of the
FR. BERNAS. It will not make any difference. I may add that there is a graduated power of the President as police by the local chief executive and local executives, the mayors. By our experience, this has spawned
Commander-in-Chief. First, he can call out such Armed Forces as may be necessary to suppress lawless warlordism, bossism and sanctuaries for vices and abuses. If the national government does not have a
violence; then he can suspend the privilege of the writ of habeas corpus, then he can impose martial law. This is mechanism to supervise these 1,500 legally, technically separate police forces, plus 61 city police forces,
a graduated sequence. fragmented police system, we will have a lot of difficulty in presenting a modern professional police force. So that
a certain amount of supervision and control will have to be exercised by the national government.
When he judges that it is necessary to impose martial law or suspend the privilege of the writ of habeas corpus,
his judgment is subject to review. We are making it subject to review by the Supreme Court and subject to For example, if a local government, a town cannot handle its peace and order problems or police problems, such
concurrence by the National Assembly. But when he exercises this lesser power of calling on the Armed Forces, as riots, conflagrations or organized crime, the national government may come in, especially if requested by the
when he says it is necessary, it is my opinion that his judgment cannot be reviewed by anybody. local executives. Under that situation, if they come in under such an extraordinary situation, they will be in
control. But if the day-to-day business of police investigation of crime, crime prevention, activities, traffic control,
xxx xxx xxx is all lodged in the mayors, and if they are in complete operational control of the day-to-day business of police
service, what the national government would control would be the administrative aspect.
MR. REGALADO. That does not require any concurrence by the legislature nor is it subject to judicial review.
xxx xxx xxx
The reason for the difference in the treatment of the aforementioned powers highlights the intent to grant the
President the widest leeway and broadest discretion in using the power to call out because it is considered as Mr. de los Reyes: so the operational control on a day-to-day basis, meaning, the usual duties being performed
the lesser and more benign power compared to the power to suspend the privilege of the writ of habeas corpus by the ordinary policemen, will be under the supervision of the local executives?
and the power to impose martial law, both of which involve the curtailment and suppression of certain basic civil
rights and individual freedoms, and thus necessitating safeguards by Congress and review by this Court. Mr. Natividad: Yes, Madam President.

x x x Thus, it is the unclouded intent of the Constitution to vest upon the President, as Commander-in-Chief of xxx xxx xxx
the Armed Forces, full discretion to call forth the military when in his judgment it is necessary to do so in order to
prevent or suppress lawless violence, invasion or rebellion. 45 (Emphasis Supplied) Mr. de los Reyes: But in exceptional cases, even the operational control can be taken over by the National
Police Commission?
In the more recent case of Constantino, Jr. v. Cuisia, 46 the Court characterized these powers as exclusive to the
President, precisely because they are of exceptional import: Mr. Natividad: If the situation is beyond the capacity of the local governments.51 (Emphases supplied)

These distinctions hold true to this day as they remain embodied in our fundamental law. There are certain Furthermore according to the framers, it is still the President who is authorized to exercise supervision and
presidential powers which arise out of exceptional circumstances, and if exercised, would involve the suspension control over the police, through the National Police Commission:
of fundamental freedoms, or at least call for the supersedence of executive prerogatives over those exercised by
PAGE 53 of 135 CONSTITUTIONAL LAW -EXECUTIVE DEPARTMENT PART 2 –SESSION 12
Mr. Rodrigo: Just a few questions. The President of the Philippines is the Commander-in-Chief of all the armed emergency, a condition which covers the entire country, and does not include emergency situations in local
forces. government units.54 This claim is belied by the clear intent of the framers that in all situations involving threats to
security, such as lawless violence, invasion or rebellion, even in localized areas, it is still the President who
Mr. Natividad: Yes, Madam President. possesses the sole authority to exercise calling-out powers. As reflected in the Journal of the Constitutional
Commission:
Mr. Rodrigo: Since the national police is not integrated with the armed forces, I do not suppose they come under
the Commander-in-Chief powers of the President of the Philippines. Thereafter, Mr. Padilla proposed on line 29 to insert the phrase OR PUBLIC DISORDER in lieu of "invasion or
rebellion." Mr. Sumulong stated that the committee could not accept the amendment because under the first
section of Section 15, the President may call out and make use of the armed forces to prevent or suppress not
Mr. Natividad: They do, Madam President. By law, they are under the supervision and control of the President of only lawless violence but even invasion or rebellion without declaring martial law. He observed that by deleting
the Philippines. "invasion or rebellion" and substituting PUBLIC DISORDER, the President would have to declare martial law
before he can make use of the armed forces to prevent or suppress lawless invasion or rebellion.
Mr. Rodrigo: Yes, but the President is not the Commander-in-Chief of the national police.
Mr. Padilla, in reply thereto, stated that the first sentence contemplates a lighter situation where there is some
Mr. Natividad: He is the President. lawless violence in a small portion of the country or public disorder in another at which times, the armed forces
can be called to prevent or suppress these incidents. He noted that the Commander-in-Chief can do so in a
Mr. Rodrigo: Yes, the Executive. But they do not come under that specific provision that the President is the minor degree but he can also exercise such powers should the situation worsen. The words "invasion or
Commander-in-Chief of all the armed forces. rebellion" to be eliminated on line 14 are covered by the following sentence which provides for "invasion or
rebellion." He maintained that the proposed amendment does not mean that under such circumstances, the
President cannot call on the armed forces to prevent or suppress the same. 55 (Emphasis supplied)
Mr. Natividad: No, not under the Commander-in-Chief provision.
III. Section 465 of the Local
Mr. Rodrigo: There are two other powers of the President. The
Government Code cannot be invoked to justify the powers enumerated under Proclamation 1-09
President has control over ministries, bureaus and offices, and supervision over local governments. Under which
does the police fall, under control or under supervision?
Respondent governor characterized the kidnapping of the three ICRC workers as a terroristic act, and used this
incident to justify the exercise of the powers enumerated under Proclamation 1-09.56 He invokes Section 465, in
Mr. Natividad: Both, Madam President. relation to Section 16, of the Local Government Code, which purportedly allows the governor to carry out
emergency measures and call upon the appropriate national law enforcement agencies for assistance. But a
Mr. Rodrigo: Control and supervision. closer look at the said proclamation shows that there is no provision in the Local Government Code nor in any
law on which the broad and unwarranted powers granted to the Governor may be based.
Mr. Natividad: Yes, in fact, the National Police Commission is under the Office of the President. 52
Petitioners cite the implementation of "General Search and Seizure including arrests in the pursuit of the
kidnappers and their supporters,"57 as being violative of the constitutional proscription on general search
In the discussions of the Constitutional Commission regarding the above provision it is clear that the framers
warrants and general seizures. Petitioners rightly assert that this alone would be sufficient to render the
never intended for local chief executives to exercise unbridled control over the police in emergency
proclamation void, as general searches and seizures are proscribed, for being violative of the rights enshrined in
situations. This is without prejudice to their authority over police units in their jurisdiction as provided by law, and
the Bill of Rights, particularly:
their prerogative to seek assistance from the police in day to day situations, as contemplated by the
Constitutional Commission. But as a civilian agency of the government, the police, through the NAPOLCOM,
properly comes within, and is subject to, the exercise by the President of the power of executive control. 53 The right of the people to be secure in their persons, houses, papers, and effects against unreasonable
searches and seizures of whatever nature and for any purpose shall be inviolable, and no search warrant or
warrant of arrest shall issue except upon probable cause to be determined personally by the judge after
iii. The provincial governor does not possess the same calling-out powers as the President
examination under oath or affirmation of the complainant and the witnesses he may produce, and particularly
describing the place to be searched and the persons or things to be seized. 58
Given the foregoing, respondent provincial governor is not endowed with the power to call upon the armed
forces at his own bidding. In issuing the assailed proclamation, Governor Tan exceeded his authority when he
In fact, respondent governor has arrogated unto himself powers exceeding even the martial law powers of the
declared a state of emergency and called upon the Armed Forces, the police, and his own Civilian Emergency
President, because as the Constitution itself declares, "A state of martial law does not suspend the operation of
Force. The calling-out powers contemplated under the Constitution is exclusive to the President. An exercise by
the Constitution, nor supplant the functioning of the civil courts or legislative assemblies, nor authorize the
another official, even if he is the local chief executive, is ultra vires, and may not be justified by the invocation of
conferment of the jurisdiction on military courts and agencies over civilians where civil courts are able to
Section 465 of the Local Government Code, as will be discussed subsequently.
function, nor automatically suspend the privilege of the writ."59

Respondents, however, justify this stance by stating that nowhere in the seminal case of David v. Arroyo, which
We find, and so hold, that there is nothing in the Local Government Code which justifies the acts sanctioned
dealt squarely with the issue of the declaration of a state of emergency, does it limit the said authority to the
under the said Proclamation. Not even Section 465 of the said Code, in relation to Section 16, which states:
President alone. Respondents contend that the ruling in David expressly limits the authority to declare a national
PAGE 54 of 135 CONSTITUTIONAL LAW -EXECUTIVE DEPARTMENT PART 2 –SESSION 12
Section 465. The Chief Executive: Powers, Duties, Functions, and Compensation. governor was to ask the assistance of the Secretary of Interior and Local Government, or such other authorized
officials, for the assistance of national law enforcement agencies.
xxx xxx xxx
The Local Government Code does not involve the diminution of central powers inherently vested in the National
(b) For efficient, effective and economical governance the purpose of which is the general welfare of the Government, especially not the prerogatives solely granted by the Constitution to the President in matters of
province and its inhabitants pursuant to Section 16 of this Code, the provincial governor shall: security and defense.

(1) Exercise general supervision and control over all programs, projects, services, and activities of the provincial The intent behind the powers granted to local government units is fiscal, economic, and administrative in
government, and in this connection, shall: nature.1âwphi1The Code is concerned only with powers that would make the delivery of basic services more
effective to the constituents,61 and should not be unduly stretched to confer calling-out powers on local
executives.
xxx xxx xxx
In the sponsorship remarks for Republic Act 7160, it was stated that the devolution of powers is a step towards
(vii) Carry out such emergency measures as may be necessary during and in the aftermath of man-made and the autonomy of local government units (LGUs), and is actually an experiment whose success heavily relies on
natural disasters and calamities; the power of taxation of the LGUs. The underpinnings of the Code can be found in Section 5, Article II of the
1973 Constitution, which allowed LGUs to create their own sources of revenue.62 During the interpellation made
(2) Enforce all laws and ordinances relative to the governance of the province and the exercise of the by Mr. Tirol addressed to Mr. de Pedro, the latter emphasized that "Decentralization is an administrative concept
appropriate corporate powers provided for under Section 22 of this Code, implement all approved policies, and the process of shifting and delegating power from a central point to subordinate levels to promote
programs, projects, services and activities of the province and, in addition to the foregoing, shall: independence, responsibility, and quicker decision-making. … (I)t does not involve any transfer of final authority
from the national to field levels, nor diminution of central office powers and responsibilities. Certain government
xxx xxx xxx agencies, including the police force, are exempted from the decentralization process because their functions are
not inherent in local government units."63

(vi) Call upon the appropriate national law enforcement agencies to suppress disorder, riot, lawless violence,
IV. Provincial governor is not authorized to convene CEF
rebellion or sedition or to apprehend violators of the law when public interest so requires and the police forces of
the component city or municipality where the disorder or violation is happening are inadequate to cope with the
situation or the violators. Pursuant to the national policy to establish one police force, the organization of private citizen armies is
proscribed. Section 24 of Article XVIII of the Constitution mandates that:
Section 16. General Welfare. - Every local government unit shall exercise the powers expressly granted, those
necessarily implied therefrom, as well as powers necessary, appropriate, or incidental for its efficient and Private armies and other armed groups not recognized by duly constituted authority shall be dismantled. All
effective governance, and those which are essential to the promotion of the general welfare. Within their paramilitary forces including Civilian Home Defense Forces (CHDF) not consistent with the citizen armed force
respective territorial jurisdictions, local government units shall ensure and support, among other things, the established in this Constitution, shall be dissolved or, where appropriate, converted into the regular force.
preservation and enrichment of culture, promote health and safety, enhance the right of the people to a
balanced ecology, encourage and support the development of appropriate and self-reliant scientific and Additionally, Section 21of Article XI states that, "The preservation of peace and order within the regions shall be
technological capabilities, improve public morals, enhance economic prosperity and social justice, promote full the responsibility of the local police agencies which shall be organized, maintained, supervised, and utilized in
employment among their residents, maintain peace and order, and preserve the comfort and convenience of accordance with applicable laws. The defense and security of the regions shall be the responsibility of the
their inhabitants. (Emphases supplied) National Government."

Respondents cannot rely on paragraph 1, subparagraph (vii) of Article 465 above, as the said provision Taken in conjunction with each other, it becomes clear that the Constitution does not authorize the organization
expressly refers to calamities and disasters, whether man-made or natural. The governor, as local chief of private armed groups similar to the CEF convened by the respondent Governor. The framers of the
executive of the province, is certainly empowered to enact and implement emergency measures during these Constitution were themselves wary of armed citizens’ groups, as shown in the following proceedings:
occurrences. But the kidnapping incident in the case at bar cannot be considered as a calamity or a disaster.
Respondents cannot find any legal mooring under this provision to justify their actions.
MR. GARCIA: I think it is very clear that the problem we have here is a paramilitary force operating under the
cloak, under the mantle of legality is creating a lot of problems precisely by being able to operate as an
Paragraph 2, subparagraph (vi) of the same provision is equally inapplicable for two reasons. First, the Armed independent private army for many regional warlords. And at the same time, this I think has been the thrust, the
Forces of the Philippines does not fall under the category of a "national law enforcement agency," to which the intent of many of the discussions and objections to the paramilitary units and the armed groups.
National Police Commission (NAPOLCOM) and its departments belong.
MR. PADILLA: My proposal covers two parts: the private armies of political warlords and other armed torces not
Its mandate is to uphold the sovereignty of the Philippines, support the Constitution, and defend the Republic recognized by constituted authority which shall be dismantled and dissolved. In my trips to the provinces, I heard
against all enemies, foreign and domestic. Its aim is also to secure the integrity of the national territory. 60 of many abuses committed by the CHDF (Civilian Home Defense Forces), specially in Escalante, Negros
Occidental. But I do not know whether a particular CHDF is approved or authorized by competent authority. If it
Second, there was no evidence or even an allegation on record that the local police forces were inadequate to is not authorized, then the CHDF will have to be dismantled. If some CHDFs, say in other provinces, are
cope with the situation or apprehend the violators. If they were inadequate, the recourse of the provincial authorized by constituted authority, by the Armed Forces of the Philippines, through the Chief of Staff or the
PAGE 55 of 135 CONSTITUTIONAL LAW -EXECUTIVE DEPARTMENT PART 2 –SESSION 12
Minister of National Defense, if they are recognized and authorized, then they will not be dismantled. But I
cannot give a categorical answer to any specific CHDF unit, only the principle that if they are armed forces which
are not authorized, then they should be dismantled. 64 (Emphasis supplied)

Thus, with the discussions in the Constitutional Commission as guide, the creation of the Civilian Emergency
Force (CEF) in the present case, is also invalid.

WHEREFORE, the instant petition is GRANTED. Judgment is rendered commanding respondents to desist from
further proceedings m implementing Proclamation No. 1, Series of 2009, and its Implementing Guidelines. The
said proclamation and guidelines are hereby declared NULL and VOID for having been issued in grave abuse of
discretion, amounting to lack or excess of jurisdiction.

SO ORDERED.
PAGE 56 of 135 CONSTITUTIONAL LAW -EXECUTIVE DEPARTMENT PART 2 –SESSION 12
EN BANC right of the Senate to compel the testimony of petitioners, the constitutional questions raised by them would have
B/GEN. (RET.) FRANCISCO V. G.R. No. 170165 come to fore. Such a scenario could have very well been presented to the Court in such manner, without the
GUDANI AND LT. COL. petitioners having had to violate a direct order from their commanding officer. Instead, the Court has to resolve
ALEXANDER F. BALUTAN whether petitioners may be subjected to military discipline on account of their defiance of a direct order of the AFP
Petitioners, Present: Chief of Staff.
PANGANIBAN, C.J.,
PUNO, The solicited writs of certiorari and prohibition do not avail; the petition must be denied.
- versus - QUISUMBING,
YNARES-SANTIAGO, I.
SANDOVAL-GUTIERREZ,
CARPIO, The petitioners are high-ranking officers of the Armed Forces of the Philippines (AFP). Both petitioners, Brigadier
AUSTRIA-MARTINEZ, General Francisco Gudani (Gen. Gudani) and Lieutenant Colonel Alexander Balutan (Col. Balutan), belonged to
LT./GEN. GENEROSO S. SENGA CORONA, the Philippine Marines. At the time of the subject incidents, both Gen. Gudani and Col. Balutan were assigned to
AS CHIEF OF STAFF OF THE CARPIO-MORALES, the Philippine Military Academy (PMA) in Baguio City, the former as the PMA Assistant Superintendent, and the
ARMED FORCES OF THE CALLEJO, SR., latter as the Assistant Commandant of Cadets. [2]
PHILIPPINES, COL. GILBERTO AZCUNA,
JOSE C. ROA AS THE PRE-TRIAL TINGA,
INVESTIGATING OFFICER, THE CHICO-NAZARIO,
PROVOST MARSHALL GENERAL GARCIA, and On 22 September 2005, Senator Rodolfo Biazon (Sen. Biazon) invited several senior officers of the AFP to appear
OF THE ARMED FORCES OF THE VELASCO, JR., JJ. at a public hearing before the Senate Committee on National Defense and Security (Senate Committee) scheduled
PHILIPPINES AND THE GENERAL on 28 September 2005. The hearing was scheduled after topics concerning the conduct of the 2004 elections
COURT-MARTIAL, emerged in the public eye, particularly allegations of massive cheating and the surfacing of copies of an audio
Respondents. excerpt purportedly of a phone conversation between President Gloria Macapagal Arroyo and an official of the
Promulgated: Commission on Elections (COMELEC) widely reputed as then COMELEC Commissioner Virgilio Garcillano. At
the time of the 2004 elections, Gen. Gudani had been designated as commander, and Col. Balutan a member, of
August 15, 2006 Joint Task Force Ranao by the AFP Southern Command. Joint Task Force Ranao was tasked with the
maintenance of peace and order during the 2004 elections in the provinces of Lanao del Norte and Lanao del
x--------------------------------------------------------------------------- x Sur.[3] `

DECISION Gen. Gudani, Col. Balutan, and AFP Chief of Staff Lieutenant General Generoso Senga (Gen. Senga) were among
the several AFP officers who received a letter invitation from Sen. Biazon to attend the 28 September
TINGA, J.: 2005 hearing. On 23 September 2005, Gen. Senga replied through a letter to Sen. Biazon that he would be unable
to attend the hearing due to a previous commitment in Brunei, but he nonetheless directed other officers from the
A most dangerous general proposition is foisted on the Court that soldiers who defy orders of their superior officers AFP who were invited to attend the hearing.[4]
are exempt
On 26 September 2005, the Office of the Chief of Staff of the AFP issued a Memorandum addressed to the
Superintendent of the PMA Gen. Cristolito P. Baloing (Gen. Baloing). It was signed by Lt. Col. Hernando DCA
from the strictures of military law and discipline if such defiance is predicated on an act otherwise valid under Iriberri in behalf of Gen. Senga.[5] Noting that Gen. Gudani and Col. Balutan had been invited to attend the Senate
civilian law. Obedience and deference to the military chain of command and the President as commander-in-chief Committee hearing on 28 September 2005, the Memorandum directed the two officers to attend the
are the cornerstones of a professional military in the firm cusp of civilian control. These values of obedience and hearing.[6] Conformably, Gen. Gudani and Col. Balutan filed their respective requests for travel authority addressed
deference expected of military officers are content-neutral, beyond the sway of the officers own sense of what is to the PMA Superintendent.
prudent or rash, or more elementally, of right or wrong. A self-righteous military invites itself as the scoundrels
activist solution to the ills of participatory democracy. On 27 September 2005, Gen. Senga wrote a letter to Sen. Biazon, requesting the postponement of the hearing
scheduled for the following day, since the AFP Chief of Staff was himself unable to attend said hearing, and that
Petitioners seek the annulment of a directive from President Gloria Macapagal-Arroyo[1] enjoining them some of the invited officers also could not attend as they were attending to other urgent operational matters. By
and other military officers from testifying before Congress without the Presidents consent. Petitioners also pray for this time, both Gen. Gudani and Col. Balutan had already departed Baguio for Manila to attend the hearing.
injunctive relief against a pending preliminary investigation against them, in preparation for possible court-martial
proceedings, initiated within the military justice system in connection with petitioners violation of the Then on the evening of 27 September 2005, at around 10:10 p.m., a message was transmitted to the PMA
aforementioned directive. Superintendent from the office of Gen. Senga, stating as follows:

The Court is cognizant that petitioners, in their defense, invoke weighty constitutional principles that PER INSTRUCTION OF HER EXCELLENCY PGMA, NO AFP PERSONNEL SHALL
center on fundamental freedoms enshrined in the Bill of Rights. Although these concerns will not be APPEAR BEFORE ANY CONGRESSIONAL OR SENATE HEARING WITHOUT HER
APPROVAL. INFORM BGEN FRANCISCO F GUDANI AFP AND LTC ALEXANDER BALUTAN
PA (GSC) ACCORDINGLY.[7]
addressed to the satisfaction of petitioners, the Court recognizes these values as of paramount importance to our
civil society, even if not determinative of the resolution of this petition. Had the relevant issue before us been the
PAGE 57 of 135 CONSTITUTIONAL LAW -EXECUTIVE DEPARTMENT PART 2 –SESSION 12
Petitioners characterize the directive from President Arroyo requiring her prior approval before any AFP personnel
The following day, Gen. Senga sent another letter to Sen. Biazon, this time informing the senator that no approval appear before Congress as a gag order, which violates the principle of separation of powers in government as it
has been granted by the President to any AFP officer to appear before the hearing scheduled on that day. interferes with the investigation of the Senate Committee conducted in aid of legislation. They also equate the gag
Nonetheless, both Gen. Gudani and Col. Balutan were present as the hearing started, and they both testified as order with culpable violation of the Constitution, particularly in relation to the publics constitutional right to
to the conduct of the 2004 elections. information and transparency in matters of public concern. Plaintively, petitioners claim that the Filipino people
have every right to hear the [petitioners] testimonies, and even if the gag order were unconstitutional, it still was
The Office of the Solicitor General (OSG), representing the respondents before this Court, has offered additional tantamount to the crime of obstruction of justice. Petitioners further argue that there was no law prohibiting them
information surrounding the testimony of Gen. Gudani and Col. Balutan. The OSG manifests that the couriers of from testifying before the Senate, and in fact, they were appearing in obeisance to the authority of Congress to
the AFP Command Center had attempted to deliver the radio message to Gen. Gudanis residence in a subdivision conduct inquiries in aid of legislation.
in Paraaque City late in the night of 27 September 2005, but they were not permitted entry by the subdivision
guards. The next day, 28 September 2005, shortly before the start of the hearing, a copy of Gen. Sengas letter to Finally, it is stressed in the petition that Gen. Gudani was no longer subject to military jurisdiction on account of
Sen. Biazon sent earlier that day was handed at the Senate by Commodore Amable B. Tolentino of the AFP Office his compulsory retirement on 4 October 2005. It is pointed out that Article 2, Title I of the Articles of War defines
for Legislative Affairs to Gen. Gudani, who replied that he already had a copy.Further, Gen. Senga called persons subject to military law as all officers and soldiers in the active service of the AFP.
Commodore Tolentino on the latters cell phone and asked to talk to Gen. Gudani, but Gen. Gudani refused. In
response, Gen. Senga instructed Commodore Tolentino to inform Gen. Gudani that it was an order, yet Gen. II.
Gudani still refused to take Gen. Sengas call.[8]
We first proceed to define the proper litigable issues. Notably, the guilt or innocence of petitioners in violating
A few hours after Gen. Gudani and Col. Balutan had concluded their testimony, the office of Gen. Senga issued a Articles 65 and 97 of the Articles of War is not an issue before this Court, especially considering that per records,
statement which noted that the two had appeared before the Senate Committee in spite of the fact that a guidance petitioners have not yet been subjected to court martial proceedings. Owing to the absence of such proceedings,
has been given that a Presidential approval should be sought prior to such an appearance; that such directive was the correct inquiry should be limited to whether respondents could properly initiate such proceedings preparatory
in keeping with the time[-]honored principle of the Chain of Command; and that the two officers disobeyed a legal to a formal court-martial, such as the aforementioned preliminary investigation, on the basis of petitioners acts
order, in violation of A[rticles of] W[ar] 65 (Willfully Disobeying Superior Officer), hence they will be subjected to surrounding their testimony before the Senate on 28 September 2005. Yet this Court, consistent with the principle
General Court Martial proceedings x x x Both Gen. Gudani and Col. Balutan were likewise relieved of their that it is not a trier of facts at first instance,[21] is averse to making any authoritative findings of fact, for that function
assignments then.[9] is first for the court-martial court to fulfill.

On the very day of the hearing, 28 September 2005, President Gloria-Macapagal-Arroyo issued Executive Order Thus, we limit ourselves to those facts that are not controverted before the Court, having been commonly alleged
No. 464 (E.O. 464). The OSG notes that the E.O. enjoined officials of the executive department including the by petitioners and the OSG (for respondents). Petitioners were called by the Senate Committee to testify in its 28
military establishment from appearing in any legislative inquiry without her approval. [10] This Court subsequently September 2005 hearing. Petitioners attended such hearing and testified before the Committee, despite the fact
ruled on the constitutionality of the said executive order in Senate v. Ermita.[11] The relevance of E.O. 464 that the day before, there was an order from Gen. Senga (which in turn was sourced per instruction from President
and Senate to the present petition shall be discussed forthwith. Arroyo) prohibiting them from testifying without the prior approval of the President. Petitioners do not precisely
admit before this Court that they had learned of such order prior to their testimony, although the OSG asserts that
In the meantime, on 30 September 2005, petitioners were directed by General Senga, through Col. Henry A. at the very least, Gen. Gudani already knew of such order before he testified.[22] Yet while this fact may be
Galarpe of the AFP Provost Marshal General, to appear before the Office of the Provost Marshal General (OPMG) ultimately material in the court-martial proceedings, it is not determinative of this petition, which as stated earlier,
on 3 October 2005 for investigation. During their appearance before Col. Galarpe, both petitioners invoked their does not proffer as an issue whether petitioners are guilty of violating the Articles of War.
right to remain silent.[12] The following day, Gen. Gudani was compulsorily retired from military service, having
reached the age of 56.[13] What the Court has to consider though is whether the violation of the aforementioned order of Gen. Senga, which
emanated from the President, could lead to any investigation for court-martial of petitioners. It has to be
In an Investigation Report dated 6 October 2005, the OPMG recommended that petitioners be charged acknowledged as a general principle[23] that AFP personnel of whatever rank are liable under military law for
with violation of Article of War 65, on willfully disobeying a superior officer, in relation to Article of War 97, on violating a direct order of an officer superior in rank. Whether petitioners did violate such an order is not for the
conduct prejudicial to the good order and military discipline. [14] As recommended, the case was referred to a Pre- Court to decide, but it will be necessary to assume, for the purposes of this petition, that petitioners did so.
Trial Investigation Officer (PTIO) preparatory to trial by the General Court Martial (GCM). [15] Consequently, on 24
October 2005, petitioners were separately served with Orders respectively addressed to them and signed by III.
respondent Col. Gilbert Jose C. Roa, the Pre-Trial Investigating Officer of the PTIO. The Orders directed
petitioners to appear in person before Col. Roa at the Pre-Trial Investigation of the Charges for violation of Articles Preliminarily, we must discuss the effect of E.O. 464 and the Courts ruling in Senate on the present
65[16] and 97[17] of Commonwealth Act No. 408,[18] and to submit their counter-affidavits and affidavits of witnesses petition. Notably, it is not alleged that petitioners were in any way called to task for violating E.O. 464, but
at the Office of the Judge Advocate General. [19] The Orders were accompanied by respective charge sheets instead, they were charged for violating the direct order of Gen. Senga not to appear before the Senate
against petitioners, accusing them of violating Articles of War 65 and 97. Committee, an order that stands independent of the executive order. Distinctions are called for, since Section
2(b) of E.O. 464 listed generals and flag officers of the Armed Forces of the Philippines and such other officers
It was from these premises that the present petition for certiorari and prohibition was filed, particularly who in the judgment of the Chief of Staff are covered by the executive privilege, as among those public officials
seeking that (1) the order of President Arroyo coursed through Gen. Senga preventing petitioners from testifying required in Section 3 of E.O. 464 to secure prior consent of the President prior to appearing before either House
before Congress without her prior approval be declared unconstitutional; (2) the charges stated in the charge of Congress. The Court in Senate declared both Section 2(b) and Section 3 void,[24] and the impression may have
sheets against petitioners be quashed; and (3) Gen. Senga, Col. Galarpe, Col. Roa, and their successors-in- been left following Senate that it settled as doctrine, that the President is prohibited from requiring military
interest or persons acting for and on their behalf or orders, be permanently enjoined from proceeding against personnel from attending congressional hearings without having first secured prior presidential consent. That
petitioners, as a consequence of their having testified before the Senate on 28 September 2005. [20] impression is wrong.
PAGE 58 of 135 CONSTITUTIONAL LAW -EXECUTIVE DEPARTMENT PART 2 –SESSION 12
Senate turned on the nature of executive privilege, a presidential prerogative which is encumbered by significant trial by court-martial ordered and held after the end of the term of the
limitations. Insofar as E.O. 464 compelled officials of the executive branch to seek prior presidential approval enlistment of the accused x x x [29]
before appearing before Congress, the notion of executive control also comes into consideration. [25] However, the
ability of the President to require a military official to secure prior consent before appearing before Congress Thus, military jurisdiction has fully attached to Gen. Gudani inasmuch as both the acts complained of and the
pertains to a wholly different and independent specie of presidential authoritythe commander-in-chief powers of initiation of the proceedings against him occurred before he compulsorily retired on 4 October 2005. We see no
the President. By tradition and jurisprudence, the commander-in-chief powers of the President are not encumbered reason to unsettle the Abadilla doctrine. The OSG also points out that under Section 28 of Presidential Decree
by the same degree of restriction as that which may attach to executive privilege or executive control. No. 1638, as amended, [a]n officer or enlisted man carried in the retired list [of the Armed Forces of the Philippines]
shall be subject to the Articles of War x x x[30] To this citation, petitioners do not offer any response, and in fact
During the deliberations in Senate, the Court was very well aware of the pendency of this petition as well as the have excluded the matter of Gen. Gudanis retirement as an issue in their subsequent memorandum.
issues raised herein. The decision in Senate was rendered with the comfort that the nullification of portions of E.O.
464 would bear no impact on the present petition since petitioners herein were not called to task for violating the IV.
executive order. Moreover, the Court was then cognizant that Senate and this case would ultimately hinge on
disparate legal issues. Relevantly, Senate purposely did not touch upon or rule on the faculty of the President, We now turn to the central issues.
under the aegis of the commander-in-chief powers[26] to require military officials from securing prior consent before
appearing before Congress. The pertinent factors in considering that question are markedly outside of those which Petitioners wish to see annulled the gag order that required them to secure presidential consent prior to
did become relevant in adjudicating the issues raised in Senate. It is in this petition that those factors come into their appearance before the Senate, claiming that it violates the constitutional right to information and transparency
play. in matters of public concern; or if not, is tantamount at least to the criminal acts of obstruction of justice and grave
coercion. However, the proper perspective from which to consider this issue entails the examination of the basis
At this point, we wish to dispose of another peripheral issue before we strike at the heart of the matter. General and authority of the President to issue such an order in the first place to members of the AFP and the determination
Gudani argues that he can no longer fall within the jurisdiction of the court-martial, considering his retirement last 4 of whether such an order is subject to any limitations.
October 2005. He cites Article 2, Title I of Commonwealth Act No. 408, which defines persons subject to military
law as, among others, all officers and soldiers in the active service of the [AFP], and points out that he is no longer The vitality of the tenet that the President is the commander-in-chief of the Armed Forces is most crucial to the
in the active service. democratic way of life, to civilian supremacy over the military, and to the general stability of our representative
system of government. The Constitution reposes final authority, control and supervision of the AFP to the
This point was settled against Gen. Gudanis position in Abadilla v. Ramos,[27] where the Court declared President, a civilian who is not a member of the armed forces, and whose duties as commander-in-chief represent
that an officer whose name was dropped from the roll of officers cannot be considered to be outside the jurisdiction only a part of the organic duties imposed upon the office, the other functions being clearly civil in nature.[31] Civilian
of military authorities when military justice proceedings were initiated against him before the termination of his supremacy over the military also countermands the notion that the military may bypass civilian authorities, such
service. Once jurisdiction has been acquired over the officer, it continues until his case is terminated. Thus, the as civil courts, on matters such as conducting warrantless searches and seizures. [32]
Court held:
Pursuant to the maintenance of civilian supremacy over the military, the Constitution has allocated specific roles
The military authorities had jurisdiction over the person of Colonel Abadilla at the time to the legislative and executive branches of government in relation to military affairs. Military appropriations, as
of the alleged offenses. This jurisdiction having been vested in the military authorities, it is with all other appropriations, are determined by Congress, as is the power to declare the existence of a state of
retained up to the end of the proceedings against Colonel Abadilla. Well-settled is the rule that war.[33] Congress is also empowered to revoke a proclamation of martial law or the suspension of the writ of habeas
jurisdiction once acquired is not lost upon the instance of the parties but continues until the case corpus.[34] The approval of the Commission on Appointments is also required before the President can promote
is terminated.[28] military officers from the rank of colonel or naval captain. [35] Otherwise, on the particulars of civilian dominance
and administration over the military, the Constitution is silent, except for thecommander-in-
chief clause which is fertile in meaning and
Citing Colonel Winthrops treatise on Military Law, the Court further stated: implication as to whatever inherent martial authority the President may possess. [36]

The commander-in-chief provision in the Constitution is denominated as Section 18, Article VII, which
We have gone through the treatise of Colonel Winthrop and We find the following begins with the simple declaration that [t]he President shall be the Commander-in-Chief of all armed forces of the
passage which goes against the contention of the petitioners, viz Philippines x x x[37] Outside explicit constitutional limitations, such as those found in Section 5, Article XVI, the
commander-in-chief clause vests on the President, as commander-in-chief, absolute authority over the persons
3. Offenders in general Attaching of jurisdiction. It has further been and actions of the members of the armed forces. Such authority includes the ability of the President to restrict the
held, and is now settled law, in regard to military offenders in general, that if travel, movement and speech of military officers, activities which may otherwise be sanctioned under civilian law.
the military jurisdiction has once dulyattached to them previous to the date of
the termination of their legal period of service, they may be brought to trial by Reference to Kapunan, Jr. v. De Villa[38] is useful in this regard. Lt. Col. Kapunan was ordered confined
court-martial after that date, their discharge being meanwhile withheld. This under house arrest by then Chief of Staff (later President) Gen. Fidel Ramos. Kapunan was also ordered, as a
principle has mostly been applied to cases where the offense was committed condition for his house arrest, that he may not issue any press statements or give any press conference during
just prior to the end of the term. In such cases the interests of discipline clearly his period of detention. The Court unanimously upheld such restrictions, noting:
forbid that the offender should go unpunished. It is held therefore that if
before the day on which his service legally terminates and his right to a
discharge is complete, proceedings with a view to trial are commenced [T]he Court is of the view that such is justified by the requirements of military discipline. It
against him as by arrest or the service of charges, the military cannot be gainsaid that certain liberties of persons in the military service, including the
jurisdiction will fully attach and once attached may be continued by a freedom of speech, may be circumscribed by rules of military discipline. Thus, to a certain
degree, individual rights may be curtailed, because the effectiveness of the military in
PAGE 59 of 135 CONSTITUTIONAL LAW -EXECUTIVE DEPARTMENT PART 2 –SESSION 12
fulfilling its duties under the law depends to a large extent on the maintenance of
discipline within its ranks. Hence, lawful orders must be followed without question and Parenthetically, it must be said that the Court is well aware that our countrys recent past is marked by regime
rules must be faithfully complied with, irrespective of a soldier's personal views on the changes wherein active military dissent from the chain of command formed a key, though not exclusive, element.
matter. It is from this viewpoint that the restrictions imposed on petitioner Kapunan, an officer in The Court is not blind to history, yet it is a judge not of history but of the Constitution. The Constitution, and indeed
the AFP, have to be considered.[39] our modern democratic order, frown in no uncertain terms on a politicized military, informed as they are on the
Any good soldier, or indeed any ROTC cadet, can attest to the fact that the military way of life circumscribes trauma of absolute martial rule. Our history might imply that a political military is part of the natural order, but this
several of the cherished freedoms of civilian life. It is part and parcel of the military package. Those who cannot view cannot be affirmed by the legal order. The evolutionary path of our young democracy necessitates a
abide by these limitations normally do not pursue a military career and instead find satisfaction in other fields; and reorientation from this view, reliant as our socio-political culture has become on it. At the same time, evolution
in fact many of those discharged from the service are inspired in their later careers precisely by their rebellion mandates a similar demand that our system of governance be more responsive to the needs and aspirations of
against the regimentation of military life. Inability or unwillingness to cope with military discipline is not a stain on the citizenry, so as to avoid an environment vulnerable to a military apparatus able at will to exert an undue
character, for the military mode is a highly idiosyncratic path which persons are not generally conscripted into, but influence in our polity.
volunteer themselves to be part of. But for those who do make the choice to be a soldier, significant concessions
to personal freedoms are expected. After all, if need be, the men and women of the armed forces may be Of possibly less gravitas, but of equal importance, is the principle that mobility of travel is another necessary
commanded upon to die for country, even against their personal inclinations. restriction on members of the military. A soldier cannot leave his/her post without the consent of the commanding
officer. The reasons are self-evident. The commanding officer has to be aware at all times of the location of the
It may be so that military culture is a remnant of a less democratic era, yet it has been fully integrated into the troops under command, so as to be able to appropriately respond to any exigencies. For the same reason,
democratic system of governance. The constitutional role of the armed forces is as protector of the people and of commanding officers have to be able to restrict the movement or travel of their soldiers, if in their judgment, their
the State.[40] Towards this end, the military must insist upon a respect for duty and a discipline without counterpart presence at place of call of duty is necessary. At times, this may lead to unsentimental, painful consequences,
in civilian life.[41] The laws and traditions governing that discipline have a long history; but they are founded on such as a soldier being denied permission to witness the birth of his first-born, or to attend the funeral of a parent.
unique military exigencies as powerful now as in the past. [42] In the end, it must be borne in mind that the armed Yet again, military life calls for considerable personal sacrifices during the period of conscription, wherein the
forces has a distinct subculture with unique needs, a specialized society separate from civilian society. [43] In the higher duty is not to self but to country.
elegant prose of the eminent British military historian, John Keegan:
Indeed, the military practice is to require a soldier to obtain permission from the commanding officer before he/she
[Warriors who fight wars have] values and skills [which] are not those of politicians and may leave his destination. A soldier who goes from the properly appointed place of duty or absents from his/her
diplomats. They are those of a world apart, a very ancient world, which exists in parallel with the command, guard, quarters, station, or camp without proper leave is subject to punishment by court-martial.[48] It is
everyday world but does not belong to it. Both worlds change over time, and the warrior world even clear from the record that petitioners had actually requested for travel authority from the PMA
adopts in step to the civilian. It follows it, however, at a distance. The distance can never be in Baguio City to Manila, to attend the Senate Hearing.[49] Even petitioners are well aware that it was necessary
closed, for the culture of the warrior can never be that of civilization itself. [44] for them to obtain permission from their superiors before they could travel to Manila to attend the Senate Hearing.

Critical to military discipline is obeisance to the military chain of command. Willful disobedience of a superior officer It is clear that the basic position of petitioners impinges on these fundamental principles we have discussed. They
is punishable by court-martial under Article 65 of the Articles of War. [45]An individual soldier is not free to ignore seek to be exempted from military justice for having traveled to the Senate to testify before the Senate Committee
the lawful orders or duties assigned by his immediate superiors. For there would be an end of all discipline if the against the express orders of Gen. Senga, the AFP Chief of Staff. If petitioners position is affirmed, a considerable
seaman and marines on board a ship of war [or soldiers deployed in the field], on a distant service, were permitted exception would be carved from the unimpeachable right of military officers to restrict the speech and movement
to act upon their own opinion of their rights [or their opinion of the of their juniors. The ruinous consequences to the chain of command and military discipline simply cannot warrant
the Courts imprimatur on petitioners position.

Presidents intent], and to throw off the authority of the commander whenever they supposed it to be unlawfully V.
exercised.[46]
Still, it would be highly myopic on our part to resolve the issue solely on generalities surrounding military discipline.
Further traditional restrictions on members of the armed forces are those imposed on free speech and After all, petitioners seek to impress on us that their acts are justified as they were responding to an invitation from
mobility. Kapunan is ample precedent in justifying that a soldier may be restrained by a superior officer from the Philippine Senate, a component of the legislative branch of government. At the same time, the order for them
speaking out on certain matters. As a general rule, the discretion of a military officer to restrain the speech of a not to testify ultimately came from the President, the head of the executive branch of government and the
soldier under his/her command will be accorded deference, with minimal regard if at all to the reason for such commander-in-chief of the armed forces.
restraint. It is integral to military discipline that the soldiers speech be with the consent and approval of the military
commander. Thus, we have to consider the question: may the President prevent a member of the armed forces from testifying
The necessity of upholding the ability to restrain speech becomes even more imperative if the soldier desires to before a legislative inquiry? We hold that the President has constitutional authority to do so, by virtue of her power
speak freely on political matters. The Constitution requires that [t]he armed forces shall be insulated from partisan as commander-in-chief, and that as a consequence a military officer who defies such injunction is liable under
politics, and that [n]o member of the military shall engage directly or indirectly in any partisan political activity, military justice. At the same time, we also hold that any chamber of Congress which seeks the appearance before
except to vote.[47] Certainly, no constitutional provision or military indoctrination will eliminate a soldiers ability to it of a military officer against the consent of the President has adequate remedies under law to compel such
form a personal political opinion, yet it is vital that such opinions be kept out of the public eye. For one, political attendance. Any military official whom Congress summons to testify before it may be compelled to do so by the
belief is a potential source of discord among people, and a military torn by political strife is incapable of fulfilling President. If the President is not so inclined, the President may be commanded by judicial order to compel the
its constitutional function as protectors of the people and of the State. For another, it is ruinous to military discipline attendance of the military officer. Final judicial orders have the force of the law of the land which the President has
to foment an atmosphere that promotes an active dislike of or dissent against the President, the commander-in- the duty to faithfully execute.[50]
chief of the armed forces. Soldiers are constitutionally obliged to obey a President they may dislike or distrust.
This fundamental principle averts the country from going the way of banana republics. Explication of these principles is in order.
PAGE 60 of 135 CONSTITUTIONAL LAW -EXECUTIVE DEPARTMENT PART 2 –SESSION 12
As discussed in Arnault, the power of inquiry, with process to enforce it, is grounded on the
necessity of information in the legislative process. If the information possessed by executive
As earlier noted, we ruled in Senate that the President may not issue a blanket requirement of prior consent on officials on the operation of their offices is necessary for wise legislation on that subject, by parity
executive officials summoned by the legislature to attend a congressional hearing. In doing so, the Court of reasoning, Congress has the right to that information and the power to compel the disclosure
recognized the considerable limitations on executive privilege, and affirmed that the privilege must be formally thereof.
invoked on specified grounds. However, the ability of the President to prevent military officers from testifying
before Congress does not turn on executive privilege, but on the Chief Executives power as commander- As evidenced by the American experience during the so-called McCarthy era, however, the right
in-chief to control the actions and speech of members of the armed forces. The Presidents prerogatives of Congress to conduct inquirites in aid of legislation is, in theory, no less susceptible to abuse
as commander-in-chief are not hampered by the same limitations as in executive privilege. than executive or judicial power. It may thus be subjected to judicial review pursuant to the
Courts certiorari powers under Section 1, Article VIII of the Constitution.
Our ruling that the President could, as a general rule, require military officers to seek presidential approval before
appearing before Congress is based foremost on the notion that a contrary rule unduly diminishes the prerogatives For one, as noted in Bengzon v. Senate Blue Ribbon Committee, the inquiry itself might not
of the President as commander-in-chief. Congress holds significant control over the armed forces in matters such properly be in aid of legislation, and thus beyond the constitutional power of Congress. Such
as budget appropriations and the approval of higher-rank promotions,[51] yet it is on the President that the inquiry could not usurp judicial functions. Parenthetically, one possible way for Congress to
Constitution vests the title as commander-in-chief and all the prerogatives and functions appertaining to the avoid such result as occurred in Bengzon is to indicate in its invitations to the public officials
position. Again, the exigencies of military discipline and the chain of command mandate that the Presidents ability concerned, or to any person for that matter, the possible needed statute which prompted the
to control the individual members of the armed forces be accorded the utmost respect. Where a military officer is need for the inquiry. Given such statement in its invitations, along with the usual indication of
torn between obeying the President and obeying the Senate, the Court will without hesitation affirm that the officer the subject of inquiry and the questions relative to and in furtherance thereof, there would be
has to choose the President. After all, the Constitution prescribes that it is the President, and not the Senate, who less room for speculation on the part of the person invited on whether the inquiry is in aid of
is the commander-in-chief of the armed forces.[52] legislation.

Section 21, Article VI likewise establishes critical safeguards that proscribe the legislative power
At the same time, the refusal of the President to allow members of the military to appear before Congress is still of inquiry. The provision requires that the inquiry be done in accordance with the Senate or
subject to judicial relief. The Constitution itself recognizes as one of the legislatures functions is the conduct of Houses duly published rules of procedure, necessarily implying the constitutional infirmity of an
inquiries in aid of legislation.[53] Inasmuch as it is ill-advised for Congress to interfere with the Presidents power as inquiry conducted without duly published rules of procedure. Section 21 also mandates that the
commander-in-chief, it is similarly detrimental for the President to unduly interfere with Congresss right to conduct rights of persons appearing in or affected by such inquiries be respected, an imposition that
legislative inquiries. The impasse did not come to pass in this petition, since petitioners testified anyway despite obligates Congress to adhere to the guarantees in the Bill of Rights.
the presidential prohibition. Yet the Court is aware that with its pronouncement today that the President has the
right to require prior consent from members of the armed forces, the clash may soon loom or actualize. These abuses are, of course, remediable before the courts, upon the proper suit filed by the
persons affected, even if they belong to the executive branch. Nonetheless, there may be
We believe and hold that our constitutional and legal order sanctions a modality by which members of the military exceptional circumstances wherein a clear pattern of abuse of the legislative power of inquiry
may be compelled to attend legislative inquiries even if the President desires otherwise, a modality which does might be established, resulting in palpable violations of the rights guaranteed to members of the
not offend the Chief Executives prerogatives as commander-in-chief. The remedy lies with the courts. executive department under the Bill of Rights. In such instances, depending on the particulars
of each case, attempts by the Executive Branch to forestall these abuses may be accorded
The fact that the executive branch is an equal, coordinate branch of government to the legislative creates judicial sanction[59].
a wrinkle to any basic rule that persons summoned to testify before Congress must do so. There is considerable
interplay between the legislative and executive branches, informed by due deference and respect as to their In Senate, the Court ruled that the President could not impose a blanket prohibition barring executive officials from
various constitutional functions. Reciprocal courtesy idealizes this relationship; hence, it is only as a last resort testifying before Congress without the Presidents consent notwithstanding the invocation of executive privilege to
that one branch seeks to compel the other to a particular mode of behavior. The judiciary, the third coordinate justify such prohibition. The Court did not rule that the power to conduct legislative inquiry ipso facto superseded
branch of government, does not enjoy a similar dynamic with either the legislative or executive branches. Whatever the claim of executive privilege, acknowledging instead that the viability of executive privilege stood on a case to
weakness inheres on judicial power due to its inability to originate national policies and legislation, such is balanced case basis. Should neither branch yield to the other branchs assertion, the constitutional recourse is to the courts,
by the fact that it is the branch empowered by the Constitution to compel obeisance to its rulings by the other as the final arbiter if the dispute. It is only the courts that can compel, with conclusiveness, attendance or non-
branches of government. attendance in legislative inquiries.

As evidenced by Arnault v. Nazareno[54] and Bengzon v. Senate Blue Ribbon Committee,[55] among Following these principles, it is clear that if the President or the Chief of Staff refuses to allow a member
others, the Court has not shirked from reviewing the exercise by Congress of its power of legislative of the AFP to appear before Congress, the legislative body seeking such testimony may seek judicial relief to
inquiry.[56] Arnault recognized that the legislative power of inquiry and the process to enforce it, is an essential and compel the attendance. Such judicial action should be directed at the heads of the executive branch or the armed
appropriate auxiliary to the legislative function. [57] On the other hand, Bengzon acknowledged that the power of forces, the persons who wield authority and control over the actions of the officers concerned. The legislative
both houses of Congress to conduct inquiries in aid of legislation is not absolute or unlimited, and its exercise is purpose of such testimony, as well as any defenses against the same whether grounded on executive privilege,
circumscribed by Section 21, Article VI of the Constitution.[58] From these premises, the Court enjoined the Senate national security or similar concerns would be accorded due judicial evaluation. All the constitutional
Blue Ribbon Committee from requiring the petitioners in Bengzon from testifying and producing evidence before considerations pertinent to either branch of government may be raised, assessed, and ultimately weighed against
the committee, holding that the inquiry in question did not involve any intended legislation. each other. And once the courts speak with finality, both branches of government have no option but to comply
with the decision of the courts, whether the effect of the decision is to their liking or disfavor.
Senate affirmed both the Arnault and Bengzon rulings. It elucidated on the constitutional scope and limitations on
the constitutional power of congressional inquiry. Thus: Courts are empowered, under the constitutional principle of judicial review, to arbitrate disputes between
the legislative and executive branches of government on the proper constitutional parameters of power. [60] This is
PAGE 61 of 135 CONSTITUTIONAL LAW -EXECUTIVE DEPARTMENT PART 2 –SESSION 12
the fair and workable solution implicit in the constitutional allocation of powers among the three branches of
government. The judicial filter helps assure that the particularities of each case would ultimately govern, rather
than any overarching principle unduly inclined towards one branch of government at the expense of the other. The
procedure may not move as expeditiously as some may desire, yet it ensures thorough deliberation of all relevant
and cognizable issues before one branch is compelled to yield to the other. Moreover, judicial review does not
preclude the legislative and executive branches from negotiating a mutually acceptable solution to the impasse.
After all, the two branches, exercising as they do functions and responsibilities that are political in nature, are free
to smooth over the thorns in their relationship with a salve of their own choosing.

And if emphasis be needed, if the courts so rule, the duty falls on the shoulders of the President,
as commander-in-chief, to authorize the appearance of the military officers before Congress. Even if the
President has earlier disagreed with the notion of officers appearing before the legislature to testify, the
Chief Executive is nonetheless obliged to comply with the final orders of the courts.

Petitioners have presented several issues relating to the tenability or wisdom of the Presidents order on
them and other military officers not to testify before Congress without the Presidents consent. Yet these issues
ultimately detract from the main point that they testified before the Senate despite an order from their commanding
officer and their commander-in-chief for them not to do so,[61] in contravention of the traditions of military discipline
which we

affirm today. The issues raised by petitioners could have very well been raised and properly adjudicated if the
proper procedure was observed. Petitioners could have been appropriately allowed to testify before the Senate
without having to countermand their Commander-in-chief and superior officer under the setup we have prescribed.

We consider the other issues raised by petitioners unnecessary to the resolution of this petition.

Petitioners may have been of the honest belief that they were defying a direct order of their Commander-in-Chief
and Commanding General in obeisance to a paramount idea formed within their consciences, which could not be
lightly ignored. Still, the Court, in turn, is guided by the superlative principle that is the Constitution, the embodiment
of the national conscience. The Constitution simply does not permit the infraction which petitioners have allegedly
committed, and moreover, provides for an orderly manner by which the same result could have been achieved
without offending constitutional principles.

WHEREFORE, the petition is DENIED. No pronouncement as to costs.

SO ORDERED.
PAGE 62 of 135 CONSTITUTIONAL LAW -EXECUTIVE DEPARTMENT PART 2 –SESSION 12
Republic of the Philippines Provincial Government (because it did not provide for interest or for any type security and it did not provide for
SUPREME COURT suretyship and comptrollership or audit to control the safe disbursement of said loan); that a total amount of
Manila P20,000,000.00 was disbursed to the aforesaid Foundation; that the transactions constitute a fraudulent scheme
to defraud the Provincial Government; and that the said Agreement is wholly unconstitutional, illegal, a immoral.
EN BANC (Annex "A", Petition)

G.R. No. 99031 October 15, 1991 On the other hand, it is the contention of respondent governor that "the funds were intended to generate
livelihood project among the residents of Tarlac and the use of the Lingkod Tarlac Foundation, Inc. was
authorized by law and considered the best alternative as a matter of judgment." (pp. 12-13, Appeal
RODOLFO D. LLAMAS, petitioner, Memorandom); that he resigned from the said Foundation in order to forestall any suspicion that he would
vs. influence it; that it is not true that the Loan Agreement did not provide for continuing audit by the Provincial
EXECUTIVE SECRETARY OSCAR ORBOS and MARIANO UN OCAMPO III, respondents. Government because the Memorandum of Agreement provides otherwise; and that the Agreement is not
manifestly and grossly disadvantageous to the Provincial Government and respondent governor did not and
Mauricio Law Office for petitioner. would not profit thereby because it provided sufficient safeguards for repayment. (Annex "A", Petition)

Ongkiko, Bucoy, Dizon & Associates for private respondent. After trial, the Secretary of the then Department of Local Government rendered a decision dated September 21,
1990, dispositive portion of which reads:

WHEREFORE, Governor Mariano Un Ocampo III is, as he hereby found guilty of having violated
PARAS, J.: Section 3(g) of Republic Act No.3019, otherwise known as the Anti-Graft and Corrupt Practices Act,
which act amounts to serious neglect of duty and/or abuse of authority, for which tilp penalty of
suspension from office for a period of ninety (90) days, effective upon the finality of this Decision, is
The case before Us calls for a determination of whether or not the President of the Philippines has the power to hereby imposed upon him. (p. 3, Petition)
grant executive clemency in administrative cases. In connection therewith, two important questions are also put
in issue, namely, whether or not the grant of executive clemency and the reason therefore, are political
questions beyond judicial review, and whether or not the questioned act was characterized by grave abuse of Parenthetically, be it noted that the Resolution imposed not a preventive suspension but a penalty of
discretion amounting to lack of jurisdiction. suspension.

Petitioner Rodolfo D. Llamas is the incumbent Vice-Governor of the Province of Tarlac and, on March 1, 1991 he Respondent govemor moved for a reconsideration of the abovequoted decision but the same was denied on
assumed, by virtue of a decision of the Office of the President, the governorship (p. 1, Petition). Private October 19, 1990. Aggrieved, he appealed the DLG decision dated September 21, 1990 and the order of denial
respondent Mariano Un Ocampo III is the incumbent Governor of the Province of Tarlac and was suspended dated October 19, 1990 to the Office of the President (O.P. Case No. 4480).
from office for a period of 90 days. Public respondent Oscar Orbos was the Executive Secretary at the time of
the filing of this petition and is being impleaded herein in that official capacity for having issued, by authority of On February 26, 1991, herein public respondent Executive Secretary issued a Resolution dismissing respondent
the President, the assailed Resolution granting executive clemency to respondent governor. governor's appeal and affirming the September 21, 1990 DLG decision.

Sometime in 1989, petiotioner, together with Tarlac Board Members Marcelino Aganon, Jr. and Arnaldo P. Subsequently, and pursuant to Sec. 66, Chapter 4 of B.P. Blg. 337, to the effect that the decision of the Office of
Dizon, filed on June 13, 1989 a verified complaint dated June 7, 1989 against respondent governor before the the President in administrative suspension of local officials shall be immediately executory without prejudice to
then Department of Local Government (DLG, for short), charging him with alleged violation of Section 203(2) (f) appeal to appropriate courts, petitioner, on March 1, 1991, took his oath of office as acting governor. Under the
203(2) (p), and 208(w), of Batas Pambansa (B.P.) Blg. 337, otherwise known as the Local Government Code, administrative suspension order, petitioner had up to May 31, 1991 as acting governor. On the same date
and other appropriate laws, among them, the Anti-Graft and Corrupt Practices ACt. Prior to that, petitoner filed (March 1, 1991), respondent govemor moved for a reconsideration of the Executive Secretary's Resolution, to
with the Office of the Omdusman a verified complainant dated November 10, 1988 against respondent governor which petitioner filed an opposition. From the allegations of the petitioner in his petition, respondent govemor
for the latter's alleged viloation of Section 3-G of Republic Act. (R.A.) No. 3019, otherwise known as the Anti- accepted his suspension and turned over his office to petitioner.
Graft and Corrupt Practices Act.
To the surprise of petitioner, however, respondent govemor on March 19, 1991, issued an "administrative order"
The complaint before the DLG, docketed as Administrative Case 10459, was subsequently tried, where both dated March 8, 1991, in which the latter signified his intention to "(continue, as I am bound to exercise my
petitioner and respondent govemor presented their respective evidence. fimctions as govemor and shall hold office at my residence," in the belief that "the pendency of my Motion for
Reconsideration precludes the coming into finality as executory the DLG decision." (Annex "E", Petition; p. 10,
Petitioner maintains that sometime in August, 1988, respondent governor, in his official capacity as Provincial Comment). And, as categorically stated in the petition, the reassumption ceremony by respondent governor was
Governor Tarlac, entered into and executed a Loan Agreement with Lingkod Tarlac Foundation, Inc., a non- held on May 21, 1991 (p. 8, Petition).
stock and non-profit organization headed by the governor himself as chairman and controlled by his brother-in-
law as executive director, trustee, and secretary; that the said Loan Agreement was never authorized and Without ruling on respondent governor's Motion for Reconsideration, public respondent issued a Resolution
approved by the Provincial Board, in direct contravention of the provisions of the Local Government Code; that dated May 15, 1991, in O.P. Case No. 4480, which reads:
the said Agreement is wholly one-sided in favor of the Foundation and grossly inimical to the interest of the
PAGE 63 of 135 CONSTITUTIONAL LAW -EXECUTIVE DEPARTMENT PART 2 –SESSION 12
This refers to the petition of Gov. Mariano Un Ocampo III of Tarlac for executive clemency, interposed the governorship in gross defiance of the suspension order), executive clemency cannot apply to him; that his
in connection with the decision of the Secretary of then Department of Local Governmen (DLG) dated rights to due process were violated because the grant of executive clemency was so sudden that he was not
21 September 1990, as affirmed in a Resolution of this Office dated 26 February 1991, suspending even notified thereof; and that despite a finding by public respondent of impropriety in the loan transaction
petitioner from office for period of ninety (90) days upon the finality of said decision. entered into by respondent governor, the former failed to justify the reduction of the penalty of suspension on the
latter. Petitioner further alleges that the exftutive clemency granted by public respondent was "the product of a
As will be recalled, the DLG Secretary imposed the penalty of suspension upon his finding that hocus-pocus strategy" (p. 1, Manifestation with Motion, etc.) because there was allegedly no real petition for the
petitioner was guilty of serious neglect of duty and/or abuse of authority for entering into a loan contract grant of executive clemency filed by respondent govemor.
— with the Lingkod Tarlac Foundation, Inc. (LTFI) — grossly/manifestly disadvantageous to Tarlac
Province. In his letter-petition of 10 May 1991, thereby pleading for a thirty (30)-day reduction of his Batas Pambansa Blg. 337 provides:
suspension, petitioner invited attention to the DLG Secretary's decision clearing him of having
personally benefitted from the questioned transaction. In the same letter, petitioner manifests serving Sec. 63. Preventive Suspension. — (1) Preventive suspension may be imposed by the Minister of Local
more than sixty (60) days of the ninety-day suspension. Previously, petitioner submitted documents and Government if the respondent is a provincial or city official, ...
letters from his constituents tending to show the relative success of his livelihood loan program pursue
under the aegis of the LTFI and/or the Foundation's credible loan repayment record. To cite some:
(2) Preventive suspension may be imposed at any time after the issues are joined, when there is
reasonable ground to believe that the respondent has committed the act or acts complained of, when
1. Certification of the Chairman,Tarlac Integrated Livelihood Cooperative, Inc., attesting to the full the evidence of culpability is strong, when the gravity of the offense s warrants, or when the
payment of its loan (P15.05 M) plus interest with LTFI; continuance in office of the respondent coul influence the witnesses or pose a threat to the safety and
integrity the records and other evidence. In all cases, preventive suspension shall not extend beyond
2. Certification of the Manager, Rural Bank of Geron (Tarlac), Inc., attesting to the gradual liquidation of sixty days after the start of said suspension.
the loan granted to family-borrowers out of funds provided by LTFI;
(3) At the expiration of sixty days, the suspended official shall be deemed reinstated in office without
3. Letter of Jover's Phil., expressing gratitude for the loan assistance extended for its export activities prejudice to the continuation the proceedings against him until its termination. (Emphasis supplied)
by LTFI;
It is admitted by petitioner that since March 1, 1991, he has assumed the governorship. A portion of the petition
4. Letter of the Tarlac Provincial Agricultural Officer i forming that the proceeds of the loan from LTFI is hereon der quoted as follows:
have bee utilized in hybrid com production; and
7. [On February 28, 1991], and in accordance with the provisions of the Local Government Code (Sec.
5. Letter of the President of the Federation of Tobacco Leaf Producers of Tarlac, Inc., informing of the 66, Chapter 4, Batas Pambansa Blg. 337), to the effect that the decision of the Office of the President
payment of 76 of the amount (P203,966.00) loaned to the Federation for tobacco production. in an administrative suspension of local officials shall be immediately executory without prejudice to
appeal to appropriate courts, Petitioner Llamas took his oath of office as acting govemor. Under the
Petitioner's act, vis-a-vis the loan to LTFI, may have been promp by an over eagerness to accelerate administrative suspension order, Llamas had up to May 31 [sic 29] 1991 as acting governor;
the delivery of livelihood services to his provincemates. As the truism goes, however, the end does not
always justify the means. Be that as it may, but without belaboring the propriety of the loan agreement 8. A copy of this oath of office is attached and made a part hereof as Annex B;
aforementioned, some measure of leniency may be accorded petitioner as the purpose of his suspen
sion may have made its mark. 9. Significantly, this oath of office was sworn to by Petitioner Llamas before Secretary Santos of the
newly created Department Interior and Local Government, as shown by the lower portion Annex B, and
WHEREFORE, Governor Mariano Un Ocampo III is hereby granted executive clemency in the sense by a picture of the oathtaking itself, attached and mad a part hereof as Annex B-1;
that his ninety-day suspension is hereby reduced to the period already served.
10. Subsequently, Petitioner Llamas and Respondent Ocampo met, where Ocampo was shown
SO ORDERED. Llamas' oath of office. During meeting, held in the presence of all department heads at the provi cial
capitol and in the presence of various local government offici and representatives of the media,
(Annex "F", Petition; pp. 25-26, Rollo) Ocampo agreed to turn over reigns of the provincial government to Petitioner;

By virtue of the aforequoted Resolution, respondent governor reassumed the governorship of the province, 11. In fact, Ocampo had asked the department heads and all other officials of the provincial
allegedly withou any notification made to the petitioner. government of Tarlac to extend their cooperation to Llamas, during the ninety days that the latter would
assume the governorship;
Petitioner posits that the issuance by public respondent of the May 15, 1991 Resolution was "whimsical,
capricious and despotic, and constituted grave abuse of discretion amounting lack of jurisdiction," (p. 6, petition) 12. And, as if this was not enough, Ocampo even made announcements in the media that he was
basically on the ground th executive clemency could be granted by the President only in criminal cases as there allowing Petitioner Llamas to perform his functions as acting governor at the Office of the Govern at the
is nothing in the statute books or even in the Constitution which allows the grant thereof in administrative cases. Capitol where he (Ocampo) used to hold office (true enough Ocampo has subsequently allowed
Petitioner also contends that since respondent governor refused to recognize his suspension (having reassumed Llamas to hold office at the of the Governor, with Ocampo even escorting the acting therein last March
4, 1991);
PAGE 64 of 135 CONSTITUTIONAL LAW -EXECUTIVE DEPARTMENT PART 2 –SESSION 12
l 3. An account of Ocampo's acceptance of his suspension and of his having turned over his office to these determinations, together with the consequences that flow therefrom, may not be traversed in the
Petitioner Llamas was published, front page, in the March 5, 1991 issue of the Manila Bulletin. A copy courts. (Willoughby on the Constitution of the United States, Vol. 3, p. 1326).
of this news account is attached and made a part hereof as Annex C);
xxx xxx xxx
14. Furthermore, various other officials, President Aquino Rep. Jose Cojuangco included, have
extended recognition to Petitions Llamas' assumption of the governorship. Llamas met with President What is generally meant, when it is said that a question is political, and not judicial, is that it is a matter
Aquino and Rep. Cojuangco and, during this meeting, the two highest officials of the land have asked which is to be exercised by the people in their primary political capacity, or that it has been specifically
Llamas to discharge his duties acting governor; delegated to some other department or particular officer of the goverrunent, with discretionary power to
act. See State vs. Cunningham, 81 Wis. 497, 51 L.R.A. 561; In Re Gunn, 50 Fan. 155; 32 Pac. 470,
15. Secretary Santos, for that matter, has issued a designation to Tarlac Senior Board Member 948, 19 L. RA. 519; Green vs. Mills, 69 Fed. 852, 16, C. CA 516, 30 L.R.A- 90; Fletcher vs. Tuttle, 151
Aganon, dated March 18, 1991, a pointing bim as acting vice governor of the province, "in view of the 111, 41, 37 N.E. 683, 25 L.R.A. 143, 42 Am. St. Rep. 220. Thus the Legislature may in its discretion
suspension of Gov. Mariano Un Ocampo III, and the assumption Vice Governor Rodolfo Llamas as determine whether it will pass a law or submit a proposed constitutional amendment to the people. The
acting governor." A copy of this designation is attached and made a part hereof as Annex D; courts have no judicial control over such matters, not merely because they involve political question,
but because they are matters which the people have by the Constitute delegated to the Legislature.
xxx xxx xxx The Governor may exercise the powers delegated to him, free from judicial control, so long as he
observes the laws and acts within the limits of the power conferred. His discretionary acts cannot be
controllable, not primarily because they are of a political nature, but because the Constitution and laws
30. ... [T]he reassumption ceremony by [Governor] Ocampo was held [in the] morning of May 21, 1991 have placed the particular matter under his control. But every officer under a constitutional government
... (pp- 2-4 & 7, Petition; pp. 3-5 & 8, Rollo) must act according to law and subject him to the restraining and controlling power of the people, acting
through the courts, as well as through the executive or the Legislature. One department is just as
It is prayed in the instant petition dated May 21, 1991 that: representative as the other, and the judiciary the department which is charged with the special duty of
determinining the limitations which the law places upon all official action. The recognition of this
b. In the meantime that this action is pending, and irnmediately upon the filing hereof, a temporary principle, unknown except in Great Britain and America, is necessary, to "the end that the government
restraining order be issued stopping the Respondents from enforcing, in any manner, the aforesaid may be one of laws and not men" — words which Webster said were the greatest contained in any
contested resolution, and Respondent Ocampo, firom continuing with his reassumption of the written constitutional document.
governorship. IN THE ALTERNATIVE, that a cease and desist order be issued against Respondent
Ocampo stopping him from continuing with hiii reassumption of the governorship. Besides, under the 1987 Constitution, the Supreme Court has been conferred an "expanded jurisdiction" to
review the decisions of the other branches and agencies of the government to determine whether or not they
Let us first deal with the issue on jurisdiction. Respondent govemor avers that since under the Constitution fiffl have acted within the bounds of the Constitution (See Art. VIII, Sec. 1, Constitution). "Yet, in the exercise
discretionary authority is granted to the President on the exercise of executive clemency, the same constitutes a thereof, the Court is to merely check whether or not the govermental branch or agency has gone beyond the
political question which is beyond judicial review. constitutional limits of its jurisdiction, not that it erred or has a different view" (Co vs. Electoral Tribunal of the
House of Representatives & Ong, G.R. Nos. 92191-92 and Balanquit vs. Electoral Tribunal of the House of
Representatives & Ong, G.R Nos. 92202-03, July 30, 1991).
Such a rule does not hold true in the case at bar. While it is true that courts cannot inquire into the manner in
which the President's discretionary powers are exercised or into the wisdom for its exercise, it is also a settled
rule that when the issue involved concerns the validity of such discretionary powers or whether said powers are In the case at bar, the nature of the question for determination is not purely political. Here, we are called upon to
within the limits prescribed by the Constitution, We will not decline to exercise our power of judicial review. And decide whether under the Constitution the President may grant executive clemency in administrative cases. We
such review does not constitute a modification or correction of the act of the President, nor does it constitute must not overlook the fact that the exercise by the President of her power of executive clemency is subject to
interference with the functions of the President. In this connection, the case of Tanada and Macapagal vs. constitutional l'um'tations. We will merely check whether the particular measure in question has been in
Cuenco, et al., 103 Phil. 1051, is very enlightening, and We quote: accordance with law. In so doing, We will not concern ourselves with the reasons or motives which actuate the
President as such is clearly beyond our power of judicial review.

Elsewhere in this treatise the well-known and well-established principle is considered that it is not within
the province of the courts to pass judgment upon the policy of legislative or executive action. Where, Petitioner's main argument is that the President may grant executive clemency only in criminal cases, based on
therefore, discretionary powers are granted by the Consfitution or by statute, the manner in which those Article VII, Section 19 of the Constitution which reads:
powers are exercised is not subject to judicial review. The courts, therefore, concern themselves only
with the question as to the existence and extent of these discretionary powers. Sec. 19. Except in cases of impeachment, or as otherwise pro vided in this Constitution, the President
may grant reprieves, commu tations, and pardons, and remit fines and forfeitures, after conviction by
As distinguished from the judicial, the legislative and executive departments are spoken of as the final judgment.
political departments of government because in very many cases their action is necessarily dictated by
considerations of public or political policy. These considerations of public or political policy of course will He shall also have the power to grant amnesty with the concurrence of a majority of all the members of
not permit the legislature to violate constitutional provisions, or the executive to exercise authority not the Congress. (Emphasis supplied)
granted him by the Constitution or by statute, but, within these limits, they do permit the departments,
separately or together, to recognize that a certain set of facts exists or that a given status exists, and According to the petitioner, the qualifying phrase "after conviction by final judgment" applies solely to criminal
cases, and no other law allows the grant of executive clemency or pardon to anyone who has been "convicted in
PAGE 65 of 135 CONSTITUTIONAL LAW -EXECUTIVE DEPARTMENT PART 2 –SESSION 12
an administrative case," allegedly because the word "conviction" refers only to criminal cases (par. 22-b, c, d, granted by the President without the favorable recommendation of the COMELEC" (Article IX, C, Section 5,
Petition). Petitioner, however, describes in his very own words, respondent governor as one who has been Constitution). If those already adjudged guilty criminally in court may be pardoned, those adjudged guilty
"convicted in an administrative case" (par. 22-a, petition). Thus, petitioner concedes that the word "conviction" administratively should likewise be extended the same benefit.
may be used either in a criminal case or in an administrative case. In Layno, Sr. vs. Sandiganbayan, 136 SCRA
536, We ruled: In criminal cases, the quantum of evidence required to convict an individual is proof beyond reasonable doubt,
but the Constitution grants to the President the power to pardon the act done by the proved criminal and in the
For misfeasance or malfeasance ... any [elective official] could ... be proceeded against administratively process exempts him from punishment therefor. On the other hand, in administrative cases, the quantum of
or ... criminally. In either case, his culpability must be established ... evidence required is mere substantial evidence to support a decision, not to mention that as to the admissibility
of evidence, administrative bodies are not bound by the technical and rigid rules of admissibility prescribed in
It is also important to note that respondent govemor's Motion for Reconsideration filed on March 1, 1991 was criminal cases. It will therefore be unjust and unfair for those found guilty administratively of some charge if the
withdrawn in his petition for the grant of executive clemency, which fact rendered the Resolution dated February same effects of pardon or executive clemency cannot be extended to them, even in the sense of modifying a
26, 1991 affirming the DLG Decision (which found respondent governor guilty of neglect of duty and/or abuse of decision to subserve the interest of the public. (p. 34, Comment of public respondent)
authority and which suspended him for ninety (90) days) final.
Of equal importance are the following provisions of Executive Order No. 292, otherwise known as the
Moreover, applying the doctrine "Ubi lex non distinguit, nec nos distinguire debemos," We cannot sustain Administrative Code of 1987, Section I, Book III of which provides:
petitioner's view. In other words, if the law does not distinguish, so We must no distinguish. The Constitution
does not distinguish between which cases executive clemency may be exercised by the President, with the sole SECTION 1. Power of Control. — The President shall have control of all the executive departments,
exclusion of impeachment cases. By the same token, if executive clemency may be exercised only in criminal bureaus, and offices. He shall ensure that the laws be faithfully executed.
cases, it would indeed be unnecessary to provide for the exclusion of impeachment cases from the coverage of
Article VII, Section 19 of the Constitution. Following petitioner's proposed interpretation, cases of impeachment SECTION 38. Definition of Administrative Relationships. — Unless otherwise expressly stated in the
are automatically excluded inasmuch as the same do not necessarily involve criminal offenses. Code or in other laws defining the special relationships of particular agencies, administrative
relationships shall be categorized and defined as follows:
In the same vein, We do not clearly see any valid and convincing reason why the President cannot grant
executive clemency in administrative cases. It is Our considered view that if the President can grant reprieves, (1) Supervision and Control. — Supervision and control shall include authority to act directly whenever a specific
commutations and pardons, and remit fines and forfeitures in criminal cases, with much more reason can she function is entrusted by law or regulation to a subordinate; direct the performance of duty; restrain the
grant executive clemency in administrative cases, which are clearly less serious than criminal offenses. commission of acts; review, approve, reverse or modify acts and decisions of subordinate officials or units;
determine priorities in the execution of plans and programs. Unless a different meaning is explicitly provided in
A number of laws impliedly or expressly recognize or support the exercise of the executive clemency in the specific law governing the relationship of particular agencies the word "control" shall encompass supervision
administrative cases. and control as defined in this paragraph. ... (emphasis supplied)

Under Sec. 43 of P.D. 807, "In meritorious cases, ..., the President may commute or remove administrative The disciplinary authority to investigate, suspend, and remove provincial or city officials devolves at the first
penalties or disabilities issued upon officers and employees, in disciplinary cases, subject to such terms and instance on the Department of Interior and Local Government (Secs. 61 and 65, B.P. Blg. 337) and ultimately on
conditions as he may impose in the interest of the service." the President (Sec. 66). Implicit in this authority, however, is the "supervision and control" power of the President
to reduce, if circumstances so warrant, the imposable penalty or to modify the suspension or removal order,
During the deliberations of the Constitutional Commission, a subject of deliberations was the proposed even "in the sense" of granting executive clemency. "Control," within the meaning of the Constitution, is the
amendment to Art. VII, Sec. 19 which reads as follows: "However, the power to grant executive clemency for power to substitute one's own judgment for that of a subordinate. Under the doctrine of Qualified Political
violation of corrupt practices laws may be limited by legislation."The Constitutional Commission, however, voted Agency, the different executive departments are mere adjuncts of the President. Their acts are presumptively
to remove the amendment, since it was in derogation of the powers of the President. As Mr. Natividad stated: the acts of the President until countermanded or reprobated by her (Vinena v. Secretary, 67 Phil. 451; Free
Telephone Workers Union vs. Minister of Labor and Employment, 108 SCRA 767 [1981]). Replying upon this
view, it is urged by the Solicitor General that in the present case, the President, in the exercise of her power of
I am also against this provision which will again chip more powers from the President. In case of other supervision and control over all executive departments, may substitute her decision for that of her subordinate,
criminals convicted in our society we extend probation to them while in this case, they have already most especially where the basis therefor would be to serve the greater public interest. It is clearly within the
been convicted and we offer mercy. The only way we can offer mercy to them is through this executive power of the President not only to grant "executive clemency" but also to reverse or modify a ruling issued by a
clemency extended to them by the President. If we still close this avenue to them, they would be subordinate against an erring public official, where a reconsideration of the facts alleged would support the
prejudiced even worse than the murderers and the more vicious killers in our society .... same. It is in this sense that the alleged executive clemency was granted, after adducing reasons that subserve
the public interest. — "the relative success of . . . livelihood loan program." (pp. 39-40, Comment of public
The proposal was primarily intended to prevent the President from protecting his cronies. Manifestly, however, respondent)
the Commission preferred to trust in the discretion of Presidents and refrained from putting additional limitations
on his clemency powers. (II RECORD of the Constitutional Commission, 392, 418-419, 524-525) We wish to stress however that when we say the President can grant executive clemency in administrative
cases, We refer only to all administrative cases in the Executive branch, not in the Judicial or Legislative
It is evident from the intent of the Constitutional Commission, therefore, that the President's executive clemency branches of the government.
powers may not be limited in terms of coverage, except as already provided in the Constitution, that is, "no
pardon, amnesty, parole, or suspension of sentence for violation of election laws, rules and regulations shall be
PAGE 66 of 135 CONSTITUTIONAL LAW -EXECUTIVE DEPARTMENT PART 2 –SESSION 12
Noteworthy is the fact that on March 1, 1991, respondent governor filed a motion for reconsideration and the Separate Opinions
same may be regarded as implicitly resolved, not only because of its withdrawal but also because of the
executive clemency which in effect reduced the penalty, conformably with the power of "control."

On petitioner's argument that private respondent's motion for reconsideration has abated the running of the CRUZ, J., dissenting:
reglementary period for finality of judgment in O.P. Case No. 4480 (that is, there being no final judgment to
speak of, the pardon granted was premature and of no effect, We reiterate the doctrine that upon acceptance of
a presidential pardon, the grantee is deemed to have waived any appeal which he may have filed. Thus, it was I concur in the result and would s the challenged resolution of May 18, 1991, on the basis only of the Fresident's
held that: control power. I think the discussion of the pardoning power is unnecessary and may even be misleading as
theponencia itself says that it was not by virtue thereof that the private respondent's penalty was reduced. The
correct approach, if I may spectfully suggest it, is to uphold the resolution solely on strength of the President's
The commutation of the penalty is impressed with legal significance. That is an exercise of executive power of "control of all the executive departments, bureaus and offices" under Article VII, Sect 17, of the
clemency embraced in the pardoning power. According to the Constitution: "The President may except Constitution.
in cases of impeachment, grant reprieves, commutations and pardons, remit fines and forfeitures and,
with the concurrence of the Batasang Pambansa, grant amnesty. "Once granted, it is binding and
effective. It serves to put an end to this appeal." (Mansanto v. Factoran, Jr., G.R. No. 78239, 170 SCRA We have held in many cases that a Cabinet member is an alter ego of the President whose acts may be
190. 196). (See also Peo v. Crisola, 129 SCRA 13) affirmed, modified reversed by the latter in his discretion. (Villena v. Sec. of Interior, 67 Phil. 451; Lacson-
Magallanes v. Paño, 21 SC 895; Gascon v. Arroyo, 178 SCRA 582; De Leon v. Carpio, 1 SCRA 457). What
happened in this case was that President Aquino saw fit to amend the decision rendered by the Secretary of
Consequently, respondent governor's acceptance of the presidential pardon "serves to put an end" to the motion Local Government on September 21, 1990, by reducing 90-day suspension imposed on Gov. Ocampo. The
for reconsideration and renders the subject decision final, that of the period already served. President had the authority to do this, and she could exercise it through Executive Secretary. His act, not having
been "reprobated disauthorized" by her, is presumed to be the act of the Preside herself.
Finally, petitioner's argument that his constitutional rights to due process were violated is uruneritorious. Pardon
has been defined as "the private, though official, act of the executive magistrate, delivered to the individual for The Court is not concerned with the wisdom of that act, on its legality. I believe the act is legal but reserve
whose benefit it is intended and not communicated officially to the court. ..." (Bernas, The Constitution of the judgment on its wisdom.
Philippines, Vol. II, First Ed. 1988, pp. 239-240, citing U.S. v. Wilson, 7 Pet. 150 [U.S. 1833]). Thus, assuming
that petitioner was not notified of the subject pardon, it is only because said notice is unnecessary. Besides,
petitioner's claim that respondent governor has not begun serve sentence is belied by his very own factual
allegations in his petition, more particularly that he served as Acting Governor of Tarlac effective from the date
he took his Oath of Office on February 28, 1991 up to the time respondent govemor reassumed the
governorship of Tarlac on May 21, 1991 (par. 30 petition). It is, therefore, error to say that private respondent did
not serve any portion of the 90-day suspension meted upon him.

We fail to see any grave abuse of discretion amounting to lack or in excess of jurisdiction committed by public
respondent.

WHEREFORE, judgment is hereby rendered: (1) DECLARING that the President did not act arbitrarily or with
abuse, much less grave abuse of discretion in issuing the May 15, 1991 Resolution granting on the grounds
mentioned therein, executive clemency to respondent governor and that, accordingly, the same is not
unconstitutional (without prejudice to criminal proceedings which have been filed or may be filed against
respondent governor), and (2) DENYING the rest of the prayers in the petition for being unmeritorious, moot and
academic. No costs.

SO ORDERED.

Fernan, C.J., Bidin, Sarmiento, Griño-Aquino, Medialdea, Regalado and Davide, Jr., JJ., concur.
Melencio-Herrera, J., is on leave.
PAGE 67 of 135 CONSTITUTIONAL LAW -EXECUTIVE DEPARTMENT PART 2 –SESSION 12
EN BANC in turn, referred the VFA to its Committee on Foreign Relations, chaired by Senator Blas F. Ople, and its Committee
on National Defense and Security, chaired by Senator Rodolfo G. Biazon, for their joint consideration and
recommendation. Thereafter, joint public hearings were held by the two Committees. [7]
On May 3, 1999, the Committees submitted Proposed Senate Resolution No. 443 [8] recommending the
[G.R. No. 138570. October 10, 2000] concurrence of the Senate to the VFA and the creation of a Legislative Oversight Committee to oversee its
implementation. Debates then ensued.
On May 27, 1999, Proposed Senate Resolution No. 443 was approved by the Senate, by a two-thirds (2/3)
vote[9] of its members. Senate Resolution No. 443 was then re-numbered as Senate Resolution No. 18.[10]
BAYAN (Bagong Alyansang Makabayan), a JUNK VFA MOVEMENT, BISHOP TOMAS MILLAMENA (Iglesia
Filipina Independiente), BISHOP ELMER BOLOCAN (United Church of Christ of the Phil.), DR. On June 1, 1999, the VFA officially entered into force after an Exchange of Notes between respondent
REYNALDO LEGASCA, MD, KILUSANG MAMBUBUKID NG PILIPINAS, KILUSANG MAYO UNO, Secretary Siazon and United States Ambassador Hubbard.
GABRIELA, PROLABOR, and the PUBLIC INTEREST LAW CENTER, petitioners, vs. EXECUTIVE
SECRETARY RONALDO ZAMORA, FOREIGN AFFAIRS SECRETARY DOMINGO SIAZON, The VFA, which consists of a Preamble and nine (9) Articles, provides for the mechanism for regulating the
DEFENSE SECRETARY ORLANDO MERCADO, BRIG. GEN. ALEXANDER AGUIRRE, SENATE circumstances and conditions under which US Armed Forces and defense personnel may be present in the
PRESIDENT MARCELO FERNAN, SENATOR FRANKLIN DRILON, SENATOR BLAS OPLE, Philippines, and is quoted in its full text, hereunder:
SENATOR RODOLFO BIAZON, and SENATOR FRANCISCO TATAD, respondents.
Article I
DECISION Definitions

BUENA, J.:
As used in this Agreement, United States personnel means United States military and civilian personnel
temporarily in the Philippines in connection with activities approved by the Philippine Government.
Confronting the Court for resolution in the instant consolidated petitions for certiorari and prohibition are
issues relating to, and borne by, an agreement forged in the turn of the last century between the Republic of the Within this definition:
Philippines and the United States of America -the Visiting Forces Agreement.
The antecedents unfold. 1. The term military personnel refers to military members of the United States Army, Navy, Marine
Corps, Air Force, and Coast Guard.
On March 14, 1947, the Philippines and the United States of America forged a Military Bases Agreement
which formalized, among others, the use of installations in the Philippine territory by United States military 2. The term civilian personnel refers to individuals who are neither nationals of, nor ordinary residents
personnel. To further strengthen their defense and security relationship, the Philippines and the United States in the Philippines and who are employed by the United States armed forces or who are
entered into a Mutual Defense Treaty on August 30, 1951. Under the treaty, the parties agreed to respond to any accompanying the United States armed forces, such as employees of the American Red Cross and
external armed attack on their territory, armed forces, public vessels, and aircraft. [1] the United Services Organization.
In view of the impending expiration of the RP-US Military Bases Agreement in 1991, the Philippines and the
United States negotiated for a possible extension of the military bases agreement. On September 16, 1991, the Article II
Philippine Senate rejected the proposed RP-US Treaty of Friendship, Cooperation and Security which, in effect, Respect for Law
would have extended the presence of US military bases in the Philippines. [2] With the expiration of the RP-US
Military Bases Agreement, the periodic military exercises conducted between the two countries were held in It is the duty of the United States personnel to respect the laws of the Republic of the Philippines and to
abeyance. Notwithstanding, the defense and security relationship between the Philippines and the United States abstain from any activity inconsistent with the spirit of this agreement, and, in particular, from any political
of America continued pursuant to the Mutual Defense Treaty. activity in the Philippines. The Government of the United States shall take all measures within its authority
to ensure that this is done.
On July 18, 1997, the United States panel, headed by US Defense Deputy Assistant Secretary for Asia
Pacific Kurt Campbell, met with the Philippine panel, headed by Foreign Affairs Undersecretary Rodolfo Severino
Jr., to exchange notes on the complementing strategic interests of the United States and the Philippines in the Article III
Asia-Pacific region. Both sides discussed, among other things, the possible elements of the Visiting Forces Entry and Departure
Agreement (VFA for brevity). Negotiations by both panels on the VFA led to a consolidated draft text, which in turn
resulted to a final series of conferences and negotiations[3]that culminated in Manila on January 12 and 13, 1998. 1. The Government of the Philippines shall facilitate the admission of United States personnel and their
Thereafter, then President Fidel V. Ramos approved the VFA, which was respectively signed by public respondent departure from the Philippines in connection with activities covered by this agreement.
Secretary Siazon and Unites States Ambassador Thomas Hubbard on February 10, 1998.
On October 5, 1998, President Joseph E. Estrada, through respondent Secretary of Foreign Affairs, ratified 2. United States military personnel shall be exempt from passport and visa regulations upon entering and
the VFA.[4] departing the Philippines.

On October 6, 1998, the President, acting through respondent Executive Secretary Ronaldo Zamora, 3. The following documents only, which shall be presented on demand, shall be required in respect of
officially transmitted to the Senate of the Philippines, [5] the Instrument of Ratification, the letter of the United States military personnel who enter the Philippines:
President[6] and the VFA, for concurrence pursuant to Section 21, Article VII of the 1987 Constitution. The Senate,
PAGE 68 of 135 CONSTITUTIONAL LAW -EXECUTIVE DEPARTMENT PART 2 –SESSION 12
(a) personal identity card issued by the appropriate United States authority showing full name, date of (1) treason;
birth, rank or grade and service number (if any), branch of service and photograph;
(2) sabotage, espionage or violation of any law relating to national defense.
(b) individual or collective document issued by the appropriate United States authority, authorizing the
travel or visit and identifying the individual or group as United States military personnel; and 3. In cases where the right to exercise jurisdiction is concurrent, the following rules shall apply:

(c) the commanding officer of a military aircraft or vessel shall present a declaration of health, and (a) Philippine authorities shall have the primary right to exercise jurisdiction over all offenses
when required by the cognizant representative of the Government of the Philippines, shall committed by United States personnel, except in cases provided for in paragraphs 1(b), 2 (b),
conduct a quarantine inspection and will certify that the aircraft or vessel is free from and 3 (b) of this Article.
quarantinable diseases. Any quarantine inspection of United States aircraft or United States (b) United States military authorities shall have the primary right to exercise jurisdiction over United
vessels or cargoes thereon shall be conducted by the United States commanding officer in States personnel subject to the military law of the United States in relation to.
accordance with the international health regulations as promulgated by the World Health
Organization, and mutually agreed procedures. (1) offenses solely against the property or security of the United States or offenses solely against
the property or person of United States personnel; and
4. United States civilian personnel shall be exempt from visa requirements but shall present, upon demand,
(2) offenses arising out of any act or omission done in performance of official duty.
valid passports upon entry and departure of the Philippines.
(c) The authorities of either government may request the authorities of the other government to
5. If the Government of the Philippines has requested the removal of any United States personnel from its waive their primary right to exercise jurisdiction in a particular case.
territory, the United States authorities shall be responsible for receiving the person concerned within
(d) Recognizing the responsibility of the United States military authorities to maintain good order
its own territory or otherwise disposing of said person outside of the Philippines.
and discipline among their forces, Philippine authorities will, upon request by the United States,
waive their primary right to exercise jurisdiction except in cases of particular importance to the
Article IV Philippines. If the Government of the Philippines determines that the case is of particular
Driving and Vehicle Registration importance, it shall communicate such determination to the United States authorities within
twenty (20) days after the Philippine authorities receive the United States request.
1. Philippine authorities shall accept as valid, without test or fee, a driving permit or license issued by the
(e) When the United States military commander determines that an offense charged by authorities
appropriate United States authority to United States personnel for the operation of military or official
of the Philippines against United states personnel arises out of an act or omission done in the
vehicles.
performance of official duty, the commander will issue a certificate setting forth such
determination. This certificate will be transmitted to the appropriate authorities of the Philippines
2. Vehicles owned by the Government of the United States need not be registered, but shall have and will constitute sufficient proof of performance of official duty for the purposes of paragraph
appropriate markings. 3(b)(2) of this Article. In those cases where the Government of the Philippines believes the
circumstances of the case require a review of the duty certificate, United States military
Article V authorities and Philippine authorities shall consult immediately. Philippine authorities at the
Criminal Jurisdiction highest levels may also present any information bearing on its validity. United States military
authorities shall take full account of the Philippine position. Where appropriate, United States
military authorities will take disciplinary or other action against offenders in official duty cases,
1. Subject to the provisions of this article: and notify the Government of the Philippines of the actions taken.

(a) Philippine authorities shall have jurisdiction over United States personnel with respect to (f) If the government having the primary right does not exercise jurisdiction, it shall notify the
offenses committed within the Philippines and punishable under the law of the Philippines. authorities of the other government as soon as possible.

(b) United States military authorities shall have the right to exercise within the Philippines all criminal (g) The authorities of the Philippines and the United States shall notify each other of the disposition
and disciplinary jurisdiction conferred on them by the military law of the United States over of all cases in which both the authorities of the Philippines and the United States have the right
United States personnel in the Philippines. to exercise jurisdiction.

2. (a) Philippine authorities exercise exclusive jurisdiction over United States personnel with respect 4. Within the scope of their legal competence, the authorities of the Philippines and United States shall
to offenses, including offenses relating to the security of the Philippines, punishable under assist each other in the arrest of United States personnel in the Philippines and in handling them
the laws of the Philippines, but not under the laws of the United States. over to authorities who are to exercise jurisdiction in accordance with the provisions of this article.

(b) United States authorities exercise exclusive jurisdiction over United States personnel with 5. United States military authorities shall promptly notify Philippine authorities of the arrest or detention
respect to offenses, including offenses relating to the security of the United States, of United States personnel who are subject of Philippine primary or exclusive jurisdiction. Philippine
punishable under the laws of the United States, but not under the laws of the Philippines. authorities shall promptly notify United States military authorities of the arrest or detention of any
United States personnel.
(c) For the purposes of this paragraph and paragraph 3 of this article, an offense relating to
security means: 6. The custody of any United States personnel over whom the Philippines is to exercise jurisdiction
shall immediately reside with United States military authorities, if they so request, from the
PAGE 69 of 135 CONSTITUTIONAL LAW -EXECUTIVE DEPARTMENT PART 2 –SESSION 12
commission of the offense until completion of all judicial proceedings. United States military 1. Except for contractual arrangements, including United States foreign military sales letters of offer
authorities shall, upon formal notification by the Philippine authorities and without delay, make such and acceptance and leases of military equipment, both governments waive any and all claims
personnel available to those authorities in time for any investigative or judicial proceedings relating against each other for damage, loss or destruction to property of each others armed forces or for
to the offense with which the person has been charged in extraordinary cases, the Philippine death or injury to their military and civilian personnel arising from activities to which this agreement
Government shall present its position to the United States Government regarding custody, which applies.
the United States Government shall take into full account. In the event Philippine judicial
proceedings are not completed within one year, the United States shall be relieved of any 2. For claims against the United States, other than contractual claims and those to which paragraph 1
obligations under this paragraph. The one-year period will not include the time necessary to appeal. applies, the United States Government, in accordance with United States law regarding foreign
Also, the one-year period will not include any time during which scheduled trial procedures are claims, will pay just and reasonable compensation in settlement of meritorious claims for damage,
delayed because United States authorities, after timely notification by Philippine authorities to loss, personal injury or death, caused by acts or omissions of United States personnel, or otherwise
arrange for the presence of the accused, fail to do so. incident to the non-combat activities of the United States forces.

7. Within the scope of their legal authority, United States and Philippine authorities shall assist each Article VII
other in the carrying out of all necessary investigation into offenses and shall cooperate in providing Importation and Exportation
for the attendance of witnesses and in the collection and production of evidence, including seizure
and, in proper cases, the delivery of objects connected with an offense.
1. United States Government equipment, materials, supplies, and other property imported into or
8. When United States personnel have been tried in accordance with the provisions of this Article and acquired in the Philippines by or on behalf of the United States armed forces in connection with
have been acquitted or have been convicted and are serving, or have served their sentence, or activities to which this agreement applies, shall be free of all Philippine duties, taxes and other
have had their sentence remitted or suspended, or have been pardoned, they may not be tried similar charges. Title to such property shall remain with the United States, which may remove such
again for the same offense in the Philippines. Nothing in this paragraph, however, shall prevent property from the Philippines at any time, free from export duties, taxes, and other similar charges.
United States military authorities from trying United States personnel for any violation of rules of The exemptions provided in this paragraph shall also extend to any duty, tax, or other similar
discipline arising from the act or omission which constituted an offense for which they were tried by charges which would otherwise be assessed upon such property after importation into, or
Philippine authorities. acquisition within, the Philippines. Such property may be removed from the Philippines, or disposed
of therein, provided that disposition of such property in the Philippines to persons or entities not
9. When United States personnel are detained, taken into custody, or prosecuted by Philippine entitled to exemption from applicable taxes and duties shall be subject to payment of such taxes,
authorities, they shall be accorded all procedural safeguards established by the law of the and duties and prior approval of the Philippine Government.
Philippines. At the minimum, United States personnel shall be entitled:
2. Reasonable quantities of personal baggage, personal effects, and other property for the personal
(a) To a prompt and speedy trial; use of United States personnel may be imported into and used in the Philippines free of all duties,
(b) To be informed in advance of trial of the specific charge or charges made against them and to taxes and other similar charges during the period of their temporary stay in the Philippines.
have reasonable time to prepare a defense; Transfers to persons or entities in the Philippines not entitled to import privileges may only be made
upon prior approval of the appropriate Philippine authorities including payment by the recipient of
(c) To be confronted with witnesses against them and to cross examine such witnesses; applicable duties and taxes imposed in accordance with the laws of the Philippines. The exportation
of such property and of property acquired in the Philippines by United States personnel shall be
(d) To present evidence in their defense and to have compulsory process for obtaining witnesses; free of all Philippine duties, taxes, and other similar charges.
(e) To have free and assisted legal representation of their own choice on the same basis as
nationals of the Philippines; Article VIII
Movement of Vessels and Aircraft
(f) To have the service of a competent interpreter; and
(g) To communicate promptly with and to be visited regularly by United States authorities, and to 1. Aircraft operated by or for the United States armed forces may enter the Philippines upon approval
have such authorities present at all judicial proceedings. These proceedings shall be public of the Government of the Philippines in accordance with procedures stipulated in implementing
unless the court, in accordance with Philippine laws, excludes persons who have no role in the arrangements.
proceedings.
2. Vessels operated by or for the United States armed forces may enter the Philippines upon approval
10. The confinement or detention by Philippine authorities of United States personnel shall be carried of the Government of the Philippines. The movement of vessels shall be in accordance with
out in facilities agreed on by appropriate Philippine and United States authorities. United States international custom and practice governing such vessels, and such agreed implementing
Personnel serving sentences in the Philippines shall have the right to visits and material assistance. arrangements as necessary.

11. United States personnel shall be subject to trial only in Philippine courts of ordinary jurisdiction, and 3. Vehicles, vessels, and aircraft operated by or for the United States armed forces shall not be subject
shall not be subject to the jurisdiction of Philippine military or religious courts. to the payment of landing or port fees, navigation or over flight charges, or tolls or other use charges,
including light and harbor dues, while in the Philippines. Aircraft operated by or for the United States
armed forces shall observe local air traffic control regulations while in the Philippines. Vessels
Article VI
owned or operated by the United States solely on United States Government non-commercial
Claims
service shall not be subject to compulsory pilotage at Philippine ports.
PAGE 70 of 135 CONSTITUTIONAL LAW -EXECUTIVE DEPARTMENT PART 2 –SESSION 12
Article IX injury as a result of the operation of the VFA.[12] Petitioners, on the other hand, counter that the validity or invalidity
Duration and Termination of the VFA is a matter of transcendental importance which justifies their standing. [13]
A party bringing a suit challenging the constitutionality of a law, act, or statute must show not only that the
This agreement shall enter into force on the date on which the parties have notified each other in writing law is invalid, but also that he has sustained or in is in immediate, or imminent danger of sustaining some direct
through the diplomatic channel that they have completed their constitutional requirements for entry into injury as a result of its enforcement, and not merely that he suffers thereby in some indefinite way. He must show
force. This agreement shall remain in force until the expiration of 180 days from the date on which either that he has been, or is about to be, denied some right or privilege to which he is lawfully entitled, or that he is
party gives the other party notice in writing that it desires to terminate the agreement. about to be subjected to some burdens or penalties by reason of the statute complained of.[14]

Via these consolidated[11] petitions for certiorari and prohibition, petitioners - as legislators, non-governmental In the case before us, petitioners failed to show, to the satisfaction of this Court, that they have sustained, or
organizations, citizens and taxpayers - assail the constitutionality of the VFA and impute to herein respondents are in danger of sustaining any direct injury as a result of the enforcement of the VFA. As taxpayers, petitioners
grave abuse of discretion in ratifying the agreement. have not established that the VFA involves the exercise by Congress of its taxing or spending powers. [15] On this
point, it bears stressing that a taxpayers suit refers to a case where the act complained of directly involves the
We have simplified the issues raised by the petitioners into the following: illegal disbursement of public funds derived from taxation.[16] Thus, in Bugnay Const. & Development Corp. vs.
Laron[17], we held:
I
x x x it is exigent that the taxpayer-plaintiff sufficiently show that he would be benefited or injured by the
Do petitioners have legal standing as concerned citizens, taxpayers, or legislators to question the judgment or entitled to the avails of the suit as a real party in interest. Before he can invoke the power of judicial
constitutionality of the VFA? review, he must specifically prove that he has sufficient interest in preventing the illegal expenditure of money
raised by taxation and that he will sustain a direct injury as a result of the enforcement of the questioned statute
II or contract.It is not sufficient that he has merely a general interest common to all members of the public.

Is the VFA governed by the provisions of Section 21, Article VII or of Section 25, Article XVIII of the Clearly, inasmuch as no public funds raised by taxation are involved in this case, and in the absence of any
Constitution? allegation by petitioners that public funds are being misspent or illegally expended, petitioners, as taxpayers, have
no legal standing to assail the legality of the VFA.
III Similarly, Representatives Wigberto Taada, Agapito Aquino and Joker Arroyo, as petitioners-legislators, do
not possess the requisite locus standi to maintain the present suit. While this Court, in Phil. Constitution
Does the VFA constitute an abdication of Philippine sovereignty? Association vs. Hon. Salvador Enriquez, [18] sustained the legal standing of a member of the Senate and the
House of Representatives to question the validity of a presidential veto or a condition imposed on an item in an
a. Are Philippine courts deprived of their jurisdiction to hear and try offenses committed by US military appropriation bull, we cannot, at this instance, similarly uphold petitioners standing as members of Congress, in
personnel? the absence of a clear showing of any direct injury to their person or to the institution to which they belong.

b. Is the Supreme Court deprived of its jurisdiction over offenses punishable by reclusion perpetua or Beyond this, the allegations of impairment of legislative power, such as the delegation of the power of
higher? Congress to grant tax exemptions, are more apparent than real. While it may be true that petitioners pointed to
provisions of the VFA which allegedly impair their legislative powers, petitioners failed however to sufficiently show
IV that they have in fact suffered direct injury.
In the same vein, petitioner Integrated Bar of the Philippines (IBP) is stripped of standing in these cases. As
Does the VFA violate: aptly observed by the Solicitor General, the IBP lacks the legal capacity to bring this suit in the absence of a board
resolution from its Board of Governors authorizing its National President to commence the present action.[19]
a. the equal protection clause under Section 1, Article III of the Constitution?
Notwithstanding, in view of the paramount importance and the constitutional significance of the issues raised
b. the Prohibition against nuclear weapons under Article II, Section 8? in the petitions, this Court, in the exercise of its sound discretion, brushes aside the procedural barrier and takes
cognizance of the petitions, as we have done in the early Emergency Powers Cases,[20] where we had occasion
c. Section 28 (4), Article VI of the Constitution granting the exemption from taxes and duties for the to rule:
equipment, materials supplies and other properties imported into or acquired in the Philippines by,
or on behalf, of the US Armed Forces?
x x x ordinary citizens and taxpayers were allowed to question the constitutionality of several executive orders
issued by President Quirino although they were involving only an indirect and general interest shared in common
with the public. The Court dismissed the objection that they were not proper parties and ruled
LOCUS STANDI that transcendental importance to the public of these cases demands that they be settled promptly and
definitely, brushing aside, if we must, technicalities of procedure. We have since then applied the
exception in many other cases. (Association of Small Landowners in the Philippines, Inc. v. Sec. of Agrarian
At the outset, respondents challenge petitioners standing to sue, on the ground that the latter have not shown Reform, 175 SCRA 343). (Underscoring Supplied)
any interest in the case, and that petitioners failed to substantiate that they have sustained, or will sustain direct
PAGE 71 of 135 CONSTITUTIONAL LAW -EXECUTIVE DEPARTMENT PART 2 –SESSION 12
This principle was reiterated in the subsequent cases of Gonzales vs. COMELEC,[21] Daza vs. a majority of the votes cast in a national referendum held for that purpose if so required by Congress, and
Singson,[22] and Basco vs. Phil. Amusement and Gaming Corporation, [23] where we emphatically held: recognized as such by the other contracting state.
It is our considered view that both constitutional provisions, far from contradicting each other, actually share
Considering however the importance to the public of the case at bar, and in keeping with the Courts duty, under some common ground. These constitutional provisions both embody phrases in the negative and thus, are
the 1987 Constitution, to determine whether or not the other branches of the government have kept themselves deemed prohibitory in mandate and character. In particular, Section 21 opens with the clause No treaty x x x, and
within the limits of the Constitution and the laws and that they have not abused the discretion given to them, the Section 25 contains the phrase shall not be allowed. Additionally, in both instances, the concurrence of the Senate
Court has brushed aside technicalities of procedure and has taken cognizance of this petition. x x x is indispensable to render the treaty or international agreement valid and effective.

Again, in the more recent case of Kilosbayan vs. Guingona, Jr.,[24] thisCourt ruled that in cases of To our mind, the fact that the President referred the VFA to the Senate under Section 21, Article VII, and that
transcendental importance, the Court may relax the standing requirements and allow a suit to prosper even the Senate extended its concurrence under the same provision, is immaterial. For in either case, whether under
where there is no direct injury to the party claiming the right of judicial review. Section 21, Article VII or Section 25, Article XVIII, the fundamental law is crystalline that the concurrence of the
Senate is mandatory to comply with the strict constitutional requirements.
Although courts generally avoid having to decide a constitutional question based on the doctrine of
separation of powers, which enjoins upon the departments of the government a becoming respect for each others On the whole, the VFA is an agreement which defines the treatment of United States troops and personnel
acts,[25] this Court nevertheless resolves to take cognizance of the instant petitions. visiting the Philippines. It provides for the guidelines to govern such visits of military personnel, and further defines
the rights of the United States and the Philippine government in the matter of criminal jurisdiction, movement of
vessel and aircraft, importation and exportation of equipment, materials and supplies.

APPLICABLE CONSTITUTIONAL PROVISION Undoubtedly, Section 25, Article XVIII, which specifically deals with treaties involving foreign military bases,
troops, or facilities, should apply in the instant case. To a certain extent and in a limited sense, however, the
provisions of section 21, Article VII will find applicability with regard to the issue and for the sole purpose of
determining the number of votes required to obtain the valid concurrence of the Senate, as will be further discussed
One focal point of inquiry in this controversy is the determination of which provision of the Constitution
hereunder.
applies, with regard to the exercise by the senate of its constitutional power to concur with the VFA.Petitioners
argue that Section 25, Article XVIII is applicable considering that the VFA has for its subject the presence of foreign It is a finely-imbedded principle in statutory construction that a special provision or law prevails over a general
military troops in the Philippines. Respondents, on the contrary, maintain that Section 21, Article VII should apply one. Lex specialis derogat generali. Thus, where there is in the same statute a particular enactment and also a
inasmuch as the VFA is not a basing arrangement but an agreement which involves merely the temporary visits general one which, in its most comprehensive sense, would include what is embraced in the former, the particular
of United States personnel engaged in joint military exercises. enactment must be operative, and the general enactment must be taken to affect only such cases within its general
language which are not within the provision of the particular enactment. [26]
The 1987 Philippine Constitution contains two provisions requiring the concurrence of the Senate on treaties
or international agreements. Section 21, Article VII, which herein respondents invoke, reads: In Leveriza vs. Intermediate Appellate Court,[27] we enunciated:

No treaty or international agreement shall be valid and effective unless concurred in by at least two-thirds of all x x x that another basic principle of statutory construction mandates that general legislation must give way to a
the Members of the Senate. special legislation on the same subject, and generally be so interpreted as to embrace only cases in which the
special provisions are not applicable (Sto. Domingo vs. de los Angeles, 96 SCRA 139), that a specific statute
Section 25, Article XVIII, provides: prevails over a general statute (De Jesus vs. People, 120 SCRA 760) and that where two statutes are of equal
theoretical application to a particular case, the one designed therefor specially should prevail (Wil Wilhensen Inc.
vs. Baluyot, 83 SCRA 38).
After the expiration in 1991 of the Agreement between the Republic of the Philippines and the United States of
America concerning Military Bases, foreign military bases, troops, or facilities shall not be allowed in the
Philippines except under a treaty duly concurred in by the senate and, when the Congress so requires, ratified Moreover, it is specious to argue that Section 25, Article XVIII is inapplicable to mere transient agreements
by a majority of the votes cast by the people in a national referendum held for that purpose, and recognized as a for the reason that there is no permanent placing of structure for the establishment of a military base. On this
treaty by the other contracting State. score, the Constitution makes no distinction between transient and permanent. Certainly, we find nothing in
Section 25, Article XVIII that requires foreign troops or facilities to be stationed or placed permanently in the
Philippines.
Section 21, Article VII deals with treatise or international agreements in general, in which case, the
concurrence of at least two-thirds (2/3) of all the Members of the Senate is required to make the subject treaty, or It is a rudiment in legal hermenuetics that when no distinction is made by law, the Court should not
international agreement, valid and binding on the part of the Philippines. This provision lays down the general rule distinguish- Ubi lex non distinguit nec nos distinguire debemos.
on treatise or international agreements and applies to any form of treaty with a wide variety of subject matter, such
as, but not limited to, extradition or tax treatise or those economic in nature. All treaties or international agreements In like manner, we do not subscribe to the argument that Section 25, Article XVIII is not controlling since no
entered into by the Philippines, regardless of subject matter, coverage, or particular designation or appellation, foreign military bases, but merely foreign troops and facilities, are involved in the VFA.Notably, a perusal of said
requires the concurrence of the Senate to be valid and effective. constitutional provision reveals that the proscription covers foreign military bases, troops, or facilities. Stated
differently, this prohibition is not limited to the entry of troops and facilities without any foreign bases being
In contrast, Section 25, Article XVIII is a special provision that applies to treaties which involve the presence established. The clause does not refer to foreign military bases, troops, or facilities collectively but treats them as
of foreign military bases, troops or facilities in the Philippines. Under this provision, the concurrence of the Senate separate and independent subjects. The use of comma and the disjunctive word or clearly signifies disassociation
is only one of the requisites to render compliance with the constitutional requirements and to consider the and independence of one thing from the others included in the enumeration, [28] such that, the provision
agreement binding on the Philippines. Section 25, Article XVIII further requires that foreign military bases, troops, contemplates three different situations - a military treaty the subject of which could be either (a) foreign bases, (b)
or facilities may be allowed in the Philippines only by virtue of a treaty duly concurred in by the Senate, ratified by
PAGE 72 of 135 CONSTITUTIONAL LAW -EXECUTIVE DEPARTMENT PART 2 –SESSION 12
foreign troops, or (c) foreign facilities - any of the three standing alone places it under the coverage of Section 25, As noted, the concurrence requirement under Section 25, Article XVIII must be construed in relation to the
Article XVIII. provisions of Section 21, Article VII. In a more particular language, the concurrence of the Senate contemplated
under Section 25, Article XVIII means that at least two-thirds of all the members of the Senate favorably vote to
To this end, the intention of the framers of the Charter, as manifested during the deliberations of the 1986 concur with the treaty-the VFA in the instant case.
Constitutional Commission, is consistent with this interpretation:
Under these circumstances, the charter provides that the Senate shall be composed of twenty-four (24)
MR. MAAMBONG. I just want to address a question or two to Commissioner Bernas. Senators.[30] Without a tinge of doubt, two-thirds (2/3) of this figure, or not less than sixteen (16) members,
This formulation speaks of three things: foreign military bases, troops or facilities. My first question is: If the favorably acting on the proposal is an unquestionable compliance with the requisite number of votes mentioned
country does enter into such kind of a treaty, must it cover the three-bases, troops or facilities-or in Section 21 of Article VII. The fact that there were actually twenty-three (23) incumbent Senators at the time the
could the treaty entered into cover only one or two? voting was made,[31] will not alter in any significant way the circumstance that more than two-thirds of the members
of the Senate concurred with the proposed VFA, even if the two-thirds vote requirement is based on this figure of
FR. BERNAS. Definitely, it can cover only one. Whether it covers only one or it covers three, the actual members (23). In this regard, the fundamental law is clear that two-thirds of the 24 Senators, or at least 16
requirement will be the same. favorable votes, suffice so as to render compliance with the strict constitutional mandate of giving concurrence to
the subject treaty.
MR. MAAMBONG. In other words, the Philippine government can enter into a treaty covering not bases
but merely troops? Having resolved that the first two requisites prescribed in Section 25, Article XVIII are present, we shall now
pass upon and delve on the requirement that the VFA should be recognized as a treaty by the United States of
FR. BERNAS. Yes. America.
MR. MAAMBONG. I cannot find any reason why the government can enter into a treaty covering only troops. Petitioners content that the phrase recognized as a treaty, embodied in section 25, Article XVIII, means that
the VFA should have the advice and consent of the United States Senate pursuant to its own constitutional
FR. BERNAS. Why not? Probably if we stretch our imagination a little bit more, we will find some. We just want
process, and that it should not be considered merely an executive agreement by the United States.
to cover everything.[29] (Underscoring Supplied)
In opposition, respondents argue that the letter of United States Ambassador Hubbard stating that the VFA
Moreover, military bases established within the territory of another state is no longer viable because of the
is binding on the United States Government is conclusive, on the point that the VFA is recognized as a treaty by
alternatives offered by new means and weapons of warfare such as nuclear weapons, guided missiles as well as
the United States of America. According to respondents, the VFA, to be binding, must only be accepted as a treaty
huge sea vessels that can stay afloat in the sea even for months and years without returning to their home
by the United States.
country. These military warships are actually used as substitutes for a land-home base not only of military aircraft
but also of military personnel and facilities. Besides, vessels are mobile as compared to a land-based military This Court is of the firm view that the phrase recognized as a treaty means that the other contracting
headquarters. party accepts or acknowledges the agreement as a treaty.[32] To require the other contractingstate, the United
States of America in this case, to submit the VFA to the United States Senate for concurrence pursuant to its
At this juncture, we shall then resolve the issue of whether or not the requirements of Section 25 were
Constitution,[33] is to accord strict meaning to the phrase.
complied with when the Senate gave its concurrence to the VFA.
Well-entrenched is the principle that the words used in the Constitution are to be given their ordinary meaning
Section 25, Article XVIII disallows foreign military bases, troops, or facilities in the country, unless the
except where technical terms are employed, in which case the significance thus attached to them prevails. Its
following conditions are sufficiently met, viz: (a) it must be under a treaty; (b) the treaty must be duly concurred
language should be understood in the sense they have in common use.[34]
in by the Senate and, when so required by congress, ratified by a majority of the votes cast by the people in a
national referendum; and (c) recognized as a treaty by the other contracting state. Moreover, it is inconsequential whether the United States treats the VFA only as an executive agreement
because, under international law, an executive agreement is as binding as a treaty. [35] To be sure, as long as the
There is no dispute as to the presence of the first two requisites in the case of the VFA. The concurrence
VFA possesses the elements of an agreement under international law, the said agreement is to be taken equally
handed by the Senate through Resolution No. 18 is in accordance with the provisions of the Constitution, whether
as a treaty.
under the general requirement in Section 21, Article VII, or the specific mandate mentioned in Section 25, Article
XVIII, the provision in the latter article requiring ratification by a majority of the votes cast in a national referendum A treaty, as defined by the Vienna Convention on the Law of Treaties, is an international instrument
being unnecessary since Congress has not required it. concluded between States in written form and governed by international law, whether embodied in a single
instrument or in two or more related instruments, and whatever its particular designation. [36] There are many other
As to the matter of voting, Section 21, Article VII particularly requires that a treaty or international
terms used for a treaty or international agreement, some of which are: act, protocol, agreement, compromis d
agreement, to be valid and effective, must be concurred in by at least two-thirds of all the members of the
arbitrage, concordat, convention, declaration, exchange of notes, pact, statute, charter and modus vivendi. All
Senate. On the other hand, Section 25, Article XVIII simply provides that the treaty be duly concurred in by the
writers, from Hugo Grotius onward, have pointed out that the names or titles of international agreements included
Senate.
under the general term treaty have little or no legal significance. Certain terms are useful, but they furnish little
Applying the foregoing constitutional provisions, a two-thirds vote of all the members of the Senate is clearly more than mere description.[37]
required so that the concurrence contemplated by law may be validly obtained and deemed present. While it is
Article 2(2) of the Vienna Convention provides that the provisions of paragraph 1 regarding the use of terms
true that Section 25, Article XVIII requires, among other things, that the treaty-the VFA, in the instant case-be duly
in the present Convention are without prejudice to the use of those terms, or to the meanings which may be given
concurred in by the Senate, it is very true however that said provision must be related and viewed in light of the
to them in the internal law of the State.
clear mandate embodied in Section 21, Article VII, which in more specific terms, requires that the concurrence of
a treaty, or international agreement, be made by a two -thirds vote of all the members of the Senate. Indeed, Thus, in international law, there is no difference between treaties and executive agreements in their binding
Section 25, Article XVIII must not be treated in isolation to section 21, Article, VII. effect upon states concerned, as long as the negotiating functionaries have remained within their
powers.[38] International law continues to make no distinction between treaties and executive agreements: they
are equally binding obligations upon nations.[39]
PAGE 73 of 135 CONSTITUTIONAL LAW -EXECUTIVE DEPARTMENT PART 2 –SESSION 12
In our jurisdiction, we have recognized the binding effect of executive agreements even without the international law as part of the law of the land and adheres to the policy of peace, equality, justice, freedom,
concurrence of the Senate or Congress. In Commissioner of Customs vs. Eastern Sea Trading,[40]we had cooperation and amity with all nations.
occasion to pronounce:
As a member of the family of nations, the Philippines agrees to be bound by generally accepted rules for the
conduct of its international relations. While the international obligation devolves upon the state and not upon any
x x x the right of the Executive to enter into binding agreements without the necessity of subsequent particular branch, institution, or individual member of its government, the Philippines is nonetheless responsible
congressional approval has been confirmed by long usage. From the earliest days of our history we have for violations committed by any branch or subdivision of its government or any official thereof. As an integral part
entered into executive agreements covering such subjects as commercial and consular relations, most-favored- of the community of nations, we are responsible to assure that our government, Constitution and laws will carry
nation rights, patent rights, trademark and copyright protection, postal and navigation arrangements and the out our international obligation.[47]Hence, we cannot readily plead the Constitution as a convenient excuse for non-
settlement of claims.The validity of these has never been seriously questioned by our courts. compliance with our obligations, duties and responsibilities under international law.

xxxxxxxxx Beyond this, Article 13 of the Declaration of Rights and Duties of States adopted by the International Law
Commission in 1949 provides: Every State has the duty to carry out in good faith its obligations arising from treaties
and other sources of international law, and it may not invoke provisions in its constitution or its laws as an excuse
Furthermore, the United States Supreme Court has expressly recognized the validity and constitutionality of for failure to perform this duty.[48]
executive agreements entered into without Senate approval. (39 Columbia Law Review, pp. 753-754) (See,
also, U.S. vs. Curtis Wright Export Corporation, 299 U.S. 304, 81 L. ed. 255; U.S. vs. Belmont, 301 U.S. Equally important is Article 26 of the convention which provides that Every treaty in force is binding upon the
324, 81 L. ed. 1134; U.S. vs. Pink, 315 U.S. 203, 86 L. ed. 796; Ozanic vs. U.S. 188 F. 2d. 288; Yale Law parties to it and must be performed by them in good faith. This is known as the principle of pacta sunt
Journal, Vol. 15 pp. 1905-1906; California Law Review, Vol. 25, pp. 670-675; Hyde on International Law servanda which preserves the sanctity of treaties and have been one of the most fundamental principles of positive
[revised Edition], Vol. 2, pp. 1405, 1416-1418; willoughby on the U.S. Constitution Law, Vol. I [2d ed.], pp. international law, supported by the jurisprudence of international tribunals.[49]
537-540; Moore, International Law Digest, Vol. V, pp. 210-218; Hackworth, International Law Digest, Vol.
V, pp. 390-407). (Italics Supplied) (Emphasis Ours)
NO GRAVE ABUSE OF DISCRETION
The deliberations of the Constitutional Commission which drafted the 1987 Constitution is enlightening and
highly-instructive:
MR. MAAMBONG. Of course it goes without saying that as far as ratification of the other state is concerned, In the instant controversy, the President, in effect, is heavily faulted for exercising a power and performing a
that is entirely their concern under their own laws. task conferred upon him by the Constitution-the power to enter into and ratify treaties.Through the expediency of
Rule 65 of the Rules of Court, petitioners in these consolidated cases impute grave abuse of discretion on the
FR. BERNAS. Yes, but we will accept whatever they say. If they say that we have done everything to make it part of the chief Executive in ratifying the VFA, and referring the same to the Senate pursuant to the provisions of
a treaty, then as far as we are concerned, we will accept it as a treaty. [41] Section 21, Article VII of the Constitution.
The records reveal that the United States Government, through Ambassador Thomas C. Hubbard, has stated On this particular matter, grave abuse of discretion implies such capricious and whimsical exercise of
that the United States government has fully committed to living up to the terms of the VFA. [42] For as long as the judgment as is equivalent to lack of jurisdiction, or, when the power is exercised in an arbitrary or despotic manner
united States of America accepts or acknowledges the VFA as a treaty, and binds itself further to comply with its by reason of passion or personal hostility, and it must be so patent and gross as to amount to an evasion of
obligations under the treaty, there is indeed marked compliance with the mandate of the Constitution. positive duty enjoined or to act at all in contemplation of law.[50]
Worth stressing too, is that the ratification, by the President, of the VFA and the concurrence of the Senate By constitutional fiat and by the intrinsic nature of his office, the President, as head of State, is the sole organ
should be taken as a clear an unequivocal expression of our nations consent to be bound by said treaty, with the and authority in the external affairs of the country. In many ways, the President is the chief architect of the nations
concomitant duty to uphold the obligations and responsibilities embodied thereunder. foreign policy; his dominance in the field of foreign relations is (then) conceded. [51] Wielding vast powers an
influence, his conduct in the external affairs of the nation, as Jefferson describes, is executive altogether."[52]
Ratification is generally held to be an executive act, undertaken by the head of the state or of the government,
as the case may be, through which the formal acceptance of the treaty is proclaimed.[43] A State may provide in As regards the power to enter into treaties or international agreements, the Constitution vests the same in
its domestic legislation the process of ratification of a treaty. The consent of the State to be bound by a treaty is the President, subject only to the concurrence of at least two-thirds vote of all the members of the Senate. In this
expressed by ratification when: (a) the treaty provides for such ratification, (b) it is otherwise established that the light, the negotiation of the VFA and the subsequent ratification of the agreement are exclusive acts which pertain
negotiating States agreed that ratification should be required, (c) the representative of the State has signed the solely to the President, in the lawful exercise of his vast executive and diplomatic powers granted him no less than
treaty subject to ratification, or (d) the intention of the State to sign the treaty subject to ratification appears from by the fundamental law itself. Into the field of negotiation the Senate cannot intrude, and Congress itself is
the full powers of its representative, or was expressed during the negotiation. [44] powerless to invade it.[53] Consequently, the acts or judgment calls of the President involving the VFA-specifically
the acts of ratification and entering into a treaty and those necessary or incidental to the exercise of such principal
In our jurisdiction, the power to ratify is vested in the President and not, as commonly believed, in the
acts - squarely fall within the sphere of his constitutional powers and thus, may not be validly struck down, much
legislature. The role of the Senate is limited only to giving or withholding its consent, or concurrence, to the
less calibrated by this Court, in the absence of clear showing of grave abuse of power or discretion.
ratification.[45]
It is the Courts considered view that the President, in ratifying the VFA and in submitting the same to the
With the ratification of the VFA, which is equivalent to final acceptance, and with the exchange of notes
Senate for concurrence, acted within the confines and limits of the powers vested in him by the Constitution. It is
between the Philippines and the United States of America, it now becomes obligatory and incumbent on our part,
of no moment that the President, in the exercise of his wide latitude of discretion and in the honest belief that the
under the principles of international law, to be bound by the terms of the agreement. Thus, no less than Section
VFA falls within the ambit of Section 21, Article VII of the Constitution, referred the VFA to the Senate for
2, Article II of the Constitution,[46] declares that the Philippines adopts the generally accepted principles of
concurrence under the aforementioned provision. Certainly, no abuse of discretion, much less a grave, patent and
whimsical abuse of judgment, may be imputed to the President in his act of ratifying the VFA and referring the
PAGE 74 of 135 CONSTITUTIONAL LAW -EXECUTIVE DEPARTMENT PART 2 –SESSION 12
same to the Senate for the purpose of complying with the concurrence requirement embodied in the fundamental
law. In doing so, the President merely performed a constitutional task and exercised a prerogative that chiefly
pertains to the functions of his office. Even if he erred in submitting the VFA to the Senate for concurrence under
the provisions of Section 21 of Article VII, instead of Section 25 of Article XVIII of the Constitution, still, the
President may not be faulted or scarred, much less be adjudged guilty of committing an abuse of discretion in
some patent, gross, and capricious manner.
For while it is conceded that Article VIII, Section 1, of the Constitution has broadened the scope of judicial
inquiry into areas normally left to the political departments to decide, such as those relating to national security, it
has not altogether done away with political questions such as those which arise in the field of foreign
relations.[54] The High Tribunals function, as sanctioned by Article VIII, Section 1, is merely (to) check whether or
not the governmental branch or agency has gone beyond the constitutional limits of its jurisdiction, not that it erred
or has a different view. In the absence of a showing (of) grave abuse of discretion amounting to lack of jurisdiction,
there is no occasion for the Court to exercise its corrective powerIt has no power to look into what it thinks is
apparent error.[55]
As to the power to concur with treaties, the constitution lodges the same with the Senate alone. Thus, once
the Senate[56] performs that power, or exercises its prerogative within the boundaries prescribed by the
Constitution, the concurrence cannot, in like manner, be viewed to constitute an abuse of power, much less grave
abuse thereof. Corollarily, the Senate, in the exercise of its discretion and acting within the limits of such power,
may not be similarly faulted for having simply performed a task conferred and sanctioned by no less than the
fundamental law.
For the role of the Senate in relation to treaties is essentially legislative in character; [57] the Senate, as an
independent body possessed of its own erudite mind, has the prerogative to either accept or reject the proposed
agreement, and whatever action it takes in the exercise of its wide latitude of discretion, pertains to the wisdom
rather than the legality of the act. In this sense, the Senate partakes a principal, yet delicate, role in keeping the
principles of separation of powers and of checks and balances alive and vigilantly ensures that these cherished
rudiments remain true to their form in a democratic government such as ours. The Constitution thus animates,
through this treaty-concurring power of the Senate, a healthy system of checks and balances indispensable toward
our nations pursuit of political maturity and growth. True enough, rudimentary is the principle that matters
pertaining to the wisdom of a legislative act are beyond the ambit and province of the courts to inquire.
In fine, absent any clear showing of grave abuse of discretion on the part of respondents, this Court- as the
final arbiter of legal controversies and staunch sentinel of the rights of the people - is then without power to conduct
an incursion and meddle with such affairs purely executive and legislative in character and nature. For the
Constitution no less, maps out the distinct boundaries and limits the metes and bounds within which each of the
three political branches of government may exercise the powers exclusively and essentially conferred to it by law.
WHEREFORE, in light of the foregoing disquisitions, the instant petitions are hereby DISMISSED.
SO ORDERED.
PAGE 75 of 135 CONSTITUTIONAL LAW -EXECUTIVE DEPARTMENT PART 2 –SESSION 12
Republic of the Philippines On December 28, 2000, the RP, through Charge dAffaires Enrique A. Manalo, signed the Rome Statute
SUPREME COURT which, by its terms, is subject to ratification, acceptance or approval by the signatory states.[6] As of the filing of
Manila the instant petition, only 92 out of the 139 signatory countries appear to have completed the ratification, approval
and concurrence process. The Philippinesis not among the 92.

EN BANC RP-US Non-Surrender Agreement

BAYAN MUNA, as represented by Rep. SATUR G.R. No. 159618 On May 9, 2003, then Ambassador Francis J. Ricciardone sent US Embassy Note No. 0470 to the
OCAMPO, Rep. CRISPIN BELTRAN, and Rep. LIZA Department of Foreign Affairs (DFA) proposing the terms of the non-surrender bilateral agreement (Agreement,
L. MAZA, Present: hereinafter) between the USA and the RP.
Petitioner, CORONA, C.J., Via Exchange of Notes No. BFO-028-03[7] dated May 13, 2003 (E/N BFO-028-03, hereinafter), the RP,
CARPIO, represented by then DFA Secretary Ople, agreed with and accepted the US proposals embodied under the US
CARPIO MORALES, Embassy Note adverted to and put in effect the Agreement with the US government. In esse, the Agreement aims
VELASCO, JR., to protect what it refers to and defines aspersons of the RP and US from frivolous and harassment suits that might
NACHURA, be brought against them in international tribunals.[8] It is reflective of the increasing pace of the strategic security
LEONARDO-DE CASTRO, and defense partnership between the two countries. As of May 2, 2003, similar bilateral agreements have been
- versus - BRION, effected by and between the US and 33 other countries.[9]
PERALTA,
BERSAMIN,
DEL CASTILLO, The Agreement pertinently provides as follows:
ABAD,
VILLARAMA, JR., 1. For purposes of this Agreement, persons are current or former Government officials,
PEREZ, employees (including contractors), or military personnel or nationals of one Party.
ALBERTO ROMULO, in his capacity as Executive MENDOZA, and
Secretary, and BLAS F. OPLE, in his capacity as SERENO, JJ. 2. Persons of one Party present in the territory of the other shall not, absent the express
Secretary of Foreign Affairs, consent of the first Party,
Respondents. Promulgated:
(a) be surrendered or transferred by any means to any international tribunal for any
February 1, 2011 purpose, unless such tribunal has been established by the UN Security Council,
x-----------------------------------------------------------------------------------------x or

DECISION (b) be surrendered or transferred by any means to any other entity or third country, or
VELASCO, JR., J.: expelled to a third country, for the purpose of surrender to or transfer to any
international tribunal, unless such tribunal has been established by the UN
The Case Security Council.

This petition[1] for certiorari, mandamus and prohibition under Rule 65 assails and seeks to nullify the 3. When the [US] extradites, surrenders, or otherwise transfers a person of the
Non-Surrender Agreement concluded by and between the Republic of the Philippines (RP) and the United States Philippines to a third country, the [US] will not agree to the surrender or transfer of that person
of America (USA). by the third country to any international tribunal, unless such tribunal has been established by
the UN Security Council, absent the express consent of the Government of the Republic of the
The Facts Philippines [GRP].

4. When the [GRP] extradites, surrenders, or otherwise transfers a person of the [USA]
Petitioner Bayan Muna is a duly registered party-list group established to represent the marginalized to a third country, the [GRP] will not agree to the surrender or transfer of that person by the third
sectors of society. Respondent Blas F. Ople, now deceased, was the Secretary of Foreign Affairs during the period country to any international tribunal, unless such tribunal has been established by the UN
material to this case. Respondent Alberto Romulo was impleaded in his capacity as then Executive Secretary. [2] Security Council, absent the express consent of the Government of the [US].

Rome Statute of the International Criminal Court 5. This Agreement shall remain in force until one year after the date on which one party
notifies the other of its intent to terminate the Agreement. The provisions of this Agreement shall
Having a key determinative bearing on this case is the Rome Statute [3] establishing the International continue to apply with respect to any act occurring, or any allegation arising, before the effective
Criminal Court (ICC) with the power to exercise its jurisdiction over persons for the most serious crimes of date of termination.
international concern x x x and shall be complementary to the national criminal jurisdictions.[4] The serious crimes
adverted to cover those considered grave under international law, such as genocide, crimes against humanity,
war crimes, and crimes of aggression.[5] In response to a query of then Solicitor General Alfredo L. Benipayo on the status of the non-surrender
agreement, Ambassador Ricciardone replied in his letter of October 28, 2003 that the exchange of diplomatic
PAGE 76 of 135 CONSTITUTIONAL LAW -EXECUTIVE DEPARTMENT PART 2 –SESSION 12
notes constituted a legally binding agreement under international law; and that, under US law, the said agreement The Courts Ruling
did not require the advice and consent of the US Senate.[10]
This petition is bereft of merit.
In this proceeding, petitioner imputes grave abuse of discretion to respondents in concluding and ratifying
the Agreement and prays that it be struck down as unconstitutional, or at least declared as without force and effect. Procedural Issue: Locus Standi of Petitioner

For their part, respondents question petitioners standing to maintain a suit and counter that Petitioner, through its three party-list representatives, contends that the issue of the validity or invalidity
the Agreement, being in the nature of an executive agreement, does not require Senate concurrence for its of the Agreement carries with it constitutional significance and is of paramount importance that justifies its
efficacy. And for reasons detailed in their comment, respondents assert the constitutionality of the Agreement. standing. Cited in this regard is what is usually referred to as the emergency powers cases, [12] in which ordinary
citizens and taxpayers were accorded the personality to question the constitutionality of executive issuances.
The Issues Locus standi is a right of appearance in a court of justice on a given question.[13] Specifically, it is a partys
personal and substantial interest in a case where he has sustained or will sustain direct injury as a result[14] of the
act being challenged, and calls for more than just a generalized grievance. [15] The term interest refers to material
I. WHETHER THE [RP] PRESIDENT AND THE [DFA] SECRETARY x x x GRAVELY ABUSED interest, as distinguished from one that is merely incidental. [16] The rationale for requiring a party who challenges
THEIR DISCRETION AMOUNTING TO LACK OR EXCESS OF JURISDICTION FOR the validity of a law or international agreement to allege such a personal stake in the outcome of the controversy
CONCLUDING THE RP-US NON SURRENDER AGREEMENT BY MEANS OF [E/N] BFO- is to assure the concrete adverseness which sharpens the presentation of issues upon which the court so largely
028-03 DATED 13 MAY 2003, WHEN THE PHILIPPINE GOVERNMENT HAS ALREADY depends for illumination of difficult constitutional questions. [17]
SIGNED THE ROME STATUTE OF THE [ICC] ALTHOUGH THIS IS PENDING
RATIFICATION BY THE PHILIPPINE SENATE. Locus standi, however, is merely a matter of procedure and it has been recognized that, in some cases,
A. Whether by entering into the x x x Agreement Respondents gravely abused their suits are not brought by parties who have been personally injured by the operation of a law or any other
discretion when they capriciously abandoned, waived and relinquished our only government act, but by concerned citizens, taxpayers, or voters who actually sue in the public
legitimate recourse through the Rome Statute of the [ICC] to prosecute and try persons interest.[18] Consequently, in a catena of cases,[19] this Court has invariably adopted a liberal stance on locus
as defined in the x x x Agreement, x x x or literally any conduit of American interests, standi.
who have committed crimes of genocide, crimes against humanity, war crimes and the
crime of aggression, thereby abdicating Philippine Sovereignty. Going by the petition, petitioners representatives pursue the instant suit primarily as concerned citizens
raising issues of transcendental importance, both for the Republic and the citizenry as a whole.
B. Whether after the signing and pending ratification of the Rome Statute of the [ICC] the
[RP] President and the [DFA] Secretary x x x are obliged by the principle of good faith When suing as a citizen to question the validity of a law or other government action, a petitioner needs
to refrain from doing all acts which would substantially impair the value of the to meet certain specific requirements before he can be clothed with standing.Francisco, Jr. v. Nagmamalasakit na
undertaking as signed. mga Manananggol ng mga Manggagawang Pilipino, Inc.[20] expounded on this requirement, thus:

C. Whether the x x x Agreement constitutes an act which defeats the object and purpose
of the Rome Statute of the International Criminal Court and contravenes the obligation In a long line of cases, however, concerned citizens, taxpayers and legislators when
of good faith inherent in the signature of the President affixed on the Rome Statute of specific requirements have been met have been given standing by this Court.
the International Criminal Court, and if so whether the x x x Agreement is void and When suing as a citizen, the interest of the petitioner assailing the constitutionality of
unenforceable on this ground. a statute must be direct and personal. He must be able to show, not only that the law or any
government act is invalid, but also that he sustained or is in imminent danger of sustaining some
D. Whether the RP-US Non-Surrender Agreement is void and unenforceable for grave direct injury as a result of its enforcement, and not merely that he suffers thereby in some
abuse of discretion amounting to lack or excess of jurisdiction in connection with its indefinite way. It must appear that the person complaining has been or is about to be denied
execution. some right or privilege to which he is lawfully entitled or that he is about to be subjected to some
burdens or penalties by reason of the statute or act complained of.In fine, when the proceeding
II. WHETHER THE RP-US NON SURRENDER AGREEMENT IS VOID AB INITIO FOR involves the assertion of a public right, the mere fact that he is a citizen satisfies the requirement
CONTRACTING OBLIGATIONS THAT ARE EITHER IMMORAL OR OTHERWISE AT of personal interest.[21]
VARIANCE WITH UNIVERSALLY RECOGNIZED PRINCIPLES OF INTERNATIONAL
LAW. In the case at bar, petitioners representatives have complied with the qualifying conditions or specific
requirements exacted under the locus standi rule. As citizens, their interest in the subject matter of the petition is
III. WHETHER THE x x x AGREEMENT IS VALID, BINDING AND EFFECTIVE WITHOUT THE direct and personal. At the very least, their assertions questioning the Agreement are made of a public right, i.e.,
CONCURRENCE BY AT LEAST TWO-THIRDS (2/3) OF ALL THE MEMBERS OF THE to ascertain that the Agreement did not go against established national policies, practices, and obligations bearing
SENATE x x x.[11] on the States obligation to the community of nations.

At any event, the primordial importance to Filipino citizens in general of the issue at hand impels the
The foregoing issues may be summarized into two: first, whether or not the Agreement was contracted Court to brush aside the procedural barrier posed by the traditional requirement of locus standi, as we have done
validly, which resolves itself into the question of whether or not respondents gravely abused their discretion in in a long line of earlier cases, notably in the old but oft-cited emergency powers cases[22] and Kilosbayan v.
concluding it; and second, whether or not the Agreement, which has not been submitted to the Senate for Guingona, Jr.[23] In cases of transcendental importance, we wrote again in Bayan v. Zamora,[24] The Court may
concurrence, contravenes and undermines the Rome Statute and other treaties. But because respondents
expectedly raised it, we shall first tackle the issue of petitioners legal standing.
PAGE 77 of 135 CONSTITUTIONAL LAW -EXECUTIVE DEPARTMENT PART 2 –SESSION 12
relax the standing requirements and allow a suit to prosper even where there is no direct injury to the party claiming Under international law, there is no difference between treaties and executive agreements in terms of
the right of judicial review. their binding effects on the contracting states concerned, [34] as long as the negotiating functionaries have remained
within their powers.[35] Neither, on the domestic sphere, can one be held valid if it violates the
Moreover, bearing in mind what the Court said in Taada v. Angara, that it will not shirk, digress from or Constitution.[36] Authorities are, however, agreed that one is distinct from another for accepted reasons apart from
abandon its sacred duty and authority to uphold the Constitution in matters that involve grave abuse of discretion the concurrence-requirement aspect.[37] As has been observed by US constitutional scholars, a treaty has greater
brought before it in appropriate cases, committed by any officer, agency, instrumentality or department of the dignity than an executive agreement, because its constitutional efficacy is beyond doubt, a treaty having behind it
government,[25] we cannot but resolve head on the issues raised before us. Indeed, where an action of any branch the authority of the President, the Senate, and the people;[38] a ratified treaty, unlike an executive agreement, takes
of government is seriously alleged to have infringed the Constitution or is done with grave abuse of discretion, it precedence over any prior statutory enactment. [39]
becomes not only the right but in fact the duty of the judiciary to settle it. As in this petition, issues are precisely
raised putting to the fore the propriety of the Agreement pending the ratification of the Rome Statute. Petitioner parlays the notion that the Agreement is of dubious validity, partaking as it does of the nature
of a treaty; hence, it must be duly concurred in by the Senate. Petitioner takes a cue from Commissioner of
Validity of the RP-US Non-Surrender Agreement Customs v. Eastern Sea Trading, in which the Court reproduced the following observations made by US legal
scholars: [I]nternational agreements involving political issues or changes of national policy and those involving
Petitioners initial challenge against the Agreement relates to form, its threshold posture being that E/N international arrangements of a permanent character usually take the form of treaties [while] those embodying
BFO-028-03 cannot be a valid medium for concluding the Agreement. adjustments of detail carrying out well established national policies and traditions and those involving
Petitioners contentionperhaps taken unaware of certain well-recognized international doctrines, arrangements of a more or less temporary nature take the form of executive agreements. [40]
practices, and jargonsis untenable. One of these is the doctrine of incorporation, as expressed in Section 2, Article
II of the Constitution, wherein the Philippines adopts the generally accepted principles of international law and Pressing its point, petitioner submits that the subject of the Agreement does not fall under any of the
international jurisprudence as part of the law of the land and adheres to the policy of peace, cooperation, and subject-categories that are enumerated in the Eastern Sea Trading case, and that may be covered by an executive
amity with all nations.[26] An exchange of notes falls into the category of inter-governmental agreements,[27] which agreement, such as commercial/consular relations, most-favored nation rights, patent rights, trademark and
is an internationally accepted form of international agreement. The United Nations Treaty Collections (Treaty copyright protection, postal and navigation arrangements and settlement of claims.
Reference Guide) defines the term as follows:
In addition, petitioner foists the applicability to the instant case of Adolfo v. CFI of Zambales and
Merchant,[41] holding that an executive agreement through an exchange of notes cannot be used to amend a
An exchange of notes is a record of a routine agreement, that has many similarities treaty.
with the private law contract. The agreement consists of the exchange of two documents, each
of the parties being in the possession of the one signed by the representative of the other. Under We are not persuaded.
the usual procedure, the accepting State repeats the text of the offering State to record its
assent. The signatories of the letters may be government Ministers, diplomats or departmental The categorization of subject matters that may be covered by international agreements mentioned
heads. The technique of exchange of notes is frequently resorted to, either because of its in Eastern Sea Trading is not cast in stone. There are no hard and fast rules on the propriety of entering, on a
speedy procedure, or, sometimes, to avoid the process of legislative approval. [28] given subject, into a treaty or an executive agreement as an instrument of international relations. The primary
consideration in the choice of the form of agreement is the parties intent and desire to craft an international
agreement in the form they so wish to further their respective interests. Verily, the matter of form takes a back seat
In another perspective, the terms exchange of notes and executive agreements have been used when it comes to effectiveness and binding effect of the enforcement of a treaty or an executive agreement, as
interchangeably, exchange of notes being considered a form of executive agreement that becomes binding the parties in either international agreement each labor under the pacta sunt servanda[42] principle.
through executive action.[29] On the other hand, executive agreements concluded by the President sometimes take
the form of exchange of notes and at other times that of more formal documents denominated agreements or As may be noted, almost half a century has elapsed since the Court rendered its decision in Eastern Sea
protocols.[30] As former US High Commissioner to the Philippines Francis B. Sayre observed in his work, The Trading. Since then, the conduct of foreign affairs has become more complex and the domain of international law
Constitutionality of TradeAgreement Acts: wider, as to include such subjects as human rights, the environment, and the sea. In fact, in the US alone, the
The point where ordinary correspondence between this and other governments ends executive agreements executed by its President from 1980 to 2000 covered subjects such as defense, trade,
and agreements whether denominated executive agreements or exchange of notes or otherwise scientific cooperation, aviation, atomic energy, environmental cooperation, peace corps, arms limitation, and
begin, may sometimes be difficult of ready ascertainment.[31] x x x nuclear safety, among others.[43] Surely, the enumeration in Eastern Sea Trading cannot circumscribe the option
It is fairly clear from the foregoing disquisition that E/N BFO-028-03be it viewed as the Non-Surrender of each state on the matter of which the international agreement format would be convenient to serve its best
Agreement itself, or as an integral instrument of acceptance thereof or as consent to be boundis a recognized interest. As Francis Sayre said in his work referred to earlier:
mode of concluding a legally binding international written contract among nations. x x x It would be useless to undertake to discuss here the large variety of executive
agreements as such concluded from time to time. Hundreds of executive agreements, other
Senate Concurrence Not Required than those entered into under the trade-agreement act, have been negotiated with foreign
governments. x x x They cover such subjects as the inspection of vessels, navigation dues,
Article 2 of the Vienna Convention on the Law of Treaties defines a treaty as an international agreement income tax on shipping profits, the admission of civil air craft, custom matters and commercial
concluded between states in written form and governed by international law, whether embodied in a single relations generally, international claims, postal matters, the registration of trademarks and
instrument or in two or more related instruments and whatever its particular designation.[32] International copyrights, etc. x x x
agreements may be in the form of (1) treaties that require legislative concurrence after executive ratification; or
(2) executive agreements that are similar to treaties, except that they do not require legislative concurrence and
are usually less formal and deal with a narrower range of subject matters than treaties. [33] And lest it be overlooked, one type of executive agreement is a treaty-authorized[44] or a treaty-
implementing executive agreement,[45] which necessarily would cover the same matters subject of the underlying
treaty.
PAGE 78 of 135 CONSTITUTIONAL LAW -EXECUTIVE DEPARTMENT PART 2 –SESSION 12
But over and above the foregoing considerations is the fact thatsave for the situation and matters Article 1
contemplated in Sec. 25, Art. XVIII of the Constitution[46]when a treaty is required, the Constitution does not classify
any subject, like that involving political issues, to be in the form of, and ratified as, a treaty. What the Constitution The Court
merely prescribes is that treaties need the concurrence of the Senate by a vote defined therein to complete the
ratification process. An International Crimininal Court (the Court) is hereby established. It x x x shall have
the power to exercise its jurisdiction over persons for the most serious crimes of international
Petitioners reliance on Adolfo[47] is misplaced, said case being inapplicable owing to different factual concern, as referred to in this Statute, and shall be complementary to national criminal
milieus. There, the Court held that an executive agreement cannot be used to amend a duly ratified and existing jurisdictions. The jurisdiction and functioning of the Court shall be governed by the provisions
treaty, i.e., the Bases Treaty. Indeed, an executive agreement that does not require the concurrence of the Senate of this Statute. (Emphasis ours.)
for its ratification may not be used to amend a treaty that, under the Constitution, is the product of the ratifying acts
of the Executive and the Senate. The presence of a treaty, purportedly being subject to amendment by an
executive agreement, does not obtain under the premises. Significantly, the sixth preambular paragraph of the Rome Statute declares that it is the duty of every
State to exercise its criminal jurisdiction over those responsible for international crimes. This provision indicates
Considering the above discussion, the Court need not belabor at length the third main issue raised, that primary jurisdiction over the so-called international crimes rests, at the first instance, with the state where the
referring to the validity and effectivity of the Agreement without the concurrence by at least two-thirds of all the crime was committed; secondarily, with the ICC in appropriate situations contemplated under Art. 17, par. 1 [55] of
members of the Senate. The Court has, in Eastern Sea Trading,[48] as reiterated in Bayan,[49] given recognition to the Rome Statute.
the obligatory effect of executive agreements without the concurrence of the Senate:
Of particular note is the application of the principle of ne bis in idem[56] under par. 3 of Art. 20, Rome
x x x [T]he right of the Executive to enter into binding agreements without the necessity Statute, which again underscores the primacy of the jurisdiction of a state vis-a-vis that of the ICC. As far as
of subsequent Congressional approval has been confirmed by long usage. From the earliest relevant, the provision states that no person who has been tried by another court for conduct x x x [constituting
days of our history, we have entered executive agreements covering such subjects as crimes within its jurisdiction] shall be tried by the [International Criminal] Court with respect to the same conduct x
commercial and consular relations, most favored-nation rights, patent rights, trademark and x x.
copyright protection, postal and navigation arrangements and the settlement of claims. The
validity of these has never been seriously questioned by our courts. The foregoing provisions of the Rome Statute, taken collectively, argue against the idea of jurisdictional
conflict between the Philippines, as party to the non-surrender agreement, and the ICC; or the idea of
the Agreement substantially impairing the value of the RPs undertaking under the Rome Statute. Ignoring for a
The Agreement Not in Contravention of the Rome Statute while the fact that the RP signed the Rome Statute ahead of the Agreement, it is abundantly clear to us that the
Rome Statute expressly recognizes the primary jurisdiction of states, like the RP, over serious crimes committed
It is the petitioners next contention that the Agreement undermines the establishment of the ICC and is within their respective borders, the complementary jurisdiction of the ICC coming into play only when the signatory
null and void insofar as it unduly restricts the ICCs jurisdiction and infringes upon the effectivity of the Rome states are unwilling or unable to prosecute.
Statute. Petitioner posits that the Agreement was constituted solely for the purpose of providing individuals or
groups of individuals with immunity from the jurisdiction of the ICC; and such grant of immunity through non- Given the above consideration, petitioners suggestionthat the RP, by entering into the Agreement,
surrender agreements allegedly does not legitimately fall within the scope of Art. 98 of the Rome Statute. It violated its duty required by the imperatives of good faith and breached its commitment under the Vienna
concludes that state parties with non-surrender agreements are prevented from meeting their obligations under Convention[57] to refrain from performing any act tending to impair the value of a treaty, e.g., the Rome Statutehas
the Rome Statute, thereby constituting a breach of Arts. 27, [50] 86,[51] 89[52] and 90[53] thereof. to be rejected outright. For nothing in the provisions of the Agreement, in relation to the Rome Statute, tends to
Petitioner stresses that the overall object and purpose of the Rome Statute is to ensure that those diminish the efficacy of the Statute, let alone defeats the purpose of the ICC. Lest it be overlooked, the Rome
responsible for the worst possible crimes are brought to justice in all cases, primarily by states, but as a last resort, Statute contains a proviso that enjoins the ICC from seeking the surrender of an erring person, should the process
by the ICC; thus, any agreementlike the non-surrender agreementthat precludes the ICC from exercising its require the requested state to perform an act that would violate some international agreement it has entered
complementary function of acting when a state is unable to or unwilling to do so, defeats the object and purpose into. We refer to Art. 98(2) of the Rome Statute, which reads:
of the Rome Statute.
Article 98
Petitioner would add that the President and the DFA Secretary, as representatives of a signatory of the
Rome Statute, are obliged by the imperatives of good faith to refrain from performing acts that substantially devalue Cooperation with respect to waiver of immunity
the purpose and object of the Statute, as signed. Adding a nullifying ingredient to the Agreement, according to and consent to surrender
petitioner, is the fact that it has an immoral purpose or is otherwise at variance with a priorly executed treaty.
xxxx
Contrary to petitioners pretense, the Agreement does not contravene or undermine, nor does it differ
from, the Rome Statute. Far from going against each other, one complements the other.As a matter of fact, the 2. The Court may not proceed with a request for surrender which would require
principle of complementarity underpins the creation of the ICC. As aptly pointed out by respondents and admitted the requested State to act inconsistently with its obligations under international agreements
by petitioners, the jurisdiction of the ICC is to be complementary to national criminal jurisdictions [of the signatory pursuant to which the consent of a sending State is required to surrender a person of that
states].[54] Art. 1 of the Rome Statute pertinently provides: State to the Court, unless the Court can first obtain the cooperation of the sending State for
the giving of consent for the surrender.
PAGE 79 of 135 CONSTITUTIONAL LAW -EXECUTIVE DEPARTMENT PART 2 –SESSION 12
Moreover, under international law, there is a considerable difference between a State-Party and a subjects of such immunity like Heads of State, diplomats and members of the armed forces
signatory to a treaty. Under the Vienna Convention on the Law of Treaties, a signatory state is only obliged to contingents of a foreign State allowed to enter another States territory. x x x
refrain from acts which would defeat the object and purpose of a treaty;[58] whereas a State-Party, on the other
hand, is legally obliged to follow all the provisions of a treaty in good faith. To be sure, the nullity of the subject non-surrender agreement cannot be predicated on the postulate that
some of its provisions constitute a virtual abdication of its sovereignty. Almost every time a state enters into an
In the instant case, it bears stressing that the Philippines is only a signatory to the Rome Statute and not international agreement, it voluntarily sheds off part of its sovereignty. The Constitution, as drafted, did not envision
a State-Party for lack of ratification by the Senate. Thus, it is only obliged to refrain from acts which would defeat a reclusive Philippines isolated from the rest of the world. It even adheres, as earlier stated, to the policy of
the object and purpose of the Rome Statute. Any argument obliging the Philippines to follow any provision in the cooperation and amity with all nations. [60]
treaty would be premature.
By their nature, treaties and international agreements actually have a limiting effect on the otherwise
As a result, petitioners argument that State-Parties with non-surrender agreements are prevented from encompassing and absolute nature of sovereignty. By their voluntary act, nations may decide to surrender or
meeting their obligations under the Rome Statute, specifically Arts. 27, 86, 89 and 90, must fail. These articles are waive some aspects of their state power or agree to limit the exercise of their otherwise exclusive and absolute
only legally binding upon State-Parties, not signatories. jurisdiction. The usual underlying consideration in this partial surrender may be the greater benefits derived from
a pact or a reciprocal undertaking of one contracting party to grant the same privileges or immunities to the
Furthermore, a careful reading of said Art. 90 would show that the Agreement is not incompatible with other. On the rationale that the Philippines has adopted the generally accepted principles of international law
the Rome Statute. Specifically, Art. 90(4) provides that [i]f the requesting State is a State not Party to this Statute as part of the law of the land, a portion of sovereignty may be waived without violating the Constitution. [61]Such
the requested State, if it is not under an international obligation to extradite the person to the requesting State, waiver does not amount to an unconstitutional diminution or deprivation of jurisdiction of Philippine courts. [62]
shall give priority to the request for surrender from the Court. x x x In applying the provision, certain undisputed
facts should be pointed out: first, the US is neither a State-Party nor a signatory to the Rome Statute; and second, Agreement Not Immoral/Not at Variance
there is an international agreement between the US and the Philippines regarding extradition or surrender of with Principles of International Law
persons, i.e., the Agreement. Clearly, even assuming that the Philippines is a State-Party, the Rome Statute still
recognizes the primacy of international agreements entered into between States, even when one of the States is
not a State-Party to the Rome Statute. Petitioner urges that the Agreement be struck down as void ab initio for imposing immoral obligations
and/or being at variance with allegedly universally recognized principles of international law. The immoral aspect
Sovereignty Limited by International Agreements proceeds from the fact that the Agreement, as petitioner would put it, leaves criminals immune from responsibility
for unimaginable atrocities that deeply shock the conscience of humanity; x x x it precludes our country from
Petitioner next argues that the RP has, through the Agreement, abdicated its sovereignty by bargaining delivering an American criminal to the [ICC] x x x. [63]
away the jurisdiction of the ICC to prosecute US nationals, government officials/employees or military personnel
who commit serious crimes of international concerns in the Philippines. Formulating petitioners argument a bit The above argument is a kind of recycling of petitioners earlier position, which, as already discussed,
differently, the RP, by entering into the Agreement, does thereby abdicate its sovereignty, abdication being done contends that the RP, by entering into the Agreement, virtually abdicated its sovereignty and in the process
by its waiving or abandoning its right to seek recourse through the Rome Statute of the ICC for erring Americans undermined its treaty obligations under the Rome Statute, contrary to international law principles. [64]
committing international crimes in the country.
The Court is not persuaded. Suffice it to state in this regard that the non-surrender agreement, as aptly
We are not persuaded. As it were, the Agreement is but a form of affirmance and confirmance of described by the Solicitor General, is an assertion by the Philippines of its desire to try and punish crimes under
the Philippines national criminal jurisdiction. National criminal jurisdiction being primary, as explained above, it is its national law. x x x The agreement is a recognition of the primacy and competence of the countrys judiciary to
always the responsibility and within the prerogative of the RP either to prosecute criminal offenses equally covered try offenses under its national criminal laws and dispense justice fairly and judiciously.
by the Rome Statute or to accede to the jurisdiction of the ICC. Thus, the Philippines may decide to try persons of
the US, as the term is understood in the Agreement, under our national criminal justice system. Or it may opt not Petitioner, we believe, labors under the erroneous impression that the Agreement would allow Filipinos
to exercise its criminal jurisdiction over its erring citizens or over US persons committing high crimes in the country and Americans committing high crimes of international concern to escape criminal trial and punishment. This is
and defer to the secondary criminal jurisdiction of the ICC over them. As to persons of the US whom the Philippines manifestly incorrect. Persons who may have committed acts penalized under the Rome Statute can be prosecuted
refuses to prosecute, the country would, in effect, accord discretion to the US to exercise either its national criminal and punished in the Philippines or in the US; or with the consent of the RP or the US, before the ICC, assuming,
jurisdiction over the person concerned or to give its consent to the referral of the matter to the ICC for trial. In the for the nonce, that all the formalities necessary to bind both countries to the Rome Statute have been met. For
same breath, the US must extend the same privilege to the Philippines with respect to persons of the RP perspective, what the Agreement contextually prohibits is the surrender by either party of individuals to
committing high crimes within US territorial jurisdiction. international tribunals, like the ICC, without the consent of the other party, which may desire to prosecute the crime
under its existing laws. With the view we take of things, there is nothing immoral or violative of international law
concepts in the act of the Philippines of assuming criminal jurisdiction pursuant to the non-surrender agreement
over an offense considered criminal by both Philippine laws and the Rome Statute.
In the context of the Constitution, there can be no serious objection to the Philippines agreeing to No Grave Abuse of Discretion
undertake the things set forth in the Agreement. Surely, one State can agree to waive jurisdictionto the extent
agreed uponto subjects of another State due to the recognition of the principle of extraterritorial immunity. What Petitioners final point revolves around the necessity of the Senates concurrence in the Agreement. And
the Court wrote in Nicolas v. Romulo[59]a case involving the implementation of the criminal jurisdiction provisions without specifically saying so, petitioner would argue that the non-surrender agreement was executed by the
of the RP-US Visiting Forces Agreementis apropos: President, thru the DFA Secretary, in grave abuse of discretion.

Nothing in the Constitution prohibits such agreements recognizing immunity from The Court need not delve on and belabor the first portion of the above posture of petitioner, the same
jurisdiction or some aspects of jurisdiction (such as custody), in relation to long-recognized having been discussed at length earlier on. As to the second portion, We wish to state that petitioner virtually faults
PAGE 80 of 135 CONSTITUTIONAL LAW -EXECUTIVE DEPARTMENT PART 2 –SESSION 12
the President for performing, through respondents, a task conferred the President by the Constitutionthe power to international tribunal is already conducting the investigation or undertaking the prosecution of such crime;
enter into international agreements. otherwise, the Philippinesmust prosecute the crime before its own courts pursuant to RA 9851.

Posing the situation of a US national under prosecution by an international tribunal for any crime under
By constitutional fiat and by the nature of his or her office, the President, as head of state and government, RA 9851, the Philippines has the option to surrender such US national to the international tribunal if it decides not
is the sole organ and authority in the external affairs of the country. [65] The Constitution vests in the President the to prosecute such US national here. The view asserts that this option of the Philippines under Sec. 17 of RA 9851
power to enter into international agreements, subject, in appropriate cases, to the required concurrence votes of is not subject to the consent of the US, and any derogation of Sec. 17 of RA 9851, such as requiring the consent
the Senate. But as earlier indicated, executive agreements may be validly entered into without such of the US before the Philippines can exercise such option, requires an amendatory law. In line with this scenario,
concurrence. As the President wields vast powers and influence, her conduct in the external affairs of the nation the view strongly argues that the Agreement prevents the Philippineswithout the consent of the USfrom
is, as Bayan would put it, executive altogether. The right of the President to enter into or ratify binding executive surrendering to any international tribunal US nationals accused of crimes covered by RA 9851, and, thus, in effect
agreements has been confirmed by long practice.[66] amends Sec. 17 of RA 9851. Consequently, the view is strongly impressed that the Agreement cannot be
embodied in a simple executive agreement in the form of an exchange of notes but must be implemented through
In thus agreeing to conclude the Agreement thru E/N BFO-028-03, then President Gloria Macapagal- an extradition law or a treaty with the corresponding formalities.
Arroyo, represented by the Secretary of Foreign Affairs, acted within the scope of the authority and discretion
vested in her by the Constitution. At the end of the day, the Presidentby ratifying, thru her deputies, the non- Moreover, consonant with the foregoing view, citing Sec. 2, Art. II of the Constitution, where
surrender agreementdid nothing more than discharge a constitutional duty and exercise a prerogative that pertains the Philippines adopts, as a national policy, the generally accepted principles of international law as part of
to her office. the law of the land, the Court is further impressed to perceive the Rome Statute as declaratory of customary
international law. In other words, the Statute embodies principles of law which constitute customary international
While the issue of ratification of the Rome Statute is not determinative of the other issues raised herein, law or custom and for which reason it assumes the status of an enforceable domestic law in the context of the
it may perhaps be pertinent to remind all and sundry that about the time this petition was interposed, such issue aforecited constitutional provision. As a corollary, it is argued that any derogation from the Rome Statute principles
of ratification was laid to rest in Pimentel, Jr. v. Office of the Executive Secretary. [67] As the Court emphasized in cannot be undertaken via a mere executive agreement, which, as an exclusive act of the executive branch, can
said case, the power to ratify a treaty, the Statute in that instance, rests with the President, subject to the only implement, but cannot amend or repeal, an existing law. The Agreement, so the argument goes, seeks to
concurrence of the Senate, whose role relative to the ratification of a treaty is limited merely to concurring in or frustrate the objects of the principles of law or alters customary rules embodied in the Rome Statute.
withholding the ratification.And concomitant with this treaty-making power of the President is his or her prerogative
to refuse to submit a treaty to the Senate; or having secured the latters consent to the ratification of the treaty, Prescinding from the foregoing premises, the view thus advanced considers the Agreement inefficacious,
refuse to ratify it.[68] This prerogative, the Court hastened to add, is the Presidents alone and cannot be encroached unless it is embodied in a treaty duly ratified with the concurrence of the Senate, the theory being that a Senate-
upon via a writ of mandamus. Barring intervening events, then, the Philippines remains to be just a signatory to ratified treaty partakes of the nature of a municipal law that can amend or supersede another law, in this instance
the Rome Statute. Under Art. 125[69] thereof, the final acts required to complete the treaty process and, thus, bring Sec. 17 of RA 9851 and the status of the Rome Statute as constitutive of enforceable domestic law under Sec. 2,
it into force, insofar as the Philippines is concerned, have yet to be done. Art. II of the Constitution.

Agreement Need Not Be in the Form of a Treaty We are unable to lend cogency to the view thus taken. For one, we find that the Agreement does not
amend or is repugnant to RA 9851. For another, the view does not clearly state what precise principles of law, if
On December 11, 2009, then President Arroyo signed into law Republic Act No. (RA) 9851, otherwise any, the Agreement alters. And for a third, it does not demonstrate in the concrete how the Agreement seeks to
known as the Philippine Act on Crimes Against International Humanitarian Law, Genocide, and Other Crimes frustrate the objectives of the principles of law subsumed in the Rome Statute.
Against Humanity. Sec. 17 of RA 9851, particularly the second paragraph thereof, provides:
Far from it, as earlier explained, the Agreement does not undermine the Rome Statute as the former
Section 17. Jurisdiction. x x x x merely reinforces the primacy of the national jurisdiction of the US and the Philippinesin prosecuting criminal
In the interest of justice, the relevant Philippine authorities may dispense with the offenses committed by their respective citizens and military personnel, among others. The jurisdiction of the ICC
investigation or prosecution of a crime punishable under this Act if another court or international pursuant to the Rome Statute over high crimes indicated thereat is clearly and unmistakably complementary to
tribunal is already conducting the investigation or undertaking the prosecution of such the national criminal jurisdiction of the signatory states.
crime. Instead, the authorities may surrender or extradite suspected or accused persons
in the Philippines to the appropriate international court, if any, or to another State Moreover, RA 9851 clearly: (1) defines and establishes the crimes against international humanitarian
pursuant to the applicable extradition laws and treaties. (Emphasis supplied.) law, genocide and other crimes against humanity; [70] (2) provides penal sanctions and criminal liability for their
commission;[71] and (3) establishes special courts for the prosecution of these crimes and for the State to exercise
primary criminal jurisdiction.[72] Nowhere in RA 9851 is there a proviso that goes against the tenor of
the Agreement.
A view is advanced that the Agreement amends existing municipal laws on the States obligation in
relation to grave crimes against the law of nations, i.e., genocide, crimes against humanity and war crimes. Relying The view makes much of the above quoted second par. of Sec. 17, RA 9851 as requiring the Philippine
on the above-quoted statutory proviso, the view posits that the Philippine is required to surrender to the proper
State to surrender to the proper international tribunal those persons accused of crimes sanctioned under said law
international tribunal those persons accused of the grave crimes defined under RA 9851, if it does not exercise its
if it does not exercise its primary jurisdiction to prosecute such persons. This view is not entirely correct, for the
primary jurisdiction to prosecute them. above quoted proviso clearly provides discretion to the Philippine State on whether to surrender or not a person
The basic premise rests on the interpretation that if it does not decide to prosecute a foreign national for
accused of the crimes under RA 9851. The statutory proviso uses the word may. It is settled doctrine in statutory
violations of RA 9851, the Philippines has only two options, to wit: (1) surrender the accused to the proper
construction that the word may denotes discretion, and cannot be construed as having mandatory effect. [73] Thus,
international tribunal; or (2) surrender the accused to another State if such surrender is pursuant to the applicable
the pertinent second pararagraph of Sec. 17, RA 9851 is simply permissive on the part of the Philippine State.
extradition laws and treaties. But the Philippines may exercise these options only in cases where another court or
PAGE 81 of 135 CONSTITUTIONAL LAW -EXECUTIVE DEPARTMENT PART 2 –SESSION 12
Besides, even granting that the surrender of a person is mandatorily required when the Philippines does (3) Which constitutes a grave breach of common Article 3 (as defined in subsection [d])
not exercise its primary jurisdiction in cases where another court or international tribunal is already conducting the when committed in the context of and in association with an armed conflict not of an
investigation or undertaking the prosecution of such crime, still, the tenor of the Agreement is not repugnant to international character; or
Sec. 17 of RA 9851. Said legal proviso aptly provides that the surrender may be made to another State pursuant (4) Of a person who, in relation to an armed conflict and contrary to the provisions of the
to the applicable extradition laws and treaties. The Agreement can already be considered a treaty following this Protocol on Prohibitions or Restrictions on the Use of Mines, Booby-Traps and Other
Courts decision in Nicolas v. Romulo[74] which cited Weinberger v. Rossi.[75] In Nicolas, We held that an executive Devices as amended at Geneva on 3 May 1996 (Protocol II as amended on 3 May
agreement is a treaty within the meaning of that word in international law and constitutes enforceable domestic 1996), when the United States is a party to such Protocol, willfully kills or causes
law vis--vis the United States.[76] serious injury to civilians.[80]

Likewise, the Philippines and the US already have an existing extradition treaty, i.e., RP-US Extradition Similarly, in December 2009, the US adopted a law that criminalized genocide, to wit:
Treaty, which was executed on November 13, 1994. The pertinent Philippine law, on the other hand, is Presidential
Decree No. 1069, issued on January 13, 1977. Thus, the Agreement, in conjunction with the RP-US Extradition 1091. Genocide
Treaty, would neither violate nor run counter to Sec. 17 of RA 9851.
(a) Basic Offense Whoever, whether in the time of peace or in time of war
The views reliance on Suplico v. Neda[77] is similarly improper. In that case, several petitions were filed and with specific intent to destroy, in whole or in substantial part, a national, ethnic,
questioning the power of the President to enter into foreign loan agreements. However, before the petitions could racial or religious group as such
be resolved by the Court, the Office of the Solicitor General filed a Manifestation and Motion averring that the (1) kills members of that group;
Philippine Government decided not to continue with the ZTE National Broadband Network Project, thus rendering (2) causes serious bodily injury to members of that group;
the petition moot. In resolving the case, the Court took judicial notice of the act of the executive department of (3) causes the permanent impairment of the mental faculties of members of the
the Philippines (the President) and found the petition to be indeed moot. Accordingly, it dismissed the petitions. group through drugs, torture, or similar techniques;
(4) subjects the group to conditions of life that are intended to cause the physical
In his dissent in the abovementioned case, Justice Carpio discussed the legal implications of an executive destruction of the group in whole or in part;
agreement. He stated that an executive agreement has the force and effect of law x x x [it] cannot amend or (5) imposes measures intended to prevent births within the group; or
repeal prior laws.[78] Hence, this argument finds no application in this case seeing as RA 9851 is a subsequent (6) transfers by force children of the group to another group;
law, not a prior one. Notably, this argument cannot be found in the ratio decidendi of the case, but only in the shall be punished as provided in subsection (b). [81]
dissenting opinion.
Arguing further, another view has been advanced that the current US laws do not cover every crime listed
The view further contends that the RP-US Extradition Treaty is inapplicable to RA 9851 for the reason within the jurisdiction of the ICC and that there is a gap between the definitions of the different crimes under
that under par. 1, Art. 2 of the RP-US Extradition Treaty, [a]n offense shall be an extraditable offense if it the US laws versus the Rome Statute. The view used a report written by Victoria K. Holt and Elisabeth W. Dallas,
is punishable under the laws in both Contracting Parties x x x,[79] and thereby concluding that while the entitled On Trial: The US Military and the International Criminal Court, as its basis.
Philippines has criminalized under RA 9851 the acts defined in the Rome Statute as war crimes, genocide and
other crimes against humanity, there is no similar legislation in the US. It is further argued that, citing U.S. v. At the outset, it should be pointed out that the report used may not have any weight or value under international
Coolidge, in the US, a person cannot be tried in the federal courts for an international crime unless Congress law. Article 38 of the Statute of the International Court of Justice (ICJ) lists the sources of international law, as
adopts a law defining and punishing the offense. follows: (1) international conventions, whether general or particular, establishing rules expressly recognized by
the contesting states; (2) international custom, as evidence of a general practice accepted as law; (3) the general
This view must fail. principles of law recognized by civilized nations; and (4) subject to the provisions of Article 59, judicial decisions
and the teachings of the most highly qualified publicists of the various nations, as subsidiary means for the
On the contrary, the US has already enacted legislation punishing the high crimes mentioned earlier. In determination of rules of law. The report does not fall under any of the foregoing enumerated sources. It cannot
fact, as early as October 2006, the US enacted a law criminalizing war crimes. Section 2441, Chapter 118, Part I, even be considered as the teachings of highly qualified publicists. A highly qualified publicist is a scholar of public
Title 18 of the United States Code Annotated (USCA) provides for the criminal offense of war crimes which is international law and the term usually refers to legal scholars or academic writers. [82] It has not been shown that
similar to the war crimes found in both the Rome Statute and RA 9851, thus: the authors[83] of this report are highly qualified publicists.

(a) Offense Whoever, whether inside or outside the United States, commits a war crime, in Assuming arguendo that the report has weight, still, the perceived gaps in the definitions of the crimes
any of the circumstances described in subsection (b), shall be fined under this title or are nonexistent. To highlight, the table below shows the definitions of genocide and war crimes under the Rome
imprisoned for life or any term of years, or both, and if death results to the victim, shall also Statute vis--vis the definitions under US laws:
be subject to the penalty of death.
(b) Circumstances The circumstances referred to in subsection (a) are that the person
committing such war crime or the victim of such war crime is a member of the Armed Forces Rome Statute US Law
of the United States or a national of the United States (as defined in Section 101 of the Article 6 1091. Genocide
Immigration and Nationality Act). Genocide
(c) Definition As used in this Section the term war crime means any conduct For the purpose of this Statute, genocide means any (a) Basic Offense Whoever, whether in the time of
(1) Defined as a grave breach in any of the international conventions signed at Geneva 12 of the following acts committed with intent to destroy, peace or in time of war and with specific intent to
August 1949, or any protocol to such convention to which the United States is a party; in whole or in part, a national, ethnical, racial or destroy, in whole or in substantial part, a national,
(2) Prohibited by Article 23, 25, 27 or 28 of the Annex to the Hague Convention IV, religious group, as such: ethnic, racial or religious group as such
Respecting the Laws and Customs of War on Land, signed 18 October 1907; (a) Killing members of the group; (1) kills members of that group;
PAGE 82 of 135 CONSTITUTIONAL LAW -EXECUTIVE DEPARTMENT PART 2 –SESSION 12
(b) Causing serious bodily or mental harm to (2) causes serious bodily injury to members of that those outlined in the UCMJ and gave strength to complementarity for the US. Small areas of
members of the group; group; potential gaps between the UCMJ and the Rome Statute, military experts argued, could be
(c) Deliberately inflicting on the group conditions of (3) causes the permanent impairment of the mental addressed through existing military laws.[87] x x x
life calculated to bring about its physical faculties of members of the group through
destruction in whole or in part; drugs, torture, or similar techniques; The report went on further to say that [a]ccording to those involved, the elements of crimes laid out in the
(d) Imposing measures intended to prevent births (4) subjects the group to conditions of life that are Rome Statute have been part of US military doctrine for decades.[88] Thus, the argument proffered cannot stand.
within the group; intended to cause the physical destruction of
(e) Forcibly transferring children of the group to the group in whole or in part; Nonetheless, despite the lack of actual domestic legislation, the US notably follows the doctrine of
another group. (5) imposes measures intended to prevent births incorporation. As early as 1900, the esteemed Justice Gray in The Paquete Habana[89]case already held
within the group; or international law as part of the law of the US, to wit:
(6) transfers by force children of the group to another
group; International law is part of our law, and must be ascertained and administered by
shall be punished as provided in subsection (b). the courts of justice of appropriate jurisdiction as often as questions of right depending upon it
Article 8 (a) Definition As used in this Section the term war are duly presented for their determination. For this purpose, where there is no treaty and no
War Crimes crime means any conduct controlling executive or legislative act or judicial decision, resort must be had to the customs
2. For the purpose of this Statute, war crimes (1) Defined as a grave breach in any of the and usages of civilized nations, and, as evidence of these, to the works of jurists and
means: international conventions signed commentators who by years of labor, research, and experience have made themselves
(a) Grave breaches of the Geneva Conventions of at Geneva12 August 1949, or any protocol to peculiarly well acquainted with the subjects of which they treat. Such works are resorted to by
12 August 1949, namely, any of the following acts such convention to which the United States is judicial tribunals, not for the speculations of their authors concerning what the law ought to be,
against persons or property protected under the a party; but for the trustworthy evidence of what the law really is. [90] (Emphasis supplied.)
provisions of the relevant Geneva Convention: x x (2) Prohibited by Article 23, 25, 27 or 28 of the
x[84] Annex to the Hague Convention IV,
(b) Other serious violations of the laws and customs Respecting the Laws and Customs of War on Thus, a person can be tried in the US for an international crime despite the lack of domestic
applicable in international armed conflict, within the Land, signed 18 October 1907; legislation. The cited ruling in U.S. v. Coolidge,[91] which in turn is based on the holding in U.S. v. Hudson,[92] only
established framework of international law, namely, (3) Which constitutes a grave breach of common applies to common law and not to the law of nations or international law. [93] Indeed, the Court in U.S. v. Hudson only
any of the following acts: Article 3 (as defined in subsection [d][85]) when considered the question, whether the Circuit Courts of the United States can exercise a common law jurisdiction
xxxx committed in the context of and in association in criminal cases.[94] Stated otherwise, there is no common law crime in the US but this is considerably different
(c) In the case of an armed conflict not of an with an armed conflict not of an international from international law.
international character, serious violations of article 3 character; or
common to the four Geneva Conventions of 12 (4) Of a person who, in relation to an armed The US doubtless recognizes international law as part of the law of the land, necessarily including
August 1949, namely, any of the following acts conflict and contrary to the provisions of the international crimes, even without any local statute.[95] In fact, years later, US courts would apply international law
committed against persons taking no active part in Protocol on Prohibitions or Restrictions on the as a source of criminal liability despite the lack of a local statute criminalizing it as such. So it was that in Ex Parte
the hostilities, including members of armed forces Use of Mines, Booby-Traps and Other Quirin[96] the US Supreme Court noted that [f]rom the very beginning of its history this Court has recognized and
who have laid down their arms and those placed Devices as amended at Geneva on 3 May applied the law of war as including that part of the law of nations which prescribes, for the conduct of war, the
hors de combat by sickness, wounds, detention or 1996 (Protocol II as amended on 3 May status, rights and duties of enemy nations as well as of enemy individuals.[97] It went on further to explain that
any other cause: 1996), when the United States is a party to Congress had not undertaken the task of codifying the specific offenses covered in the law of war, thus:
xxxx such Protocol, willfully kills or causes serious
(d) Paragraph 2 (c) applies to armed conflicts not of injury to civilians.[86] It is no objection that Congress in providing for the trial of such offenses has not itself
an international character and thus does not apply to undertaken to codify that branch of international law or to mark its precise boundaries,
situations of internal disturbances and tensions, or to enumerate or define by statute all the acts which that law condemns. An Act of
such as riots, isolated and sporadic acts of violence Congress punishing the crime of piracy as defined by the law of nations is an appropriate
or other acts of a similar nature. exercise of its constitutional authority, Art. I, s 8, cl. 10, to define and punish the offense since it
(e) Other serious violations of the laws and customs has adopted by reference the sufficiently precise definition of international law. x x x Similarly
applicable in armed conflicts not of an international by the reference in the 15th Article of War to offenders or offenses that x x x by the law of war
character, within the established framework of may be triable by such military commissions. Congress has incorporated by reference, as within
international law, namely, any of the following acts: x the jurisdiction of military commissions, all offenses which are defined as such by the law of war
x x. x x x, and which may constitutionally be included within that jurisdiction. [98] x x x (Emphasis
supplied.)
Evidently, the gaps pointed out as to the definition of the crimes are not present. In fact, the report itself stated as
much, to wit: This rule finds an even stronger hold in the case of crimes against humanity. It has been held that
genocide, war crimes and crimes against humanity have attained the status of customary international law. Some
Few believed there were wide differences between the crimes under the jurisdiction of even go so far as to state that these crimes have attained the status of jus cogens.[99]
the Court and crimes within the Uniform Code of Military Justice that would expose US
personnel to the Court. Since US military lawyers were instrumental in drafting the elements of Customary international law or international custom is a source of international law as stated in the
crimes outlined in the Rome Statute, they ensured that most of the crimes were consistent with Statute of the ICJ.[100] It is defined as the general and consistent practice of states recognized and followed by
them from a sense of legal obligation. [101] In order to establish the customary status of a particular norm, two
PAGE 83 of 135 CONSTITUTIONAL LAW -EXECUTIVE DEPARTMENT PART 2 –SESSION 12
elements must concur: State practice, the objective element; and opinio juris sive necessitates, the subjective
element.[102]
Duration therefore is not the most important element. More important is the consistency
State practice refers to the continuous repetition of the same or similar kind of acts or norms by and the generality of the practice. x x x
States.[103] It is demonstrated upon the existence of the following elements: (1) generality; (2) uniformity and
consistency; and (3) duration.[104] While, opinio juris, the psychological element, requires that the state practice or xxxx
norm be carried out in such a way, as to be evidence of a belief that this practice is rendered obligatory by the
existence of a rule of law requiring it.[105] Once the existence of state practice has been established, it becomes
necessary to determine why states behave the way they do. Do states behave the way they
The term jus cogens means the compelling law.[106] Corollary, a jus cogens norm holds the highest do because they consider it obligatory to behave thus or do they do it only as a matter of
hierarchical position among all other customary norms and principles.[107] As a result, jus cogens norms are courtesy? Opinio juris, or the belief that a certain form of behavior is obligatory, is what makes
deemed peremptory and non-derogable.[108] When applied to international crimes, jus cogens crimes have been practice an international rule. Without it, practice is not law.[116](Emphasis added.)
deemed so fundamental to the existence of a just international legal order that states cannot derogate from them,
even by agreement.[109]

These jus cogens crimes relate to the principle of universal jurisdiction, i.e., any state may exercise Evidently, there is, as yet, no overwhelming consensus, let alone prevalent practice, among the different
countries in the world that the prosecution of internationally recognized crimes of genocide, etc. should be
jurisdiction over an individual who commits certain heinous and widely condemned offenses, even when no other
handled by a particular international criminal court.
recognized basis for jurisdiction exists.[110] The rationale behind this principle is that the crime committed is so
egregious that it is considered to be committed against all members of the international community[111] and thus
Absent the widespread/consistent-practice-of-states factor, the second or the psychological element
granting every State jurisdiction over the crime. [112]
must be deemed non-existent, for an inquiry on why states behave the way they do presupposes, in the first place,
that they are actually behaving, as a matter of settled and consistent practice, in a certain manner. This implicitly
Therefore, even with the current lack of domestic legislation on the part of the US, it still has both the doctrine of
requires belief that the practice in question is rendered obligatory by the existence of a rule of law requiring
incorporation and universal jurisdiction to try these crimes.
it.[117] Like the first element, the second element has likewise not been shown to be present.
Consequently, no matter how hard one insists, the ICC, as an international tribunal, found in the Rome
Further, the Rome Statute itself rejects the concept of universal jurisdiction over the crimes enumerated
Statute is not declaratory of customary international law.
therein as evidenced by it requiring State consent. [118] Even further, the Rome Statute specifically and
unequivocally requires that: This Statute is subject to ratification, acceptance or approval by signatory
The first element of customary international law, i.e., established, widespread, and consistent practice
States.[119] These clearly negate the argument that such has already attained customary status.
on the part of States,[113] does not, under the premises, appear to be obtaining as reflected in this simple reality: As
of October 12, 2010, only 114[114] States have ratified the Rome Statute, subsequent to its coming into force eight
More importantly, an act of the executive branch with a foreign government must be afforded great
(8) years earlier, or on July 1, 2002. The fact that 114 States out of a total of 194[115] countries in the world, or
respect. The power to enter into executive agreements has long been recognized to be lodged with the President.
roughly 58.76%, have ratified the Rome Statute casts doubt on whether or not the perceived principles contained
As We held in Neri v. Senate Committee on Accountability of Public Officers and Investigations, [t]he power to
in the Statute have attained the status of customary law and should be deemed as obligatory international law. The
enter into an executive agreement is in essence an executive power. This authority of the President to enter into
numbers even tend to argue against the urgency of establishing international criminal courts envisioned in the
executive agreements without the concurrence of the Legislature has traditionally been recognized in Philippine
Rome Statute. Lest it be overlooked, the Philippines, judging by the action or inaction of its top officials, does not
even feel bound by the Rome Statute. Res ipsa loquitur. More than eight (8) years have elapsed since the jurisprudence.[120]The rationale behind this principle is the inviolable doctrine of separation of powers among the
legislative, executive and judicial branches of the government. Thus, absent any clear contravention of the law,
Philippine representative signed the Statute, but the treaty has not been transmitted to the Senate for the
courts should exercise utmost caution in declaring any executive agreement invalid.
ratification process.

And this brings us to what Fr. Bernas, S.J. aptly said respecting the application of the concurring
elements, thus:

Custom or customary international law means a general and consistent practice of


states followed by them from a sense of legal obligation [opinio juris] x x x. This statement In light of the above consideration, the position or view that the challenged RP-US Non-Surrender
Agreement ought to be in the form of a treaty, to be effective, has to be rejected.
contains the two basic elements of custom: the material factor, that is how the states behave,
and the psychological factor or subjective factor, that is, why they behave the way they do.
WHEREFORE, the petition for certiorari, mandamus and prohibition is hereby DISMISSED for lack of
merit. No costs.
xxxx

The initial factor for determining the existence of custom is the actual behavior of states. This SO ORDERED.
includes several elements: duration, consistency, and generality of the practice of states.

The required duration can be either short or long. x x x

xxxx
PAGE 84 of 135 CONSTITUTIONAL LAW -EXECUTIVE DEPARTMENT PART 2 –SESSION 12
2. The conduct of this training Exercise is in accordance with pertinent United Nations resolutions against global
EN BANC terrorism as understood by the respective parties.
[G.R. No. 151445. April 11, 2002]
ARTHUR D. LIM and PAULINO R. ERSANDO, petitioners, vs. HONORABLE EXECUTIVE SECRETARY 3. No permanent US basing and support facilities shall be established. Temporary structures such as those for
as alter ego of HER EXCELLENCY GLORIA MACAPAGAL-ARROYO, and HONORABLE ANGELO troop billeting, classroom instruction and messing may be set up for use by RP and US Forces during the
REYES in his capacity as Secretary of National Defense, respondents. Exercise.
SANLAKAS and PARTIDO NG MANGGAGAWA, petitioners-intervenors, vs. GLORIA MACAPAGAL-
ARROYO, ALBERTO ROMULO, ANGELO REYES, respondents.
4. The Exercise shall be implemented jointly by RP and US Exercise Co-Directors under the authority of the
Chief of Staff, AFP. In no instance will US Forces operate independently during field training exercises
DECISION (FTX). AFP and US Unit Commanders will retain command over their respective forces under the overall
DE LEON, JR., J.: authority of the Exercise Co-Directors. RP and US participants shall comply with operational instructions of the
APP during the FTX.

This case involves a petition for certiorari and prohibition as well as a petition-in-intervention, praying that
respondents be restrained from proceeding with the so-called Balikatan 02-1 and that after due notice and hearing, 5. The exercise shall be conducted and completed within a period of not more than six months, with the
that judgment be rendered issuing a permanent writ of injunction and/or prohibition against the deployment of U.S. projected participation of 660 US personnel and 3,800 RP Forces. The Chief of Staff, AFP shall direct the
troops in Basilan and Mindanao for being illegal and in violation of the Constitution. Exercise Co-Directors to wind up and terminate the Exercise and other activities within the six month Exercise
period.
The facts are as follows:
6. The Exercise is a mutual counter-terrorism advising, assisting and training Exercise relative to Philippine
Beginning January of this year 2002, personnel from the armed forces of the United States of America started
efforts against the ASG, and will be conducted on the Island of Basilan. Further advising, assisting and training
arriving in Mindanao to take part, in conjunction with the Philippine military, in Balikatan 02-1. These so-called
exercises shall be conducted in Malagutay and the Zamboanga area. Related activities in Cebu will be for
Balikatan exercises are the largest combined training operations involving Filipino and American troops. In theory,
support of the Exercise.
they are a simulation of joint military maneuvers pursuant to the Mutual Defense Treaty, [1] a bilateral defense
agreement entered into by the Philippines and the United States in 1951.
7. Only 160 US Forces organized in 12-man Special Forces Teams shall be deployed with AFP field
Prior to the year 2002, the last Balikatan was held in 1995. This was due to the paucity of any formal commanders. The US teams shall remain at the Battalion Headquarters and, when approved, Company Tactical
agreement relative to the treatment of United States personnel visiting the Philippines. In the meantime, the headquarters where they can observe and assess the performance of the APP Forces.
respective governments of the two countries agreed to hold joint exercises on a reduced scale. The lack of
consensus was eventually cured when the two nations concluded the Visiting Forces Agreement (VFA) in 1999.
8. US exercise participants shall not engage in combat, without prejudice to their right of self-defense.
The entry of American troops into Philippine soil is proximately rooted in the international anti-terrorism
campaign declared by President George W. Bush in reaction to the tragic events that occurred on September 11, 9. These terms of Reference are for purposes of this Exercise only and do not create additional legal obligations
2001. On that day, three (3) commercial aircrafts were hijacked, flown and smashed into the twin towers of the between the US Government and the Republic of the Philippines.
World Trade Center in New York City and the Pentagon building in Washington, D.C. by terrorists with alleged
links to the al-Qaeda (the Base), a Muslim extremist organization headed by the infamous Osama bin Laden. Of
II. EXERCISE LEVEL
no comparable historical parallels, these acts caused billions of dollars worth of destruction of property and
incalculable loss of hundreds of lives.
1. TRAINING
On February 1, 2002, petitioners Arthur D. Lim and Paulino P. Ersando filed this petition for certiorari and
prohibition, attacking the constitutionality of the joint exercise. [2] They were joined subsequently by SANLAKAS
and PARTIDO NG MANGGAGAWA, both party-list organizations, who filed a petition-in-intervention on February a. The Exercise shall involve the conduct of mutual military assisting, advising and training of RP and US Forces
11, 2002. with the primary objective of enhancing the operational capabilities of both forces to combat terrorism.

Lim and Ersando filed suit in their capacities as citizens, lawyers and taxpayers. SANLAKAS and PARTIDO, b. At no time shall US Forces operate independently within RP territory.
on the other hand, aver that certain members of their organization are residents of Zamboanga and Sulu, and
hence will be directly affected by the operations being conducted in Mindanao. They likewise pray for a relaxation
on the rules relative to locus standi citing the unprecedented importance of the issue involved. c. Flight plans of all aircraft involved in the exercise will comply with the local air traffic regulations.

On February 7, 2002 the Senate conducted a hearing on the Balikatan exercise wherein Vice-President 2. ADMINISTRATION & LOGISTICS
Teofisto T. Guingona, Jr., who is concurrently Secretary of Foreign Affairs, presented the Draft Terms of Reference
(TOR).[3] Five days later, he approved the TOR, which we quote hereunder:
a. RP and US participants shall be given a country and area briefing at the start of the Exercise. This briefing
shall acquaint US Forces on the culture and sensitivities of the Filipinos and the provisions of the VFA. The
I. POLICY LEVEL briefing shall also promote the full cooperation on the part of the RP and US participants for the successful
conduct of the Exercise.
1. The Exercise shall be Consistent with the Philippine Constitution and all its activities shall be in consonance
with the laws of the land and the provisions of the RP-US Visiting Forces Agreement (VFA).
PAGE 85 of 135 CONSTITUTIONAL LAW -EXECUTIVE DEPARTMENT PART 2 –SESSION 12
b. RP and US participating forces may share, in accordance with their respective laws and regulations, in the petitioners are premature, as they are based only on a fear of future violation of the Terms of Reference. Even
use of their resources, equipment and other assets. They will use their respective logistics channels. petitioners resort to a special civil action for certiorari is assailed on the ground that the writ may only issue on the
basis of established facts.
c. Medical evaluation shall be jointly planned and executed utilizing RP and US assets and resources. Apart from these threshold issues, the Solicitor General claims that there is actually no question of
constitutionality involved. The true object of the instant suit, it is said, is to obtain an interpretation of the VFA. The
d. Legal liaison officers from each respective party shall be appointed by the Exercise Directors. Solicitor General asks that we accord due deference to the executive determination that Balikatan 02-1 is covered
by the VFA, considering the Presidents monopoly in the field of foreign relations and her role as commander-in-
3. PUBLIC AFFAIRS chief of the Philippine armed forces.
Given the primordial importance of the issue involved, it will suffice to reiterate our view on this point in a
a. Combined RP-US Information Bureaus shall be established at the Exercise Directorate in Zamboanga City related case:
and at GHQ, AFP in Camp Aguinaldo, Quezon City.
Notwithstanding, in view of the paramount importance and the constitutional significance of the issues
b. Local media relations will be the concern of the AFP and all public affairs guidelines shall be jointly developed raised in the petitions, this Court, in the exercise of its sound discretion, brushes aside the procedural
by RP and US Forces. barrier and takes cognizance of the petitions, as we have done in the early Emergency Powers
Cases, where we had occasion to rule:
c. Socio-Economic Assistance Projects shall be planned and executed jointly by RP and US Forces in
accordance with their respective laws and regulations, and in consultation with community and local government x x x ordinary citizens and taxpayers were allowed to question the constitutionality of several
officials. executive orders issued by President Quirino although they were involving only an indirect and
general interest shared in common with the public. The Court dismissed the objection that they were
Contemporaneously, Assistant Secretary for American Affairs Minerva Jean A. Falcon and United not proper parties and ruled that transcendental importance to the public of these cases
States Charge d Affaires Robert Fitts signed the Agreed Minutes of the discussion between the Vice-President demands that they be settled promptly and definitely, brushing aside, if we must,
and Assistant Secretary Kelly.[4] technicalities of procedure. We have since then applied the exception in many other
cases. [citation omitted]
Petitioners Lim and Ersando present the following arguments:
I This principle was reiterated in the subsequent cases of Gonzales vs. COMELEC, Daza vs. Singson,
and Basco vs. Phil. Amusement and Gaming Corporation, where we emphatically held:

THE PHILIPPINES AND THE UNITED STATES SIGNED THE MUTUAL DEFENSE TREATY (MDT) in 1951 TO
PROVIDE MUTUAL MILITARY ASSISTANCE IN ACCORDANCE WITH THE CONSTITUTIONAL PROCESSES Considering however the importance to the public of the case at bar, and in keeping with the Courts
OF EACH COUNTRY ONLY IN THE CASE OF AN ARMED ATTACK BY AN EXTERNAL AGGRESSOR, duty, under the 1987 Constitution, to determine whether or not the other branches of the government
MEANING A THIRD COUNTRY AGAINST ONE OF THEM. have kept themselves within the limits of the Constitution and the laws that they have not abused the
discretion given to them, the Court has brushed aside technicalities of procedure and has taken
cognizance of this petition. xxx
BY NO STRETCH OF THE IMAGINATION CAN IT BE SAID THAT THE ABU SAYYAF BANDITS IN BASILAN
CONSTITUTE AN EXTERNAL ARMED FORCE THAT HAS SUBJECT THE PHILIPPINES TO AN ARMED
EXTERNAL ATTACK TO WARRANT U.S. MILITARY ASSISTANCE UNDER THE MDT OF 1951. Again, in the more recent case of Kilosbayan vs. Guingona, Jr., this Court ruled that in cases of
transcendental importance, the court may relax the standing requirements and allow a suit to prosper
even where there is no direct injury to the party claiming the right of judicial review.
II
Although courts generally avoid having to decide a constitutional question based on the doctrine of separation of
NEITHER DOES THE VFA OF 1999 AUTHORIZE AMERICAN SOLDIERS TO ENGAGE IN COMBAT powers, which enjoins upon the departments of the government a becoming respect for each others acts, this
OPERATIONS IN PHILIPPINE TERRITORY, NOT EVEN TO FIRE BACK IF FIRED UPON. Court nevertheless resolves to take cognizance of the instant petitions. [6]

Substantially the same points are advanced by petitioners SANLAKAS and PARTIDO. Hence, we treat with similar dispatch the general objection to the supposed prematurity of the action. At any
rate, petitioners' concerns on the lack of any specific regulation on the latitude of activity US personnel may
In his Comment, the Solicitor General points to infirmities in the petitions regarding, inter alia, Lim and
undertake and the duration of their stay has been addressed in the Terms of Reference.
Ersandos standing to file suit, the prematurity of the action, as well as the impropriety of availing of certiorari to
ascertain a question of fact. Anent their locus standi, the Solicitor General argues that first, they may not file suit The holding of Balikatan 02-1 must be studied in the framework of the treaty antecedents to which the
in their capacities as taxpayers inasmuch as it has not been shown that Balikatan 02-1 involves the exercise of Philippines bound itself. The first of these is the Mutual Defense Treaty (MDT, for brevity). The MDT has been
Congress taxing or spending powers. Second, their being lawyers does not invest them with sufficient personality described as the core of the defense relationship between the Philippines and its traditional ally, the United
to initiate the case, citing our ruling in Integrated Bar of the Philippines v. Zamora.[5] Third, Lim and Ersando States. Its aim is to enhance the strategic and technological capabilities of our armed forces through joint training
have failed to demonstrate the requisite showing of direct personal injury. We agree. with its American counterparts; the Balikatan is the largest such training exercise directly supporting the MDTs
objectives. It is this treaty to which the VFA adverts and the obligations thereunder which it seeks to reaffirm.
It is also contended that the petitioners are indulging in speculation. The Solicitor General is of the view that
since the Terms of Reference are clear as to the extent and duration of Balikatan 02-1, the issues raised by
PAGE 86 of 135 CONSTITUTIONAL LAW -EXECUTIVE DEPARTMENT PART 2 –SESSION 12
The lapse of the US-Philippine Bases Agreement in 1992 and the decision not to renew it created a vacuum Article 32
in US-Philippine defense relations, that is, until it was replaced by the Visiting Forces Agreement. It should be
recalled that on October 10, 2000, by a vote of eleven to three, this court upheld the validity of the VFA. [7] The Supplementary means of interpretation
VFA provides the regulatory mechanism by which United States military and civilian personnel [may visit]
temporarily in the Philippines in connection with activities approved by the Philippine Government. It contains
provisions relative to entry and departure of American personnel, driving and vehicle registration, criminal Recourse may be had to supplementary means of interpretation, including the preparatory work of the treaty and
jurisdiction, claims, importation and exportation, movement of vessels and aircraft, as well as the duration of the the circumstances of its conclusion, in order to confirm the meaning resulting from the application of article 31, or
agreement and its termination. It is the VFA which gives continued relevance to the MDT despite the passage of to determine the meaning when the interpretation according to article 31:
years. Its primary goal is to facilitate the promotion of optimal cooperation between American and Philippine
military forces in the event of an attack by a common foe. (a) leaves the meaning ambiguous or obscure; or
The first question that should be addressed is whether Balikatan 02-1 is covered by the Visiting Forces
Agreement. To resolve this, it is necessary to refer to the VFA itself. Not much help can be had therefrom, (b) leads to a result which is manifestly absurd or unreasonable.
unfortunately, since the terminology employed is itself the source of the problem. The VFA permits United States
personnel to engage, on an impermanent basis, in activities, the exact meaning of which was left undefined. The It is clear from the foregoing that the cardinal rule of interpretation must involve an examination of the text,
expression is ambiguous, permitting a wide scope of undertakings subject only to the approval of the Philippine which is presumed to verbalize the parties intentions. The Convention likewise dictates what may be used as aids
government.[8] The sole encumbrance placed on its definition is couched in the negative, in that United States to deduce the meaning of terms, which it refers to as the context of the treaty, as well as other elements may be
personnel must abstain from any activity inconsistent with the spirit of this agreement, and in particular, from any taken into account alongside the aforesaid context. As explained by a writer on the Convention,
political activity.[9] All other activities, in other words, are fair game.
We are not left completely unaided, however. The Vienna Convention on the Law of Treaties, which contains [t]he Commissions proposals (which were adopted virtually without change by the conference and are now
provisos governing interpretations of international agreements, state: reflected in Articles 31 and 32 of the Convention) were clearly based on the view that the text of a treaty must be
presumed to be the authentic expression of the intentions of the parties; the Commission accordingly came
down firmly in favour of the view that the starting point of interpretation is the elucidation of the meaning of the
SECTION 3. INTERPRETATION OF TREATIES text, not an investigation ab initio into the intentions of the parties. This is not to say that the travaux
prparatoires of a treaty, or the circumstances of its conclusion, are relegated to a subordinate, and wholly
Article 31 ineffective, role. As Professor Briggs points out, no rigid temporal prohibition on resort to travaux prparatoires of
a treaty was intended by the use of the phrase supplementary means of interpretation in what is now Article 32
General rule of interpretation of the Vienna Convention. The distinction between the general rule of interpretation and the supplementary
means of interpretation is intended rather to ensure that the supplementary means do not constitute an
alternative, autonomous method of interpretation divorced from the general rule. [10]
1. A treaty shall be interpreted in good faith in accordance with the ordinary meaning to be given to the terms of
the treaty in their context and in the light of its object and purpose.
The Terms of Reference rightly fall within the context of the VFA.
2. The context for the purpose of the interpretation of a treaty shall comprise, in addition to the text, including its After studied reflection, it appeared farfetched that the ambiguity surrounding the meaning of the word
preamble and annexes: activities arose from accident. In our view, it was deliberately made that way to give both parties a certain leeway
in negotiation. In this manner, visiting US forces may sojourn in Philippine territory for purposes other than
(a) any agreement relating to the treaty which was made between all the parties in connexion military. As conceived, the joint exercises may include training on new techniques of patrol and surveillance to
with the conclusion of the treaty; protect the nations marine resources, sea search-and-rescue operations to assist vessels in distress, disaster
relief operations, civic action projects such as the building of school houses, medical and humanitarian missions,
and the like.
(b) any instrument which was made by one or more parties in connexion with the conclusion of
the treaty and accepted by the other parties as an instrument related to the party. Under these auspices, the VFA gives legitimacy to the current Balikatan exercises. It is only logical to assume
that Balikatan 02-1, a mutual anti-terrorism advising, assisting and training exercise, falls under the umbrella of
3. There shall be taken into account, together with the context: sanctioned or allowable activities in the context of the agreement. Both the history and intent of the Mutual Defense
Treaty and the VFA support the conclusion that combat-related activities as opposed to combat itself such as the
one subject of the instant petition, are indeed authorized.
(a) any subsequent agreement between the parties regarding the interpretation of the treaty or
the application of its provisions; That is not the end of the matter, though. Granted that Balikatan 02-1 is permitted under the terms of the
VFA, what may US forces legitimately do in furtherance of their aim to provide advice, assistance and training in
(b) any subsequent practice in the application of the treaty which establishes the agreement of the global effort against terrorism? Differently phrased, may American troops actually engage in combat in
the parties regarding its interpretation; Philippine territory? The Terms of Reference are explicit enough. Paragraph 8 of section I stipulates that US
exercise participants may not engage in combat except in self-defense. We wryly note that this sentiment is
admirable in the abstract but difficult in implementation. The target of Balikatan 02-1, the Abu Sayyaf, cannot
(c) any relevant rules of international law applicable in the relations between the parties. reasonably be expected to sit idly while the battle is brought to their very doorstep. They cannot be expected to
pick and choose their targets for they will not have the luxury of doing so. We state this point if only to signify our
4. A special meaning shall be given to a term if it is established that the parties so intended. awareness that the parties straddle a fine line, observing the honored legal maxim Nemo potest facere per alium
PAGE 87 of 135 CONSTITUTIONAL LAW -EXECUTIVE DEPARTMENT PART 2 –SESSION 12
quod non potest facere per directum.[11] The indirect violation is actually petitioners worry, that in reality, Balikatan by a majority of the votes cast by the people in a national referendum held for that purpose, and recognized as a
02-1 is actually a war principally conducted by the United States government, and that the provision on self- treaty by the other contracting state.
defense serves only as camouflage to conceal the true nature of the exercise. A clear pronouncement on this
matter thereby becomes crucial. The aforequoted provisions betray a marked antipathy towards foreign military presence in the country, or of
In our considered opinion, neither the MDT nor the VFA allow foreign troops to engage in an offensive war foreign influence in general. Hence, foreign troops are allowed entry into the Philippines only by way of direct
on Philippine territory. We bear in mind the salutary proscription stated in the Charter of the United Nations, to wit: exception. Conflict arises then between the fundamental law and our obligations arising from international
agreements.
Article 2 A rather recent formulation of the relation of international law vis--vis municipal law was expressed in Philip
Morris, Inc. v. Court of Appeals,[13] to wit:
The Organization and its Members, in pursuit of the Purposes stated in Article 1, shall act in accordance with the
following Principles. xxx Withal, the fact that international law has been made part of the law of the land does not by any means imply
the primacy of international law over national law in the municipal sphere. Under the doctrine of incorporation as
xxx xxx xxx xxx applied in most countries, rules of international law are given a standing equal, not superior, to national
legislation.
4. All Members shall refrain in their international relations from the threat or use of force against the territorial
integrity or political independence of any state, or in any other manner inconsistent with the Purposes of the This is not exactly helpful in solving the problem at hand since in trying to find a middle ground, it favors neither
United Nations. one law nor the other, which only leaves the hapless seeker with an unsolved dilemma. Other more traditional
approaches may offer valuable insights.
xxx xxx xxx xxx From the perspective of public international law, a treaty is favored over municipal law pursuant to the
principle of pacta sunt servanda. Hence, [e]very treaty in force is binding upon the parties to it and must be
In the same manner, both the Mutual Defense Treaty and the Visiting Forces Agreement, as in all other performed by them in good faith.[14] Further, a party to a treaty is not allowed to invoke the provisions of its internal
treaties and international agreements to which the Philippines is a party, must be read in the context of the 1987 law as justification for its failure to perform a treaty. [15]
Constitution. In particular, the Mutual Defense Treaty was concluded way before the present Charter, though it
nevertheless remains in effect as a valid source of international obligation. The present Constitution contains key Our Constitution espouses the opposing view. Witness our jurisdiction as stated in section 5 of Article VIII:
provisions useful in determining the extent to which foreign military troops are allowed in Philippine territory. Thus,
in the Declaration of Principles and State Policies, it is provided that: The Supreme Court shall have the following powers:
xxx xxx xxx xxx
xxx xxx xxx xxx
SEC. 2. The Philippines renounces war as an instrument of national policy, adopts the generally accepted
principles of international law as part of the law of the land and adheres to the policy of peace, equality, justice, (2) Review, revise, reverse, modify, or affirm on appeal or certiorari, as the law or the Rules of Court may
freedom, cooperation, and amity with all nations. provide, final judgments and order of lower courts in:

xxx xxx xxx xxx (A) All cases in which the constitutionality or validity of any treaty, international or executive agreement, law,
presidential decree, proclamation, order, instruction, ordinance, or regulation is in question.
SEC. 7. The State shall pursue an independent foreign policy. In its relations with other states the paramount
consideration shall be national sovereignty, territorial integrity, national interest, and the right to self- xxx xxx xxx xxx
determination.
In Ichong v. Hernandez,[16] we ruled that the provisions of a treaty are always subject to qualification or
amendment by a subsequent law, or that it is subject to the police power of the State. In Gonzales v.
SEC. 8. The Philippines, consistent with the national interest, adopts and pursues a policy of freedom from Hechanova,[17]
nuclear weapons in the country.
xxx As regards the question whether an international agreement may be invalidated by our courts, suffice it to
xxx xxx xxx xxx say that the Constitution of the Philippines has clearly settled it in the affirmative, by providing, in Section 2 of
The Constitution also regulates the foreign relations powers of the Chief Executive when it provides that [n]o Article VIII thereof, that the Supreme Court may not be deprived of its jurisdiction to review, revise, reverse,
treaty or international agreement shall be valid and effective unless concurred in by at least two-thirds of all the modify, or affirm on appeal, certiorari, or writ of error as the law or the rules of court may provide, final judgments
members of the Senate.[12] Even more pointedly, the Transitory Provisions state: and decrees of inferior courts in (1) All cases in which the constitutionality or validity of any treaty, law,
ordinance, or executive order or regulation is in question. In other words, our Constitution authorizes the
nullification of a treaty, not only when it conflicts with the fundamental law, but, also, when it runs counter to an
Sec. 25. After the expiration in 1991 of the Agreement between the Republic of the Philippines and the United act of Congress.
States of America concerning Military Bases, foreign military bases, troops or facilities shall not be allowed in the
Philippines except under a treaty duly concurred in by the Senate and, when the Congress so requires, ratified
PAGE 88 of 135 CONSTITUTIONAL LAW -EXECUTIVE DEPARTMENT PART 2 –SESSION 12
The foregoing premises leave us no doubt that US forces are prohibited from engaging in an offensive war Desiring further to strengthen their present efforts for collective defense for the preservation of peace and security
on Philippine territory. pending the development of a more comprehensive system of regional security in the Pacific Area,
Agreeing that nothing in this present instrument shall be considered or interpreted as in any way or sense altering
Yet a nagging question remains: are American troops actively engaged in combat alongside Filipino soldiers or diminishing any existing agreements or understandings between the United States of America and the Republic
under the guise of an alleged training and assistance exercise? Contrary to what petitioners would have us do, of the Philippines,
we cannot take judicial notice of the events transpiring down south, [18] as reported from the saturation coverage Have agreed as follows:
of the media. As a rule, we do not take cognizance of newspaper or electronic reports per se, not because of any ARTICLE I.
issue as to their truth, accuracy, or impartiality, but for the simple reason that facts must be established in The Parties undertake, as set forth in the Charter of the United Nations, to settle any international disputes in
accordance with the rules of evidence. As a result, we cannot accept, in the absence of concrete proof, petitioners which they may be involved by peaceful means in such a manner that international peace and security and justice
allegation that the Arroyo government is engaged in doublespeak in trying to pass off as a mere training exercise are not endangered and to refrain in their international relations from the threat or use of force in any manner
an offensive effort by foreign troops on native soil. The petitions invite us to speculate on what is really happening inconsistent with the purpose of the United Nations.
in Mindanao, to issue, make factual findings on matters well beyond our immediate perception, and this we are ARTICLE II.
understandably loath to do. In order more effectively to achieve the objective of this Treaty, the Parties separately and jointly by self-help and
It is all too apparent that the determination thereof involves basically a question of fact. On this point, we mutual aid will maintain and develop their individual and collective capacity to resist armed attack.
must concur with the Solicitor General that the present subject matter is not a fit topic for a special civil action ARTICLE III.
for certiorari. We have held in too many instances that questions of fact are not entertained in such a remedy. The The Parties, through their Foreign Ministers or their deputies, will consult together from time to time regarding the
sole object of the writ is to correct errors of jurisdiction or grave abuse of discretion. The phrase grave abuse of implementation of this Treaty and whenever in the opinion of either of them the territorial integrity, political
discretion has a precise meaning in law, denoting abuse of discretion too patent and gross as to amount to an independence or security of either of the Parties is threatened by external armed attack in the Pacific.
evasion of a positive duty, or a virtual refusal to perform the duty enjoined or act in contemplation of law, or where ARTICLE IV.
the power is exercised in an arbitrary and despotic manner by reason of passion and personal hostility. [19] Each Party recognizes that an armed attack in the Pacific Area on either of the Parties would be dangerous to its
own peace and safety and declares that it would act to meet the common dangers in accordance with its
In this connection, it will not be amiss to add that the Supreme Court is not a trier of facts.[20] constitutional processes.
Any such armed attack and all measures taken as a result thereof shall be immediately reported to the Security
Under the expanded concept of judicial power under the Constitution, courts are charged with the duty to Council of the United Nations. Such measures shall be terminated when the Security Council has taken the
determine whether or not there has been a grave abuse of discretion amounting to lack or excess of jurisdiction measures necessary to restore and maintain international peace and security.
on the part of any branch or instrumentality of the government.[21] From the facts obtaining, we find that the holding ARTICLE V.
of Balikatan 02-1 joint military exercise has not intruded into that penumbra of error that would otherwise call for For the purpose of Article IV, an armed attack on either of the Parties is deemed to include an attack on the
correction on our part. In other words, respondents in the case at bar have not committed grave abuse of discretion metropolitan territory of either of the Parties, or on the island territories under its jurisdiction in the Pacific or on its
amounting to lack or excess of jurisdiction. armed forces, public vessels or aircraft used in the Pacific.
ARTICLE VI.
WHEREFORE, the petition and the petition-in-intervention are hereby DISMISSED without prejudice to the
This Treaty does not affect and shall not be interpreted as affecting in any way the rights and obligations of the
filing of a new petition sufficient in form and substance in the proper Regional Trial Court.
Parties under the Charter of the United Nations or the responsibility of the United Nations for the maintenance of
SO ORDERED. international peace and security.
ARTICLE VII.
Bellosillo, Melo, Mendoza, Quisumbing, and Carpio, JJ., concur. This Treaty shall be ratified by the United States of America and the Republic of the Philippines in accordance
Davide, Jr., C.J., and Puno, J., join the main and separate opinion of J. Panganiban. with their respective constitutional processes and will come into force when instruments of ratification thereof have
Vitug, J., in the result. been exchanged by them at Manila.
Kapunan, J., see dissenting opinion. ARTICLE VIII.
Panganiban, J., see separate opinion. This Treaty shall remain in force indefinitely. Either Party may terminate it one year after notice has been given to
Ynares-Santiago, and Sandoval-Gutierrez, JJ., join the dissenting opinion of J. Kapunan. the other party.
IN WITNESS WHEREOF the undersigned Plenipotentiaries have signed this Treaty.
DONE in duplicate at Washington this thirtieth day of August, 1951.
xxx xxx xxx xxx
[1]For ready reference, the text of the treaty is reproduced herein: [2] The day before, the first petition in connection with the joint military enterprise was filed -- G.R. No. 151433,
MUTUAL DEFENSE TREATY entitled In the Matter of Declaration as Constitutional and Legal the Balikatan RP-US Military Exercises. Petitioner
BETWEEN THE REPUBLIC OF THE PHILIPPINES therein Atty. Eduardo B. Inlayo manifested that he would be perfectly comfortable should the Court merely note
AND THE UNITED STATES OF AMERICA his petition. We did not oblige him; in a Resolution dated February 12, 2002, we dismissed his petition on the
30 August 1951 grounds of insufficiency in form and substance and lack of jurisdiction. After extending a hearty Valentines greeting
The parties to this Treaty, to the Court en banc, Atty. Inlayo promised to laminate the aforesaid resolution as a testimonial of his once upon
Reaffirming their faith in the purposes and principles of the Charter of the United Nations and their desire to live a time participation in an issue of national consequence.
in peace with all peoples and all Governments, and desiring to strengthen the fabric of peace in the Pacific Area, [3] Annex 1 of the Comment.
Recalling with mutual pride the historic relationship which brought their two peoples together in a common bond [4] Annex 2 of the Comment. The Minutes state:
of sympathy and mutual ideals to fight side-by-side against imperialist aggression during the last war, Secretary Guingona and Assistant Secretary Kelly welcomed the holding of Balikatan 02-1 exercise (the Exercise)
Desiring to declare publicly and formally their sense of unity and their common determination to defend themselves and the conclusion of the Terms of Reference for the Exercise. Assistant Secretary Kelly thanked Secretary
against external armed attack, so that no potential aggressor could be under the illusion that either of them stands Guingona for Secretary Guingonas personal approval of the Terms of Reference.
alone in the Pacific Area,
PAGE 89 of 135 CONSTITUTIONAL LAW -EXECUTIVE DEPARTMENT PART 2 –SESSION 12
Both Secretary Guingona and Assistant Secretary Kelly emphasized the importance of cooperating, within the [20] Hervas v. Court of Appeals, 319 SCRA 776 (1999); Valmonte v. Court of Appeals, 303 SCRA 278 (1999).
bounds provided for by their respective constitutions and laws, in the fight against international terrorism. [21] Article VIII, section 1
Both Secretary Guingona and Assistant Secretary Kelly expressed the belief that the Exercise shall not in any way
contribute to any escalation of other conflicts in Mindanao, shall not adversely affect the progress of ongoing peace
negotiations between the Government of the Philippines and other parties, and shall not put at risk the friendly
relations between the Philippines and its neighbors as well as with other states. Secretary Guingona stated that
he had in mind the ongoing peace negotiations with the NDF and the MILF and he emphasized that it is important
to make sure that the Exercise shall not in any way hinder those negotiations.
Both Secretary Guingona and Assistant Secretary Kelly stated that they look forward to the realization of the nearly
US$100 million in security assistance for fiscal years 2001-2002 agreed upon between H.E. President Gloria
Macapagal-Arroyo and H.E. President George W. Bush last November 2001.
Secretary Guingona stated that the Philippines welcomes the assistance that the U.S. will be providing, saying
that while Filipino soldier does not lack experience, courage and determination, they could benefit from additional
knowledge and updated military technologies.
Assistant Secretary Kelly said that he is glad the U.S. is able to provide advice, assistance and training and
reiterated the policy position expressed by H.E. President George W. Bush during his State of the Nation Address
that U.S. forces are in the Philippines to advise, assist and train Philippine military forces.
Both Secretary Guingona and Assistant Secretary Kelly reiterated that, as provided in the Terms of Reference,
U.S. Forces shall not engage in combat during the Exercise, except in accordance with their right to act in self-
defense.
Both Secretary Guingona and Assistant Secretary Kelly reiterated that, pursuant to Article II of the Visiting Forces
Agreement, U.S. forces are bound to respect the laws of the Philippines during the Exercise.
Both Secretary Guingona and Assistant Secretary Kelly recognized that, pursuant to Article VI of the Visiting
Forces Agreement, both the U.S. and Philippine Governments waive any and all claims against the other for any
deaths or injuries to their military and civilian personnel from the Exercise.
Secretary Guingona and Assistant Secretary Kelly designated Ambassador Minerva Falcon and Charge d Affaires,
a.i. Robert Fitts to initial these minutes.
Both secretary Guingona and Assistant Secretary Kelly agreed to consult from time to time on matters relating to
the Exercise as well as on other matters.
[5] 338 SCRA 81, 100-101 (2000).
[6] BAYAN, et. al. v. Zamora, 342 SCRA 449 (2000).
[7] BAYAN, et. al. v. Zamora, et. al., 342 SCRA 449 (2000).
[8] Article I [Definitions], VFA.
[9] Article II [Respect for Law], VFA.
[10] I.M. SINCLAIR, THE VIENNA CONVENTION ON THE LAW OF TREATIES 71-72 (1973).
[11] No one is allowed to do indirectly what he is prohibited to do directly.
[12] Sec. 21, Art. VII.
[13] 224 SCRA 576, 593 (1993).
[14] Vienna Convention on the Law of Treaties, art. 26.
[15] Id, art. 27. However, this is without prejudice to the provisions of art. 46 of the convention, which provides:

1. A State may not invoke the fact that its consent to be bound by a treaty has been expressed in violation of a
provision of its internal law regarding competence to conclude treaties as invalidating its consent unless that
violation was manifest and concerned a rule of its internal law of fundamental importance.
2. A violation is manifest if it would be objectively evident to any State conducting itself in the manner in accordance
with normal practice and in good faith.
[16] 101 Phil. 1155, 1191 (1957).
[17] 9 SCRA 230, 242 (1963).
[18] Pertinent sections of Rule 129 provide: SECTION 1. Judicial notice, when mandatory.A court shall take judicial

notice, without the introduction of evidence, of the existence and territorial extent of states, their political history,
forms of government and symbols of nationality, the law of nations, the admiralty and maritime courts of the world
and their seals, the political constitution and history of the Philippines, the official acts of the legislative, executive
and judicial departments of the Philippines, the laws of nature, the measure of time, and the geographical divisions.
Likewise, it is also provided in the next succeeding section: SEC. 2. Judicial notice, when discretionary.A court
may take judicial notice of matters which are of public knowledge, or are capable of unquestionable demonstration,
or ought to be known to judges because of their judicial functions.
[19] Sanchez v. National Labor Relations Commission, 312 SCRA 727 (1999).
PAGE 90 of 135 CONSTITUTIONAL LAW -EXECUTIVE DEPARTMENT PART 2 –SESSION 12
Nicolas vs. Romulo, GR 175888, Feb. 11, 2009 The prosecution having presented sufficient evidence against accused L/CPL. DANIEL
J. SMITH, also of the US Marine Corps at the USS Essex, this Court hereby finds him GUILTY
DECISION BEYOND REASONABLE DOUBT of the crime of RAPE defined under Article 266-A, paragraph
1 (a) of the Revised Penal Code, as amended by R.A. 8353, and, in accordance with Article
AZCUNA, J.: 266-B, first paragraph thereof, hereby sentences him to suffer the penalty of reclusion
perpetua together with the accessory penalties provided for under Article 41 of the same Code.
These are petitions for certiorari, etc. as special civil actions and/or for review of the Decision of the Court
of Appeals in Lance Corporal Daniel J. Smith v. Hon. Benjamin T. Pozon, et al., in CA-G.R. SP No. 97212, Pursuant to Article V, paragraph No. 10, of the Visiting Forces Agreement entered into
dated January 2, 2007. by the Philippines and the United States, accused L/CPL. DANIEL J. SMITH shall serve his
sentence in the facilities that shall, thereafter, be agreed upon by appropriate Philippine and
The facts are not disputed. United States authorities. Pending agreement on such facilities, accused L/CPL. DANIEL J.
SMITH is hereby temporarily committed to the Makati City Jail.
Respondent Lance Corporal (L/CPL) Daniel Smith is a member of the United States Armed Forces. He
was charged with the crime of rape committed against a Filipina, petitioner herein, sometime on November 1, Accused L/CPL. DANIEL J. SMITH is further sentenced to indemnify complainant
2005, as follows: SUZETTE S. NICOLAS in the amount of P50,000.00 as compensatory damages
plus P50,000.00 as moral damages.
The undersigned accused LCpl. Daniel Smith, Ssgt. Chad Brian Carpentier, Dominic
Duplantis, Keith Silkwood and Timoteo L. Soriano, Jr. of the crime of Rape under Article 266-A SO ORDERED.[2]
of the Revised Penal Code, as amended by Republic Act 8353, upon a complaint under oath
filed by Suzette S. Nicolas, which is attached hereto and made an integral part hereof as Annex
A, committed as follows: As a result, the Makati court ordered Smith detained at the Makati jail until further orders.

That on or about the First (1st) day of November 2005, inside the On December 29, 2006, however, defendant Smith was taken out of the Makati jail by a contingent of
Subic Bay Freeport Zone, Olongapo City and within the jurisdiction of this Philippine law enforcement agents, purportedly acting under orders of the Department of the Interior and Local
Honorable Court, the above-named accuseds (sic), being then members of Government, and brought to a facility for detention under the control of the United States government, provided
the United States Marine Corps, except Timoteo L. Soriano, Jr., conspiring, for under new agreements between the Philippines and the United States, referred to as the Romulo-Kenney
confederating together and mutually helping one another, with lewd design Agreement of December 19, 2006 which states:
and by means of force, threat and intimidation, with abuse of superior strength
and taking advantage of the intoxication of the victim, did then and there The Government of the Republic of the Philippines and the Government of the United States of
willfully, unlawfully and feloniously sexually abuse and have sexual America agree that, in accordance with the Visiting Forces Agreement signed between our two
intercourse with or carnal knowledge of one Suzette S. Nicolas, a 22-year old nations, Lance Corporal Daniel J. Smith, United States Marine Corps, be returned to U.S.
unmarried woman inside a Starex Van with Plate No. WKF-162, owned by military custody at the U.S. Embassy in Manila.
Starways Travel and Tours, with Office address at 8900 P. Victor St.,
Guadalupe, Makati City, and driven by accused Timoteo L. Soriano, Jr., (Sgd.) KRISTIE A. KENNEY (Sgd.) ALBERTO G. ROMULO
against the will and consent of the said Suzette S. Nicolas, to her damage Representative of the United States Representative of the Republic
and prejudice. of America of the Philippines

CONTRARY TO LAW.[1] DATE: 12-19-06 DATE: December 19, 2006__

Pursuant to the Visiting Forces Agreement (VFA) between the Republic of the Philippines and the United and the Romulo-Kenney Agreement of December 22, 2006 which states:
States, entered into on February 10, 1998, the United States, at its request, was granted custody of defendant
Smith pending the proceedings. The Department of Foreign Affairs of the Republic of the Philippines and the Embassy of
the United States of America agree that, in accordance with the Visiting Forces Agreement
During the trial, which was transferred from the Regional Trial Court (RTC) of Zambales to the RTC of signed between the two nations, upon transfer of Lance Corporal Daniel J. Smith, United States
Makati for security reasons, the United States Government faithfully complied with its undertaking to bring Marine Corps, from the Makati City Jail, he will be detained at the first floor, Rowe (JUSMAG)
defendant Smith to the trial court every time his presence was required. Building, U.S. Embassy Compound in a room of approximately 10 x 12 square feet. He will be
guarded round the clock by U.S. military personnel. The Philippine police and jail authorities,
On December 4, 2006, the RTC of Makati, following the end of the trial, rendered its Decision, finding under the direct supervision of the Philippine Department of Interior and Local Government
defendant Smith guilty, thus: (DILG) will have access to the place of detention to ensure the United States is in compliance
with the terms of the VFA.
WHEREFORE, premises considered, for failure of the prosecution to adduce sufficient
evidence against accused S/SGT. CHAD BRIAN CARPENTER, L/CPL. KEITH SILKWOOD
AND L/CPL. DOMINIC DUPLANTIS, all of the US Marine Corps assigned at the USS Essex, The matter was brought before the Court of Appeals which decided on January 2, 2007, as follows:
are hereby ACQUITTED to the crime charged.
PAGE 91 of 135 CONSTITUTIONAL LAW -EXECUTIVE DEPARTMENT PART 2 –SESSION 12
WHEREFORE, all the foregoing considered, we resolved to DISMISS the petition for
having become moot.[3] Applying the provision to the situation involved in these cases, the question is whether or not the presence
of US Armed Forces in Philippine territory pursuant to the VFA is allowed under a treaty duly concurred in by the
Hence, the present actions. Senate xxx and recognized as a treaty by the other contracting State.

The petitions were heard on oral arguments on September 19, 2008, after which the parties submitted This Court finds that it is, for two reasons.
their memoranda.
First, as held in Bayan v. Zamora,[5] the VFA was duly concurred in by the Philippine Senate and has
Petitioners contend that the Philippines should have custody of defendant L/CPL Smith because, first of been recognized as a treaty by the United States as attested and certified by the duly authorized representative
all, the VFA is void and unconstitutional. of the United States government.

This issue had been raised before, and this Court resolved in favor of the constitutionality of the VFA. This The fact that the VFA was not submitted for advice and consent of the United States Senate does not
was in Bayan v. Zamora,[4] brought by Bayan, one of petitioners in the present cases. detract from its status as a binding international agreement or treaty recognized by the said State. For this is a
matter of internal United States law. Notice can be taken of the internationally known practice by the United
Against the barriers of res judicata vis--vis Bayan, and stare decisis vis--vis all the parties, the reversal States of submitting to its Senate for advice and consent agreements that are policymaking in nature, whereas
of the previous ruling is sought on the ground that the issue is of primordial importance, involving the sovereignty those that carry out or further implement these policymaking agreements are merely submitted to Congress, under
of the Republic, as well as a specific mandate of the Constitution. the provisions of the so-called CaseZablocki Act, within sixty days from ratification. [6]

The provision of the Constitution is Art. XVIII, Sec. 25 which states: The second reason has to do with the relation between the VFA and the RP-US Mutual Defense Treaty
of August 30, 1951. This earlier agreement was signed and duly ratified with the concurrence of both the Philippine
Sec. 25. After the expiration in 1991 of the Agreement between the Philippines and the Senate and the United States Senate.
United States of America concerning Military Bases, foreign military bases, troops, or facilities
shall not be allowed in the Philippines except under a treaty duly concurred in by the Senate The RP-US Mutual Defense Treaty states:[7]
and, when the Congress so requires, ratified by a majority of the votes cast by the people in a
national referendum held for that purpose, and recognized as a treaty by the other contracting MUTUAL DEFENSE TREATY BETWEEN THE REPUBLIC OF THE PHILIPPINES AND
State. THE UNITED STATES OF AMERICA. Signed at Washington, August 30, 1951.

The Parties of this Treaty


The reason for this provision lies in history and the Philippine experience in regard to the United
States military bases in the country. Reaffirming their faith in the purposes and principles of the Charter of the United Nations and
their desire to live in peace with all peoples and all governments, and desiring to strengthen the
It will be recalled that under the Philippine Bill of 1902, which laid the basis for the Philippine fabric of peace in the Pacific area.
Commonwealth and, eventually, for the recognition of independence, the United States agreed to cede to the
Philippines all the territory it acquired from Spain under the Treaty of Paris, plus a few islands later added to its Recalling with mutual pride the historic relationship which brought their two peoples together in
realm, except certain naval ports and/or military bases and facilities, which the United States retained for itself. a common bond of sympathy and mutual ideals to fight side-by-side against imperialist
aggression during the last war.
This is noteworthy, because what this means is that Clark and Subic and the other places in
the Philippines covered by the RP-US Military Bases Agreement of 1947 were not Philippine territory, as they were Desiring to declare publicly and formally their sense of unity and their common
excluded from the cession and retained by the US. determination to defend themselves against external armed attack, so that no potential
aggressor could be under the illusion that either of them stands alone in the Pacific area.
Accordingly, the Philippines had no jurisdiction over these bases except to the extent allowed by
the United States. Furthermore, the RP-US Military Bases Agreement was never advised for ratification by the Desiring further to strengthen their present efforts for collective defense for the
United States Senate, a disparity in treatment, because the Philippines regarded it as a treaty and had it concurred preservation of peace and security pending the development of a more comprehensive
in by our Senate. system of regional security in the Pacific area.

Subsequently, the United States agreed to turn over these bases to the Philippines; and with the Agreeing that nothing in this present instrument shall be considered or interpreted as in any way
expiration of the RP-US Military Bases Agreement in 1991, the territory covered by these bases were finally ceded or sense altering or diminishing any existing agreements or understandings between the
to the Philippines. Republic of the Philippinesand the United States of America.

To prevent a recurrence of this experience, the provision in question was adopted in the 1987 Have agreed as follows:
Constitution.
ARTICLE I. The parties undertake, as set forth in the Charter of the United Nations, to settle
The provision is thus designed to ensure that any agreement allowing the presence of foreign military any international disputes in which they may be involved by peaceful means in such a manner
bases, troops or facilities in Philippine territory shall be equally binding on the Philippines and the foreign sovereign that international peace and security and justice are not endangered and to refrain in their
State involved. The idea is to prevent a recurrence of the situation in which the terms and conditions governing international relation from the threat or use of force in any manner inconsistent with the purposes
the presence of foreign armed forces in our territory were binding upon us but not upon the foreign State. of the United Nations.
PAGE 92 of 135 CONSTITUTIONAL LAW -EXECUTIVE DEPARTMENT PART 2 –SESSION 12
instrument agreed upon to provide for the joint RP-US military exercises, is simply an implementing agreement to
ARTICLE II. In order more effectively to achieve the objective of this Treaty, the Parties the main RP-US Military Defense Treaty. The Preamble of the VFA states:
separately and jointly by self-help and mutual aid will maintain and develop their
individual and collective capacity to resist armed attack. The Government of the United States of America and the Government of the Republic of
the Philippines,
ARTICLE III. The Parties, through their Foreign Ministers or their deputies, will consult together
from time to time regarding the implementation of this Treaty and whenever in the opinion of Reaffirming their faith in the purposes and principles of the Charter of the United Nations and
either of them the territorial integrity, political independence or security of either of the Parties is their desire to strengthen international and regional security in the Pacific area;
threatened by external armed attack in the Pacific.
Reaffirming their obligations under the Mutual Defense Treaty of August 30, 1951;
ARTICLE IV. Each Party recognizes that an armed attack in the Pacific area on either of the
parties would be dangerous to its own peace and safety and declares that it would act to meet Noting that from time to time elements of the United States armed forces may visit the
the common dangers in accordance with its constitutional processes. Republic of the Philippines;

Any such armed attack and all measures taken as a result thereof shall be immediately reported Considering that cooperation between the United States and the Republic of
to the Security Council of the United Nations. Such measures shall be terminated when the the Philippines promotes their common security interests;
Security Council has taken the measures necessary to restore and maintain international peace
and security. Recognizing the desirability of defining the treatment of United States personnel visiting the
Republic of the Philippines;
ARTICLE V. For the purpose of Article IV, an armed attack on either of the Parties is deemed
to include an armed attack on the metropolitan territory of either of the Parties, or on the island Have agreed as follows:[9]
territories under its jurisdiction in the Pacific Ocean, its armed forces, public vessels or aircraft
in the Pacific. Accordingly, as an implementing agreement of the RP-US Mutual Defense Treaty, it was not necessary
to submit the VFA to the US Senate for advice and consent, but merely to the US Congress under the
ARTICLE VI. This Treaty does not affect and shall not be interpreted as affecting in any way the CaseZablocki Act within 60 days of its ratification. It is for this reason that the US has certified that it recognizes
rights and obligations of the Parties under the Charter of the United Nations or the responsibility the VFA as a binding international agreement, i.e., a treaty, and this substantially complies with the requirements
of the United Nations for the maintenance of international peace and security. of Art. XVIII, Sec. 25 of our Constitution. [10]

ARTICLE VII. This Treaty shall be ratified by the Republic of the Philippines and the United The provision of Art. XVIII, Sec. 25 of the Constitution, is complied with by virtue of the fact that the
Nations of America in accordance with their respective constitutional processes and will come presence of the US Armed Forces through the VFA is a presence allowed under the RP-US Mutual Defense
into force when instruments of ratification thereof have been exchanged by them at Manila. Treaty. Since the RP-US Mutual Defense Treaty itself has been ratified and concurred in by both the Philippine
Senate and the US Senate, there is no violation of the Constitutional provision resulting from such presence.
ARTICLE VIII. This Treaty shall remain in force indefinitely. Either Party may terminate it one
year after notice has been given to the other party. The VFA being a valid and binding agreement, the parties are required as a matter of international law
to abide by its terms and provisions.
IN WITHNESS WHEREOF the undersigned Plenipotentiaries have signed this Treaty.
The VFA provides that in cases of offenses committed by the members of the US Armed Forces in
DONE in duplicate at Washington this thirtieth day of August, 1951. the Philippines, the following rules apply:

For the Republic of the Philippines:


(Sgd.) CARLOS P. ROMULO Article V
(Sgd.) JOAQUIN M. ELIZALDE Criminal Jurisdiction
(Sgd.) VICENTE J. FRANCISCO
(Sgd.) DIOSDADO MACAPAGAL xxx
6. The custody of any United States personnel over whom the Philippines is to exercise
For the United States of America: jurisdiction shall immediately reside with United States military authorities, if they so request,
from the commission of the offense until completion of all judicial proceedings. United
(Sgd.) DEAN ACHESON States military authorities shall, upon formal notification by the Philippine authorities and without
(Sgd.) JOHN FOSTER DULLES delay, make such personnel available to those authorities in time for any investigative or judicial
(Sgd.) TOM CONNALLY proceedings relating to the offense with which the person has been charged. In extraordinary
(Sgd.) ALEXANDER WILEY[8] cases, the Philippine Government shall present its position to the United States Government
regarding custody, which the United States Government shall take into full account. In the event
Philippine judicial proceedings are not completed within one year, the United Statesshall be
Clearly, therefore, joint RP-US military exercises for the purpose of developing the capability to resist an relieved of any obligations under this paragraph. The one year period will not include the time
armed attack fall squarely under the provisions of the RP-US Mutual Defense Treaty. The VFA, which is the necessary to appeal. Also, the one year period will not include any time during which scheduled
PAGE 93 of 135 CONSTITUTIONAL LAW -EXECUTIVE DEPARTMENT PART 2 –SESSION 12
trial procedures are delayed because United States authorities, after timely notification by Next, the Court addresses the recent decision of the United States Supreme Court in Medellin v. Texas (
Philippine authorities to arrange for the presence of the accused, fail to do so. 552 US ___ No. 06-984, March 25, 2008), which held that treaties entered into by the United States are not
automatically part of their domestic law unless these treaties are self-executing or there is an implementing
Petitioners contend that these undertakings violate another provision of the Constitution, namely, that legislation to make them enforceable.
providing for the exclusive power of this Court to adopt rules of procedure for all courts in the Philippines (Art. VIII,
Sec. 5[5]). They argue that to allow the transfer of custody of an accused to a foreign power is to provide for a On February 3, 2009, the Court issued a Resolution, thus:
different rule of procedure for that accused, which also violates the equal protection clause of the Constitution (Art.
III, Sec. 1.). G.R. No. 175888 (Suzette Nicolas y Sombilon v. Alberto Romulo, et al.); G.R. No. 176051
(Jovito R. Salonga, et al. v. Daniel Smith, et al.); and G.R. No. 176222 (Bagong
Again, this Court finds no violation of the Constitution. Alyansang Makabayan [BAYAN], et al. v. President Gloria Macapagal-Arroyo, et al.).

The equal protection clause is not violated, because there is a substantial basis for a different treatment The parties, including the Solicitor General, are required to submit within three (3) days a
of a member of a foreign military armed forces allowed to enter our territory and all other accused. [11] Comment/Manifestation on the following points:

The rule in international law is that a foreign armed forces allowed to enter ones territory is immune from 1. What is the implication on the RP-US Visiting Forces Agreement of the recent US
local jurisdiction, except to the extent agreed upon. The Status of Forces Agreements involving foreign military Supreme Court decision in Jose Ernesto Medellin v. Texas, dated March 25, 2008, to
units around the world vary in terms and conditions, according to the situation of the parties involved, and reflect the effect that treaty stipulations that are not self-executory can only be enforced
their bargaining power. But the principle remains, i.e., the receiving State can exercise jurisdiction over the forces pursuant to legislation to carry them into effect; and that, while treaties may comprise
of the sending State only to the extent agreed upon by the parties. [12] international commitments, they are not domestic law unless Congress has enacted
implementing statutes or the treaty itself conveys an intention that it be self-executory
As a result, the situation involved is not one in which the power of this Court to adopt rules of procedure and is ratified on these terms?
is curtailed or violated, but rather one in which, as is normally encountered around the world, the laws (including
rules of procedure) of one State do not extend or apply except to the extent agreed upon to subjects of another 2. Whether the VFA is enforceable in the US as domestic law, either because it is
State due to the recognition of extraterritorial immunity given to such bodies as visiting foreign armed forces. self-executory or because there exists legislation to implement it.

Nothing in the Constitution prohibits such agreements recognizing immunity from jurisdiction or some 3. Whether the RP-US Mutual Defense Treaty of August 30, 1951 was concurred in
aspects of jurisdiction (such as custody), in relation to long-recognized subjects of such immunity like Heads of by the US Senate and, if so, is there proof of the US Senate advice and consent
State, diplomats and members of the armed forces contingents of a foreign State allowed to enter another States resolution? Peralta, J., no part.
territory. On the contrary, the Constitution states that the Philippines adopts the generally accepted principles of
international law as part of the law of the land. (Art. II, Sec. 2). After deliberation, the Court holds, on these points, as follows:

Applying, however, the provisions of VFA, the Court finds that there is a different treatment when it comes First, the VFA is a self-executing Agreement, as that term is defined in Medellin itself, because the parties
to detention as against custody. The moment the accused has to be detained, e.g., after conviction, the rule that intend its provisions to be enforceable, precisely because the Agreement is intended to carry out obligations and
governs is the following provision of the VFA: undertakings under the RP-US Mutual Defense Treaty. As a matter of fact, the VFA has been implemented and
executed, with the US faithfully complying with its obligation to produce L/CPL Smith before the court during the
Article V trial.
Criminal Jurisdiction
Secondly, the VFA is covered by implementing legislation, namely, the Case-Zablocki Act, USC Sec. 112(b),
xxx inasmuch as it is the very purpose and intent of the US Congress that executive agreements registered under this
Sec. 10. The confinement or detention by Philippine authorities of United Act within 60 days from their ratification be immediately implemented. The parties to these present cases do not
States personnel shall be carried out in facilities agreed on by question the fact that the VFA has been registered under the Case-Zablocki Act.
appropriate Philippines and United States authorities. United Statespersonnel serving
sentences in the Philippines shall have the right to visits and material assistance. In sum, therefore, the VFA differs from the Vienna Convention on Consular Relations and the Avena decision of
the International Court of Justice (ICJ), subject matter of the Medellin decision.The Convention and the ICJ
decision are not self-executing and are not registrable under the Case-Zablocki Act, and thus lack legislative
It is clear that the parties to the VFA recognized the difference between custody during the trial and implementing authority.
detention after conviction, because they provided for a specific arrangement to cover detention. And this specific
arrangement clearly states not only that the detention shall be carried out in facilities agreed on by authorities of Finally, the RP-US Mutual Defense Treaty was advised and consented to by the US Senate on March
both parties, but also that the detention shall be by Philippine authorities. Therefore, the Romulo-Kenney 20, 1952, as reflected in the US Congressional Record, 82nd Congress, Second Session, Vol. 98 Part 2, pp. 2594-
Agreements of December 19 and 22, 2006, which are agreements on the detention of the accused in the United 2595.
States Embassy, are not in accord with the VFA itself because such detention is not by Philippine authorities.
The framers of the Constitution were aware that the application of international law in domestic courts
Respondents should therefore comply with the VFA and negotiate with representatives of the United varies from country to country.
States towards an agreement on detention facilities under Philippine authorities as mandated by Art. V, Sec. 10
of the VFA. As Ward N. Ferdinandusse states in his Treatise, DIRECT APPLICATION OF INTERNATIONAL
CRIMINAL LAW IN NATIONAL COURTS, some countries require legislation whereas others do not.
PAGE 94 of 135 CONSTITUTIONAL LAW -EXECUTIVE DEPARTMENT PART 2 –SESSION 12
It was not the intention of the framers of the 1987 Constitution, in adopting Article XVIII, Sec. 25, to require
the other contracting State to convert their system to achieve alignment and parity with ours. It was simply required
that the treaty be recognized as a treaty by the other contracting State. With that, it becomes for both parties a
binding international obligation and the enforcement of that obligation is left to the normal recourse and processes
under international law.

Furthermore, as held by the US Supreme Court in Weinberger v. Rossi,[13] an executive agreement is a


treaty within the meaning of that word in international law and constitutes enforceable domestic law vis--
vis the United States. Thus, the US Supreme Court in Weinberger enforced the provisions of the executive
agreement granting preferential employment to Filipinos in the US Bases here.

Accordingly, there are three types of treaties in the American system:

1. Art. II, Sec. 2 treaties These are advised and consented to by the US Senate in accordance
with Art. II, Sec. 2 of the US Constitution.

2. ExecutiveCongressional Agreements: These are joint agreements of the President and


Congress and need not be submitted to the Senate.

3. Sole Executive Agreements. These are agreements entered into by the President. They are
to be submitted to Congress within sixty (60) days of ratification under the provisions of the
Case-Zablocki Act, after which they are recognized by the Congress and may be implemented.

As regards the implementation of the RP-US Mutual Defense Treaty, military aid or assistance has been
given under it and this can only be done through implementing legislation. The VFA itself is another form of
implementation of its provisions.

WHEREFORE, the petitions are PARTLY GRANTED, and the Court of Appeals Decision in CA-G.R. SP
No. 97212 dated January 2, 2007 is MODIFIED. The Visiting Forces Agreement (VFA) between the Republic of
the Philippines and the United States, entered into on February 10, 1998, is UPHELD as constitutional, but the
Romulo-Kenney Agreements of December 19 and 22, 2006 are DECLARED not in accordance with the VFA,
and respondent Secretary of Foreign Affairs is hereby ordered to forthwith negotiate with the United States
representatives for the appropriate agreement on detention facilities under Philippine authorities as provided in
Art. V, Sec. 10 of the VFA, pending which the status quo shall be maintained until further orders by this Court.

The Court of Appeals is hereby directed to resolve without delay the related matters pending therein,
namely, the petition for contempt and the appeal of L/CPL Daniel Smith from the judgment of conviction.

No costs.

SO ORDERED.
PAGE 95 of 135 CONSTITUTIONAL LAW -EXECUTIVE DEPARTMENT PART 2 –SESSION 12
EN BANC SERENO, J.:

January 12, 2016 The petitions1 before this Court question the constitutionality of the Enhanced Defense Cooperation Agreement
(EDCA) between the Republic of the Philippines and the United States of America (U.S.). Petitioners allege that
G.R. No. 212426 respondents committed grave abuse of discretion amounting to lack or excess of jurisdiction when they entered
into EDCA with the U.S.,2 claiming that the instrument violated multiple constitutional provisions. 3 In reply,
respondents argue that petitioners lack standing to bring the suit. To support the legality of their actions,
RENE A.V. SAGUISAG, WIGBERTO E. TAÑADA, FRANCISCO "DODONG" NEMENZO, JR., SR. MARY respondents invoke the 1987 Constitution, treaties, and judicial precedents. 4
JOHN MANANZAN, PACIFICO A. AGABIN, ESTEBAN "STEVE" SALONGA, H. HARRY L. ROQUE, JR.,
EVALYN G. URSUA, EDRE U. OLALIA, DR. CAROL PAGADUAN-ARAULLO, DR. ROLAND SIMBULAN,
AND TEDDY CASIÑO, Petitioners, A proper analysis of the issues requires this Court to lay down at the outset the basic parameters of the
vs. constitutional powers and roles of the President and the Senate in respect of the above issues. A more detailed
EXECUTIVE SECRETARY PAQUITO N. OCHOA, JR., DEPARTMENT OF NATIONAL DEFENSE discussion of these powers and roles will be made in the latter portions.
SECRETARY VOLTAIRE GAZMIN, DEPARTMENT OF FOREIGN AFFAIRS SECRETARY ALBERT DEL
ROSARIO, JR., DEPARTMENT OF BUDGET AND MANAGEMENT SECRETARY FLORENCIO ABAD, AND I. BROAD CONSTITUTIONAL CONTEXT OF THE POWERS OF THE PRESIDENT: DEFENSE, FOREIGN
ARMED FORCES OF THE PHILIPPINES CHIEF OF STAFF GENERAL EMMANUEL T. RELATIONS, AND EDCA
BAUTISTA, Respondents.
A. The Prime Duty of the State and the Consolidation of Executive Power in the President
x-----------------------x
Mataimtim kong pinanunumpaan (o pinatotohanan) na tutuparin ko nang buong katapatan at sigasig ang aking
G.R. No. 212444 mga tungkulin bilang Pangulo (o Pangalawang Pangulo o Nanunungkulang Pangulo) ng Pilipinas,
pangangalagaan at ipagtatanggol ang kanyang Konstitusyon, ipatutupad ang mga batas nito, magiging
BAGONG ALYANSANG MAKABAYAN (BAYAN), REPRESENTED BY ITS SECRETARY GENERAL makatarungan sa bawat tao, at itatalaga ang aking sarili sa paglilingkod sa Bansa. Kasihan nawa aka ng Diyos.
RENATO M. REYES, JR., BAYAN MUNA PARTY-LIST REPRESENTATIVES NERI J. COLMENARES AND
CARLOS ZARATE, GABRIELA WOMEN'S PARTY-LIST REPRESENTATIVES LUZ ILAGAN AND - Panunumpa sa Katungkulan ng Pangulo ng Pilipinas ayon sa Saligang Batas 5
EMERENCIANA DE JESUS, ACT TEACHERS PARTY-LIST REPRESENTATIVE ANTONIO L. TINIO,
ANAKPAWIS PARTY-LIST REPRESENTATIVE FERNANDO HICAP, KABATAAN PARTY-LIST The 1987 Constitution has "vested the executive power in the President of the Republic of the
REPRESENTATIVE TERRY RIDON, MAKABAYANG KOALISYON NG MAMAMAYAN (MAKABAYAN), Philippines."6 While the vastness of the executive power that has been consolidated in the person of the
REPRESENTED BY SATURNINO OCAMPO AND LIZA MAZA, BIENVENIDO LUMBERA, JOEL C. President cannot be expressed fully in one provision, the Constitution has stated the prime duty of the
LAMANGAN, RAFAEL MARIANO, SALVADOR FRANCE, ROGELIO M. SOLUTA, AND CLEMENTE G. government, of which the President is the head:
BAUTISTA, Petitioners,
vs.
DEPARTMENT OF NATIONAL DEFENSE (DND) SECRETARY VOLTAIRE GAZMIN, DEPARTMENT OF The prime duty of the Government is to serve and protect the people. The Government may call upon the
FOREIGN AFFAIRS SECRETARY ALBERT DEL ROSARIO, EXECUTIVE SECRETARY PAQUITO N. people to defend the State and, in the fulfillment thereof, all citizens may be required, under conditions provided
OCHOA, JR., ARMED FORCES OF THE PHILIPPINES CHIEF OF STAFF GENERAL EMMANUEL T. by law, to render personal military or civil service. 7 (Emphases supplied)
BAUTISTA, DEFENSE UNDERSECRETARY PIO LORENZO BATINO, AMBASSADOR LOURDES
YPARRAGUIRRE, AMBASSADOR J. EDUARDO MALAYA, DEPARTMENT OF JUSTICE B. The duty to protect the territory and the citizens of the Philippines, the power to call upon the people
UNDERSECRETARY FRANCISCO BARAAN III, AND DND ASSISTANT SECRETARY FOR STRATEGIC to defend the State, and the President as Commander-in-Chief
ASSESSMENTS RAYMUND JOSE QUILOP AS CHAIRPERSON AND MEMBERS, RESPECTIVELY, OF THE
NEGOTIATING PANEL FOR THE PHILIPPINES ON EDCA, Respondents. The duty to protect the State and its people must be carried out earnestly and effectively throughout the whole
territory of the Philippines in accordance with the constitutional provision on national territory. Hence, the
x-----------------------x President of the Philippines, as the sole repository of executive power, is the guardian of the Philippine
archipelago, including all the islands and waters embraced therein and all other territories over which it has
KILUSANG MAYO UNO, REPRESENTED BY ITS CHAIRPERSON, ELMER LABOG, CONFEDERATION sovereignty or jurisdiction. These territories consist of its terrestrial, fluvial, and aerial domains; including its
FOR UNITY, RECOGNITION AND ADVANCEMENT OF GOVERNMENT EMPLOYEES (COURAGE), territorial sea, the seabed, the subsoil, the insular shelves, and other submarine areas; and the waters around,
REPRESENTED BY ITS NATIONAL PRESIDENT FERDINAND GAITE, NATIONAL FEDERATION OF between, and connecting the islands of the archipelago, regardless of their breadth and dimensions.8
LABOR UNIONS-KILUSANG MAYO UNO, REPRESENTED BY ITS NATIONAL PRESIDENT JOSELITO
USTAREZ, NENITA GONZAGA, VIOLETA ESPIRITU, VIRGINIA FLORES, AND ARMANDO TEODORO, To carry out this important duty, the President is equipped with authority over the Armed Forces of the
JR., Petitioners-in-Intervention, Philippines (AFP),9 which is the protector of the people and the state. The AFP's role is to secure the
RENE A.Q. SAGUISAG, JR., Petitioner-in-Intervention. sovereignty of the State and the integrity of the national territory. 10 In addition, the Executive is constitutionally
empowered to maintain peace and order; protect life, liberty, and property; and promote the general welfare. 11
DECISION
PAGE 96 of 135 CONSTITUTIONAL LAW -EXECUTIVE DEPARTMENT PART 2 –SESSION 12
In recognition of these powers, Congress has specified that the President must oversee, ensure, and reinforce The responsibility of the President when it comes to treaties and international agreements under the present
our defensive capabilities against external and internal threats 12 and, in the same vein, ensure that the country is Constitution is therefore shared with the Senate. This shared role, petitioners claim, is bypassed by EDCA.
adequately prepared for all national and local emergencies arising from natural and man-made disasters.13
II. HISTORICAL ANTECEDENTS OF EDCA
To be sure, this power is limited by the Constitution itself. To illustrate, the President may call out the AFP to
prevent or suppress instances of lawless violence, invasion or rebellion,14 but not suspend the privilege of the A. U.S. takeover of Spanish colonization and its military bases, and the transition to Philippine
writ of habeas corpus for a period exceeding 60 days, or place the Philippines or any part thereof under martial independence
law exceeding that same span. In the exercise of these powers, the President is also duty-bound to submit a
report to Congress, in person or in writing, within 48 hours from the proclamation of martial law or the
suspension of the privilege of the writ of habeas corpus; and Congress may in turn revoke the proclamation or The presence of the U.S. military forces in the country can be traced to their pivotal victory in the 1898 Battle of
suspension. The same provision provides for the Supreme Court's review of the factual basis for the Manila Bay during the Spanish-American War.24 Spain relinquished its sovereignty over the Philippine Islands in
proclamation or suspension, as well as the promulgation of the decision within 30 days from filing. favor of the U.S. upon its formal surrender a few months later.25 By 1899, the Americans had consolidated a
military administration in the archipelago.26
C. The power and duty to conduct foreign relations
When it became clear that the American forces intended to impose colonial control over the Philippine Islands,
General Emilio Aguinaldo immediately led the Filipinos into an all-out war against the U.S.27 The Filipinos were
The President also carries the mandate of being the sole organ in the conduct of foreign relations.15 Since every ultimately defeated in the Philippine-American War, which lasted until 1902 and led to the downfall of the first
state has the capacity to interact with and engage in relations with other sovereign states,16 it is but logical that Philippine Republic.28 The Americans henceforth began to strengthen their foothold in the country. 29 They took
every state must vest in an agent the authority to represent its interests to those other sovereign states. over and expanded the former Spanish Naval Base in Subic Bay, Zambales, and put up a cavalry post called
Fort Stotsenberg in Pampanga, now known as Clark Air Base.30
The conduct of foreign relations is full of complexities and consequences, sometimes with life and death
significance to the nation especially in times of war. It can only be entrusted to that department of government When talks of the eventual independence of the Philippine Islands gained ground, the U.S. manifested the
which can act on the basis of the best available information and can decide with decisiveness. x x x It is also the desire to maintain military bases and armed forces in the country. 31 The U.S. Congress later enacted the Hare-
President who possesses the most comprehensive and the most confidential information about foreign countries Hawes-Cutting Act of 1933, which required that the proposed constitution of an independent Philippines
for our diplomatic and consular officials regularly brief him on meaningful events all over the world. He has also recognize the right of the U.S. to maintain the latter's armed forces and military bases.32 The Philippine
unlimited access to ultra-sensitive military intelligence data. In fine, the presidential role in foreign affairs is Legislature rejected that law, as it also gave the U.S. the power to unilaterally designate any part of Philippine
dominant and the President is traditionally accorded a wider degree of discretion in the conduct of foreign affairs. territory as a permanent military or naval base of the U.S. within two years from complete independence. 33
The regularity, nay, validity of his actions are adjudged under less stringent standards, lest their judicial
repudiation lead to breach of an international obligation, rupture of state relations, forfeiture of confidence,
national embarrassment and a plethora of other problems with equally undesirable consequences. 17 The U.S. Legislature subsequently crafted another law called the Tydings-McDuffie Act or the Philippine
Independence Act of 1934. Compared to the old Hare-Hawes-Cutting Act, the new law provided for the
surrender to the Commonwealth Government of "all military and other reservations" of the U.S. government in
The role of the President in foreign affairs is qualified by the Constitution in that the Chief Executive must give the Philippines, except "naval reservations and refueling stations."34 Furthermore, the law authorized the U.S.
paramount importance to the sovereignty of the nation, the integrity of its territory, its interest, and the right of the President to enter into negotiations for the adjustment and settlement of all questions relating to naval
sovereign Filipino people to self-determination.18 In specific provisions, the President's power is also limited, or reservations and fueling stations within two years after the Philippines would have gained
at least shared, as in Section 2 of Article II on the conduct of war; Sections 20 and 21 of Article VII on foreign independence.35 Under the Tydings-McDuffie Act, the U.S. President would proclaim the American withdrawal
loans, treaties, and international agreements; Sections 4(2) and 5(2)(a) of Article VIII on the judicial review of and surrender of sovereignty over the islands 10 years after the inauguration of the new government in the
executive acts; Sections 4 and 25 of Article XVIII on treaties and international agreements entered into prior to Philippines.36 This law eventually led to the promulgation of the 1935 Philippine Constitution.
the Constitution and on the presence of foreign military troops, bases, or facilities.
The original plan to surrender the military bases changed.37 At the height of the Second World War, the
D. The relationship between the two major presidential functions and the role of the Senate Philippine and the U.S. Legislatures each passed resolutions authorizing their respective Presidents to negotiate
the matter of retaining military bases in the country after the planned withdrawal of the U.S.38 Subsequently, in
Clearly, the power to defend the State and to act as its representative in the international sphere inheres in the 1946, the countries entered into the Treaty of General Relations, in which the U.S. relinquished all control and
person of the President. This power, however, does not crystallize into absolute discretion to craft whatever sovereignty over the Philippine Islands, except the areas that would be covered by the American military bases
instrument the Chief Executive so desires. As previously mentioned, the Senate has a role in ensuring that in the country.39 This treaty eventually led to the creation of the post-colonial legal regime on which would hinge
treaties or international agreements the President enters into, as contemplated in Section 21 of Article VII of the the continued presence of U.S. military forces until 1991: the Military Bases Agreement (MBA) of 1947, the
Constitution, obtain the approval of two-thirds of its members. Military Assistance Agreement of 1947, and the Mutual Defense Treaty (MDT) of 1951.40

Previously, treaties under the 1973 Constitution required ratification by a majority of the Batasang B. Former legal regime on the presence of U.S. armed forces in the territory of an independent
Pambansa,19except in instances wherein the President "may enter into international treaties or agreements as Philippines (1946-1991)
the national welfare and interest may require."20 This left a large margin of discretion that the President could
use to bypass the Legislature altogether. This was a departure from the 1935 Constitution, which explicitly gave Soon after the Philippines was granted independence, the two countries entered into their first military
the President the power to enter into treaties only with the concurrence of two-thirds of all the Members of the arrangement pursuant to the Treaty of General Relations - the 1947 MBA.41 The Senate concurred on the
Senate.21 The 1987 Constitution returned the Senate's power22 and, with it, the legislative's traditional role in premise of "mutuality of security interest,"42 which provided for the presence and operation of 23 U.S. military
foreign affairs.23
PAGE 97 of 135 CONSTITUTIONAL LAW -EXECUTIVE DEPARTMENT PART 2 –SESSION 12
bases in the Philippines for 99 years or until the year 2046.43 The treaty also obliged the Philippines to negotiate Beginning in January 2002, U.S. military and civilian personnel started arriving in Mindanao to take part in joint
with the U.S. to allow the latter to expand the existing bases or to acquire new ones as military necessity might military exercises with their Filipino counterparts. 78 Called Balikatan, these exercises involved trainings aimed at
require.44 simulating joint military maneuvers pursuant to the MDT.79

A number of significant amendments to the 1947 MBA were made.45 With respect to its duration, the parties In the same year, the Philippines and the U.S. entered into the Mutual Logistics Support Agreement to "further
entered into the Ramos-Rusk Agreement of 1966, which reduced the term of the treaty from 99 years to a total the interoperability, readiness, and effectiveness of their respective military forces"80 in accordance with the
of 44 years or until 1991.46 Concerning the number of U.S. military bases in the country, the Bohlen-Serrano MDT, the Military Assistance Agreement of 1953, and the VFA.81 The new agreement outlined the basic terms,
Memorandum of Agreement provided for the return to the Philippines of 17 U.S. military bases covering a total conditions, and procedures for facilitating the reciprocal provision of logistics support, supplies, and services
area of 117,075 hectares.47 Twelve years later, the U.S. returned Sangley Point in Cavite City through an between the military forces of the two countries.82 The phrase "logistics support and services" includes billeting,
exchange of notes.48 Then, through the Romulo-Murphy Exchange of Notes of 1979, the parties agreed to the operations support, construction and use of temporary structures, and storage services during an approved
recognition of Philippine sovereignty over Clark and Subic Bases and the reduction of the areas that could be activity under the existing military arrangements.83 Already extended twice, the agreement will last until 2017.84
used by the U.S. military.49 The agreement also provided for the mandatory review of the treaty every five
years.50 In 1983, the parties revised the 1947 MBA through the Romualdez-Armacost Agreement.51 The revision D. The Enhanced Defense Cooperation Agreement
pertained to the operational use of the military bases by the U.S. government within the context of Philippine
sovereignty,52 including the need for prior consultation with the Philippine government on the former' s use of the
bases for military combat operations or the establishment of long-range missiles.53 EDCA authorizes the U.S. military forces to have access to and conduct activities within certain "Agreed
Locations" in the country. It was not transmitted to the Senate on the executive's understanding that to do so
was no longer necessary.85 Accordingly, in June 2014, the Department of Foreign Affairs (DFA) and the U.S.
Pursuant to the legislative authorization granted under Republic Act No. 9,54 the President also entered into the Embassy exchanged diplomatic notes confirming the completion of all necessary internal requirements for the
1947 Military Assistance Agreement55 with the U.S. This executive agreement established the conditions under agreement to enter into force in the two countries.86
which U.S. military assistance would be granted to the Philippines, 56 particularly the provision of military arms,
ammunitions, supplies, equipment, vessels, services, and training for the latter's defense forces. 57 An exchange
of notes in 1953 made it clear that the agreement would remain in force until terminated by any of the parties. 58 According to the Philippine government, the conclusion of EDCA was the result of intensive and comprehensive
negotiations in the course of almost two years. 87 After eight rounds of negotiations, the Secretary of National
Defense and the U.S. Ambassador to the Philippines signed the agreement on 28 April 2014. 88 President
To further strengthen their defense and security relationship,59 the Philippines and the U.S. next entered into the Benigno S. Aquino III ratified EDCA on 6 June 2014. 89 The OSG clarified during the oral arguments90 that the
MDT in 1951. Concurred in by both the Philippine60 and the U.S.61 Senates, the treaty has two main Philippine and the U.S. governments had yet to agree formally on the specific sites of the Agreed Locations
features: first, it allowed for mutual assistance in maintaining and developing their individual and collective mentioned in the agreement.
capacities to resist an armed attack;62 and second, it provided for their mutual self-defense in the event of an
armed attack against the territory of either party. 63 The treaty was premised on their recognition that an armed
attack on either of them would equally be a threat to the security of the other. 64 Two petitions for certiorari were thereafter filed before us assailing the constitutionality of EDCA. They primarily
argue that it should have been in the form of a treaty concurred in by the Senate, not an executive agreement.
C. Current legal regime on the presence of U.S. armed forces in the country
On 10 November 2015, months after the oral arguments were concluded and the parties ordered to file their
respective memoranda, the Senators adopted Senate Resolution No. (SR) 105. 91 The resolution expresses the
In view of the impending expiration of the 1947 MBA in 1991, the Philippines and the U.S. negotiated for a "strong sense"92 of the Senators that for EDCA to become valid and effective, it must first be transmitted to the
possible renewal of their defense and security relationship. 65 Termed as the Treaty of Friendship, Cooperation Senate for deliberation and concurrence.
and Security, the countries sought to recast their military ties by providing a new framework for their defense
cooperation and the use of Philippine installations.66 One of the proposed provisions included an arrangement in
which U.S. forces would be granted the use of certain installations within the Philippine naval base in III. ISSUES
Subic.67 On 16 September 1991, the Senate rejected the proposed treaty. 68
Petitioners mainly seek a declaration that the Executive Department committed grave abuse of discretion in
The consequent expiration of the 1947 MBA and the resulting paucity of any formal agreement dealing with the entering into EDCA in the form of an executive agreement. For this reason, we cull the issues before us:
treatment of U.S. personnel in the Philippines led to the suspension in 1995 of large-scale joint military
exercises.69In the meantime, the respective governments of the two countries agreed70 to hold joint exercises at A. Whether the essential requisites for judicial review are present
a substantially reduced level.71 The military arrangements between them were revived in 1999 when they
concluded the first Visiting Forces Agreement (VFA). 72 B. Whether the President may enter into an executive agreement on foreign military bases,
troops, or facilities
As a "reaffirm[ation] [of the] obligations under the MDT,"73 the VFA has laid down the regulatory mechanism for
the treatment of U.S. military and civilian personnel visiting the country. 74 It contains provisions on the entry and C. Whether the provisions under EDCA are consistent with the Constitution, as well as with
departure of U.S. personnel; the purpose, extent, and limitations of their activities; criminal and disciplinary existing laws and treaties
jurisdiction; the waiver of certain claims; the importation and exportation of equipment, materials, supplies, and
other pieces of property owned by the U.S. government; and the movement of U.S. military vehicles, vessels,
and aircraft into and within the country.75 The Philippines and the U.S. also entered into a second counterpart IV. DISCUSSION
agreement (VFA II), which in turn regulated the treatment of Philippine military and civilian personnel visiting the
U.S.76 The Philippine Senate concurred in the first VFA on 27 May 1999.77 A. Whether the essential requisites for judicial review have been satisfied
PAGE 98 of 135 CONSTITUTIONAL LAW -EXECUTIVE DEPARTMENT PART 2 –SESSION 12
Petitioners are hailing this Court's power of judicial review in order to strike down EDCA for violating the The power of judicial review has since been strengthened in the 1987 Constitution. The scope of that power has
Constitution. They stress that our fundamental law is explicit in prohibiting the presence of foreign military forces been extended to the determination of whether in matters traditionally considered to be within the sphere of
in the country, except under a treaty concurred in by the Senate. Before this Court may begin to analyze the appreciation of another branch of government, an exercise of discretion has been attended with grave
constitutionality or validity of an official act of a coequal branch of government, however, petitioners must show abuse.97 The expansion of this power has made the political question doctrine "no longer the insurmountable
that they have satisfied all the essential requisites for judicial review. 93 obstacle to the exercise of judicial power or the impenetrable shield that protects executive and legislative
actions from judicial inquiry or review."98
Distinguished from the general notion of judicial power, the power of judicial review specially refers to both the
authority and the duty of this Court to determine whether a branch or an instrumentality of government has acted This moderating power, however, must be exercised carefully and only if it cannot be completely avoided. We
beyond the scope of the latter's constitutional powers. 94 As articulated in Section 1, Article VIII of the stress that our Constitution is so incisively designed that it identifies the spheres of expertise within which the
Constitution, the power of judicial review involves the power to resolve cases in which the questions concern the different branches of government shall function and the questions of policy that they shall resolve.99 Since the
constitutionality or validity of any treaty, international or executive agreement, law, presidential decree, power of judicial review involves the delicate exercise of examining the validity or constitutionality of an act of a
proclamation, order, instruction, ordinance, or regulation. 95 In Angara v. Electoral Commission, this Court coequal branch of government, this Court must continually exercise restraint to avoid the risk of supplanting the
exhaustively discussed this "moderating power" as part of the system of checks and balances under the wisdom of the constitutionally appointed actor with that of its own. 100
Constitution. In our fundamental law, the role of the Court is to determine whether a branch of government has
adhered to the specific restrictions and limitations of the latter's power:96 Even as we are left with no recourse but to bare our power to check an act of a coequal branch of government -
in this case the executive - we must abide by the stringent requirements for the exercise of that power under the
The separation of powers is a fundamental principle in our system of government. It obtains not through express Constitution. Demetria v. Alba101 and Francisco v. House of Representatives102 cite the "pillars" of the limitations
provision but by actual division in our Constitution. Each department of the government has exclusive on the power of judicial review as enunciated in the concurring opinion of U.S. Supreme Court Justice Brandeis
cognizance of matters within its jurisdiction, and is supreme within its own sphere. But it does not follow in Ashwander v. Tennessee Valley Authority.103 Francisco104 redressed these "pillars" under the following
from the fact that the three powers are to be kept separate and distinct that the Constitution intended them to be categories:
absolutely unrestrained and independent of each other. The Constitution has provided for an elaborate
system of checks and balances to secure coordination in the workings of the various departments of the 1. That there be absolute necessity of deciding a case
government. x x x. And the judiciary in turn, with the Supreme Court as the final arbiter, effectively checks
the other departments in the exercise of its power to determine the law, and hence to declare executive
and legislative acts void if violative of the Constitution. 2. That rules of constitutional law shall be formulated only as required by the facts of the case

xxxx 3. That judgment may not be sustained on some other ground

As any human production, our Constitution is of course lacking perfection and perfectibility, but as much as it 4. That there be actual injury sustained by the party by reason of the operation of the statute
was within the power of our people, acting through their delegates to so provide, that instrument which is the
expression of their sovereignty however limited, has established a republican government intended to 5. That the parties are not in estoppel
operate and function as a harmonious whole, under a system of checks and balances, and subject to
specific limitations and restrictions provided in the said instrument. The Constitution sets forth in no 6. That the Court upholds the presumption of constitutionality
uncertain language the restrictions and limitations upon governmental powers and agencies. If these
restrictions and limitations are transcended it would be inconceivable if the Constitution had not
provided for a mechanism by which to direct the course of government along constitutional channels, (Emphases supplied)
for then the distribution of powers would be mere verbiage, the bill of rights mere expressions of
sentiment, and the principles of good government mere political apothegms. Certainly, the limitations and These are the specific safeguards laid down by the Court when it exercises its power of judicial
restrictions embodied in our Constitution are real as they should be in any living constitution. x x x. In our case, review.105 Guided by these pillars, it may invoke the power only when the following four stringent requirements
this moderating power is granted, if not expressly, by clear implication from section 2 of article VIII of [the 1935] are satisfied: (a) there is an actual case or controversy; (b) petitioners possess locus standi; (c) the question of
Constitution. constitutionality is raised at the earliest opportunity; and (d) the issue of constitutionality is the lis mota of the
case.106 Of these four, the first two conditions will be the focus of our discussion.
The Constitution is a definition of the powers of government. Who is to determine the nature, scope and extent
of such powers? The Constitution itself has provided for the instrumentality of the judiciary as the rational 1. Petitioners have shown the presence of an actual case or controversy.
way. And when the judiciary mediates to allocate constitutional boundaries, it does not assert any
superiority over the other departments; it does not in reality nullify or invalidate an act of the legislature,
The OSG maintains107 that there is no actual case or controversy that exists, since the Senators have not been
but only asserts the solemn and sacred obligation assigned to it by the Constitution to determine
deprived of the opportunity to invoke the privileges of the institution they are representing. It contends that the
conflicting claims of authority under the Constitution and to establish for the parties in an actual
nonparticipation of the Senators in the present petitions only confirms that even they believe that EDCA is a
controversy the rights which that instrument secures and guarantees to them. This is in truth all that is
binding executive agreement that does not require their concurrence.
involved in what is termed "judicial supremacy" which properly is the power of judicial review under the
Constitution. x x x x. (Emphases supplied)
It must be emphasized that the Senate has already expressed its position through SR 105. 108 Through the
Resolution, the Senate has taken a position contrary to that of the OSG. As the body tasked to participate in
PAGE 99 of 135 CONSTITUTIONAL LAW -EXECUTIVE DEPARTMENT PART 2 –SESSION 12
foreign affairs by ratifying treaties, its belief that EDCA infringes upon its constitutional role indicates that an that they have legal standing, they claim121 that the case they have filed is a concerned citizen's suit. But aside
actual controversy - albeit brought to the Court by non-Senators, exists. from general statements that the petitions involve the protection of a public right, and that their constitutional
rights as citizens would be violated, they fail to make any specific assertion of a particular public right that would
Moreover, we cannot consider the sheer abstention of the Senators from the present proceedings as basis for be violated by the enforcement of EDCA. For their failure to do so, the present petitions cannot be
finding that there is no actual case or controversy before us. We point out that the focus of this requirement is considered by the Court as citizens' suits that would justify a disregard of the aforementioned
the ripeness for adjudication of the matter at hand, as opposed to its being merely conjectural or requirements.
anticipatory.109 The case must involve a definite and concrete issue involving real parties with conflicting legal
rights and legal claims admitting of specific relief through a decree conclusive in nature. 110 It should not equate In claiming that they have legal standing as taxpayers, petitioners 122 aver that the implementation of EDCA
with a mere request for an opinion or advice on what the law would be upon an abstract, hypothetical, or would result in the unlawful use of public funds. They emphasize that Article X(1) refers to an appropriation of
contingent state of facts.111 As explained in Angara v. Electoral Commission:112 funds; and that the agreement entails a waiver of the payment of taxes, fees, and rentals. During the oral
arguments, however, they admitted that the government had not yet appropriated or actually disbursed public
[The] power of judicial review is limited to actual cases and controversies to be exercised after full funds for the purpose of implementing the agreement. 123 The OSG, on the other hand, maintains that petitioners
opportunity of argument by the parties, and limited further to the constitutional question raised or the very lis cannot sue as taxpayers.124Respondent explains that EDCA is neither meant to be a tax measure, nor is it
mota presented. Any attempt at abstraction could only lead to dialectics and barren legal questions and directed at the disbursement of public funds.
to sterile conclusions of wisdom, justice or expediency of legislation. More than that, courts accord the
presumption of constitutionality to legislative enactments, not only because the legislature is presumed to abide A taxpayer's suit concerns a case in which the official act complained of directly involves the illegal disbursement
by the Constitution but also because the judiciary in the determination of actual cases and controversies of public funds derived from taxation.125 Here, those challenging the act must specifically show that they have
must reflect the wisdom and justice of the people as expressed through their representatives in the sufficient interest in preventing the illegal expenditure of public money, and that they will sustain a direct injury as
executive and legislative departments of the government. (Emphases supplied) a result of the enforcement of the assailed act. 126 Applying that principle to this case, they must establish that
EDCA involves the exercise by Congress of its taxing or spending powers.127
We find that the matter before us involves an actual case or controversy that is already ripe for adjudication. The
Executive Department has already sent an official confirmation to the U.S. Embassy that "all internal We agree with the OSG that the petitions cannot qualify as taxpayers' suits. We emphasize that a taxpayers' suit
requirements of the Philippines x x x have already been complied with."113 By this exchange of diplomatic notes, contemplates a situation in which there is already an appropriation or a disbursement of public funds. 128 A
the Executive Department effectively performed the last act required under Article XII(l) of EDCA before the reading of Article X(l) of EDCA would show that there has been neither an appropriation nor an authorization of
agreement entered into force. Section 25, Article XVIII of the Constitution, is clear that the presence of foreign disbursement of funds. The cited provision reads:
military forces in the country shall only be allowed by virtue of a treaty concurred in by the Senate. Hence, the
performance of an official act by the Executive Department that led to the entry into force of an executive All obligations under this Agreement are subject to the availability of appropriated funds authorized for these
agreement was sufficient to satisfy the actual case or controversy requirement. purposes. (Emphases supplied)

2. While petitioners Saguisag et. al., do not have legal standing, they nonetheless raise issues involving This provision means that if the implementation of EDCA would require the disbursement of public funds, the
matters of transcendental importance. money must come from appropriated funds that are specifically authorized for this purpose. Under the
agreement, before there can even be a disbursement of public funds, there must first be a legislative
The question of locus standi or legal standing focuses on the determination of whether those assailing the action. Until and unless the Legislature appropriates funds for EDCA, or unless petitioners can pinpoint
governmental act have the right of appearance to bring the matter to the court for adjudication. 114 They must a specific item in the current budget that allows expenditure under the agreement, we cannot at this time
show that they have a personal and substantial interest in the case, such that they have sustained or are in rule that there is in fact an appropriation or a disbursement of funds that would justify the filing of a
immediate danger of sustaining, some direct injury as a consequence of the enforcement of the challenged taxpayers' suit.
governmental act.115 Here, "interest" in the question involved must be material - an interest that is in issue and
will be affected by the official act - as distinguished from being merely incidental or general.116 Clearly, it would Petitioners Bayan et al. also claim129 that their co-petitioners who are party-list representatives have the standing
be insufficient to show that the law or any governmental act is invalid, and that petitioners stand to suffer in some to challenge the act of the Executive Department, especially if it impairs the constitutional prerogatives, powers,
indefinite way.117 They must show that they have a particular interest in bringing the suit, and that they have and privileges of their office. While they admit that there is no incumbent Senator who has taken part in the
been or are about to be denied some right or privilege to which they are lawfully entitled, or that they are about present petition, they nonetheless assert that they also stand to sustain a derivative but substantial injury as
to be subjected to some burden or penalty by reason of the act complained of. 118 The reason why those who legislators. They argue that under the Constitution, legislative power is vested in both the Senate and the House
challenge the validity of a law or an international agreement are required to allege the existence of a personal of Representatives; consequently, it is the entire Legislative Department that has a voice in determining whether
stake in the outcome of the controversy is "to assure the concrete adverseness which sharpens the presentation or not the presence of foreign military should be allowed. They maintain that as members of the Legislature, they
of issues upon which the court so largely depends for illumination of difficult constitutional questions." 119 have the requisite personality to bring a suit, especially when a constitutional issue is raised.

The present petitions cannot qualify as citizens', taxpayers', or legislators' suits; the Senate as a body has the The OSG counters130 that petitioners do not have any legal standing to file the suits concerning the lack of
requisite standing, but considering that it has not formally filed a pleading to join the suit, as it merely conveyed Senate concurrence in EDCA. Respondent emphasizes that the power to concur in treaties and international
to the Supreme Court its sense that EDCA needs the Senate's concurrence to be valid, petitioners continue to agreements is an "institutional prerogative" granted by the Constitution to the Senate. Accordingly, the OSG
suffer from lack of standing. argues that in case of an allegation of impairment of that power, the injured party would be the Senate as an
institution or any of its incumbent members, as it is the Senate's constitutional function that is allegedly being
In assailing the constitutionality of a governmental act, petitioners suing as citizens may dodge the requirement violated.
of having to establish a direct and personal interest if they show that the act affects a public right. 120 In arguing
PAGE 100 of 135 CONSTITUTIONAL LAW -EXECUTIVE DEPARTMENT PART 2 –SESSION 12
The legal standing of an institution of the Legislature or of any of its Members has already been recognized by cognizance of the suit. It may do so in spite of the inability of the petitioners to show that they have been
this Court in a number of cases.131 What is in question here is the alleged impairment of the constitutional duties personally injured by the operation of a law or any other government act.
and powers granted to, or the impermissible intrusion upon the domain of, the Legislature or an institution
thereof.132 In the case of suits initiated by the legislators themselves, this Court has recognized their standing to While this Court has yet to thoroughly delineate the outer limits of this doctrine, we emphasize that not every
question the validity of any official action that they claim infringes the prerogatives, powers, and privileges vested other case, however strong public interest may be, can qualify as an issue of transcendental importance. Before
by the Constitution in their office.133 As aptly explained by Justice Perfecto in Mabanag v. Lopez Vito:134 it can be impelled to brush aside the essential requisites for exercising its power of judicial review, it must at the
very least consider a number of factors: (1) the character of the funds or other assets involved in the case; (2)
Being members of Congress, they are even duty bound to see that the latter act within the bounds of the the presence of a clear case of disregard of a constitutional or statutory prohibition by the public respondent
Constitution which, as representatives of the people, they should uphold, unless they are to commit a agency or instrumentality of the government; and (3) the lack of any other party that has a more direct and
flagrant betrayal of public trust. They are representatives of the sovereign people and it is their sacred duty to specific interest in raising the present questions. 141
see to it that the fundamental law embodying the will of the sovereign people is not trampled upon.
(Emphases supplied) An exhaustive evaluation of the memoranda of the parties, together with the oral arguments, shows that
petitioners have presented serious constitutional issues that provide ample justification for the Court to set aside
We emphasize that in a legislators' suit, those Members of Congress who are challenging the official act have the rule on standing. The transcendental importance of the issues presented here is rooted in the Constitution
standing only to the extent that the alleged violation impinges on their right to participate in the exercise of the itself. Section 25, Article XVIII thereof, cannot be any clearer: there is a much stricter mechanism required before
powers of the institution of which they are members. 135 Legislators have the standing "to maintain inviolate the foreign military troops, facilities, or bases may be allowed in the country. The DFA has already confirmed to the
prerogatives, powers, and privileges vested by the Constitution in their office and are allowed to sue to question U.S. Embassy that "all internal requirements of the Philippines x x x have already been complied with."142 It
the validity of any official action, which they claim infringes their prerogatives as legislators." 136 As legislators, behooves the Court in this instance to take a liberal stance towards the rule on standing and to determine
they must clearly show that there was a direct injury to their persons or the institution to which they belong. 137 forthwith whether there was grave abuse of discretion on the part of the Executive Department.

As correctly argued by respondent, the power to concur in a treaty or an international agreement is an We therefore rule that this case is a proper subject for judicial review.
institutional prerogative granted by the Constitution to the Senate, not to the entire Legislature. In Pimentel v.
Office of the Executive Secretary, this Court did not recognize the standing of one of the petitioners therein who B. Whether the President may enter into an executive agreement on foreign military bases,
was a member of the House of Representatives. The petition in that case sought to compel the transmission to troops, or facilities
the Senate for concurrence of the signed text of the Statute of the International Criminal Court. Since that
petition invoked the power of the Senate to grant or withhold its concurrence in a treaty entered into by the
Executive Department, only then incumbent Senator Pimentel was allowed to assert that authority of the Senate C. Whether the provisions under EDCA are consistent with the Constitution, as well as with
of which he was a member. existing laws and treaties

Therefore, none of the initial petitioners in the present controversy has the standing to maintain the suits Issues B and C shall be discussed together infra.
as legislators.
1. The role of the President as the executor of the law includes the duty to defend the State, for which
Nevertheless, this Court finds that there is basis for it to review the act of the Executive for the following reasons. purpose he may use that power in the conduct of foreign relations

In any case, petitioners raise issues involving matters of transcendental importance. Historically, the Philippines has mirrored the division of powers in the U.S. government. When the Philippine
government was still an agency of the Congress of the U.S., it was as an agent entrusted with powers
categorized as executive, legislative, and judicial, and divided among these three great branches. 143 By this
Petitioners138 argue that the Court may set aside procedural technicalities, as the present petition tackles issues division, the law implied that the divided powers cannot be exercised except by the department given the
that are of transcendental importance. They point out that the matter before us is about the proper exercise of power.144
the Executive Department's power to enter into international agreements in relation to that of the Senate to
concur in those agreements. They also assert that EDCA would cause grave injustice, as well as irreparable
violation of the Constitution and of the Filipino people's rights. This divide continued throughout the different versions of the Philippine Constitution and specifically vested the
supreme executive power in the Governor-General of the Philippines,145 a position inherited by the President of
the Philippines when the country attained independence. One of the principal functions of the supreme executive
The OSG, on the other hand, insists139 that petitioners cannot raise the mere fact that the present petitions is the responsibility for the faithful execution of the laws as embodied by the oath of office. 146 The oath of the
involve matters of transcendental importance in order to cure their inability to comply with the constitutional President prescribed by the 1987 Constitution reads thus:
requirement of standing. Respondent bewails the overuse of "transcendental importance" as an exception to the
traditional requirements of constitutional litigation. It stresses that one of the purposes of these requirements is
to protect the Supreme Court from unnecessary litigation of constitutional questions. I do solemnly swear (or affirm) that I will faithfully and conscientiously fulfill my duties as President (or
Vice-President or Acting President) of the Philippines, preserve and defend its Constitution, execute its laws,
do justice to every man, and consecrate myself to the service of the Nation. So help me God. (In case of
In a number of cases,140 this Court has indeed taken a liberal stance towards the requirement of legal standing, affirmation, last sentence will be omitted.)147 (Emphases supplied)
especially when paramount interest is involved. Indeed, when those who challenge the official act are able to
craft an issue of transcendental significance to the people, the Court may exercise its sound discretion and take
PAGE 101 of 135 CONSTITUTIONAL LAW -EXECUTIVE DEPARTMENT PART 2 –SESSION 12
This Court has interpreted the faithful execution clause as an obligation imposed on the President, and not a intended that the Governor-General should be saddled with the responsibility of administering the government
separate grant of power.148 Section 1 7, Article VII of the Constitution, expresses this duty in no uncertain terms and of executing the laws but shorn of the power to do so. The interests of the Philippines will be best served by
and includes it in the provision regarding the President's power of control over the executive department, viz: strict adherence to the basic principles of constitutional government.

The President shall have control of all the executive departments, bureaus, and offices. He shall ensure that the In light of this constitutional duty, it is the President's prerogative to do whatever is legal and necessary for
laws be faithfully executed. Philippine defense interests. It is no coincidence that the constitutional provision on the faithful execution clause
was followed by that on the President's commander-in-chief powers,164 which are specifically granted during
The equivalent provisions in the next preceding Constitution did not explicitly require this oath from the extraordinary events of lawless violence, invasion, or rebellion. And this duty of defending the country is
President. In the 1973 Constitution, for instance, the provision simply gives the President control over the unceasing, even in times when there is no state of lawlesss violence, invasion, or rebellion. At such times, the
ministries.149 A similar language, not in the form of the President's oath, was present in the 1935 Constitution, President has full powers to ensure the faithful execution of the laws.
particularly in the enumeration of executive functions. 150 By 1987, executive power was codified not only in the
Constitution, but also in the Administrative Code: 151 It would therefore be remiss for the President and repugnant to the faithful-execution clause of the Constitution
to do nothing when the call of the moment requires increasing the military's defensive capabilities, which could
SECTION 1. Power of Control. - The President shall have control of all the executive departments, bureaus, and include forging alliances with states that hold a common interest with the Philippines or bringing an international
offices. He shall ensure that the laws be faithfully executed. (Emphasis supplied) suit against an offending state.

Hence, the duty to faithfully execute the laws of the land is inherent in executive power and is intimately related The context drawn in the analysis above has been termed by Justice Arturo D. Brion's Dissenting Opinion as the
to the other executive functions. These functions include the faithful execution of the law in autonomous beginning of a "patent misconception."165 His dissent argues that this approach taken in analyzing the
regions;152 the right to prosecute crimes;153 the implementation of transportation projects;154 the duty to ensure President's role as executor of the laws is preceded by the duty to preserve and defend the Constitution, which
compliance with treaties, executive agreements and executive orders; 155 the authority to deport undesirable was allegedly overlooked.166
aliens;156 the conferment of national awards under the President's jurisdiction; 157 and the overall administration
and control of the executive department. 158 In arguing against the approach, however, the dissent grossly failed to appreciate the nuances of the analysis, if
read holistically and in context. The concept that the President cannot function with crippled hands and therefore
These obligations are as broad as they sound, for a President cannot function with crippled hands, but must be can disregard the need for Senate concurrence in treaties 167 was never expressed or implied. Rather, the
capable of securing the rule of law within all territories of the Philippine Islands and be empowered to do so appropriate reading of the preceding analysis shows that the point being elucidated is the reality that the
within constitutional limits. Congress cannot, for instance, limit or take over the President's power to adopt President's duty to execute the laws and protect the Philippines is inextricably interwoven with his foreign affairs
implementing rules and regulations for a law it has enacted.159 powers, such that he must resolve issues imbued with both concerns to the full extent of his powers, subject only
to the limits supplied by law. In other words, apart from an expressly mandated limit, or an implied limit by virtue
of incompatibility, the manner of execution by the President must be given utmost deference. This approach is
More important, this mandate is self-executory by virtue of its being inherently executive in nature. 160 As Justice not different from that taken by the Court in situations with fairly similar contexts.
Antonio T. Carpio previously wrote,161
Thus, the analysis portrayed by the dissent does not give the President authority to bypass constitutional
[i]f the rules are issued by the President in implementation or execution of self-executory constitutional powers safeguards and limits. In fact, it specifies what these limitations are, how these limitations are triggered, how
vested in the President, the rule-making power of the President is not a delegated legislative power. The most these limitations function, and what can be done within the sphere of constitutional duties and limitations of the
important self-executory constitutional power of the President is the President's constitutional duty and mandate President.
to "ensure that the laws be faithfully executed." The rule is that the President can execute the law without any
delegation of power from the legislature.
Justice Brion's dissent likewise misinterprets the analysis proffered when it claims that the foreign relations
power of the President should not be interpreted in isolation.168 The analysis itself demonstrates how the foreign
The import of this characteristic is that the manner of the President's execution of the law, even if not affairs function, while mostly the President's, is shared in several instances, namely in Section 2 of Article II on
expressly granted by the law, is justified by necessity and limited only by law, since the President must the conduct of war; Sections 20 and 21 of Article VII on foreign loans, treaties, and international agreements;
"take necessary and proper steps to carry into execution the law."162 Justice George Malcolm states this Sections 4(2) and 5(2)(a) of Article VIII on the judicial review of executive acts; Sections 4 and 25 of Article XVIII
principle in a grand manner:163 on treaties and international agreements entered into prior to the Constitution and on the presence of foreign
military troops, bases, or facilities.
The executive should be clothed with sufficient power to administer efficiently the affairs of state. He should have
complete control of the instrumentalities through whom his responsibility is discharged. It is still true, as said by In fact, the analysis devotes a whole subheading to the relationship between the two major presidential functions
Hamilton, that "A feeble executive implies a feeble execution of the government. A feeble execution is but and the role of the Senate in it.
another phrase for a bad execution; and a government ill executed, whatever it may be in theory, must be in
practice a bad government." The mistakes of State governments need not be repeated here.
This approach of giving utmost deference to presidential initiatives in respect of foreign affairs is not novel to the
Court. The President's act of treating EDCA as an executive agreement is not the principal power being
xxxx analyzed as the Dissenting Opinion seems to suggest. Rather, the preliminary analysis is in reference to the
expansive power of foreign affairs. We have long treated this power as something the Courts must not unduly
Every other consideration to one side, this remains certain - The Congress of the United States clearly intended restrict. As we stated recently in Vinuya v. Romulo:
that the Governor-General's power should be commensurate with his responsibility. The Congress never
PAGE 102 of 135 CONSTITUTIONAL LAW -EXECUTIVE DEPARTMENT PART 2 –SESSION 12
To be sure, not all cases implicating foreign relations present political questions, and courts certainly possess SECTION 25. After the expiration in 1991 of the Agreement between the Republic of the Philippines and the
the authority to construe or invalidate treaties and executive agreements. However, the question whether the United States of America concerning Military Bases, foreign military bases, troops, or facilities shall not be
Philippine government should espouse claims of its nationals against a foreign government is a foreign relations allowed in the Philippines except under a treaty duly concurred in by the Senate and, when the Congress so
matter, the authority for which is demonstrably committed by our Constitution not to the courts but to the political requires, ratified by a majority of the votes cast by the people in a national referendum held for that purpose, and
branches. In this case, the Executive Department has already decided that it is to the best interest of the country recognized as a treaty by the other contracting State.
to waive all claims of its nationals for reparations against Japan in the Treaty of Peace of 1951. The wisdom of
such decision is not for the courts to question. Neither could petitioners herein assail the said determination by It is quite plain that the Transitory Provisions of the 1987 Constitution intended to add to the basic requirements
the Executive Department via the instant petition for certiorari. of a treaty under Section 21 of Article VII. This means that both provisions must be read as additional limitations
to the President's overarching executive function in matters of defense and foreign relations.
In the seminal case of US v. Curtiss-Wright Export Corp., the US Supreme Court held that "[t]he President is the
sole organ of the nation in its external relations, and its sole representative with foreign relations." 3. The President, however, may enter into an executive agreement on foreign military bases, troops, or
facilities, if (a) it is not the instrument that allows the presence of foreign military bases, troops, or
It is quite apparent that if, in the maintenance of our international relations, embarrassment - facilities; or (b) it merely aims to implement an existing law or treaty.
perhaps serious embarrassment - is to be avoided and success for our aims achieved,
congressional legislation which is to be made effective through negotiation and inquiry within Again we refer to Section 25, Article XVIII of the Constitution:
the international field must often accord to the President a degree of discretion and
freedom from statutory restriction which would not be admissible where domestic
affairs alone involved. Moreover, he, not Congress, has the better opportunity of knowing the SECTION 25. After the expiration in 1991 of the Agreement between the Republic of the Philippines and the
conditions which prevail in foreign countries, and especially is this true in time of war. He has United States of America concerning Military Bases, foreign military bases, troops, or facilities shall not be
his confidential sources of information. He has his agents in the form of diplomatic, consular allowed in the Philippines except under a treaty duly concurred in by the Senate and, when the Congress
and other officials .... so requires, ratified by a majority of the votes cast by the people in a national referendum held for that purpose,
and recognized as a treaty by the other contracting State. (Emphases supplied)
This ruling has been incorporated in our jurisprudence through Bavan v. Executive
Secretary and Pimentel v. Executive Secretary; its overreaching principle was, perhaps, best articulated in In view of this provision, petitioners argue170 that EDCA must be in the form of a "treaty" duly concurred in by the
(now Chief) Justice Puno's dissent in Secretary of Justice v. Lantion: Senate. They stress that the Constitution is unambigous in mandating the transmission to the Senate of all
international agreements concluded after the expiration of the MBA in 1991 - agreements that concern the
presence of foreign military bases, troops, or facilities in the country. Accordingly, petitioners maintain that the
. . . The conduct of foreign relations is full of complexities and consequences, sometimes with Executive Department is not given the choice to conclude agreements like EDCA in the form of an executive
life and death significance to the nation especially in times of war. It can only be entrusted to agreement.
that department of government which can act on the basis of the best available information
and can decide with decisiveness .... It is also the President who possesses the most
comprehensive and the most confidential information about foreign countries for our diplomatic This is also the view of the Senate, which, through a majority vote of 15 of its members - with 1 against and 2
and consular officials regularly brief him on meaningful events all over the world. He has also abstaining - says in SR 105171 that EDCA must be submitted to the Senate in the form of a treaty for
unlimited access to ultra-sensitive military intelligence data. In fine, the presidential role in concurrence by at least two-thirds of all its members.
foreign affairs is dominant and the President is traditionally accorded a wider degree of
discretion in the conduct of foreign affairs. The regularity, nay, validity of his actions The Senate cites two constitutional provisions (Article VI, Section 21 and Article XVIII, Section 25) to support its
are adjudged under less stringent standards, lest their judicial repudiation lead to position. Compared with the lone constitutional provision that the Office of the Solicitor General (OSG) cites,
breach of an international obligation, rupture of state relations, forfeiture of confidence, which is Article XVIII, Section 4(2), which includes the constitutionality of "executive agreement(s)" among the
national embarrassment and a plethora of other problems with equally undesirable cases subject to the Supreme Court's power of judicial review, the Constitution clearly requires submission of
consequences.169 (Emphases supplied) EDCA to the Senate. Two specific provisions versus one general provision means that the specific provisions
prevail. The term "executive agreement" is "a term wandering alone in the Constitution, bereft of provenance
Understandably, this Court must view the instant case with the same perspective and understanding, knowing and an unidentified constitutional mystery."
full well the constitutional and legal repercussions of any judicial overreach.
The author of SR 105, Senator Miriam Defensor Santiago, upon interpellation even added that the MDT, which
2. The plain meaning of the Constitution prohibits the entry of foreign military bases, troops or facilities, the Executive claims to be partly implemented through EDCA, is already obsolete.
except by way of a treaty concurred in by the Senate - a clear limitation on the President's dual role as
defender of the State and as sole authority in foreign relations. There are two insurmountable obstacles to this Court's agreement with SR 105, as well as with the comment on
interpellation made by Senator Santiago.
Despite the President's roles as defender of the State and sole authority in foreign relations, the 1987
Constitution expressly limits his ability in instances when it involves the entry of foreign military bases, troops or First, the concept of "executive agreement" is so well-entrenched in this Court's pronouncements on the powers
facilities. The initial limitation is found in Section 21 of the provisions on the Executive Department: "No treaty or of the President. When the Court validated the concept of "executive agreement," it did so with full knowledge of
international agreement shall be valid and effective unless concurred in by at least two-thirds of all the Members the Senate's role in concurring in treaties. It was aware of the problematique of distinguishing when an
of the Senate." The specific limitation is given by Section 25 of the Transitory Provisions, the full text of which international agreement needed Senate concurrence for validity, and when it did not; and the Court continued to
reads as follows:
PAGE 103 of 135 CONSTITUTIONAL LAW -EXECUTIVE DEPARTMENT PART 2 –SESSION 12
validate the existence of "executive agreements" even after the 1987 Constitution.172 This follows a long line of The verb "allow" is followed by the word "in," which is a preposition used to indicate "place or position in space
similar decisions upholding the power of the President to enter into an executive agreement. 173 or anything having material extension: Within the limits or bounds of, within (any place or thing)." 183 That
something is the Philippines, which is the noun that follows.
Second, the MDT has not been rendered obsolescent, considering that as late as 2009, 174 this Court continued
to recognize its validity. It is evident that the constitutional restriction refers solely to the initial entry of the foreign military bases, troops,
or facilities. Once entry is authorized, the subsequent acts are thereafter subject only to the limitations provided
Third, to this Court, a plain textual reading of Article XIII, Section 25, inevitably leads to the conclusion that it by the rest of the Constitution and Philippine law, and not to the Section 25 requirement of validity through a
applies only to a proposed agreement between our government and a foreign government, whereby military treaty.
bases, troops, or facilities of such foreign government would be "allowed" or would "gain entry" Philippine
territory. The VFA has already allowed the entry of troops in the Philippines. This Court stated in Lim v. Executive
Secretary:
Note that the provision "shall not be allowed" is a negative injunction. This wording signifies that the President is
not authorized by law to allow foreign military bases, troops, or facilities to enter the Philippines, except under a After studied reflection, it appeared farfetched that the ambiguity surrounding the meaning of the word "activities"
treaty concurred in by the Senate. Hence, the constitutionally restricted authority pertains to the entry of the arose from accident. In our view, it was deliberately made that way to give both parties a certain leeway in
bases, troops, or facilities, and not to the activities to be done after entry. negotiation. In this manner, visiting US forces may sojourn in Philippine territory for purposes other than
military. As conceived, the joint exercises may include training on new techniques of patrol and surveillance to
Under the principles of constitutional construction, of paramount consideration is the plain meaning of the protect the nation's marine resources, sea search-and-rescue operations to assist vessels in distress, disaster
language expressed in the Constitution, or the verba legis rule.175 It is presumed that the provisions have been relief operations, civic action projects such as the building of school houses, medical and humanitarian missions,
carefully crafted in order to express the objective it seeks to attain.176 It is incumbent upon the Court to refrain and the like.
from going beyond the plain meaning of the words used in the Constitution. It is presumed that the framers and
the people meant what they said when they said it, and that this understanding was reflected in the Constitution Under these auspices, the VFA gives legitimacy to the current Balikatan exercises. It is only logical to assume
and understood by the people in the way it was meant to be understood when the fundamental law was that "Balikatan 02-1," a "mutual anti- terrorism advising, assisting and training exercise," falls under the umbrella
ordained and promulgated.177 As this Court has often said: of sanctioned or allowable activities in the context of the agreement. Both the history and intent of the Mutual
Defense Treaty and the VFA support the conclusion that combat-related activities -as opposed to combat itself-
We look to the language of the document itself in our search for its meaning. We do not of course stop there, but such as the one subject of the instant petition, are indeed authorized. 184 (Emphasis supplied)
that is where we begin. It is to be assumed that the words in which constitutional provisions are couched express
the objective sought to be attained. They are to be given their ordinary meaning except where technical Moreover, the Court indicated that the Constitution continues to govern the conduct of foreign military troops in
terms are employed in which case the significance thus attached to them prevails. As the Constitution is not the Philippines,185 readily implying the legality of their initial entry into the country.
primarily a lawyer's document, it being essential for the rule of law to obtain that it should ever be present in
the people's consciousness, its language as much as possible should be understood in the sense they The OSG emphasizes that EDCA can be in the form of an executive agreement, since it merely involves
have in common use. What it says according to the text of the provision to be construed compels acceptance "adjustments in detail" in the implementation of the MDT and the VFA.186 It points out that there are existing
and negates the power of the courts to alter it, based on the postulate that the framers and the people mean treaties between the Philippines and the U.S. that have already been concurred in by the Philippine Senate and
what they say. Thus, these are the cases where the need for construction is reduced to a have thereby met the requirements of the Constitution under Section 25. Because of the status of these prior
minimum.178(Emphases supplied) agreements, respondent emphasizes that EDCA need not be transmitted to the Senate.

It is only in those instances in which the constitutional provision is unclear, ambiguous, or silent that further The aforecited Dissenting Opinion of Justice Brion disagrees with the ponencia's application of verba
construction must be done to elicit its meaning.179 In Ang Bagong Bayani-OFW v. Commission on legis construction to the words of Article XVIII, Section 25. 187 It claims that the provision is "neither plain, nor that
Elections,180 we reiterated this guiding principle: simple."188 To buttress its disagreement, the dissent states that the provision refers to a historical incident, which
is the expiration of the 1947 MBA.189 Accordingly, this position requires questioning the circumstances that led to
it [is] safer to construe the Constitution from what appears upon its face. The proper interpretation the historical event, and the meaning of the terms under Article XVIII, Section 25.
therefore depends more on how it was understood by the people adopting it than in the framers'
understanding thereof. (Emphases supplied) This objection is quite strange. The construction technique of verba legis is not inapplicable just because a
provision has a specific historical context. In fact, every provision of the Constitution has a specific historical
The effect of this statement is surprisingly profound, for, if taken literally, the phrase "shall not be allowed in the context. The purpose of constitutional and statutory construction is to set tiers of interpretation to guide the Court
Philippines" plainly refers to the entry of bases, troops, or facilities in the country. The Oxford English as to how a particular provision functions. Verba legis is of paramount consideration, but it is not the only
Dictionary defines the word "allow" as a transitive verb that means "to permit, enable"; "to give consent to the consideration. As this Court has often said:
occurrence of or relax restraint on (an action, event, or activity)"; "to consent to the presence or attendance of (a
person)"; and, when with an adverbial of place, "to permit (a person or animal) to go, come, or be in, out, near, We look to the language of the document itself in our search for its meaning. We do not of course stop there,
etc."181 Black's Law Dictionary defines the term as one that means "[t]o grant, approve, or permit."182 but that is where we begin. It is to be assumed that the words in which constitutional provisions are couched
express the objective sought to be attained. They are to be given their ordinary meaning except where
technical terms are employed in which case the significance thus attached to them prevails. As the
Constitution is not primarily a lawyer's document, it being essential for the rule of law to obtain that it should ever
PAGE 104 of 135 CONSTITUTIONAL LAW -EXECUTIVE DEPARTMENT PART 2 –SESSION 12
be present in the people's consciousness, its language as much as possible should be understood in the (2) Review, revise, reverse, modify, or affirm on appeal or certiorari, as the law or the Rules of Court
sense they have in common use. What it says according to the text of the provision to be construed compels may provide, final judgments and orders of lower courts in:
acceptance and negates the power of the courts to alter it, based on the postulate that the framers and the
people mean what they say. Thus, these are the cases where the need for construction is reduced to a (a) All cases in which the constitutionality or
minimum.190(Emphases supplied) validity of any treaty, international or executive agreement, law, presidential decree,
proclamation, order, instruction, ordinance, or regulation is in question. (Emphases supplied)
As applied, verba legis aids in construing the ordinary meaning of terms. In this case, the phrase being
construed is "shall not be allowed in the Philippines" and not the preceding one referring to "the expiration in In Commissioner of Customs v. Eastern Sea Trading, executive agreements are defined as "international
1991 of the Agreement between the Republic of the Philippines and the United States of America concerning agreements embodying adjustments of detail carrying out well-established national policies and traditions and
Military Bases, foreign military bases, troops, or facilities." It is explicit in the wording of the provision itself that those involving arrangements of a more or less temporary nature."204 In Bayan Muna v. Romulo, this Court
any interpretation goes beyond the text itself and into the discussion of the framers, the context of the further clarified that executive agreements can cover a wide array of subjects that have various scopes and
Constitutional Commission's time of drafting, and the history of the 1947 MBA. Without reference to these purposes.205 They are no longer limited to the traditional subjects that are usually covered by executive
factors, a reader would not understand those terms. However, for the phrase "shall not be allowed in the agreements as identified in Eastern Sea Trading. The Court thoroughly discussed this matter in the following
Philippines," there is no need for such reference. The law is clear. No less than the Senate understood this when manner:
it ratified the VFA.
The categorization of subject matters that may be covered by international
4. The President may generally enter into executive agreements subject to limitations defined by the agreementsmentioned in Eastern Sea Trading is not cast in stone. x x x.
Constitution and may be in furtherance of a treaty already concurred in by the Senate.
As may be noted, almost half a century has elapsed since the Court rendered its decision in Eastern Sea
We discuss in this section why the President can enter into executive agreements. Trading. Since then, the conduct of foreign affairs has become more complex and the domain of
international law wider, as to include such subjects as human rights, the environment, and the sea. In fact, in
It would be helpful to put into context the contested language found in Article XVIII, Section 25. Its more exacting the US alone, the executive agreements executed by its President from 1980 to 2000 covered subjects such
requirement was introduced because of the previous experience of the country when its representatives felt as defense, trade, scientific cooperation, aviation, atomic energy, environmental cooperation, peace
compelled to consent to the old MBA.191 They felt constrained to agree to the MBA in fulfilment of one of the corps, arms limitation, and nuclear safety, among others. Surely, the enumeration in Eastern Sea
major conditions for the country to gain independence from the U.S. 192 As a result of that experience, a second Trading cannot circumscribe the option of each state on the matter of which the international agreement
layer of consent for agreements that allow military bases, troops and facilities in the country is now articulated in format would be convenient to serve its best interest. As Francis Sayre said in his work referred to earlier:
Article XVIII of our present Constitution.
. . . It would be useless to undertake to discuss here the large variety of executive agreements as such
This second layer of consent, however, cannot be interpreted in such a way that we completely ignore the intent concluded from time to time. Hundreds of executive agreements, other than those entered into under the
of our constitutional framers when they provided for that additional layer, nor the vigorous statements of this trade-agreement act, have been negotiated with foreign governments. . . . They cover such subjects as the
Court that affirm the continued existence of that class of international agreements called "executive inspection of vessels, navigation dues, income tax on shipping profits, the admission of civil air craft, custom
agreements." matters and commercial relations generally, international claims, postal matters, the registration of trademarks
and copyrights, etc .... (Emphases Supplied)
The power of the President to enter into binding executive agreements without Senate concurrence is already
well-established in this jurisdiction.193 That power has been alluded to in our present and past Constitutions,194 in One of the distinguishing features of executive agreements is that their validity and effectivity are not affected by
various statutes,195 in Supreme Court decisions,196 and during the deliberations of the Constitutional a lack of Senate concurrence.206 This distinctive feature was recognized as early as in Eastern Sea
Commission.197 They cover a wide array of subjects with varying scopes and purposes, 198 including those that Trading (1961), viz:
involve the presence of foreign military forces in the country. 199
Treaties are formal documents which require ratification with the approval of two-thirds of the
As the sole organ of our foreign relations200 and the constitutionally assigned chief architect of our foreign Senate. Executive agreements become binding through executive action without the need of a vote by
policy,201the President is vested with the exclusive power to conduct and manage the country's interface with the Senate or by Congress.
other states and governments. Being the principal representative of the Philippines, the Chief Executive speaks
and listens for the nation; initiates, maintains, and develops diplomatic relations with other states and xxxx
governments; negotiates and enters into international agreements; promotes trade, investments, tourism and
other economic relations; and settles international disputes with other states. 202
[T]he right of the Executive to enter into binding agreements without the necessity of subsequent
Congressional approval has been confirmed by long usage. From the earliest days of our history we have
As previously discussed, this constitutional mandate emanates from the inherent power of the President to enter entered into executive agreements covering such subjects as commercial and consular relations, most-favored-
into agreements with other states, including the prerogative to conclude binding executive agreements that do nation rights, patent rights, trademark and copyright protection, postal and navigation arrangements and the
not require further Senate concurrence. The existence of this presidential power 203 is so well-entrenched that settlement of claims. The validity of these has never been seriously questioned by our courts. (Emphases
Section 5(2)(a), Article VIII of the Constitution, even provides for a check on its exercise. As expressed below, Supplied)
executive agreements are among those official governmental acts that can be the subject of this Court's power
of judicial review:
PAGE 105 of 135 CONSTITUTIONAL LAW -EXECUTIVE DEPARTMENT PART 2 –SESSION 12
That notion was carried over to the present Constitution. In fact, the framers specifically deliberated on whether Agreements with respect to the registration of trademarks have been concluded by the executive of various
the general term "international agreement" included executive agreements, and whether it was necessary to countries under the Act of Congress of March 3, 1881 (21 Stat. 502) . . . International agreements involving
include an express proviso that would exclude executive agreements from the requirement of Senate political issues or changes of national policy and those involving international agreements of a permanent
concurrence. After noted constitutionalist Fr. Joaquin Bernas quoted the Court's ruling in Eastern Sea character usually take the form of treaties. But international agreements embodying adjustments of detail,
Trading, the Constitutional Commission members ultimately decided that the term "international agreements" as carrying out well established national policies and traditions and those involving arrangements of a more
contemplated in Section 21, Article VII, does not include executive agreements, and that a proviso is no longer or less temporary nature usually take the form of executive agreements.
needed. Their discussion is reproduced below:207
MR. ROMULO: Is the Commissioner, therefore, excluding the executive agreements?
MS. AQUINO: Madam President, first I would like a clarification from the Committee. We have retained the
words "international agreement" which I think is the correct judgment on the matter because an international FR. BERNAS: What we are referring to, therefore, when we say international agreements which need
agreement is different from a treaty. A treaty is a contract between parties which is in the nature of international concurrence by at least two-thirds are those which are permanent in nature.
agreement and also a municipal law in the sense that the people are bound. So there is a conceptual difference.
However, I would like to be clarified if the international agreements include executive agreements.
MS. AQUINO: And it may include commercial agreements which are executive agreements essentially but which
are proceeding from the authorization of Congress. If that is our understanding, then I am willing to withdraw that
MR. CONCEPCION: That depends upon the parties. All parties to these international negotiations stipulate the amendment.
conditions which are necessary for the agreement or whatever it may be to become valid or effective as regards
the parties.
FR. BERNAS: If it is with prior authorization of Congress, then it does not need subsequent
concurrence by Congress.
MS. AQUINO: Would that depend on the parties or would that depend on the nature of the executive
agreement? According to common usage, there are two types of executive agreement: one is purely
proceeding from an executive act which affects external relations independent of the legislative and MS. AQUINO: In that case, I am withdrawing my amendment.
the other is an executive act in pursuance of legislative authorization. The first kind might take the form of
just conventions or exchanges of notes or protocol while the other, which would be pursuant to the MR. TINGSON: Madam President.
legislative authorization, may be in the nature of commercial agreements.
THE PRESIDENT: Is Commissioner Aquino satisfied?
MR. CONCEPCION: Executive agreements are generally made to implement a treaty already enforced or
to determine the details for the implementation of the treaty. We are speaking of executive agreements, not MS. AQUINO: Yes. There is already an agreement among us on the definition of "executive agreements"
international agreements. and that would make unnecessary any explicit proviso on the matter.

MS. AQUINO: I am in full agreement with that, except that it does not cover the first kind of executive agreement xxx
which is just protocol or an exchange of notes and this would be in the nature of reinforcement of claims of a
citizen against a country, for example.
MR. GUINGONA: I am not clear as to the meaning of "executive agreements" because I heard that these
executive agreements must rely on treaties. In other words, there must first be treaties.
MR. CONCEPCION: The Commissioner is free to require ratification for validity insofar as the Philippines is
concerned.
MR. CONCEPCION: No, I was speaking about the common use, as executive agreements being the
implementation of treaties, details of which do not affect the sovereignty of the State.
MS. AQUINO: It is my humble submission that we should provide, unless the Committee explains to us
otherwise, an explicit proviso which would except executive agreements from the requirement of
concurrence of two-thirds of the Members of the Senate. Unless I am enlightened by the Committee I MR. GUINGONA: But what about the matter of permanence, Madam President? Would 99 years be considered
propose that tentatively, the sentence should read. "No treaty or international agreement EXCEPT EXECUTIVE permanent? What would be the measure of permanency? I do not conceive of a treaty that is going to be
AGREEMENTS shall be valid and effective." forever, so there must be some kind of a time limit.

FR. BERNAS: I wonder if a quotation from the Supreme Court decision [in Eastern Sea Trading] might MR. CONCEPCION: I suppose the Commissioner's question is whether this type of agreement should be
help clarify this: included in a provision of the Constitution requiring the concurrence of Congress.

The right of the executive to enter into binding agreements without the necessity of subsequent MR. GUINGONA: It depends on the concept of the executive agreement of which I am not clear. If the
Congressional approval has been confirmed by long usage. From the earliest days of our history, we have executive agreement partakes of the nature of a treaty, then it should also be included.
entered into executive agreements covering such subjects as commercial and consular relations, most favored
nation rights, patent rights, trademark and copyright protection, postal and navigation arrangements and the MR. CONCEPCION: Whether it partakes or not of the nature of a treaty, it is within the power of the
settlement of claims. The validity of this has never been seriously questioned by our Courts. Constitutional Commission to require that.
PAGE 106 of 135 CONSTITUTIONAL LAW -EXECUTIVE DEPARTMENT PART 2 –SESSION 12
MR. GUINGONA: Yes. That is why I am trying to clarify whether the words "international agreements" level as a statute.217 If there is an irreconcilable conflict, a later law or treaty takes precedence over one that is
would include executive agreements. prior.218 An executive agreement is treated differently. Executive agreements that are inconsistent with either a
law or a treaty are considered ineffective. 219 Both types of international agreement are nevertheless subject to
MR. CONCEPCION: No, not necessarily; generally no. the supremacy of the Constitution.220

xxx This rule does not imply, though, that the President is given carte blanche to exercise this discretion. Although
the Chief Executive wields the exclusive authority to conduct our foreign relations, this power must still be
exercised within the context and the parameters set by the Constitution, as well as by existing domestic and
MR. ROMULO: I wish to be recognized first. I have only one question. Do we take it, therefore, that as far as the international laws. There are constitutional provisions that restrict or limit the President's prerogative in
Committee is concerned, the term "international agreements" does not include the term "executive concluding international agreements, such as those that involve the following:
agreements" as read by the Commissioner in that text?
a. The policy of freedom from nuclear weapons within Philippine territory221
FR. BERNAS: Yes. (Emphases Supplied)
b. The fixing of tariff rates, import and export quotas, tonnage and wharfage dues, and other duties or
The inapplicability to executive agreements of the requirements under Section 21 was again recognized imposts, which must be pursuant to the authority granted by Congress 222
in Bayan v. Zamora and in Bayan Muna v. Romulo. These cases, both decided under the aegis of the present
Constitution, quoted Eastern Sea Trading in reiterating that executive agreements are valid and binding even
without the concurrence of the Senate. c. The grant of any tax exemption, which must be pursuant to a law concurred in by a majority of all the
Members of Congress223
Executive agreements may dispense with the requirement of Senate concurrence because of the legal mandate
with which they are concluded. As culled from the afore-quoted deliberations of the Constitutional Commission, d. The contracting or guaranteeing, on behalf of the Philippines, of foreign loans that must be
past Supreme Court Decisions, and works of noted scholars,208 executive agreements merely involve previously concurred in by the Monetary Board224
arrangements on the implementation of existing policies, rules, laws, or agreements. They are concluded (1) to
adjust the details of a treaty;209 (2) pursuant to or upon confirmation by an act of the Legislature; 210 or (3) in the e. The authorization of the presence of foreign military bases, troops, or facilities in the country must be
exercise of the President's independent powers under the Constitution.211 The raison d'etre of executive in the form of a treaty duly concurred in by the Senate.225
agreements hinges on prior constitutional or legislative authorizations.
f. For agreements that do not fall under paragraph 5, the concurrence of the Senate is required, should
The special nature of an executive agreement is not just a domestic variation in international agreements. the form of the government chosen be a treaty.
International practice has accepted the use of various forms and designations of international agreements,
ranging from the traditional notion of a treaty - which connotes a formal, solemn instrument - to engagements 5. The President had the choice to enter into EDCA by way of an executive agreement or a treaty.
concluded in modem, simplified forms that no longer necessitate ratification. 212 An international agreement may
take different forms: treaty, act, protocol, agreement, concordat, compromis d'arbitrage, convention, covenant,
declaration, exchange of notes, statute, pact, charter, agreed minute, memorandum of agreement, modus No court can tell the President to desist from choosing an executive agreement over a treaty to embody an
vivendi, or some other form.213 Consequently, under international law, the distinction between a treaty and an international agreement, unless the case falls squarely within Article VIII, Section 25.
international agreement or even an executive agreement is irrelevant for purposes of determining international
rights and obligations. As can be gleaned from the debates among the members of the Constitutional Commission, they were aware
that legally binding international agreements were being entered into by countries in forms other than a treaty. At
However, this principle does not mean that the domestic law distinguishing treaties, international the same time, it is clear that they were also keen to preserve the concept of "executive agreements" and the
agreements, and executive agreements is relegated to a mere variation in form, or that the constitutional right of the President to enter into such agreements.
requirement of Senate concurrence is demoted to an optional constitutional directive. There remain two very
important features that distinguish treaties from executive agreements and translate them into terms of art in the What we can glean from the discussions of the Constitutional Commissioners is that they understood the
domestic setting. following realities:

First, executive agreements must remain traceable to an express or implied authorization under the Constitution, 1. Treaties, international agreements, and executive agreements are all constitutional manifestations of
statutes, or treaties. The absence of these precedents puts the validity and effectivity of executive agreements the conduct of foreign affairs with their distinct legal characteristics.
under serious question for the main function of the Executive is to enforce the Constitution and the laws enacted
by the Legislature, not to defeat or interfere in the performance of these rules.214 In turn, executive agreements a. Treaties are formal contracts between the Philippines and other States-parties, which are in
cannot create new international obligations that are not expressly allowed or reasonably implied in the law they the nature of international agreements, and also of municipal laws in the sense of their binding
purport to implement. nature.226

Second, treaties are, by their very nature, considered superior to executive agreements. Treaties are products of b. International agreements are similar instruments, the provisions of which may require the
the acts of the Executive and the Senate215 unlike executive agreements, which are solely executive ratification of a designated number of parties thereto. These agreements involving political
actions.216Because of legislative participation through the Senate, a treaty is regarded as being on the same issues or changes in national policy, as well as those involving international agreements of a
PAGE 107 of 135 CONSTITUTIONAL LAW -EXECUTIVE DEPARTMENT PART 2 –SESSION 12
permanent character, usually take the form of treaties. They may also include commercial x x x. As the President wields vast powers and influence, her conduct in the external affairs of the nation is,
agreements, which are executive agreements essentially, but which proceed from previous as Bayan would put it, "executive altogether." The right of the President to enter into or ratify binding
authorization by Congress, thus dispensing with the requirement of concurrence by the executive agreements has been confirmed by long practice.
Senate.227
In thus agreeing to conclude the Agreement thru E/N BF0-028-03, then President Gloria Macapagal-Arroyo,
c. Executive agreements are generally intended to implement a treaty already enforced or to represented by the Secretary of Foreign Affairs, acted within the scope of the authority and discretion
determine the details of the implementation thereof that do not affect the sovereignty of the vested in her by the Constitution. At the end of the day, the President - by ratifying, thru her deputies, the
State.228 non-surrender agreement - did nothing more than discharge a constitutional duty and exercise a
prerogative that pertains to her office. (Emphases supplied)
2. Treaties and international agreements that cannot be mere executive agreements must, by
constitutional decree, be concurred in by at least two-thirds of the Senate. Indeed, in the field of external affairs, the President must be given a larger measure of authority and wider
discretion, subject only to the least amount of checks and restrictions under the Constitution.229 The rationale
3. However, an agreement - the subject of which is the entry of foreign military troops, bases, or behind this power and discretion was recognized by the Court in Vinuya v. Executive Secretary, cited earlier.230
facilities - is particularly restricted. The requirements are that it be in the form of a treaty concurred in by
the Senate; that when Congress so requires, it be ratified by a majority of the votes cast by the people Section 9 of Executive Order No. 459, or the Guidelines in the Negotiation of International Agreements and its
in a national referendum held for that purpose; and that it be recognized as a treaty by the other Ratification, thus, correctly reflected the inherent powers of the President when it stated that the DFA "shall
contracting State. determine whether an agreement is an executive agreement or a treaty."

4. Thus, executive agreements can continue to exist as a species of international agreements. Accordingly, in the exercise of its power of judicial review, the Court does not look into whether an international
agreement should be in the form of a treaty or an executive agreement, save in cases in which the Constitution
That is why our Court has ruled the way it has in several cases. or a statute requires otherwise. Rather, in view of the vast constitutional powers and prerogatives granted to the
President in the field of foreign affairs, the task of the Court is to determine whether the international agreement
is consistent with the applicable limitations.
In Bayan Muna v. Romulo, we ruled that the President acted within the scope of her constitutional authority and
discretion when she chose to enter into the RP-U.S. Non-Surrender Agreement in the form of an executive
agreement, instead of a treaty, and in ratifying the agreement without Senate concurrence. The Court en 6. Executive agreements may cover the matter of foreign military forces if it merely involves detail
banc discussed this intrinsic presidential prerogative as follows: adjustments.

Petitioner parlays the notion that the Agreement is of dubious validity, partaking as it does of the nature of a The practice of resorting to executive agreements in adjusting the details of a law or a treaty that already deals
treaty; hence, it must be duly concurred in by the Senate. x x x x. Pressing its point, petitioner submits that the with the presence of foreign military forces is not at all unusual in this jurisdiction. In fact, the Court has already
subject of the Agreement does not fall under any of the subject-categories that xx x may be covered by an implicitly acknowledged this practice in Lim v. Executive Secretary.231 In that case, the Court was asked to
executive agreement, such as commercial/consular relations, most-favored nation rights, patent rights, scrutinize the constitutionality of the Terms of Reference of the Balikatan 02-1 joint military exercises, which
trademark and copyright protection, postal and navigation arrangements and settlement of claims. sought to implement the VFA. Concluded in the form of an executive agreement, the Terms of Reference
detailed the coverage of the term "activities" mentioned in the treaty and settled the matters pertaining to the
construction of temporary structures for the U.S. troops during the activities; the duration and location of the
The categorization of subject matters that may be covered by international agreements mentioned in Eastern exercises; the number of participants; and the extent of and limitations on the activities of the U.S. forces. The
Sea Trading is not cast in stone. There are no hard and fast rules on the propriety of entering, on a given Court upheld the Terms of Reference as being consistent with the VFA. It no longer took issue with the fact that
subject, into a treaty or an executive agreement as an instrument of international relations. The primary the Balikatan Terms of Reference was not in the form of a treaty concurred in by the Senate, even if it dealt with
consideration in the choice of the form of agreement is the parties' intent and desire to craft an the regulation of the activities of foreign military forces on Philippine territory.
international agreement in the form they so wish to further their respective interests. Verily, the matter of
form takes a back seat when it comes to effectiveness and binding effect of the enforcement of a treaty or an
executive agreement, as the parties in either international agreement each labor under the pacta sunt In Nicolas v. Romulo,232 the Court again impliedly affirmed the use of an executive agreement in an attempt to
servanda principle. adjust the details of a provision of the VFA. The Philippines and the U.S. entered into the Romulo-Kenney
Agreement, which undertook to clarify the detention of a U.S. Armed Forces member, whose case was pending
appeal after his conviction by a trial court for the crime of rape. In testing the validity of the latter agreement, the
xxxx Court precisely alluded to one of the inherent limitations of an executive agreement: it cannot go beyond the
terms of the treaty it purports to implement. It was eventually ruled that the Romulo-Kenney Agreement was "not
But over and above the foregoing considerations is the fact that - save for the situation and matters in accord" with the VFA, since the former was squarely inconsistent with a provision in the treaty requiring that
contemplated in Sec. 25, Art. XVIII of the Constitution - when a treaty is required, the Constitution does not the detention be "by Philippine authorities." Consequently, the Court ordered the Secretary of Foreign Affairs to
classify any subject, like that involving political issues, to be in the form of, and ratified as, a comply with the VFA and "forthwith negotiate with the United States representatives for the appropriate
treaty. What the Constitution merely prescribes is that treaties need the concurrence of the Senate by a vote agreement on detention facilities under Philippine authorities as provided in Art. V, Sec. 10 of the VFA. " 233
defined therein to complete the ratification process.
Culling from the foregoing discussions, we reiterate the following pronouncements to guide us in resolving the
xxxx present controversy:
PAGE 108 of 135 CONSTITUTIONAL LAW -EXECUTIVE DEPARTMENT PART 2 –SESSION 12
1. Section 25, Article XVIII of the Constitution, contains stringent requirements that must be fulfilled by The first difference emphasized is that EDCA does not only regulate visits as the VFA does, but allows
the international agreement allowing the presence of foreign military bases, troops, or facilities in the temporary stationing on a rotational basis of U.S. military personnel and their contractors in physical locations
Philippines: (a) the agreement must be in the form of a treaty, and (b) it must be duly concurred in by with permanent facilities and pre-positioned military materiel.
the Senate.
This argument does not take into account that these permanent facilities, while built by U.S. forces, are to be
2. If the agreement is not covered by the above situation, then the President may choose the form of owned by the Philippines once constructed.243 Even the VFA allowed construction for the benefit of U.S. forces
the agreement (i.e., either an executive agreement or a treaty), provided that the agreement dealing during their temporary visits.
with foreign military bases, troops, or facilities is not the principal agreement that first allows their entry
or presence in the Philippines. The second difference stated by the dissent is that EDCA allows the prepositioning of military materiel, which
can include various types of warships, fighter planes, bombers, and vessels, as well as land and amphibious
3. The executive agreement must not go beyond the parameters, limitations, and standards set by the vehicles and their corresponding ammunition.244
law and/or treaty that the former purports to implement; and must not unduly expand the international
obligation expressly mentioned or necessarily implied in the law or treaty. However, the VFA clearly allows the same kind of equipment, vehicles, vessels, and aircraft to be brought into
the country. Articles VII and VIII of the VFA contemplates that U.S. equipment, materials, supplies, and other
4. The executive agreement must be consistent with the Constitution, as well as with existing laws and property are imported into or acquired in the Philippines by or on behalf of the U.S. Armed Forces; as are
treaties. vehicles, vessels, and aircraft operated by or for U.S. forces in connection with activities under the VFA. These
provisions likewise provide for the waiver of the specific duties, taxes, charges, and fees that correspond to
In light of the President's choice to enter into EDCA in the form of an executive agreement, respondents carry these equipment.
the burden of proving that it is a mere implementation of existing laws and treaties concurred in by the Senate.
EDCA must thus be carefully dissected to ascertain if it remains within the legal parameters of a valid executive The third difference adverted to by the Justice Leonen's dissent is that the VFA contemplates the entry of troops
agreement. for training exercises, whereas EDCA allows the use of territory for launching military and paramilitary operations
conducted in other states.245 The dissent of Justice Teresita J. Leonardo-De Castro also notes that VFA was
7. EDCA is consistent with the content, purpose, and framework of the MDT and the VFA intended for non-combat activides only, whereas the entry and activities of U.S. forces into Agreed Locations
were borne of military necessity or had a martial character, and were therefore not contemplated by the VFA. 246
The starting point of our analysis is the rule that "an executive agreement xx x may not be used to amend a
treaty."234 In Lim v. Executive Secretary and in Nicolas v. Romulo, the Court approached the question of the This Court's jurisprudence however established in no uncertain terms that combat-related activities, as opposed
validity of executive agreements by comparing them with the general framework and the specific provisions of to actual combat, were allowed under the MDT and VFA, viz:
the treaties they seek to implement.
Both the history and intent of the Mutual Defense Treaty and the VFA support the conclusion that combat-
In Lim, the Terms of Reference of the joint military exercises was scrutinized by studying "the framework of the related activities as opposed to combat itself such as the one subject of the instant petition, are indeed
treaty antecedents to which the Philippines bound itself,"235 i.e., the MDT and the VFA. The Court proceeded to authorized.247
examine the extent of the term "activities" as contemplated in Articles 1 236 and II237 of the VFA. It later on found
that the term "activities" was deliberately left undefined and ambiguous in order to permit "a wide scope of Hence, even if EDCA was borne of military necessity, it cannot be said to have strayed from the intent of the
undertakings subject only to the approval of the Philippine government"238 and thereby allow the parties "a VFA since EDCA's combat-related components are allowed under the treaty.
certain leeway in negotiation."239 The Court eventually ruled that the Terms of Reference fell within the
sanctioned or allowable activities, especially in the context of the VFA and the MDT. Moreover, both the VFA and EDCA are silent on what these activities actually are. Both the VFA and EDCA deal
with the presence of U.S. forces within the Philippines, but make no mention of being platforms for activity
The Court applied the same approach to Nicolas v. Romulo. It studied the provisions of the VFA on custody and beyond Philippine territory. While it may be that, as applied, military operations under either the VFA or EDCA
detention to ascertain the validity of the Romulo-Kenney Agreement.240 It eventually found that the two would be carried out in the future the scope of judicial review does not cover potential breaches of discretion but
international agreements were not in accord, since the Romulo-Kenney Agreement had stipulated that U.S. only actual occurrences or blatantly illegal provisions. Hence, we cannot invalidate EDCA on the basis of the
military personnel shall be detained at the U.S. Embassy Compound and guarded by U.S. military personnel, potentially abusive use of its provisions.
instead of by Philippine authorities. According to the Court, the parties "recognized the difference between
custody during the trial and detention after conviction."241 Pursuant to Article V(6) of the VFA, the custody of a The fourth difference is that EDCA supposedly introduces a new concept not contemplated in the VFA or the
U.S. military personnel resides with U.S. military authorities during trial. Once there is a finding of guilt, Article MDT: Agreed Locations, Contractors, Pre-positioning, and Operational Control.248
V(l0) requires that the confinement or detention be "by Philippine authorities."
As previously mentioned, these points shall be addressed fully and individually in the latter analysis of EDCA's
Justice Marvic M.V.F. Leonen's Dissenting Opinion posits that EDCA "substantially modifies or amends the provisions. However, it must already be clarified that the terms and details used by an implementing agreement
VFA"242and follows with an enumeration of the differences between EDCA and the VFA. While these arguments need not be found in the mother treaty. They must be sourced from the authority derived from the treaty, but are
will be rebutted more fully further on, an initial answer can already be given to each of the concerns raised by his not necessarily expressed word-for-word in the mother treaty. This concern shall be further elucidated in this
dissent. Decision.
PAGE 109 of 135 CONSTITUTIONAL LAW -EXECUTIVE DEPARTMENT PART 2 –SESSION 12
The fifth difference highlighted by the Dissenting Opinion is that the VFA does not have provisions that may be "United States forces" means the entity comprising United States personnel and all property, equipment,
construed as a restriction on or modification of obligations found in existing statues, including the jurisdiction of and materiel of the United States Armed Forces present in the territory of the Philippines. 253
courts, local autonomy, and taxation. Implied in this argument is that EDCA contains such restrictions or
modifications.249 "United States contractors" means companies and firms, and their employees, under contract or
subcontract to or on behalf of the United States Department of Defense. United States contractors
This last argument cannot be accepted in view of the clear provisions of EDCA. Both the VFA and EDCA ensure are not included as part of the definition of United States personnel in this Agreement, including within the
Philippine jurisdiction in all instances contemplated by both agreements, with the exception of those outlined by context of the VFA.254
the VFA in Articles III-VI. In the VFA, taxes are clearly waived whereas in EDCA, taxes are assumed by the
government as will be discussed later on. This fact does not, therefore, produce a diminution of jurisdiction on United States forces may contract for any materiel, supplies, equipment, and services (including
the part of the Philippines, but rather a recognition of sovereignty and the rights that attend it, some of which construction) to be furnished or undertaken in the territory of the Philippines without restriction as to choice of
may be waived as in the cases under Articles III-VI of the VFA. contractor, supplier, or person who provides such materiel, supplies, equipment, or services. Such contracts
shall be solicited, awarded, and administered in accordance with the laws and regulations of the United
Taking off from these concerns, the provisions of EDCA must be compared with those of the MDT and the VFA, States.255 (Emphases Supplied)
which are the two treaties from which EDCA allegedly draws its validity.
A thorough evaluation of how EDCA is phrased clarities that the agreement does not deal with the entry
"Authorized presence" under the VFA versus "authorized activities" under EDCA: (1) U.S. personnel and into the country of U.S. personnel and contractors per se. While Articles I(l)(b)256 and II(4)257 speak of "the
(2) U.S. contractors right to access and use" the Agreed Locations, their wordings indicate the presumption that these groups have
already been allowed entry into Philippine territory, for which, unlike the VFA, EDCA has no specific provision.
The OSG argues250 that EDCA merely details existing policies under the MDT and the VFA. It explains that Instead, Article II of the latter simply alludes to the VFA in describing U.S. personnel, a term defined under
EDCA articulates the principle of defensive preparation embodied in Article II of the MDT; and seeks to enhance Article I of the treaty as follows:
the defensive, strategic, and technological capabilities of both parties pursuant to the objective of the treaty to
strengthen those capabilities to prevent or resist a possible armed attack. Respondent also points out that EDCA As used in this Agreement, "United States personnel" means United States military and civilian personnel
simply implements Article I of the VFA, which already allows the entry of U.S. troops and personnel into the temporarily in the Philippines in connection with activities approved by the Philippine Government. Within this
country. Respondent stresses this Court's recognition in Lim v. Executive Secretary that U.S. troops and definition:
personnel are authorized to conduct activities that promote the goal of maintaining and developing their defense
capability. 1. The term "military personnel" refers to military members of the United States Army,
Navy, Marine Corps, Air Force, and Coast Guard.
Petitioners contest251 the assertion that the provisions of EDCA merely implement the MDT. According to them,
the treaty does not specifically authorize the entry of U.S. troops in the country in order to maintain and develop 2. The term "civilian personnel" refers to individuals who are neither nationals of nor
the individual and collective capacities of both the Philippines and the U.S. to resist an armed attack. They ordinarily resident in the Philippines and who are employed by the United States armed
emphasize that the treaty was concluded at a time when there was as yet no specific constitutional prohibition forces or who are accompanying the United States armed forces, such as employees of
on the presence of foreign military forces in the country. the American Red Cross and the United Services Organization.258

Petitioners also challenge the argument that EDCA simply implements the VFA. They assert that the agreement Article II of EDCA must then be read with Article III of the VFA, which provides for the entry accommodations to
covers only short-term or temporary visits of U.S. troops "from time to time" for the specific purpose be accorded to U.S. military and civilian personnel:
of combined military exercises with their Filipino counterparts. They stress that, in contrast, U.S. troops are
allowed under EDCA to perform activities beyond combined military exercises, such as those enumerated in
Articles 111(1) and IV(4) thereof. Furthermore, there is some degree of permanence in the presence of U.S. 1. The Government of the Philippines shall facilitate the admission of United States personnel and
troops in the country, since the effectivity of EDCA is continuous until terminated. They proceed to argue that their departure from the Philippines in connection with activities covered by this agreement.
while troops have a "rotational" presence, this scheme in fact fosters their permanent presence.
2. United States military personnel shall be exempt from passport and visa regulations upon
a. Admission of U.S. military and civilian personnel into Philippine territory is already allowed under the VFA enteringand departing the Philippines.

We shall first deal with the recognition under EDCA of the presence in the country of three distinct classes of 3. The following documents only, which shall be required in respect of United States military personnel
individuals who will be conducting different types of activities within the Agreed Locations: (1) U.S. military who enter the Philippines; xx xx.
personnel; (2) U.S. civilian personnel; and (3) U.S. contractors. The agreement refers to them as follows:
4. United States civilian personnel shall be exempt from visa requirements but shall present, upon
"United States personnel" means United States military and civilian personnel temporarily in the territory demand, valid passports upon entry and departure of the Philippines. (Emphases Supplied)
of the Philippines in connection with activities approved by the Philippines, as those terms are defined in
the VFA.252 By virtue of Articles I and III of the VFA, the Philippines already allows U.S. military and civilian personnel to be
"temporarily in the Philippines," so long as their presence is "in connection with activities approved by the
Philippine Government." The Philippines, through Article III, even guarantees that it shall facilitate the admission
PAGE 110 of 135 CONSTITUTIONAL LAW -EXECUTIVE DEPARTMENT PART 2 –SESSION 12
of U.S. personnel into the country and grant exemptions from passport and visa regulations. The VFA does not Article III
even limit their temporary presence to specific locations.
The Parties, through their Foreign Ministers or their deputies, will consult together from time to time
Based on the above provisions, the admission and presence of U.S. military and civilian personnel in regarding the implementation of this Treaty and whenever in the opinion of either of them the territorial
Philippine territory are already allowed under the VFA, the treaty supposedly being implemented by integrity, political independence or security of either of the Parties is threatened by external armed attack in the
EDCA. What EDCA has effectively done, in fact, is merely provide the mechanism to identify the locations in Pacific.
which U.S. personnel may perform allowed activities pursuant to the VFA. As the implementing agreement, it
regulates and limits the presence of U.S. personnel in the country. VISITING FORCES AGREEMENT

b. EDCA does not provide the legal basis for admission of U.S. contractors into Philippine territory; their entry Preamble
must be sourced from extraneous Philippine statutes and regulations for the admission of alien employees or
business persons.
xxx
Of the three aforementioned classes of individuals who will be conducting certain activities within the Agreed
Locations, we note that only U.S. contractors are not explicitly mentioned in the VFA. This does not mean, Reaffirming their obligations under the Mutual Defense Treaty of August 30, 1951;
though, that the recognition of their presence under EDCA is ipso facto an amendment of the treaty, and that
there must be Senate concurrence before they are allowed to enter the country. Noting that from time to time elements of the United States armed forces may visit the Republic of the
Philippines;
Nowhere in EDCA are U.S. contractors guaranteed immediate admission into the Philippines. Articles III and IV,
in fact, merely grant them the right of access to, and the authority to conduct certain activities within the Agreed Considering that cooperation between the United States and the Republic of the Philippines promotes their
Locations. Since Article II(3) of EDCA specifically leaves out U.S. contractors from the coverage of the VFA, they common security interests;
shall not be granted the same entry accommodations and privileges as those enjoyed by U.S. military and
civilian personnel under the VFA. xxx

Consequently, it is neither mandatory nor obligatory on the part of the Philippines to admit U.S. contractors into Article I - Definitions
the country.259 We emphasize that the admission of aliens into Philippine territory is "a matter of pure permission
and simple tolerance which creates no obligation on the part of the government to permit them to stay."260 Unlike
U.S. personnel who are accorded entry accommodations, U.S. contractors are subject to Philippine immigration As used in this Agreement, "United States personnel" means United States military and civilian personnel
laws.261The latter must comply with our visa and passport regulations 262 and prove that they are not subject to temporarily in the Philippines in connection with activities approved by the Philippine Government. Within
exclusion under any provision of Philippine immigration laws.263 The President may also deny them entry this definition: xx x
pursuant to his absolute and unqualified power to prohibit or prevent the admission of aliens whose presence in
the country would be inimical to public interest. 264 Article II - Respect for Law

In the same vein, the President may exercise the plenary power to expel or deport U.S. contractors 265 as may be It is the duty of United States personnel to respect the laws of the Republic of the Philippines and to
necessitated by national security, public safety, public health, public morals, and national interest.266 They may abstain from any activity inconsistent with the spirit of this agreement, and, in particular, from any political
also be deported if they are found to be illegal or undesirable aliens pursuant to the Philippine Immigration activity in the Philippines. The Government of the United States shall take all measures within its authority to
Act267 and the Data Privacy Act.268 In contrast, Article 111(5) of the VFA requires a request for removal from the ensure that this is done.
Philippine government before a member of the U.S. personnel may be "dispos[ed] xx x outside of the
Philippines."
Article VII - Importation and Exportation

c. Authorized activities of U.S. military and civilian personnel within Philippine territory are in furtherance of the
1. United States Government equipment, materials, supplies, and other property imported into or
MDT and the VFA
acquired in the Philippines by or on behalf of the United States armed forces in connection with activities to
which this agreement applies, shall be free of all Philippine duties, taxes and other similar charges. Title to
We begin our analysis by quoting the relevant sections of the MDT and the VFA that pertain to the activities in such property shall remain with the United States, which may remove such property from the Philippines at any
which U.S. military and civilian personnel may engage: time, free from export duties, taxes, and other similar charges. x x x.

MUTUAL DEFENSE TREATY Article VIII - Movement of Vessels and Aircraft

Article II 1. Aircraft operated by or for the United States armed forces may enter the Philippines upon approval of
the Government of the Philippines in accordance with procedures stipulated in implementing arrangements.
In order more effectively to achieve the objective of this Treaty, the Parties separately and jointly byself-help
and mutual aid will maintain and develop their individual and collective capacity to resist armed attack.
PAGE 111 of 135 CONSTITUTIONAL LAW -EXECUTIVE DEPARTMENT PART 2 –SESSION 12
2. Vessels operated by or for the United States armed forces may enter the Philippines upon approval of After studied reflection, it appeared farfetched that the ambiguity surrounding the meaning of the word
the Government of the Philippines. The movement of vessels shall be in accordance with international "activities" arose from accident. In our view, it was deliberately made that way to give both parties a
custom and practice governing such vessels, and such agreed implementing arrangements as certain leeway in negotiation. In this manner, visiting US forces may sojourn in Philippine territory for
necessary. x x x (Emphases Supplied) purposes other than military. As conceived, the joint exercises may include training on new techniques of
patrol and surveillance to protect the nation's marine resources, sea search-and-rescue operations to assist
Manifest in these provisions is the abundance of references to the creation of further "implementing vessels in distress, disaster relief operations, civic action projects such as the building of school houses, medical
arrangements" including the identification of "activities [to be] approved by the Philippine Government." To and humanitarian missions, and the like.
determine the parameters of these implementing arrangements and activities, we referred to the content,
purpose, and framework of the MDT and the VFA. Under these auspices, the VFA gives legitimacy to the current Balikatan exercises. It is only logical to assume
that "Balikatan 02-1," a "mutual anti-terrorism advising, assisting and training exercise," falls under the
By its very language, the MDT contemplates a situation in which both countries shall engage in joint activities, so umbrella of sanctioned or allowable activities in the context of the agreement. Both the history and intent
that they can maintain and develop their defense capabilities. The wording itself evidently invites a reasonable of the Mutual Defense Treaty and the VFA support the conclusion that combat-related activities - as opposed to
construction that the joint activities shall involve joint military trainings, maneuvers, and exercises. Both the combat itself- such as the one subject of the instant petition, are indeed authorized. (Emphases Supplied)
interpretation269 and the subsequent practice270 of the parties show that the MDT independently allows joint
military exercises in the country. Lim v. Executive Secretary271 and Nicolas v. Romulo272 recognized The joint report of the Senate committees on foreign relations and on national defense and security further
that Balikatan exercises, which are activities that seek to enhance and develop the strategic and technological explains the wide range and variety of activities contemplated in the VFA, and how these activities shall be
capabilities of the parties to resist an armed attack, "fall squarely under the provisions of the RP-US identified:277
MDT."273 In Lim, the Court especially noted that the Philippines and the U.S. continued to conduct joint military
exercises even after the expiration of the MBA and even before the conclusion of the VFA. 274 These activities These joint exercises envisioned in the VFA are not limited to combat-related activities; they have a wide
presumably related to the Status of Forces Agreement, in which the parties agreed on the status to be accorded range and variety. They include exercises that will reinforce the AFP's ability to acquire new techniques of
to U.S. military and civilian personnel while conducting activities in the Philippines in relation to the MDT.275 patrol and surveillance to protect the country's maritime resources; sea-search and rescue operations to
assist ships in distress; and disaster-relief operations to aid the civilian victims of natural calamities, such as
Further, it can be logically inferred from Article V of the MDT that these joint activities may be conducted on earthquakes, typhoons and tidal waves.
Philippine or on U.S. soil. The article expressly provides that the term armed attack includes "an armed attack on
the metropolitan territory of either of the Parties, or on the island territories under its jurisdiction in the xxxx
Pacific or on its armed forces, public vessels or aircraft in the Pacific." Surely, in maintaining and developing
our defense capabilities, an assessment or training will need to be performed, separately and jointly by self-help
and mutual aid, in the territories of the contracting parties. It is reasonable to conclude that the assessment of Joint activities under the VFA will include combat maneuvers; training in aircraft maintenance and equipment
defense capabilities would entail understanding the terrain, wind flow patterns, and other environmental factors repair; civic-action projects; and consultations and meetings of the Philippine-U.S. Mutual Defense Board. It is at
unique to the Philippines. the level of the Mutual Defense Board-which is headed jointly by the Chief of Staff of the AFP and the
Commander in Chief of the U.S. Pacific Command-that the VFA exercises are planned. Final approval of any
activity involving U.S. forces is, however, invariably given by the Philippine Government.
It would also be reasonable to conclude that a simulation of how to respond to attacks in vulnerable areas would
be part of the training of the parties to maintain and develop their capacity to resist an actual armed attack and
to test and validate the defense plan of the Philippines. It is likewise reasonable to imagine that part of the xxxx
training would involve an analysis of the effect of the weapons that may be used and how to be prepared for the
eventuality. This Court recognizes that all of this may require training in the area where an armed attack might Siazon clarified that it is not the VFA by itself that determines what activities will be conductedbetween the
be directed at the Philippine territory. armed forces of the U.S. and the Philippines. The VFA regulates and provides the legal framework for the
presence, conduct and legal status of U.S. personnel while they are in the country for visits, joint exercises
The provisions of the MDT must then be read in conjunction with those of the VFA. and other related activities. (Emphases Supplied)

Article I of the VFA indicates that the presence of U.S. military and civilian personnel in the Philippines is "in What can be gleaned from the provisions of the VFA, the joint report of the Senate committees on
connection with activities approved by the Philippine Government." While the treaty does not expressly foreign relations and on national defense and security, and the ruling of this Court in Lim is that the
enumerate or detail the nature of activities of U.S. troops in the country, its Preamble makes explicit references "activities" referred to in the treaty are meant to be specified and identified infurther agreements. EDCA
to the reaffirmation of the obligations of both countries under the MDT. These obligations include the is one such agreement.
strengthening of international and regional security in the Pacific area and the promotion of common security
interests. EDCA seeks to be an instrument that enumerates the Philippine-approved activities of U.S. personnel referred to
in the VFA. EDCA allows U.S. military and civilian personnel to perform "activities approved by the Philippines,
The Court has already settled in Lim v. Executive Secretary that the phrase "activities approved by the Philippine as those terms are defined in the VFA"278 and clarifies that these activities include those conducted within the
Government" under Article I of the VFA was intended to be ambiguous in order to afford the parties flexibility to Agreed Locations:
adjust the details of the purpose of the visit of U.S. personnel. 276 In ruling that the Terms of Reference for
the Balikatan Exercises in 2002 fell within the context of the treaty, this Court explained: 1. Security cooperation exercises; joint and combined training activities; humanitarian assistance and disaster
relief activities; and such other activities as may be agreed upon by the Parties279
PAGE 112 of 135 CONSTITUTIONAL LAW -EXECUTIVE DEPARTMENT PART 2 –SESSION 12
2. Training; transit; support and related activities; refueling of aircraft; bunkering of vessels; temporary US exercise participants shall not engage in combat, without prejudice to their right of self-defense.
maintenance of vehicles, vessels, and aircraft; temporary accommodation of personnel; communications;
prepositioning of equipment, supplies, and materiel; deployment of forces and materiel; and such other activities These terms of Reference are for purposes of this Exercise only and do not create additional legal obligations
as the Parties may agree280 between the US Government and the Republic of the Philippines.

3. Exercise of operational control over the Agreed Locations for construction activities and other types of activity, II. EXERCISE LEVEL
including alterations and improvements thereof281
1. TRAINING
4. Exercise of all rights and authorities within the Agreed Locations that are necessary for their operational
control or defense, including the adoption of apfropriate measures to protect U.S. forces and contractors282
a. The Exercise shall involve the conduct of mutual military assisting, advising and
trainingof RP and US Forces with the primary objective of enhancing the operational
5. Use of water, electricity, and other public utilities283 capabilities of both forces to combat terrorism.

6. Operation of their own telecommunication systems, including the utilization of such means and services as b. At no time shall US Forces operate independently within RP territory.
are required to ensure the full ability to operate telecommunication systems, as well as the use of the necessary
radio spectrum allocated for this purpose284
c. Flight plans of all aircraft involved in the exercise will comply with the local air traffic
regulations.
According to Article I of EDCA, one of the purposes of these activities is to maintain and develop, jointly and by
mutual aid, the individual and collective capacities of both countries to resist an armed attack. It further states
that the activities are in furtherance of the MDT and within the context of the VFA. 2. ADMINISTRATION & LOGISTICS

We note that these planned activities are very similar to those under the Terms of Reference285 mentioned xxxx
in Lim. Both EDCA and the Terms of Reference authorize the U.S. to perform the following: (a) participate in
training exercises; (b) retain command over their forces; (c) establish temporary structures in the country; (d) a. RP and US participating forces may share, in accordance with their respective laws and regulations, in the
share in the use of their respective resources, equipment and other assets; and (e) exercise their right to self- use of their resources, equipment and other assets. They will use their respective logistics channels. x x
defense. We quote the relevant portion of the Terms and Conditions as follows:286 x. (Emphases Supplied)

I. POLICY LEVEL After a thorough examination of the content, purpose, and framework of the MDT and the VFA, we find that
EDCA has remained within the parameters set in these two treaties. Just like the Terms of Reference mentioned
xxxx in Lim, mere adjustments in detail to implement the MDT and the VFA can be in the form of executive
agreements.
No permanent US basing and support facilities shall be established. Temporary structures such as those
for troop billeting, classroom instruction and messing may be set up for use by RP and US Forces Petitioners assert287 that the duration of the activities mentioned in EDCA is no longer consistent with the
during the Exercise. temporary nature of the visits as contemplated in the VFA. They point out that Article XII(4) of EDCA has an
initial term of 10 years, a term automatically renewed unless the Philippines or the U.S. terminates the
agreement. According to petitioners, such length of time already has a badge of permanency.
The Exercise shall be implemented jointly by RP and US Exercise Co-Directors under the authority of the Chief
of Staff, AFP. In no instance will US Forces operate independently during field training exercises (FTX). AFP
and US Unit Commanders will retain command over their respective forces under the overall authority of In connection with this, Justice Teresita J. Leonardo-De Castro likewise argues in her Concurring and Dissenting
the Exercise Co-Directors. RP and US participants shall comply with operational instructions of the AFP during Opinion that the VFA contemplated mere temporary visits from U.S. forces, whereas EDCA allows an unlimited
the FTX. period for U.S. forces to stay in the Philippines.288

The exercise shall be conducted and completed within a period of not more than six months, with the projected However, the provisions of EDCA directly contradict this argument by limiting itself to 10 years of effectivity.
participation of 660 US personnel and 3,800 RP Forces. The Chief of Staff, AFP shall direct the Exercise Co- Although this term is automatically renewed, the process for terminating the agreement is unilateral and the right
Directors to wind up and terminate the Exercise and other activities within the six month Exercise period. to do so automatically accrues at the end of the 10 year period. Clearly, this method does not create a
permanent obligation.
The Exercise is a mutual counter-terrorism advising, assisting and training Exercise relative to Philippine
efforts against the ASG, and will be conducted on the Island of Basilan. Further advising, assisting and training Drawing on the reasoning in Lim, we also believe that it could not have been by chance that the VFA does not
exercises shall be conducted in Malagutay and the Zamboanga area. Related activities in Cebu will be for include a maximum time limit with respect to the presence of U.S. personnel in the country. We construe this
support of the Exercise. lack of specificity as a deliberate effort on the part of the Philippine and the U.S. governments to leave out this
aspect and reserve it for the "adjustment in detail" stage of the implementation of the treaty. We interpret the
subsequent, unconditional concurrence of the Senate in the entire text of the VFA as an implicit grant to the
xx xx.
PAGE 113 of 135 CONSTITUTIONAL LAW -EXECUTIVE DEPARTMENT PART 2 –SESSION 12
President of a margin of appreciation in determining the duration of the "temporary" presence of U.S. personnel external affairs; and who has unrestricted access to highly classified military intelligence data294 that may
in the country. threaten the life of the nation. Thus, if after a geopolitical prognosis of situations affecting the country, a belief is
engendered that a much longer period of military training is needed, the President must be given ample
Justice Brion's dissent argues that the presence of U.S. forces under EDCA is "more permanent" in discretion to adopt necessary measures including the flexibility to set an extended timetable.
nature.289However, this argument has not taken root by virtue of a simple glance at its provisions on the
effectivity period. EDCA does not grant permanent bases, but rather temporary rotational access to facilities for Due to the sensitivity and often strict confidentiality of these concerns, we acknowledge that the President may
efficiency. As Professor Aileen S.P. Baviera notes: not always be able to candidly and openly discuss the complete situation being faced by the nation. The Chief
Executive's hands must not be unduly tied, especially if the situation calls for crafting programs and setting
The new EDCA would grant American troops, ships and planes rotational access to facilities of the Armed timelines for approved activities. These activities may be necessary for maintaining and developing our capacity
Forces of the Philippines – but not permanent bases which are prohibited under the Philippine Constitution - with to resist an armed attack, ensuring our national sovereignty and territorial integrity, and securing our national
the result of reducing response time should an external threat from a common adversary crystallize. 290 interests. If the Senate decides that the President is in the best position to define in operational terms the
meaning of temporary in relation to the visits, considered individually or in their totality, the Court must respect
that policy decision. If the Senate feels that there is no need to set a time limit to these visits, neither should we.
EDCA is far from being permanent in nature compared to the practice of states as shown in other defense
cooperation agreements. For example, Article XIV(l) of the U.S.-Romania defense agreement provides the
following: Evidently, the fact that the VFA does not provide specificity in regard to the extent of the "temporary" nature of
the visits of U.S. personnel does not suggest that the duration to which the President may agree is unlimited.
Instead, the boundaries of the meaning of the term temporary in Article I of the treaty must be measured
This Agreement is concluded for an indefinite period and shall enter into force in accordance with the internal depending on the purpose of each visit or activity. 295 That purpose must be analyzed on a case-by-case basis
laws of each Party x x x. (emphasis supplied) depending on the factual circumstances surrounding the conclusion of the implementing agreement. While the
validity of the President's actions will be judged under less stringent standards, the power of this Court to
Likewise, Article 36(2) of the US-Poland Status of Forces Agreement reads: determine whether there was grave abuse of discretion remains unimpaired.

This Agreement has been concluded for an indefinite period of time. It may be terminated by written d. Authorized activities performed by US. contractors within Philippine territory - who were legitimately permitted
notification by either Party and in that event it terminates 2 years after the receipt of the notification. (emphasis to enter the country independent of EDCA - are subject to relevant Philippine statutes and regulations and must
supplied) be consistent with the MDT and the VFA

Section VIII of US.-Denmark Mutual Support Agreement similarly provides: Petitioners also raise296 concerns about the U.S. government's purported practice of hiring private security
contractors in other countries. They claim that these contractors - one of which has already been operating in
8.1 This Agreement, which consists of a Preamble, SECTIONs I-VIII, and Annexes A and B, shall become Mindanao since 2004 - have been implicated in incidents or scandals in other parts of the globe involving
effective on the date of the last signature affixed below and shall remain in force until terminated by the rendition, torture and other human rights violations. They also assert that these contractors employ paramilitary
Parties, provided that it may be terminated by either Party upon 180 days written notice of its intention to do so forces in other countries where they are operating.
to the other Party. (emphasis supplied)
Under Articles III and IV of EDCA, U.S. contractors are authorized to perform only the following activities:
On the other hand, Article XXI(3) of the US.-Australia Force Posture Agreement provides a longer initial term:
1. Training; transit; support and related activities; refueling of aircraft; bunkering of vessels; temporary
3. This Agreement shall have an initial term of 25 years and thereafter shall continue in force, but may be maintenance of vehicles, vessels, and aircraft; temporary accommodation of personnel;
terminated by either Party at any time upon one year's written notice to the other Party through diplomatic communications; prepositioning of equipment, supplies, and materiel; deployment of forces and
channels. (emphasis supplied) materiel; and such other activities as the Parties may agree297

The phrasing in EDCA is similar to that in the U.S.-Australia treaty but with a term less than half of that is 2. Prepositioning and storage of defense equipment, supplies, and materiel, including delivery,
provided in the latter agreement. This means that EDCA merely follows the practice of other states in not management, inspection, use, maintenance, and removal of such equipment, supplies and materiel 298
specifying a non-extendible maximum term. This practice, however, does not automatically grant a badge of
permanency to its terms. Article XII(4) of EDCA provides very clearly, in fact, that its effectivity is for an initial 3. Carrying out of matters in accordance with, and to the extent permissible under, U.S. laws,
term of 10 years, which is far shorter than the terms of effectivity between the U.S. and other states. It is simply regulations, and policies299
illogical to conclude that the initial, extendible term of 10 years somehow gives EDCA provisions a permanent
character. EDCA requires that all activities within Philippine territory be in accordance with Philippine law. This means that
certain privileges denied to aliens are likewise denied to foreign military contractors. Relevantly, providing
The reasoning behind this interpretation is rooted in the constitutional role of the President who, as Commander- security300and carrying, owning, and possessing firearms301 are illegal for foreign civilians.
in-Chief of our armed forces, is the principal strategist of the nation and, as such, duty-bound to defend our
national sovereignty and territorial integrity;291 who, as chief architect of our foreign relations, is the head The laws in place already address issues regarding the regulation of contractors. In the 2015 Foreign
policymaker tasked to assess, ensure, and protect our national security and interests; 292 who holds the most Investment Negative list,302 the Executive Department has already identified corporations that have equity
comprehensive and most confidential information about foreign countries293 that may affect how we conduct our restrictions in Philippine jurisdiction. Of note is No. 5 on the list - private security agencies that cannot have any
PAGE 114 of 135 CONSTITUTIONAL LAW -EXECUTIVE DEPARTMENT PART 2 –SESSION 12
foreign equity by virtue of Section 4 of Republic Act No. 5487;303 and No. 15, which regulates contracts for the Article VII of EDCA authorizes U.S. forces to use public utilities and to operate their own telecommunications
construction of defense-related structures based on Commonwealth Act No. 541. system.

Hence, any other entity brought into the Philippines by virtue of EDCA must subscribe to corporate and civil a. Preliminary point on badges of exclusivity
requirements imposed by the law, depending on the entity's corporate structure and the nature of its business.
As a preliminary observation, petitioners have cherry-picked provisions of EDCA by presenting so-called
That Philippine laws extraneous to EDCA shall govern the regulation of the activities of U.S. contractors has "badges of exclusivity," despite the presence of contrary provisions within the text of the agreement itself.
been clear even to some of the present members of the Senate.
First, they clarify the word "return" in Article V(2) of EDCA. However, the use of the word "return" is within the
For instance, in 2012, a U.S. Navy contractor, the Glenn Marine, was accused of spilling fuel in the waters off context of a lengthy provision. The provision as a whole reads as follows:
Manila Bay.304 The Senate Committee on Foreign Relations and the Senate Committee on Environment and
Natural Resources chairperson claimed environmental and procedural violations by the contractor.305 The U.S. The United States shall return to the Philippines any Agreed Locations, or any portion thereof, including non-
Navy investigated the contractor and promised stricter guidelines to be imposed upon its contractors. 306 The relocatable structures and assemblies constructed, modified, or improved by the United States, once no longer
statement attributed to Commander Ron Steiner of the public affairs office of the U.S. Navy's 7th Fleet - that required by United States forces for activities under this Agreement. The Parties or the Designated Authorities
U.S. Navy contractors are bound by Philippine laws - is of particular relevance. The statement acknowledges not shall consult regarding the terms of return of any Agreed Locations, including possible compensation for
just the presence of the contractors, but also the U.S. position that these contractors are bound by the local laws improvements or construction.
of their host state. This stance was echoed by other U.S. Navy representatives. 307
The context of use is "required by United States forces for activities under this Agreement." Therefore, the return
This incident simply shows that the Senate was well aware of the presence of U.S. contractors for the purpose of of an Agreed Location would be within the parameters of an activity that the Mutual Defense Board (MDB) and
fulfilling the terms of the VFA. That they are bound by Philippine law is clear to all, even to the U.S. the Security Engagement Board (SEB) would authorize. Thus, possession by the U.S. prior to its return of the
Agreed Location would be based on the authority given to it by a joint body co-chaired by the "AFP Chief of Staff
As applied to EDCA, even when U.S. contractors are granted access to the Agreed Locations, all their activities and Commander, U.S. PACOM with representatives from the Philippines' Department of National Defense and
must be consistent with Philippine laws and regulations and pursuant to the MDT and the VFA. Department of Foreign Affairs sitting as members."313 The terms shall be negotiated by both the Philippines and
the U.S., or through their Designated Authorities. This provision, seen as a whole, contradicts petitioners'
While we recognize the concerns of petitioners, they do not give the Court enough justification to strike down interpretation of the return as a "badge of exclusivity." In fact, it shows the cooperation and partnership aspect of
EDCA. In Lim v. Executive Secretary, we have already explained that we cannot take judicial notice of claims EDCA in full bloom.
aired in news reports, "not because of any issue as to their truth, accuracy, or impartiality, but for the simple
reason that facts must be established in accordance with the rules of evidence."308 What is more, we cannot Second, the term "unimpeded access" must likewise be viewed from a contextual perspective. Article IV(4)
move one step ahead and speculate that the alleged illegal activities of these contractors in other countries states that U.S. forces and U.S. contractors shall have "unimpeded access to Agreed Locations for all matters
would take place in the Philippines with certainty. As can be seen from the above discussion, making sure that relating to the prepositioning and storage of defense equipment, supplies, and materiel, including delivery,
U.S. contractors comply with Philippine laws is a function of law enforcement. EDCA does not stand in the way management, inspection, use, maintenance, and removal of such equipment, supplies and materiel."
of law enforcement.
At the beginning of Article IV, EDCA states that the Philippines gives the U.S. the authority to bring in these
Nevertheless, we emphasize that U.S. contractors are explicitly excluded from the coverage of the VFA. As equipment, supplies, and materiel through the MDB and SEB security mechanism. These items are owned by
visiting aliens, their entry, presence, and activities are subject to all laws and treaties applicable within the the U.S.,314 are exclusively for the use of the U.S.315 and, after going through the joint consent mechanisms of
Philippine territory. They may be refused entry or expelled from the country if they engage in illegal or the MDB and the SEB, are within the control of the U.S. 316 More importantly, before these items are considered
undesirable activities. There is nothing that prevents them from being detained in the country or being subject to prepositioned, they must have gone through the process of prior authorization by the MDB and the SEB and
the jurisdiction of our courts. Our penal laws,309 labor laws,310 and immigrations laws311 apply to them and given proper notification to the AFP.317
therefore limit their activities here. Until and unless there is another law or treaty that specifically deals with their
entry and activities, their presence in the country is subject to unqualified Philippine jurisdiction. Therefore, this "unimpeded access" to the Agreed Locations is a necessary adjunct to the ownership, use, and
control of the U.S. over its own equipment, supplies, and materiel and must have first been allowed by the joint
EDCA does not allow the presence of U.S.-owned or -controlled military facilities and bases in the mechanisms in play between the two states since the time of the MDT and the VFA. It is not the use of the
Philippines Agreed Locations that is exclusive per se; it is mere access to items in order to exercise the rights of ownership
granted by virtue of the Philippine Civil Code.318
Petitioners Saguisag et al. claim that EDCA permits the establishment of U.S. military bases through the
"euphemistically" termed "Agreed Locations. "312 Alluding to the definition of this term in Article II(4) of EDCA, As for the view that EDCA authorizes U.S. forces to use public utilities and to operate their own
they point out that these locations are actually military bases, as the definition refers to facilities and areas to telecommunications system, it will be met and answered in part D, infra.
which U.S. military forces have access for a variety of purposes. Petitioners claim that there are several badges
of exclusivity in the use of the Agreed Locations by U.S. forces. First, Article V(2) of EDCA alludes to a "return" Petitioners also point out319 that EDCA is strongly reminiscent of and in fact bears a one-to-one correspondence
of these areas once they are no longer needed by U.S. forces, indicating that there would be some transfer of with the provisions of the 1947 MBA. They assert that both agreements (a) allow similar activities within the
use. Second, Article IV(4) ofEDCA talks about American forces' unimpeded access to the Agreed Locations for area; (b) provide for the same "species of ownership" over facilities; and (c) grant operational control over the
all matters relating to the prepositioning and storage of U.S. military equipment, supplies, and materiel. Third,
PAGE 115 of 135 CONSTITUTIONAL LAW -EXECUTIVE DEPARTMENT PART 2 –SESSION 12
entire area. Finally, they argue320 that EDCA is in fact an implementation of the new defense policy of the U.S.
The Government of the Republic of Affirming that the Parties share an
According to them, this policy was not what was originally intended either by the MDT or by the VFA. the Philippines (hereinafter referred to as understanding for the United States not to
the Philippines) grants to the Government establish a permanent military presence
On these points, the Court is not persuaded. of the United States of America or base in the territory of the
(hereinafter referred to as the United Philippines;
The similar activities cited by petitioners321 simply show that under the MBA, the U.S. had the right to construct, States) the right to retain the use of the
operate, maintain, utilize, occupy, garrison, and control the bases. The so-called parallel provisions of EDCA bases in the Philippines listed in Annex A xxxx
allow only operational control over the Agreed Locations specifically for construction activities. They do not allow attached hereto.
the overarching power to operate, maintain, utilize, occupy, garrison, and control a base with full discretion. Recognizing that all United States
EDCA in fact limits the rights of the U.S. in respect of every activity, including construction, by giving the MDB 1947 MBA, Art. XVII(2): access to and use of facilities and areas
and the SEB the power to determine the details of all activities such as, but not limited to, operation, will be at the invitation of the
maintenance, utility, occupancy, garrisoning, and control. 322 All buildings and structures which Philippines and with full respect for the
are erected by the United States in the Philippine Constitution and Philippine
The "species of ownership" on the other hand, is distinguished by the nature of the property. For immovable bases shall be the property of the United laws;
property constructed or developed by the U.S., EDCA expresses that ownership will automatically be vested to States and may be removed by it before
the Philippines.323 On the other hand, for movable properties brought into the Philippines by the U.S., EDCA the expiration of this Agreement or the xxxx
provides that ownership is retained by the latter. In contrast, the MBA dictates that the U.S. retains ownership earlier relinquishment of the base on which
over immovable and movable properties. the structures are situated. There shall be
no obligation on the part of the Philippines EDCA, Art. II(4):
To our mind, both EDCA and the MBA simply incorporate what is already the law of the land in the Philippines. or of the United States to rebuild or repair
The Civil Code's provisions on ownership, as applied, grant the owner of a movable property full rights over that any destruction or damage inflicted from "Agreed Locations" means facilities and
property, even if located in another person's property.324 any cause whatsoever on any of the said areas that are provided by the
buildings or structures owned or used by Government of the Philippines through
the United States in the bases. x x x x. the AFP and that United States forces,
The parallelism, however, ends when the situation involves facilities that can be considered immovable. Under United States contractors, and others as
the MBA, the U.S. retains ownership if it paid for the facility. 325 Under EDCA, an immovable is owned by the
1946 Treaty of Gen. Relations, Art. I: mutually agreed, shall have the right to
Philippines, even if built completely on the back of U.S. funding. 326 This is consistent with the constitutional
access and use pursuant to this
prohibition on foreign land ownership.327 Agreement. Such Agreed Locations may
The United States of America agrees to be listed in an annex to be appended to
Despite the apparent similarity, the ownership of property is but a part of a larger whole that must be considered withdraw and surrender, and does hereby this Agreement, and may be further
before the constitutional restriction is violated. Thus, petitioners' points on operational control will be given more withdraw and surrender, all rights of described in implementing arrangements.
attention in the discussion below. The arguments on policy are, however, outside the scope of judicial review possession, supervision, jurisdiction,
and will not be discussed control or sovereignty existing and
exercised by the United States of EDCA, Art. V:
America in and over the territory and the
Moreover, a direct comparison of the MBA and EDCA will result in several important distinctions that would allay people of the Philippine Islands, except 1. The Philippines shall retain ownership
suspicion that EDCA is but a disguised version of the MBA. the use of such bases, necessary of and title to Agreed Locations.
appurtenances to such bases, and the
b. There are substantial matters that the US. cannot do under EDCA, but which it was authorized to do under rights incident thereto, as the United
xxxx
the 1947 MBA States of America, by agreement with the
Republic of the Philippines may deem
necessary to retain for the mutual 4. All buildings, non-relocatable
The Philippine experience with U.S. military bases under the 1947 MBA is simply not possible under EDCA for a structures, and assemblies affixed to
number of important reasons. protection of the Republic of the
Philippines and of the United States of the land in the Agreed
America. x x x. Locations, including ones altered or
First, in the 1947 MBA, the U.S. retained all rights of jurisdiction in and over Philippine territory occupied by improved by United States forces, remain
American bases. In contrast, the U.S. under EDCA does not enjoy any such right over any part of the Philippines the property of the
in which its forces or equipment may be found. Below is a comparative table between the old treaty and EDCA: Philippines.Permanent buildings
constructed by United States forces
become the property of the Philippines,
1947 MBA/ 1946 Treaty of General EDCA once constructed, but shall be used by
Relations United States forces until no longer
1947 MBA, Art. I(1): EDCA, preamble: required by United States forces.
PAGE 116 of 135 CONSTITUTIONAL LAW -EXECUTIVE DEPARTMENT PART 2 –SESSION 12
The Philippine Designated Authority
and its authorized
representative shall have access to the
Second, in the bases agreement, the U.S. and the Philippines were visibly not on equal footing when it came to entire area of the Agreed Locations.
deciding whether to expand or to increase the number of bases, as the Philippines may be compelled to Such access shall be provided promptly
negotiate with the U.S. the moment the latter requested an expansion of the existing bases or to acquire consistent with operational safety and
additional bases. In EDCA, U.S. access is purely at the invitation of the Philippines. security requirements in accordance with
agreed procedures developed by the
Parties.
1947 MBA/ 1946 Treaty of General EDCA
Relations
Fourth, in the bases agreement, the U.S. retained the right, power, and authority over the establishment, use,
1947 MBA, Art.I(3): EDCA, preamble:
operation, defense, and control of military bases, including the limits of territorial waters and air space adjacent
to or in the vicinity of those bases. The only standard used in determining the extent of its control was military
The Philippines agree to enter into Recognizing that all United States necessity. On the other hand, there is no such grant of power or authority under EDCA. It merely allows the U.S.
negotiations with the United States at access to and use of facilities and areas to exercise operational control over the construction of Philippine-owned structures and facilities:
the latter's request, to permit the United will be at the invitation of the
States to expand such bases, to Philippines and with full respect for the
exchange such bases for other bases, to Philippine Constitution and Philippine 1947 MBA EDCA
acquire additional bases, or relinquish laws;
rights to bases, as any of such exigencies 1947 MBA, Art.I(2): EDCA, Art. III(4):
may be required by military necessity. xxxx
The Philippines agrees to permit the The Philippines hereby grants to the
1946 Treaty of Gen. Relations, Art. I: United States, upon notice to the United States, through bilateral security
EDCA. Art. II(4): Philippines, to use such of those mechanisms, such as the MDB and
bases listed in Annex B as the United SEB, operational control of Agreed
The United States of America agrees to "Agreed Locations" means facilities and States determines to be required by Locations for construction
withdraw and surrender, and does hereby areas that are provided by the military necessity. activities and authority to undertake
withdraw and surrender, all rights of Government of the Philippines through such activities on, and make alterations
possession, supervision, jurisdiction, the AFP and that United States forces, and improvements to, Agreed
control or sovereignty existing and 1947 MBA, Art. III(1):
United States contractors, and others as Locations. United States forces shall
exercised by the United States of mutually agreed, shall have the right to consult on issues regarding such
America in and over the territory and the access and use pursuant to this It is mutually agreed that the United construction, alterations, and
people of the Philippine Islands, except Agreement. Such Agreed Locations may Statesshall have the rights, power and improvements based on the Parties'
the use of such bases, necessary be listed in an annex to be appended to authority within the bases which shared intent that the technical
appurtenances to such bases, and the this Agreement, and may be further are necessary for the establishment, requirements and construction standards
rights incident thereto, as the United described in implementing arrangements. use, operation and defense thereof or of any such projects undertaken by or on
States of America, by agreement with appropriate for the control thereof and behalf of United States forces should be
the Republic of the Philippines may all the rights, power and authority within consistent with the requirements and
deem necessary to retain for the mutual the limits of territorial waters and air standards of both Parties.
protection of the Republic of the space adjacent to, or in the vicinity of,
Philippines and of the United States of the bases which are necessary to
America. x x x. provide access to them, or appropriate
for their control.

Third, in EDCA, the Philippines is- guaranteed access over the entire area of the Agreed Locations. On the
other hand, given that the U.S. had complete control over its military bases under the 1947 MBA, the treaty did Fifth, the U.S. under the bases agreement was given the authority to use Philippine territory for additional
not provide for any express recognition of the right of access of Philippine authorities. Without that provision and staging areas, bombing and gunnery ranges. No such right is given under EDCA, as seen below:
in light of the retention of U.S. sovereignty over the old military bases, the U.S. could effectively prevent
Philippine authorities from entering those bases.
1947 MBA EDCA

1947 MBA EDCA 1947 MBA, Art. VI: EDCA, Art. III(1):

No equivalent provision. EDCA, Art. III(5): The United States shall, subject to With consideration of the views of
previous agreement with the Philippines, the Parties,
PAGE 117 of 135 CONSTITUTIONAL LAW -EXECUTIVE DEPARTMENT PART 2 –SESSION 12
have the right to use land and coastal the Philippines hereby authorizes and authority: x x x x to improve and transit or temporary access by United
sea areas of appropriate size and location agrees that United States forces, United deepen the harbors, channels, States forces to public land and facilities
for periodic maneuvers, for additional States contractors, and vehicles, vessels, entrances and anchorages, and to (including roads, ports, and airfields),
staging areas, bombing and gunnery and aircraft operated by or for United construct or maintain necessary including those owned or controlled by
ranges, and for such intermediate States forces may conduct the following roadsand bridges affording access to the local governments, and to other land and
airfields as may be required for safe and activities with respect to Agreed Locations: bases. facilities (including roads, ports, and
efficient air operations. Operations in such training; transit; support and related airfields).
areas shall be carried on with due regard activities; refueling of aircraft; bunkering of
and safeguards for the public safety. vessels; temporary maintenance of
vehicles, vessels, and aircraft; temporary Eighth, in the 1947 MBA, the U.S. was granted the automatic right to use any and all public utilities, services
accommodation of personnel; and facilities, airfields, ports, harbors, roads, highways, railroads, bridges, viaducts, canals, lakes, rivers, and
1947 MBA, Art.I(2):
communications; prepositioning of streams in the Philippines in the same manner that Philippine military forces enjoyed that right. No such
equipment, supplies, and materiel; arrangement appears in EDCA. In fact, it merely extends to U.S. forces temporary access to public land and
The Philippines agrees to permit the deploying forces and materiel; and such facilities when requested:
United States, upon notice to the other activities as the Parties may agree.
Philippines, to use such of those
bases listed in Annex B as the United 1947 MBA EDCA
States determines to be required by
military necessity. 1947 MBA, Art. VII: EDCA, Art. III(2):

It is mutually agreed that the United When requested, the Designated


Sixth, under the MBA, the U.S. was given the right, power, and authority to control and prohibit the movement States may employ and use for United Authority of the Philippines shall assist
and operation of all types of vehicles within the vicinity of the bases. The U.S. does not have any right, power, or States military forces any and all public in facilitating transit or temporary
authority to do so under EDCA. utilities, other services and facilities, access by United States forces to public
airfields, ports, harbors, roads, highways, land and facilities (including roads, ports,
railroads, bridges, viaducts, canals, lakes, and airfields), including those owned or
1947 MBA EDCA rivers and streams in the Philippines under controlled by local governments, and to
1947 MBA, Art. 111(2)(c) No equivalent provision. conditions no less favorable than other land and facilities (including roads,
those that may be applicablefrom time to ports, and airfields).
time to the military forces of the
Such rights, power and authority shall Philippines.
include, inter alia, the right, power and
authority: x x x x to control (including the
right to prohibit) in so far as may be Ninth, under EDCA, the U.S. no longer has the right, power, and authority to construct, install, maintain, and
required for the efficient operation and employ any type of facility, weapon, substance, device, vessel or vehicle, or system unlike in the old treaty.
safety of the bases, and within the limits of EDCA merely grants the U.S., through bilateral security mechanisms, the authority to undertake construction,
military necessity, anchorages, alteration, or improvements on the Philippine-owned Agreed Locations.
moorings, landings, takeoffs,
movements and operation of ships and
water-borne craft, aircraft and other 1947 MBA EDCA
vehicles on water, in the air or on land
comprising 1947 MBA, Art. III(2)(e): EDCA, Art. III(4):

Such rights, power and authority shall The Philippines hereby grants to
Seventh, under EDCA, the U.S. is merely given temporary access to land and facilities (including roads, ports, include, inter alia, the right, power and the United States, through bilateral
and airfields). On the other hand, the old treaty gave the U.S. the right to improve and deepen the harbors, authority: x x x x to construct, install, security mechanisms, such as the MDB
channels, entrances, and anchorages; and to construct or maintain necessary roads and bridges that would maintain, and employ on any base any and SEB, operational control of Agreed
afford it access to its military bases. type of facilities, weapons, substance, Locations for construction activities
device, vessel or vehicle on or under the and authority to undertake such
ground, in the air or on or under the water activities on, and make alterations and
1947 MBA EDCA that may be requisite or appropriate, improvements to, Agreed
1947 MBA, Art. III(2)(b): EDCA, Art. III(2): including meteorological systems, aerial Locations. United States forces shall
and water navigation lights, radio and consult on issues regarding such
radar apparatus and electronic devices, of construction, alterations, and
Such rights, power and authority shall When requested, the Designated Authority improvements based on the Parties'
include, inter alia, the right, power and of the Philippines shall assist in facilitating
PAGE 118 of 135 CONSTITUTIONAL LAW -EXECUTIVE DEPARTMENT PART 2 –SESSION 12
any desired power, type of emission and shared intent that the technical a contract with the United States approved by the Philippines, as those
frequency. requirements and construction standards together with their families, and terms are defined in the VFA.
of any such projects undertaken by or on technical personnel of other
behalf of United States forces should be nationalities (not being persons excluded x xx x
consistent with the requirements and by the laws of the Philippines) in
standards of both Parties. connection with the construction,
maintenance, or operation of the bases. 3. "United States contractors" means
The United States shall make suitable companies and firms, and their employees,
Tenth, EDCA does not allow the U.S. to acquire, by condemnation or expropriation proceedings, real property arrangements so that such persons may under contract or subcontract to or on
belonging to any private person. The old military bases agreement gave this right to the U.S. as seen below: be readily identified and their status behalf of the United States Department of
established when necessary by the Defense. United States contractors are not
Philippine authorities. Such persons, other included as part of
1947 MBA EDCA the definition of United States
than members of the United States armed
forces in uniform, shall present their travel personnel in this Agreement, including
1947 MBA, Art. XXII(l): No equivalent provision. within the context of the VFA.
documents to the appropriate Philippine
authorities for visas, it being understood
Whenever it is necessary to acquire by that no objection will be made to their
travel to the Philippines as non-
condemnation or expropriation immigrants.
proceedings real property belonging to
any private persons, associations or
corporations located in bases named in Twelfth, EDCA does not allow the U.S. to exercise jurisdiction over any offense committed by any person within
Annex A and Annex B in order to carry out the Agreed Locations, unlike in the former military bases:
the purposes of this Agreement, the
Philippines will institute and prosecute
1947 MBA EDCA
such condemnation or expropriation
proceedings in accordance with the laws of 1947 MBA, Art. XIII(l)(a): No equivalent provision.
the Philippines. The United States agrees
to reimburse the Philippines for all the
reasonable expenses, damages and costs The Philippines consents that the United
therebv incurred, including the value of the
property as determined by the Court. In States shall have the right to exercise
addition, subject to the mutual agreement jurisdiction over the following offenses:
of the two Governments, the United States (a) Any offense committed by any
will reimburse the Philippines for the person within any base except where the
reasonable costs of transportation and offender and offended parties are both
removal of any occupants displaced or Philippine citizens (not members of the
ejected by reason of the condemnation or armed forces of the United States on
expropriation. active duty) or the offense is against the
security of the Philippines.

Eleventh, EDCA does not allow the U.S. to unilaterally bring into the country non-Philippine nationals who are
under its employ, together with their families, in connection with the construction, maintenance, or operation of Thirteenth, EDCA does not allow the U.S. to operate military post exchange (PX) facilities, which is free of
the bases. EDCA strictly adheres to the limits under the VFA. customs duties and taxes, unlike what the expired MBA expressly allowed. Parenthetically, the PX store has
become the cultural icon of U.S. military presence in the country.

1947 MBA EDCA


1947 MBA EDCA
1947 MBA, Art. XI(l): EDCA, Art. II:
1947 MBA, Art. XVIII(l): No equivalent provision.
It is mutually agreed that the United 1. "United States personnel" means
States shall have the right to bring into United States military and civilian It is mutually agreed that the United
the Philippines members of the United personneltemporarily in the territory of the States
States military forces and the United Philippines in connection with activities
States nationals employed by or under
PAGE 119 of 135 CONSTITUTIONAL LAW -EXECUTIVE DEPARTMENT PART 2 –SESSION 12
shall have the right to establish on 150 hectares Wallace Air Station
bases, free of all licenses; fees; sales,
excise or other taxes, or 400 hectares John Hay Air Station
imposts; Government
agencies, including concessions, such 15,000 hectares Subic Naval Base
as sales commissaries and post
exchanges; messes and social clubs, for
the exclusive use of the United States 1,000 hectares San Miguel Naval Communication
military forces and authorized civilian
personnel and their families. The 750 hectares Radio Transmitter in Capas, Tarlac
merchandise or services sold or dispensed
by such agencies shall be free of all 900 hectares Radio Bigot Annex at Bamban, Tarlac329
taxes, duties and inspection by the
Philippine authorities. Administrative
measures shall be taken by the The Bases Conversion and Development Act of 1992 described its coverage in its Declaration of Policies:
appropriate authorities of the United States
to prevent the resale of goods which are Sec. 2. Declaration of Policies. - It is hereby declared the policy of the Government to accelerate the sound and
sold under the provisions of this Article to balanced conversion into alternative productive uses of the Clark and Subic military reservations and their
persons not entitled to buy goods at such extensions (John Hay Station, Wallace Air Station, O'Donnell Transmitter Station, San Miguel Naval
agencies and, generally, to prevent abuse Communications Station and Capas Relay Station), to raise funds by the sale of portions of Metro Manila military
of the privileges granted under this Article. camps, and to apply said funds as provided herein for the development and conversion to productive civilian use
There shall be cooperation between such of the lands covered under the 194 7 Military Bases Agreement between the Philippines and the United States
authorities and the Philippines to this end. of America, as amended.330

In sum, EDCA is a far cry from a basing agreement as was understood by the people at the time that the 1987 The result of the debates and subsequent voting is Section 25, Article XVIII of the Constitution, which specifically
Constitution was adopted. restricts, among others, foreign military facilities or bases. At the time of its crafting of the Constitution, the 1986
Constitutional Commission had a clear idea of what exactly it was restricting. While the term "facilities and
bases" was left undefined, its point of reference was clearly those areas covered by the 1947 MBA as amended.
Nevertheless, a comprehensive review of what the Constitution means by "foreign military bases" and "facilities"
is required before EDCA can be deemed to have passed judicial scrutiny.
Notably, nearly 30 years have passed since then, and the ever-evolving world of military technology and
geopolitics has surpassed the understanding of the Philippine people in 1986. The last direct military action of
c. The meaning of military facilities and bases the U.S. in the region was the use of Subic base as the staging ground for Desert Shield and Desert Storm
during the Gulf War.331In 1991, the Philippine Senate rejected the successor treaty of the 1947 MBA that would
An appreciation of what a military base is, as understood by the Filipino people in 1987, would be vital in have allowed the continuation of U.S. bases in the Philippines.
determining whether EDCA breached the constitutional restriction.
Henceforth, any proposed entry of U.S. forces into the Philippines had to evolve likewise, taking into
Prior to the drafting of the 1987 Constitution, the last definition of "military base" was provided under Presidential consideration the subsisting agreements between both parties, the rejection of the 1991 proposal, and a
Decree No. (PD) 1227.328 Unlawful entry into a military base is punishable under the decree as supported by concrete understanding of what was constitutionally restricted. This trend birthed the VFA which, as discussed,
Article 281 of the Revised Penal Code, which itself prohibits the act of trespass. has already been upheld by this Court.

Section 2 of the law defines the term in this manner: "'[M]ilitary base' as used in this decree means any military, The latest agreement is EDCA, which proposes a novel concept termed "Agreed Locations."
air, naval, or coast guard reservation, base, fort, camp, arsenal, yard, station, or installation in the Philippines."
By definition, Agreed Locations are
Commissioner Tadeo, in presenting his objections to U.S. presence in the Philippines before the 1986
Constitutional Commission, listed the areas that he considered as military bases: facilities and areas that are provided by the Government of the Philippines through the AFP and that United
States forces, United States contractors, and others as mutually agreed, shall have the right to access and use
1,000 hectares Camp O'Donnel pursuant to this Agreement. Such Agreed Locations may be listed in an annex to be appended to this
Agreement, and may be further described in implementing arrangements.332
20,000 hectares Crow Valley Weapon's Range
Preliminarily, respondent already claims that the proviso that the Philippines shall retain ownership of and title to
the Agreed Locations means that EDCA is "consistent with Article II of the VFA which recognizes Philippine
55,000 hectares Clark Air Base
sovereignty and jurisdiction over locations within Philippine territory. 333
PAGE 120 of 135 CONSTITUTIONAL LAW -EXECUTIVE DEPARTMENT PART 2 –SESSION 12
By this interpretation, respondent acknowledges that the contention of petitioners springs from an understanding magnets of nuclear attack or are deterrents to such attack; 2) whether an alliance or mutual defense treaty is a
that the Agreed Locations merely circumvent the constitutional restrictions. Framed differently, the bone of derogation of our national sovereignty; 3) whether criticism of us by Russia, Vietnam and North Korea is
contention is whether the Agreed Locations are, from a legal perspective, foreign military facilities or bases. This outweighed by the support for us of the ASEAN countries, the United States, South Korea, Taiwan, Australia
legal framework triggers Section 25, Article XVIII, and makes Senate concurrence a sine qua non. and New Zealand; and 4) whether the social, moral and legal problems spawned by the military bases and their
operations can be compensated by the economic benefits outlined in papers which have been furnished recently
Article III of EDCA provides for Agreed Locations, in which the U.S. is authorized by the Philippines to "conduct to all of us.335
the following activities: "training; transit; support and related activities; refueling of aircraft; bunkering of vessels;
temporary maintenance of vehicles, vessels and aircraft; temporary accommodation of personnel; xxxx
communications; prepositioning of equipment, supplies and materiel; deploying forces and materiel; and such
other activities as the Parties may agree." Of course, one side of persuasion has submitted categorical, unequivocal and forceful assertions of their
positions. They are entitled to the luxury of the absolutes. We are urged now to adopt the proposed
This creation of EDCA must then be tested against a proper interpretation of the Section 25 restriction. declaration as a "golden," "unique" and "last" opportunity for Filipinos to assert their sovereign
rights. Unfortunately, I have never been enchanted by superlatives, much less for the applause of the moment
d. Reasons for the constitutional requirements and legal standards for constitutionally compatible military bases or the ovation of the hour. Nor do I look forward to any glorious summer after a winter of political discontent.
and facilities Hence, if I may join Commissioner Laurel, I also invoke a caveat not only against the tyranny of labels but also
the tyranny of slogans.336
Section 25 does not define what is meant by a "foreign military facility or base." While it specifically alludes to
U.S. military facilities and bases that existed during the framing of the Constitution, the provision was clearly xxxx
meant to apply to those bases existing at the time and to any future facility or base. The basis for the restriction
must first be deduced from the spirit of the law, in order to set a standard for the application of its text, given the SPEECH OF COMMISSIONER SUAREZ337
particular historical events preceding the agreement.
MR. SUAREZ: Thank you, Madam President.
Once more, we must look to the 1986 Constitutional Commissioners to glean, from their collective wisdom, the
intent of Section 25. Their speeches are rich with history and wisdom and present a clear picture of what they I am quite satisfied that the crucial issues involved in the resolution of the problem of the removal of foreign
considered in the crafting the provision. bases from the Philippines have been adequately treated by previous speakers. Let me, therefore, just
recapitulate the arguments adduced in favor of a foreign bases-free Philippines:
SPEECH OF COMMISSIONER REGALADO334
1. That every nation should be free to shape its own destiny without outside interference;
xxxx
2. That no lasting peace and no true sovereignty would ever be achieved so long as there are
We have been regaled here by those who favor the adoption of the anti-bases provisions with what purports to foreign military forces in our country;
be an objective presentation of the historical background of the military bases in the Philippines. Care appears,
however, to have been taken to underscore the inequity in their inception as well as their 3. That the presence of foreign military bases deprives us of the very substance of national
implementation, as to seriously reflect on the supposed objectivity of the report. Pronouncements of military sovereigntyand this is a constant source of national embarrassment and an insult to our national
and civilian officials shortly after World War II are quoted in support of the proposition on neutrality; regrettably, dignity and selfrespect as a nation;
the implication is that the same remains valid today, as if the world and international activity stood still for the last
40 years.
4. That these foreign military bases unnecessarily expose our country to devastating nuclear
attacks;
We have been given inspired lectures on the effect of the presence of the military bases on our
sovereignty - whether in its legal or political sense is not clear - and the theory that any country with
foreign bases in its territory cannot claim to be fully sovereign or completely independent. I was not 5. That these foreign military bases create social problems and are designed to perpetuate the
aware that the concepts of sovereignty and independence have now assumed the totality principle, such that a strangle-hold of United States interests in our national economy and development;
willing assumption of some delimitations in the exercise of some aspects thereof would put that State in a lower
bracket of nationhood. 6. That the extraterritorial rights enjoyed by these foreign bases operate to deprive our country of
jurisdiction over civil and criminal offenses committed within our own national territory and against
xxxx Filipinos;

We have been receiving a continuous influx of materials on the pros and cons on the advisability of having 7. That the bases agreements are colonial impositions and dictations upon our helpless country; and
military bases within our shores. Most of us who, only about three months ago, were just mulling the prospects
of these varying contentions are now expected, like armchair generals, to decide not only on the geopolitical 8. That on the legal viewpoint and in the ultimate analysis, all the bases agreements are null and
aspects and contingent implications of the military bases but also on their political, social, economic and cultural void ab initio, especially because they did not count the sovereign consent and will of the Filipino
impact on our national life. We are asked to answer a plethora of questions, such as: 1) whether the bases are people.338
PAGE 121 of 135 CONSTITUTIONAL LAW -EXECUTIVE DEPARTMENT PART 2 –SESSION 12
xxxx In the case of the Philippines and the other Southeast Asian nations, the presence of American troops in the
country is a projection of America's security interest. Enrile said that nonetheless, they also serve, although in an
In the real sense, Madam President, if we in the Commission could accommodate the provisions I have cited, incidental and secondary way, the security interest of the Republic of the Philippines and the region. Yes, of
what is our objection to include in our Constitution a matter as priceless as the nationalist values we cherish? A course, Mr. Enrile also echoes the sentiments of most of us in this Commission, namely: It is ideal for us as an
matter of the gravest concern for the safety and survival of this nation indeed deserves a place in our independent and sovereign nation to ultimately abrogate the RP-US military treaty and, at the right time,
Constitution. build our own air and naval might.347

xxxx xxxx

x x x Why should we bargain away our dignity and our self-respect as a nation and the future of generations to Allow me to say in summation that I am for the retention of American military bases in the Philippines
come with thirty pieces of silver?339 provided that such an extension from one period to another shall be concluded upon concurrence of the
parties, and such extension shall be based on justice, the historical amity of the people of the
Philippines and the United States and their common defense interest.348
SPEECH OF COMMISSIONER BENNAGEN340
SPEECH OF COMMISSIONER ALONTO349
xxxx
xxxx
The underlying principle of military bases and nuclear weapons wherever they are found and whoever owns
them is that those are for killing people or for terrorizing humanity. This objective by itself at any point in
history is morally repugnant. This alone is reason enough for us to constitutionalize the ban on foreign military Madam President, sometime ago after this Commission started with this task of framing a constitution, I read a
bases and on nuclear weapons.341 statement of President Aquino to the effect that she is for the removal of the U.S. military bases in this country
but that the removal of the U.S. military bases should not be done just to give way to other foreign bases. Today,
there are two world superpowers, both vying to control any and all countries which have importance to their
SPEECH OF COMMISSIONER BACANI342 strategy for world domination. The Philippines is one such country.

xxxx Madam President, I submit that I am one of those ready to completely remove any vestiges of the days of
enslavement, but not prepared to erase them if to do so would merely leave a vacuum to be occupied by a far
x x x Hence, the remedy to prostitution does not seem to be primarily to remove the bases because even if worse type.350
the bases are removed, the girls mired in poverty will look for their clientele elsewhere. The remedy to the
problem of prostitution lies primarily elsewhere - in an alert and concerned citizenry, a healthy economy and a SPEECH OF COMMISSIONER GASCON351
sound education in values.343
xxxx
SPEECH OF COMMISSIONER JAMIR344
Let us consider the situation of peace in our world today. Consider our brethren in the Middle East, in Indo-
xxxx China, Central America, in South Africa - there has been escalation of war in some of these areas because of
foreign intervention which views these conflicts through the narrow prism of the East-West conflict. The United
One of the reasons advanced against the maintenance of foreign military bases here is that they impair States bases have been used as springboards for intervention in some of these conflicts. We should not
portions of our sovereignty. While I agree that our country's sovereignty should not be impaired, I also hold allow ourselves to be party to the warlike mentality of these foreign interventionists. We must always be
the view that there are times when it is necessary to do so according to the imperatives of national interest. on the side of peace – this means that we should not always rely on military solution.352
There are precedents to this effect. Thus, during World War II, England leased its bases in the West Indies and
in Bermuda for 99 years to the United States for its use as naval and air bases. It was done in consideration of xxxx
50 overaged destroyers which the United States gave to England for its use in the Battle of the Atlantic.
x x x The United States bases, therefore, are springboards for intervention in our own internal affairs and
A few years ago, England gave the Island of Diego Garcia to the United States for the latter's use as a naval in the affairs of other nations in this region.
base in the Indian Ocean. About the same time, the United States obtained bases in Spain, Egypt and Israel. In
doing so, these countries, in effect, contributed to the launching of a preventive defense posture against possible
trouble in the Middle East and in the Indian Ocean for their own protection.345 xxxx

SPEECH OF COMMISSIONER TINGSON346 Thus, I firmly believe that a self-respecting nation should safeguard its fundamental freedoms which should
logically be declared in black and white in our fundamental law of the land - the Constitution. Let us express
our desire for national sovereignty so we may be able to achieve national self-determination. Let us
xxxx express our desire for neutrality so that we may be able to follow active nonaligned independent foreign policies.
PAGE 122 of 135 CONSTITUTIONAL LAW -EXECUTIVE DEPARTMENT PART 2 –SESSION 12
Let us express our desire for peace and a nuclear-free zone so we may be able to pursue a healthy and tranquil Third, the continued exercise by the United States of extraterritoriality despite the condemnations of such
existence, to have peace that is autonomous and not imposed. 353 practice by the world community of nations in the light of overwhelming international approval of eradicating all
vestiges of colonialism.358
xxxx
xxxx
SPEECH OF COMMISSIONER TADEO354
Sixth, the deification of a new concept called pragmatic sovereignty, in the hope that such can be wielded to
Para sa magbubukid, ano ha ang kahulugan ng U.S. military bases? Para sa magbubukid, ang kahulugan force the United States government to concede to better terms and conditions concerning the military bases
nito ay pagkaalipin. Para sa magbubukid, ang pananatili ng U.S. military bases ay tinik sa dibdib ng agreement, including the transfer of complete control to the Philippine government of
sambayanang Pilipinong patuloy na nakabaon. Para sa sambayanang magbubukid, ang ibig sabihin ng U.S. the U.S. facilities, while in the meantime we have to suffer all existing indignities and disrespect towards our
military bases ay batong pabigat na patuloy na pinapasan ng sambayanang Pilipino. Para sa sambayanang rights as a sovereign nation.
magbubukid, ang pananatili ng U.S. military bases ay isang nagdudumilat na katotohanan ng patuloy na
paggahasa ng imperyalistang Estados Unidos sa ating Inang Bayan - economically, politically and xxxx
culturally. Para sa sambayanang magbubukid ang U.S. military bases ay kasingkahulugan ng nuclear
weapon - ang kahulugan ay magneto ng isang nuclear war. Para sa sambayanang magbubukid, ang Eighth, the utter failure of this forum to view the issue of foreign military bases as essentially a question
kahulugan ng U.S. military bases ay isang salot.355 of sovereignty which does not require in-depth studies or analyses and which this forum has, as a constituent
assembly drafting a constitution, the expertise and capacity to decide on except that it lacks the political will that
SPEECH OF COMMISSIONER QUESADA356 brought it to existence and now engages in an elaborate scheme of buck-passing.

xxxx xxxx

The drift in the voting on issues related to freeing ourselves from the instruments of domination and Without any doubt we can establish a new social order in our country, if we reclaim, restore, uphold and defend
subservience has clearly been defined these past weeks. our national sovereignty. National sovereignty is what the military bases issue is all about. It is only the
sovereign people exercising their national sovereignty who can design an independent course and take full
xxxx control of their national destiny.359

So for the record, Mr. Presiding Officer, I would like to declare my support for the committee's position to SPEECH OF COMMISSIONER P ADILLA360
enshrine in the Constitution a fundamental principle forbidding foreign military bases, troops or facilities in any
part of the Philippine territory as a clear and concrete manifestation of our inherent right to national self- xxxx
determination, independence and sovereignty.
Mr. Presiding Officer, in advocating the majority committee report, specifically Sections 3 and 4 on neutrality,
Mr. Presiding Officer, I would like to relate now these attributes of genuine nationhood to the social cost of nuclear and bases-free country, some views stress sovereignty of the Republic and even invoke survival of
allowing foreign countries to maintain military bases in our country. Previous speakers have dwelt on this the Filipino nation and people.361
subject, either to highlight its importance in relation to the other issues or to gloss over its significance and !llake
this a part of future negotiations.357 REBUTTAL OF COMMISSIONER NOLLEDO362

xxxx xxxx

Mr. Presiding Officer, I feel that banning foreign military bases is one of the solutions and is the response of the The anachronistic and ephemeral arguments against the provisions of the committee report to dismantle the
Filipino people against this condition and other conditions that have already been clearly and emphatically American bases after 1991 only show the urgent need to free our country from the entangling alliance with
discussed in past deliberations. The deletion, therefore, of Section 3 in the Constitution we are drafting will have any power bloc.363
the following implications:
xxxx
First, the failure of the Constitutional Commission to decisively respond to the continuing violation of our
territorial integrity via the military bases agreement which permits the retention of U.S. facilities within
the Philippine soil over which our authorities have no exclusive jurisdiction contrary to the accepted xx x Mr. Presiding Officer, it is not necessary for us to possess expertise to know that the so-called RP-US
definition of the exercise of sovereignty. Bases Agreement will expire in 1991, that it infringes on our sovereignty and jurisdiction as well as national
dignity and honor, that it goes against the UN policy of disarmament and that it constitutes unjust intervention
in our internal affairs.364 (Emphases Supplied)
Second, consent by this forum, this Constitutional Commission, to an exception in the application of a
provision in the Bill of Rights that we have just drafted regarding equal application of the laws of the land to all
inhabitants, permanent or otherwise, within its territorial boundaries.
PAGE 123 of 135 CONSTITUTIONAL LAW -EXECUTIVE DEPARTMENT PART 2 –SESSION 12
The Constitutional Commission eventually agreed to allow foreign military bases, troops, or facilities, subject to Philippine consent and cooperation.373 Finally, the Philippine Designated Authority or a duly designated
the provisions of Section 25. It is thus important to read its discussions carefully. From these discussions, we representative is given access to the Agreed Locations. 374
can deduce three legal standards that were articulated by the Constitutional Commission Members. These are
characteristics of any agreement that the country, and by extension this Court, must ensure are observed. We To our mind, these provisions do not raise the spectre of U.S. control, which was so feared by the Constitutional
can thereby determine whether a military base or facility in the Philippines, which houses or is accessed by Commission. In fact, they seem to have been the product of deliberate negotiation from the point of view of the
foreign military troops, is foreign or remains a Philippine military base or facility. The legal standards we find Philippine government, which balanced constitutional restrictions on foreign military bases and facilities against
applicable are: independence from foreign control, sovereignty and applicable law, and national security and the security needs of the country. In the 1947 MBA, the U.S. forces had "the right, power and authority x x x to
territorial integrity. construct (including dredging and filling), operate, maintain, utilize, occupy, garrison and control the
bases."375 No similarly explicit provision is present in EDCA.
i. First standard: independence from foreign control
Nevertheless, the threshold for allowing the presence of foreign military facilities and bases has been raised by
Very clearly, much of the opposition to the U.S. bases at the time of the Constitution's drafting was aimed at the present Constitution. Section 25 is explicit that foreign military bases, troops, or facilities shall not be allowed
asserting Philippine independence from the U.S., as well as control over our country's territory and military. in the Philippines, except under a treaty duly concurred in by the Senate. Merely stating that the Philippines
would retain ownership would do violence to the constitutional requirement if the Agreed Locations were simply
Under the Civil Code, there are several aspects of control exercised over property. to become a less obvious manifestation of the U.S. bases that were rejected in 1991.

Property is classified as private or public.365 It is public if "intended for public use, such as roads, canals, rivers, When debates took place over the military provisions of the Constitution, the committee rejected a specific
torrents, ports and bridges constructed by the State, banks, shores, roadsteads, and others of similar provision proposed by Commissioner Sarmiento. The discussion illuminates and provides context to the 1986
character[,]" or "[t]hose which belong to the State, without being for public use, and are intended for some public Constitutional Commission's vision of control and independence from the U.S., to wit:
service or for the development of the national wealth. "366
MR. SARMIENTO: Madam President, my proposed amendment reads as follows: "THE STATE SHALL
Quite clearly, the Agreed Locations are contained within a property for public use, be it within a government ESTABLISH AND MAINTAIN AN INDEPENDENT AND SELF-RELIANT ARMED FORCES OF THE
military camp or property that belongs to the Philippines.1avvphi1 PHILIPPINES." Allow me to briefly explain, Madam President. The Armed Forces of the Philippines is a vital
component of Philippine society depending upon its training, orientation and support. It will either be the people's
protector or a staunch supporter of a usurper or tyrant, local and foreign interest. The Armed Forces of the
Once ownership is established, then the rights of ownership flow freely. Article 428 of the Civil Code provides Philippines' past and recent experience shows it has never been independent and self-reliant. Facts, data
that "[t]he owner has the right to enjoy and dispose of a thing, without other limitations than those established by and statistics will show that it has been substantially dependent upon a foreign power. In March 1968,
law." Moreover, the owner "has also a right of action against the holder and possessor of the thing in order to Congressman Barbero, himself a member of the Armed Forces of the Philippines, revealed top secret
recover it." documents showing what he described as U.S. dictation over the affairs of the Armed Forces of the
Philippines. He showed that under existing arrangements, the United States unilaterally determines not
Philippine civil law therefore accords very strong rights to the owner of property, even against those who hold the only the types and quantity of arms and equipments that our armed forces would have, but also the time
property. Possession, after all, merely raises a disputable presumption of ownership, which can be contested when these items are to be made available to us. It is clear, as he pointed out, that the composition,
through normal judicial processes.367 capability and schedule of development of the Armed Forces of the Philippines is under the effective
control of the U.S. government.376 (Emphases supplied)
In this case, EDCA explicitly provides that ownership of the Agreed Locations remains with the Philippine
govemment.368 What U.S. personnel have a right to, pending mutual agreement, is access to and use of these Commissioner Sarmiento proposed a motherhood statement in the 1987 Constitution that would assert
locations.369 "independent" and "self-reliant" armed forces. This proposal was rejected by the committee, however. As
Commissioner De Castro asserted, the involvement of the Philippine military with the U.S. did not, by
The right of the owner of the property to allow access and use is consistent with the Civil Code, since the owner itself, rob the Philippines of its real independence. He made reference to the context of the times: that the
may dispose of the property in whatever way deemed fit, subject to the limits of the law. So long as the right of limited resources of the Philippines and the current insurgency at that time necessitated a strong military
ownership itself is not transferred, then whatever rights are transmitted by agreement does not completely divest relationship with the U.S. He said that the U.S. would not in any way control the Philippine military despite this
the owner of the rights over the property, but may only limit them in accordance with law. relationship and the fact that the former would furnish military hardware or extend military assistance and
training to our military. Rather, he claimed that the proposal was in compliance with the treaties between the two
states.
Hence, even control over the property is something that an owner may transmit freely. This act does not
translate into the full transfer of ownership, but only of certain rights. In Roman Catholic Apostolic Administrator
of Davao, Inc. v. Land Registration Commission, we stated that the constitutional proscription on property MR. DE CASTRO: If the Commissioner will take note of my speech on U.S. military bases on 12 September
ownership is not violated despite the foreign national's control over the property. 370 1986, I spoke on the selfreliance policy of the armed forces. However, due to very limited resources, the only
thing we could do is manufacture small arms ammunition. We cannot blame the armed forces. We have to
blame the whole Republic of the Philippines for failure to provide the necessary funds to make the Philippine
EDCA, in respect of its provisions on Agreed Locations, is essentially a contract of use and access. Under its Armed Forces self-reliant. Indeed that is a beautiful dream. And I would like it that way. But as of this time,
pertinent provisions, it is the Designated Authority of the Philippines that shall, when requested, assist in fighting an insurgency case, a rebellion in our country - insurgency - and with very limited funds and very limited
facilitating transit or access to public land and facilities. 371 The activities carried out within these locations are number of men, it will be quite impossible for the Philippines to appropriate the necessary funds
subject to agreement as authorized by the Philippine govemment. 372 Granting the U.S. operational control over therefor. However, if we say that the U.S. government is furnishing us the military hardware, it is not
these locations is likewise subject to EDCA' s security mechanisms, which are bilateral procedures involving
PAGE 124 of 135 CONSTITUTIONAL LAW -EXECUTIVE DEPARTMENT PART 2 –SESSION 12
control of our armed forces or of our government. It is in compliance with the Mutual Defense Treaty. It is A basic textual construction would show that the word "their," as understood above, is a possessive pronoun for
under the military assistance program that it becomes the responsibility of the United States to furnish us the the subject "they," a third-person personal pronoun in plural form. Thus, "their" cannot be used for a non-
necessary hardware in connection with the military bases agreement. Please be informed that there are three (3) personal subject such as "Agreed Locations." The simple grammatical conclusion is that "their" refers to the
treaties connected with the military bases agreement; namely: the RP-US Military Bases Agreement, the Mutual previous third-person plural noun, which is "United States forces." This conclusion is in line with the definition of
Defense Treaty and the Military Assistance Program. operational control.

My dear Commissioner, when we enter into a treaty and we are furnished the military hardware pursuant a. U.S. operational control as the exercise of authority over U.S. personnel, and not over the Agreed Locations
to that treaty, it is not in control of our armed forces nor control of our government. True indeed, we have
military officers trained in the U.S. armed forces school. This is part of our Military Assistance Program, but it Operational control, as cited by both petitioner and respondents, is a military term referring to
does not mean that the minds of our military officers are for the U.S. government, no. I am one of those who took
four courses in the United States schools, but I assure you, my mind is for the Filipino people. Also, while we are
sending military officers to train or to study in U.S. military schools, we are also sending our officers to study in [t]he authority to perform those functions of command over subordinate forces involving organizing and
other military schools such as in Australia, England and in Paris. So, it does not mean that when we send employing commands and forces, assigning tasks, designating objective, and giving authoritative direction
military officers to United States schools or to other military schools, we will be under the control of that country. necessary to accomplish the mission.383
We also have foreign officers in our schools, we in the Command and General Staff College in Fort Bonifacio
and in our National Defense College, also in Fort Bonifacio.377 (Emphases supplied) At times, though, operational control can mean something slightly different. In JUSMAG Philippines v. National
Labor Relations Commission, the Memorandum of Agreement between the AFP and JUSMAG Philippines
This logic was accepted in Tañada v. Angara, in which the Court ruled that independence does not mean the defined the term as follows:384
absence of foreign participation:
The term "Operational Control" includes, but is not limited to, all personnel administrative actions, such as: hiring
Furthermore, the constitutional policy of a "self-reliant and independent national economy" does not recommendations; firing recommendations; position classification; discipline; nomination and approval of
necessarily rule out the entry of foreign investments, goods and services. It contemplates neither incentive awards; and payroll computation.
"economic seclusion" nor "mendicancy in the international community." As explained by Constitutional
Commissioner Bernardo Villegas, sponsor of this constitutional policy: Clearly, traditional standards define "operational control" as personnel control. Philippine law, for instance,
deems operational control as one exercised by police officers and civilian authorities over their subordinates and
Economic self reliance is a primary objective of a developing country that is keenly aware of overdependence on is distinct from the administrative control that they also exercise over police subordinates.385 Similarly, a
external assistance for even its most basic needs. It does not mean autarky or economic seclusion; rather, it municipal mayor exercises operational control over the police within the municipal government, 386 just as city
means avoiding mendicancy in the international community. Independence refers to the freedom from undue mayor possesses the same power over the police within the city government.387
foreign control of the national economy, especially in such strategic industries as in the development of natural
resources and public utilities.378 (Emphases supplied) Thus, the legal concept of operational control involves authority over personnel in a commander-subordinate
relationship and does not include control over the Agreed Locations in this particular case. Though not
The heart of the constitutional restriction on foreign military facilities and bases is therefore the assertion of necessarily stated in EDCA provisions, this interpretation is readily implied by the reference to the taking of
independence from the U.S. and other foreign powers, as independence is exhibited by the degree of foreign "appropriate measures to protect United States forces and United States contractors."
control exerted over these areas.1âwphi1 The essence of that independence is self-governance and self-
control.379 Independence itself is "[t]he state or condition of being free from dependence, subjection, or control. It is but logical, even necessary, for the U.S. to have operational control over its own forces, in much the same
"380 way that the Philippines exercises operational control over its own units.

Petitioners assert that EDCA provides the U.S. extensive control and authority over Philippine facilities and For actual operations, EDCA is clear that any activity must be planned and pre-approved by the MDB-
locations, such that the agreement effectively violates Section 25 of the 1987 Constitution. 381 SEB.388 This provision evinces the partnership aspect of EDCA, such that both stakeholders have a say on how
its provisions should be put into effect.
Under Article VI(3) of EDCA, U.S. forces are authorized to act as necessary for "operational control and
defense." The term "operational control" has led petitioners to regard U.S. control over the Agreed Locations as b. Operational control vis-à-vis effective command and control
unqualified and, therefore, total.382 Petitioners contend that the word "their" refers to the subject "Agreed
Locations." Petitioners assert that beyond the concept of operational control over personnel, qualifying access to the Agreed
Locations by the Philippine Designated Authority with the phrase "consistent with operational safety and security
This argument misreads the text, which is quoted below: requirements in accordance with agreed procedures developed by the Parties" leads to the conclusion that the
U.S. exercises effective control over the Agreed Locations. 389 They claim that if the Philippines exercises
United States forces are authorized to exercise all rights and authorities within Agreed Locations that are possession of and control over a given area, its representative should not have to be authorized by a special
necessary for their operational control or defense, including taking appropriate measure to protect United States provision.390
forces and United States contractors. The United States should coordinate such measures with appropriate
authorities of the Philippines. For these reasons, petitioners argue that the "operational control" in EDCA is the "effective command and
control" in the 1947 MBA.391 In their Memorandum, they distinguish effective command and control from
PAGE 125 of 135 CONSTITUTIONAL LAW -EXECUTIVE DEPARTMENT PART 2 –SESSION 12
operational control in U.S. parlance.392 Citing the Doctrine for the Armed Forces of the United States, Joint c. Limited operational control over the Agreed Locations only for construction activitites
Publication 1, "command and control (C2)" is defined as "the exercise of authority and direction by a properly
designated commander over assigned and attached forces in the accomplishment of the mission x x As petitioners assert, EDCA indeed contains a specific provision that gives to the U.S. operational control within
x."393 Operational control, on the other hand, refers to "[t]hose functions of command over assigned forces the Agreed Locations during construction activities.401 This exercise of operational control is premised upon the
involving the composition of subordinate forces, the assignment of tasks, the designation of objectives, the approval by the MDB and the SEB of the construction activity through consultation and mutual agreement on the
overall control of assigned resources, and the full authoritative direction necessary to accomplish the requirements and standards of the construction, alteration, or improvement.402
mission."394
Despite this grant of operational control to the U.S., it must be emphasized that the grant is only for construction
Two things demonstrate the errors in petitioners' line of argument. activities. The narrow and limited instance wherein the U.S. is given operational control within an Agreed
Location cannot be equated with foreign military control, which is so abhorred by the Constitution.
Firstly, the phrase "consistent with operational safety and security requirements in accordance with agreed
procedures developed by the Parties" does not add any qualification beyond that which is already imposed by The clear import of the provision is that in the absence of construction activities, operational control over the
existing treaties. To recall, EDCA is based upon prior treaties, namely the VFA and the MDT. 395 Treaties are in Agreed Location is vested in the Philippine authorities. This meaning is implicit in the specific grant of
themselves contracts from which rights and obligations may be claimed or waived.396 In this particular case, the operational control only during construction activities. The principle of constitutional construction, "expressio
Philippines has already agreed to abide by the security mechanisms that have long been in place between the unius est exclusio alterius," means the failure to mention the thing becomes the ground for inferring that it was
U.S. and the Philippines based on the implementation of their treaty relations. 397 deliberately excluded.403Following this construction, since EDCA mentions the existence of U.S. operational
control over the Agreed Locations for construction activities, then it is quite logical to conclude that it is not
Secondly, the full document cited by petitioners contradicts the equation of "operational control" with "effective exercised over other activities.
command and control," since it defines the terms quite differently, viz:398
Limited control does not violate the Constitution. The fear of the commissioners was total control, to the point
Command and control encompasses the exercise of authority, responsibility, and direction by a commander over that the foreign military forces might dictate the terms of their acts within the Philippines. 404 More important,
assigned and attached forces to accomplish the mission. Command at all levels is the art of motivating and limited control does not mean an abdication or derogation of Philippine sovereignty and legal jurisdiction over
directing people and organizations into action to accomplish missions. Control is inherent in command. To the Agreed Locations. It is more akin to the extension of diplomatic courtesies and rights to diplomatic
control is to manage and direct forces and functions consistent with a commander's command authority. Control agents,405 which is a waiver of control on a limited scale and subject to the terms of the treaty.
of forces and functions helps commanders and staffs compute requirements, allocate means, and integrate
efforts. Mission command is the preferred method of exercising C2. A complete discussion of tenets, This point leads us to the second standard envisioned by the framers of the Constitution: that the Philippines
organization, and processes for effective C2 is provided in Section B, "Command and Control of Joint Forces," of must retain sovereignty and jurisdiction over its territory.
Chapter V "Joint Command and Control."
ii. Second standard: Philippine sovereignty and applicable law
Operational control is defined thus:399
EDCA states in its Preamble the "understanding for the United States not to establish a permanent military
OPCON is able to be delegated from a lesser authority than COCOM. It is the authority to perform those presence or base in the territory of the Philippines." Further on, it likewise states the recognition that "all United
functions of command over subordinate forces involving organizing and employing commands and forces, States access to and use of facilities and areas will be at the invitation of the Philippines and with full respect for
assigning tasks, designating objectives, and giving authoritative direction over all aspects of military operations the Philippine Constitution and Philippine laws."
and joint training necessary to accomplish the mission. It should be delegated to and exercised by the
commanders of subordinate organizations; normally, this authority is exercised through subordinate JFCs,
Service, and/or functional component commanders. OPCON provides authority to organize and employ The sensitivity of EDCA provisions to the laws of the Philippines must be seen in light of Philippine sovereignty
commands and forces as the commander considers necessary to accomplish assigned missions. It does not and jurisdiction over the Agreed Locations.
include authoritative direction for logistics or matters of administration, discipline, internal organization, or unit
training. These elements of COCOM must be specifically delegated by the CCDR. OPCON does include the Sovereignty is the possession of sovereign power,406 while jurisdiction is the conferment by law of power and
authority to delineate functional responsibilities and operational areas of subordinate JFCs. authority to apply the law.407 Article I of the 1987 Constitution states:

Operational control is therefore the delegable aspect of combatant command, while command and control is the The national territory comprises the Philippine archipelago, with all the islands and waters embraced therein,
overall power and responsibility exercised by the commander with reference to a mission. Operational control is and all other territories over which the Philippines has sovereignty or jurisdiction, consisting of its terrestrial,
a narrower power and must be given, while command and control is plenary and vested in a commander. fluvial, and aerial domains, including its territorial sea, the seabed, the subsoil, the insular shelves, and other
Operational control does not include the planning, programming, budgeting, and execution process input; the submarine areas. The waters around, between, and connecting the islands of the archipelago, regardless of
assignment of subordinate commanders; the building of relationships with Department of Defense agencies; or their breadth and dimensions, form part of the internal waters of the Philippines. (Emphasis supplied)
the directive authority for logistics, whereas these factors are included in the concept of command and control. 400
From the text of EDCA itself, Agreed Locations are territories of the Philippines that the U.S. forces are allowed
This distinction, found in the same document cited by petitioners, destroys the very foundation of the arguments to access and use.408 By withholding ownership of these areas and retaining unrestricted access to them, the
they have built: that EDCA is the same as the MBA. government asserts sovereignty over its territory. That sovereignty exists so long as the Filipino people exist. 409
PAGE 126 of 135 CONSTITUTIONAL LAW -EXECUTIVE DEPARTMENT PART 2 –SESSION 12
Significantly, the Philippines retains primary responsibility for security with respect to the Agreed held to specific standards of conduct that require them to distinguish between combatants and non-
Locations.410Hence, Philippine law remains in force therein, and it cannot be said that jurisdiction has been combatants,420 as embodied by the Geneva Conventions and their Additional Protocols.421
transferred to the U.S. Even the previously discussed necessary measures for operational control and defense
over U.S. forces must be coordinated with Philippine authorities.411 Corollary to this point, Professor John Woodcliffe, professor of international law at the University of Leicester,
noted that there is no legal consensus for what constitutes a base, as opposed to other terms such as "facilities"
Jurisprudence bears out the fact that even under the former legal regime of the MBA, Philippine laws continue to or "installation."422 In strategic literature, "base" is defined as an installation "over which the user State has a
be in force within the bases.412 The difference between then and now is that EDCA retains the primary right to exclusive control in an extraterritorial sense."423 Since this definition would exclude most foreign military
jurisdiction of the Philippines over the security of the Agreed Locations, an important provision that gives it actual installations, a more important distinction must be made.
control over those locations. Previously, it was the provost marshal of the U.S. who kept the peace and enforced
Philippine law in the bases. In this instance, Philippine forces act as peace officers, in stark contrast to the 1947 For Woodcliffe, a type of installation excluded from the definition of "base" is one that does not fulfill a combat
MBA provisions on jurisdiction.413 role. He cites an example of the use of the territory of a state for training purposes, such as to obtain experience
in local geography and climactic conditions or to carry out joint exercises.424 Another example given is an
iii. Third standard: must respect national security and territorial integrity advanced communications technology installation for purposes of information gathering and
communication.425 Unsurprisingly, he deems these non-combat uses as borderline situations that would be
The last standard this Court must set is that the EDCA provisions on the Agreed Locations must not impair or excluded from the functional understanding of military bases and installations.426
threaten the national security and territorial integrity of the Philippines.
By virtue of this ambiguity, the laws of war dictate that the status of a building or person is presumed to be
This Court acknowledged in Bayan v. Zamora that the evolution of technology has essentially rendered the prior protected, unless proven otherwise.427 Moreover, the principle of distinction requires combatants in an armed
notion of permanent military bases obsolete. conflict to distinguish between lawful targets 428 and protected targets.429 In an actual armed conflict between the
U.S. and a third state, the Agreed Locations cannot be considered U.S. territory, since ownership of territory
even in times of armed conflict does not change.430
Moreover, military bases established within the territory of another state is no longer viable because of the
alternatives offered by new means and weapons of warfare such as nuclear weapons, guided missiles as well as
huge sea vessels that can stay afloat in the sea even for months and years without returning to their home Hence, any armed attack by forces of a third state against an Agreed Location can only be legitimate under
country. These military warships are actually used as substitutes for a land-home base not only of military international humanitarian law if it is against a bona fide U.S. military base, facility, or installation that directly
aircraft but also of military personnel and facilities. Besides, vessels are mobile as compared to a land-based contributes to the military effort of the U.S. Moreover, the third state's forces must take all measures to ensure
military headquarters.414 that they have complied with the principle of distinction (between combatants and non-combatants).

The VFA serves as the basis for the entry of U.S. troops in a limited scope. It does not allow, for instance, the re- There is, then, ample legal protection for the Philippines under international law that would ensure its territorial
establishment of the Subic military base or the Clark Air Field as U.S. military reservations. In this context, integrity and national security in the event an Agreed Location is subjected to attack. As EDCA stands, it does
therefore, this Court has interpreted the restrictions on foreign bases, troops, or facilities as three independent not create the situation so feared by petitioners - one in which the Philippines, while not participating in an armed
restrictions. In accord with this interpretation, each restriction must have its own qualification. conflict, would be legitimately targeted by an enemy of the U.S.431

Petitioners quote from the website http://en.wikipedia.org to define what a military base is. 415 While the source is In the second place, this is a policy question about the wisdom of allowing the presence of U.S. personnel within
not authoritative, petitioners make the point that the Agreed Locations, by granting access and use to U.S. our territory and is therefore outside the scope of judicial review.
forces and contractors, are U.S. bases under a different name. 416 More important, they claim that the Agreed
Locations invite instances of attack on the Philippines from enemies of the U.S.417 Evidently, the concept of giving foreign troops access to "agreed" locations, areas, or facilities within the military
base of another sovereign state is nothing new on the international plane. In fact, this arrangement has been
We believe that the raised fear of an attack on the Philippines is not in the realm of law, but of politics and policy. used as the framework for several defense cooperation agreements, such as in the following:
At the very least, we can say that under international law, EDCA does not provide a legal basis for a justified
attack on the Philippines. 1. 2006 U.S.-Bulgaria Defense Cooperation Agreement432

In the first place, international law disallows any attack on the Agreed Locations simply because of the presence 2. 2009 U.S.-Colombia Defense Cooperation Agreement433
of U.S. personnel. Article 2(4) of the United Nations Charter states that "All Members shall refrain in their
international relations from the threat or use of force against the territorial integrity or political independence of 3. 2009 U.S.-Poland Status of Forces Agreement434
any state, or in any other manner inconsistent with the Purposes of the United Nations." 418 Any unlawful attack
on the Philippines breaches the treaty, and triggers Article 51 of the same charter, which guarantees the
inherent right of individual or collective self-defence. 4. 2014 U.S.-Australia Force Posture Agreement435

Moreover, even if the lawfulness of the attack were not in question, international humanitarian law standards 5. 2014 U.S.-Afghanistan Security and Defense Cooperation Agreement436
prevent participants in an armed conflict from targeting non-participants. International humanitarian law, which is
the branch of international law applicable to armed conflict, expressly limits allowable military conduct exhibited In all of these arrangements, the host state grants U.S. forces access to their military bases. 437 That access is
by forces of a participant in an armed conflict. 419 Under this legal regime, participants to an armed conflict are without rental or similar costs to the U.S. 438 Further, U.S. forces are allowed to undertake construction activities
PAGE 127 of 135 CONSTITUTIONAL LAW -EXECUTIVE DEPARTMENT PART 2 –SESSION 12
in, and make alterations and improvements to, the agreed locations, facilities, or areas. 439 As in EDCA, the host Therefore, there is no basis to invalidate EDCA on fears that it increases the threat to our national security. If
states retain ownership and jurisdiction over the said bases. 440 anything, EDCA increases the likelihood that, in an event requiring a defensive response, the Philippines will be
prepared alongside the U.S. to defend its islands and insure its territorial integrity pursuant to a relationship built
In fact, some of the host states in these agreements give specific military-related rights to the U.S. For example, on the MDT and VFA.
under Article IV(l) of the US.-Bulgaria Defense Cooperation Agreement, "the United States forces x x x are
authorized access to and may use agreed facilities and areas x x x for staging and deploying of forces and 8. Others issues and concerns raised
materiel, with the purpose of conducting x x x contingency operations and other missions, including those
undertaken in the framework of the North Atlantic Treaty." In some of these agreements, host countries allow A point was raised during the oral arguments that the language of the MDT only refers to mutual help and
U.S. forces to construct facilities for the latter’s exclusive use.441 defense in the Pacific area.453 We believe that any discussion of the activities to be undertaken under EDCA vis-
a-vis the defense of areas beyond the Pacific is premature. We note that a proper petition on that issue must be
Troop billeting, including construction of temporary structures, is nothing new. In Lim v. Executive Secretary, the filed before we rule thereon. We also note that none of the petitions or memoranda has attempted to discuss this
Court already upheld the Terms of Reference of Balikatan 02-1, which authorized U.S. forces to set up issue, except only to theorize that the U.S. will not come to our aid in the event of an attack outside of the
"[t]emporary structures such as those for troop billeting, classroom instruction and messing x x x during the Pacific. This is a matter of policy and is beyond the scope of this judicial review.
Exercise." Similar provisions are also in the Mutual Logistics Support Agreement of 2002 and 2007, which are
essentially executive agreements that implement the VFA, the MDT, and the 1953 Military Assistance In reference to the issue on telecommunications, suffice it to say that the initial impression of the facility adverted
Agreement. These executive agreements similarly tackle the "reciprocal provision of logistic support, supplies, to does appear to be one of those that require a public franchise by way of congressional action under Section
and services,"442 which include "[b ]illeting, x x x operations support (and construction and use of temporary 11, Article XII of the Constitution. As respondents submit, however, the system referred to in the agreement
structures incident to operations support), training services, x x x storage services, x x x during an approved does not provide telecommunications services to the public for compensation. 454 It is clear from Article VIl(2) of
activity."443 These logistic supplies, support, and services include temporary use of "nonlethal items of military EDCA that the telecommunication system is solely for the use of the U.S. and not the public in general, and that
equipment which are not designated as significant military equipment on the U.S. Munitions List, during an this system will not interfere with that which local operators use. Consequently, a public franchise is no longer
approved activity."444 The first Mutual Logistics Support Agreement has lapsed, while the second one has been necessary.
extended until 2017 without any formal objection before this Court from the Senate or any of its members.
Additionally, the charge that EDCA allows nuclear weapons within Philippine territory is entirely speculative. It is
The provisions in EDCA dealing with Agreed Locations are analogous to those in the aforementioned executive noteworthy that the agreement in fact specifies that the prepositioned materiel shall not include nuclear
agreements. Instead of authorizing the building of temporary structures as previous agreements have done, weapons.455Petitioners argue that only prepositioned nuclear weapons are prohibited by EDCA; and that,
EDCA authorizes the U.S. to build permanent structures or alter or improve existing ones for, and to be owned therefore, the U.S. would insidiously bring nuclear weapons to Philippine territory.456 The general prohibition on
by, the Philippines.445 EDCA is clear that the Philippines retains ownership of altered or improved facilities and nuclear weapons, whether prepositioned or not, is already expressed in the 1987 Constitution.457 It would be
newly constructed permanent or non-relocatable structures.446 Under EDCA, U.S. forces will also be allowed to unnecessary or superfluous to include all prohibitions already in the Constitution or in the law through a
use facilities and areas for "training; x x x; support and related activities; x x x; temporary accommodation of document like EDCA.
personnel; communications" and agreed activities.447
Finally, petitioners allege that EDCA creates a tax exemption, which under the law must originate from
Concerns on national security problems that arise from foreign military equipment being present in the Congress. This allegation ignores jurisprudence on the government's assumption of tax liability. EDCA simply
Philippines must likewise be contextualized. Most significantly, the VFA already authorizes the presence states that the taxes on the use of water, electricity, and public utilities are for the account of the Philippine
of U.S. military equipment in the country. Article VII of the VFA already authorizes the U.S. to import into or Government.458 This provision creates a situation in which a contracting party assumes the tax liability of the
acquire in the Philippines "equipment, materials, supplies, and other property" that will be used "in connection other.459 In National Power Corporation v. Province of Quezon, we distinguished between enforceable and
with activities" contemplated therein. The same section also recognizes that "[t]itle to such property shall remain" unenforceable stipulations on the assumption of tax liability. Afterwards, we concluded that an enforceable
with the US and that they have the discretion to "remove such property from the Philippines at any time." assumption of tax liability requires the party assuming the liability to have actual interest in the property
taxed.460 This rule applies to EDCA, since the Philippine Government stands to benefit not only from the
There is nothing novel, either, in the EDCA provision on the prepositioning and storing of "defense equipment, structures to be built thereon or improved, but also from the joint training with U.S. forces, disaster preparation,
supplies, and materiel,"448 since these are sanctioned in the VFA. In fact, the two countries have already entered and the preferential use of Philippine suppliers.461 Hence, the provision on the assumption of tax liability does
into various implementing agreements in the past that are comparable to the present one. The Balikatan 02- not constitute a tax exemption as petitioners have posited.
1 Terms of Reference mentioned in Lim v. Executive Secretary specifically recognizes that Philippine and U.S.
forces "may share x x x in the use of their resources, equipment and other assets." Both the 2002 and 2007 Additional issues were raised by petitioners, all relating principally to provisions already sufficiently addressed
Mutual Logistics Support Agreements speak of the provision of support and services, including the "construction above. This Court takes this occasion to emphasize that the agreement has been construed herein as to
and use of temporary structures incident to operations support" and "storage services" during approved absolutely disauthorize the violation of the Constitution or any applicable statute. On the contrary, the
activities.449 These logistic supplies, support, and services include the "temporary use of x x x nonlethal items of applicability of Philippine law is explicit in EDCA.
military equipment which are not designated as significant military equipment on the U.S. Munitions List, during
an approved activity."450Those activities include "combined exercises and training, operations and other
deployments" and "cooperative efforts, such as humanitarian assistance, disaster relief and rescue operations, EPILOGUE
and maritime anti-pollution operations" within or outside Philippine territory.451 Under EDCA, the equipment,
supplies, and materiel that will be prepositioned at Agreed Locations include "humanitarian assistance and The fear that EDCA is a reincarnation of the U.S. bases so zealously protested by noted personalities in
disaster relief equipment, supplies, and materiel. "452 Nuclear weapons are specifically excluded from the Philippine history arises not so much from xenophobia, but from a genuine desire for self-determination,
materiel that will be prepositioned. nationalism, and above all a commitment to ensure the independence of the Philippine Republic from any
foreign domination.
PAGE 128 of 135 CONSTITUTIONAL LAW -EXECUTIVE DEPARTMENT PART 2 –SESSION 12
Mere fears, however, cannot curtail the exercise by the President of the Philippines of his Constitutional
prerogatives in respect of foreign affairs. They cannot cripple him when he deems that additional security
measures are made necessary by the times. As it stands, the Philippines through the Department of Foreign
Affairs has filed several diplomatic protests against the actions of the People's Republic of China in the West
Philippine Sea;462 initiated arbitration against that country under the United Nations Convention on the Law of
the Sea;463 is in the process of negotiations with the Moro Islamic Liberation Front for peace in Southern
Philippines,464 which is the subject of a current case before this Court; and faces increasing incidents of
kidnappings of Filipinos and foreigners allegedly by the Abu Sayyaf or the New People's Army. 465 The Philippine
military is conducting reforms that seek to ensure the security and safety of the nation in the years to come.466 In
the future, the Philippines must navigate a world in which armed forces fight with increasing sophistication in
both strategy and technology, while employing asymmetric warfare and remote weapons.

Additionally, our country is fighting a most terrifying enemy: the backlash of Mother Nature. The Philippines is
one of the countries most directly affected and damaged by climate change. It is no coincidence that the record-
setting tropical cyclone Yolanda (internationally named Haiyan), one of the most devastating forces of nature the
world has ever seen hit the Philippines on 8 November 2013 and killed at least 6,000 people. 467 This
necessitated a massive rehabilitation project. 468 In the aftermath, the U.S. military was among the first to extend
help and support to the Philippines.

That calamity brought out the best in the Filipinos as thousands upon thousands volunteered their help, their
wealth, and their prayers to those affected. It also brought to the fore the value of having friends in the
international community.

In order to keep the peace in its archipelago in this region of the world, and to sustain itself at the same time
against the destructive forces of nature, the Philippines will need friends. Who they are, and what form the
friendships will take, are for the President to decide. The only restriction is what the Constitution itself expressly
prohibits. It appears that this overarching concern for balancing constitutional requirements against the dictates
of necessity was what led to EDCA.

As it is, EDCA is not constitutionally infirm. As an executive agreement, it remains consistent with existing laws
and treaties that it purports to implement.

WHEREFORE, we hereby DISMISS the petitions.

SO ORDERED.
PAGE 129 of 135 CONSTITUTIONAL LAW -EXECUTIVE DEPARTMENT PART 2 –SESSION 12
EN BANC SERENO, CJ:

July 26, 2016 The Motion for Reconsideration before us seeks to reverse the Decision of this Court in Saguisag et. al., v.
Executive Secretary dated 12 January 2016.1 The petitions in Sasguisag, et. al.2 had questioned the
G.R. No. 212426 constitutionality of the Enhanced Defense Cooperation Agreement (EDCA) between the Republic of the
Philippines and the United States of America (U.S.). There, this Court ruled that the petitions be dismissed. 3
RENE A.V. SAGUISAG, WIGBERTO E. TAÑADA, FRANCISCO "DODONG" NEMENZO, JR., SR. MARY
JOHN MANANZAN, PACIFICO A. AGABIN, ESTEBAN "STEVE" SALONGA, H. HARRY L. ROQUE, JR., On 3 February 2016, petitioners in the Decision filed the instant Motion, asking for a reconsideration of the
EVALYN G. URSUA, EDRE U. OLALIA, DR. CAROL PAGADUAN-ARAULLO, DR. ROLAND SIMBULAN, Decision in Saguisag, et. al., questioning the ruling of the Court on both procedural and substantive grounds, viz:
and TEDDY CASIÑO, Petitioners,
vs WHEREFORE, premises considered, petitioners respectfully pray that the Honorable Court RECONSIDER,
EXECUTIVE SECRETARY PAQUITO N. OCHOA, JR., DEPARTMENT OF NATIONAL DEFENSE REVERSE, AND SET- ASIDE its Decision dated January 12, 2016, and issue a new Decision GRANTING the
SECRETARY VOLTAIRE GAZMIN, DEPARTMENT OF FOREIGN AFFAIRS SECRETARY ALBERT DEL instant consolidated petitions by declaring the Enhanced Defense Cooperation Agreement (EDCA) entered into
ROSARIO, JR., DEPARTMENT OF BUDGET AND MANAGEMENT SECRETARY FLORENCIO ABAD, and by the respondents for the Philippine government, with the United States of America, UNCONSTITUTIONAL
ARMED FORCES OF THE PHILIPPINES CHIEF OF STAFF GENERAL EMMANUEL T. AND INVALID and to permanently enjoin its implementation.
BAUTISTA, Respondents.
Other fonns of relief just and equitable under the premises are likewise prayed for.
x-----------------------x
At the outset, petitioners questioned the procedural findings of the Court despite acknowledging the fact that the
G.R. No. 212444 Court had given them standing to sue.4 Therefore this issue is now irrelevant and academic, and deserves no
reconsideration.
BAGONG ALY ANSANG MAKABAYAN (BAYAN), REPRESENTED BY ITS SECRETARY GENERAL
RENATO M. REYES, JR., BAYAN MUNA PARTY-LIST REPRESENTATIVES NERI J. COLMENARES, and As for the substantive grounds, petitioners claim this Court erred when it ruled that EDCA was not a treaty. 5 In
CARLOS ZARATE, GABRIELA WOMEN'S PARTY-LIST REPRESENTATIVES LUZ ILAGAN AND connection to this, petitioners move that EDCA must be in the form of a treaty in order to comply with the
EMERENCIANA DE JESUS, ACT TEACHERS PARTY-LIST REPRESENTATIVE ANTONIO L. TINIO, constitutional restriction under Section 25, Article· XVIII of the 1987 Constitution on foreign military bases,
ANAKPAWIS PARTY-LIST REPRESENTATIVE FERNANDO HICAP, KABATAAN PARTY-LIST troops, and facilities.6 Additionally, they reiterate their arguments on the issues of telecommunications, taxation,
REPRESENTATIVE TERRY RIDON, MAKABAYANG KOALISYON NG MAMAMAYAN (MAKABAYAN), and nuclear weapons.7
REPRESENTED BY SATURNINO OCAMPO, and LIZA MAZA, BIENVENIDO LUMBERA, JOEL C.
LAMANGAN, RAFAEL MARIANO, SALVADOR FRANCE, ROGELIO M. SOLUTA, and CLEMENTE G. We deny the Motion for Reconsideration.
BAUTISTA, Petitioners,
vs.
DEPARTMENT OF NATIONAL DEFENSE (DND) SECRETARY VOLTAIRE GAZMIN, DEPARTMENT OF Petitioners do not present new arguments to buttress their claims of error on the part of this Court. They have
FOREIGN AFFAIRS SECRETARY ALBERT DEL ROSARIO, EXECUTIVE SECRETARY PAQUITO N. rehashed their prior arguments and made them responsive to the structure of the Decision in Saguisag, yet the
OCHOA, JR., ARMED FORCES OF THE PHILIPPINES CHIEF OF STAFF GENERAL EMMANUEL T. points being made are the same.
BAUTISTA, DEFENSE UNDERSECRETARY PIO LORENZO BATINO, AMBASSADOR LOURDES
YPARRAGUIRRE, AMBASSADOR J. EDUARDO MALAYA, DEPARTMENT OF JUSTICE However, certain claims made by petitioners must be addressed.
UNDERSECRETARY FRANCISCO BARAAN III, and DND ASSISTANT SECRETARY FOR STRATEGIC
ASSESSMENTS RAYMUND JOSE QUILOP AS CHAIRPERSON AND MEMBERS, RESPECTIVELY, OF THE
On verba legis interpretation
NEGOTIATING PANEL FOR THE PHILIPPINES ON EDCA, Respondents.

Petitioners assert that this Court contradicted itself when it interpreted the word "allowed in" to refer to the initial
x-----------------------x entry of foreign bases, troops, and facilities, based on the fact that the plain meaning of the provision in question
referred to prohibiting the return of foreign bases, troops, and facilities except under a treaty concurred in by the
KILUSANG MAYO UNO, REPRESENTED BY ITS CHAIRPERSON, ELMER LABOG, CONFEDERATION Senate.8
FOR UNITY, RECOGNITION AND ADVANCEMENT OF GOVERNMENT EMPLOYEES (COURAGE),
REPRESENTED BY ITS NATIONAL PRESIDENT FERDINAND GAITE, NATIONAL FEDERATION OF This argument fails to consider the function and application of the verba legis rule.
LABOR UNIONS-KILUSANG MAYO UNO, REPRESENTED BY ITS NATIONAL PRESIDENT JOSELITO
USTAREZ, NENITA GONZAGA, VIOLETA ESPIRITU, VIRGINIA FLORES, and ARMANDO TEODORO,
JR., Petitioners-in-Intervention, Firstly, verba legis is a mode of construing the provisions of law as they stand.9 This takes into account the
RENE A.Q. SAGUISAG, JR., Petitioner-in-Intervention. language of the law, which is in English, and therefore includes reference to the meaning of the words based on
the actual use of the word in the language.
RESOLUTION
Secondly, by interpreting "allowed in" as referring to an initial entry, the Court has simply applied the plain
meaning of the words in the particular provision.10 Necessarily, once entry has been established by a subsisting
PAGE 130 of 135 CONSTITUTIONAL LAW -EXECUTIVE DEPARTMENT PART 2 –SESSION 12
treaty, latter instances of entry need not be embodied by a separate treaty. After all, the Constitution did not and MDT. They reiterate their arguments that EDCA's provisions fall outside the allegedly limited scope of the
state that foreign military bases, troops, and facilities shall not subsist or exist in the Philippines. VFA and MDT because it provides a wider arrangement than the VFA for military bases, troops, and facilities,
and it allows the establishment of U.S. military bases. 20
Petitioners' own interpretation and application of the verba legis rule will in fact result in an absurdity, which legal
construction strictly abhors.11 If this Court accept the essence of their argument that every instance of entry by Specifically, petitioners cite the terms of the VFA referring to "joint exercises,"21 such that arrangements
foreign bases, troops, and facilities must be set out in detail in a new treaty, then the resulting bureaucratic involving the individual States-parties such as exclusive use of prepositioned materiel are not covered by the
impossibility of negotiating a treaty for the entry of a head of State's or military officer's security detail, meetings VFA. More emphatically, they state that prepositioning itself as an activity is not allowed under the VFA. 22
of foreign military officials in the country, and indeed military exercises such as Balikatan will occupy much of, if
not all of the official working time by various government agencies. This is precisely the reason why any valid Evidently, petitioners left out of their quote the portion of the Decision which cited the Senate report on the VFA.
mode of interpretation must take into account how the law is exercised and its goals effected. 12 Ut res magis The full quote reads as follows:
valeat quam pereat.
Siazon clarified that it is not the VFA by itself that determines what activities will be conducted between the
The Constitution cannot be viewed solely as a list of prohibitions and limitations on governmental power, but armed forces of the U.S. and the Philippines. The VFA regulates and provides the legal framework for the
rather as an instrument providing the process of structuring government in order that it may effectively serve the presence, conduct and legal status of U.S. personnel while they are in the country for visits, joint exercises
people.13It is not simply a set of rules, but an entire legal framework for Philippine society. and other related activities.23

In this particular case, we find that EDCA did not go beyond the framework. The entry of US troops has long Quite clearly, the VFA contemplated activities beyond joint exercises, which this Court had already recognized
been authorized under a valid and subsisting treaty, which is the Visiting Forces Agreement (VFA). 14 Reading and alluded to in Lim v. Executive Secretary,24even though the Court in that case was faced with a challenge to
the VFA along with the longstanding Mutual Defense Treaty (MDT)15 led this Court to the conclusion that an the Terms of Reference of a specific type of joint exercise, the Balikatan Exercise.
executive agreement such as the EDCA was well within the bounds of the obligations imposed by both treaties.
One source petitioners used to make claims on the limitation of the VFA to joint exercises is the alleged
On strict construction of an exception Department of Foreign Affairs (DFA) Primer on the VFA, which they claim states that:

This Court agrees with petitioners' cited jurisprudence that exceptions are strictly construed.16 However, their Furthermore, the VFA does not involve access arrangements for United States armed forces or the pre-
patent misunderstanding of the Decision and the confusion this creates behooves this Court to address this positioning in the country of U.S. armaments and war materials. The agreement is about personnel and not
argument. equipment or supplies.25

To be clear, the Court did not add an exception to Section 25 Article XVIII. The general rule is that foreign Unfortunately, the uniform resource locator link cited by petitioners is inaccessible. However, even if we grant its
bases, troops, and facilities are not allowed in the Philippines. 17 The exception to this is authority granted to the veracity, the text of the VFA itself belies such a claim. Article I of the VFA states that "[a ]s used in this
foreign state in the form of a treaty duly concurred in by the Philippine Senate. 18 Agreement, "United States personnel" means United States military and civilian personnel temporarily in the
Philippines in connection with activities approved by the Philippine Government."26 These "activities" were, as
It is in the operation of this exception that the Court exercised its power of review. The lengthy legal analysis stated in Lim, left to further implementing agreements. It is true that Article VII on Importation did not indicate
resulted in a proper categorization of EDCA: an executive cagreement authorized by treaty. This Court pre-positioned materiel, since it referred to "United States Government equipment, materials, supplies, and other
undeniably considered the arguments asserting that EDCA was, in fact, a treaty and not an executive property imported into or acquired in the Philippines by or on behalf of the United States armed forces in
agreement, but these arguments fell flat before the stronger legal position that EDCA merely implemented the connection with activities to which this agreement applies[.]" 27
VFA and MDT. As we stated in the Decision:
Nonetheless, neither did the text of the VFA indicate "joint exercises" as the only activity, or even as one of
x x x [I]t must already be clarified that the terms and details used by an implementing agreement need not be those activities authorized by the treaty. In fact, the Court had previously noted that
found in the mother treaty. They must be sourced from the authority derived from the treaty, but are not
necessarily expressed word-for-word in the mother treaty.19 [n]ot much help can be had therefrom [VFA], unfortunately, since the terminology employed is itself the source of
the problem. The VFA permits United States personnel to engage, on an impermanent basis, in "activities," the
Hence, the argument that the Court added an exception to the law is erroneous and potentially misleading. The exact meaning of which was left undefined. The expression is ambiguous, permitting a wide scope of
parties, both petitioners and respondents must therefore read the Decision carefully in order to fully comply with undertakings subject only to the approval of the Philippine government. The sole encumbrance placed on its
its disposition. definition is couched in the negative, in that United States personnel must "abstain from any activity inconsistent
with the spirit of this agreement, and in particular, from any political activity." All other activities, in other words,
On EDCA as a treaty are fair game.28

The principal reason for the Motion for Reconsideration is evidently petitioners' disagreement with the Decision Moreover, even if the DFA Primer was accurate, properly cited, and offered as evidence, it is quite clear that the
that EDCA implements the VFA DFA's opinion on the VFA is not legally binding nor conclusive. 29 It is the exclusive duty of the Court to interpret
with finality what the VFA can or cannot allow according to its provisions. 30
PAGE 131 of 135 CONSTITUTIONAL LAW -EXECUTIVE DEPARTMENT PART 2 –SESSION 12
In addition to this, petitioners detail their objections to EDCA in a similar way to their original petition, claiming treaty are considered ineffective. Both types of international agreement are nevertheless subject to the
that the VFA and MDT did not allow EDCA to contain the following provisions: supremacy of the Constitution.33 (Emphasis supplied, citations omitted)

1. Agreed Locations Subsequently, the Decision goes to great lengths to illustrate the source of EDCA's validity, in that as an
executive agreement it fell within the parameters of the VFA and MDT, and seamlessly merged with the whole
2. Rotational presence of personnel web of Philippine law. We need not restate the arguments here. It suffices to state that this Court remains
unconvinced that EDCA deserves treaty status under the law.
3. U.S. contractors
On EDCA as basing agreement
4. Activities of U.S. contractors31
Petitioners claim that the Decision did not consider the similarity of EDCA to the previous Military Bases
Agreement (MBA) as grounds to declare it unconstitutional.34
We ruled in Saguisag, et. al. that the EDCA is not a treaty despite the presence of these provisions. The very
nature of EDCA, its provisions and subject matter, indubitably categorize it as an executive agreement - a class
of agreement that is not covered by the Article XVIII Section 25 restriction - in painstaking detail.32 To partially Firstly, the Court has discussed this issue in length and there is no need to rehash the analysis leading towards
quote the Decision: the conclusion that EDCA is different from the MBA or any basing agreement for that matter.

Executive agreements may dispense with the requirement of Senate concurrence because of the legal mandate Secondly, the new issues raised by petitioners are not weighty enough to overturn the legal distinction between
with which they are concluded. As culled from the afore-quoted deliberations of the Constitutional Commission, EDCA and the MBA.
past Supreme Court Decisions, and works of noted scholars, executive agreements merely involve
arrangements on the implementation of existing policies, rules, laws, or agreements. They are concluded (1) to In disagreeing with the Court in respect of the MBA's jurisdictional provisions, petitioners cite an exchange of
adjust the details of a treaty; (2) pursuant to or upon confirmation by an act of the Legislature; or (3) in the notes categorized as an "amendment" to the MBA, as if to say it operated as a new treaty and should be read
exercise of the President's independent powers under the Constitution. The raison d'etre of executive into the MBA.35
agreements hinges on prior constitutional or legislative authorizations.
This misleadingly equates an exchange of notes with an amendatory treaty. Diplomatic exchanges of notes are
The special nature of an executive agreement is not just a domestic variation in international agreements. not treaties but rather formal communication tools on routine agreements, akin to private law contracts, for the
International practice has accepted the use of various forms and designations of international agreements, executive branch.36 This cannot truly amend or change the terms of the treaty, 37 but merely serve as private
ranging from the traditional notion of a treaty - which connotes a formal, solemn instrument - to engagements contracts between the executive branches of government. They cannot ipso facto amend treaty obligations
concluded in modern, simplified forms that no longer necessitate ratification. An international agreement may between States, but may be treaty-authorized or treaty-implementing.38
take different forms: treaty, act, protocol, agreement, concordat, compromis d'arbitrage, convention, covenant,
declaration, exchange of notes, statute, pact, charter, agreed minute, memorandum of agreement, modus Hence, it is correct to state that the MBA as the treaty did not give the Philippines jurisdiction over the bases
vivendi, or some other form. Consequently, under international law, the distinction between a treaty and an because its provisions on U.S. jurisdiction were explicit. What the exchange of notes did provide was effectively
international agreement or even an executive agreement is irrelevant for purposes of determining international a contractual waiver of the jurisdictional rights granted to the U.S. under the MBA, but did not amend the treaty
rights and obligations. itself.

However, this principle does not mean that the domestic law distinguishing treaties, international Petitioners reassert that EDCA provisions on operational control, access to Agreed Locations, various rights and
agreements, and executive agreements is relegated to a mere variation in form, or that the constitutional authorities granted to the US "ensures, establishes, and replicates what MBA had provided." 39 However, as
requirement of Senate concurrence is demoted to an optional constitutional directive. There remain two very thoroughly and individually discussed in Saguisag, et. al., the significant differences taken as a whole result in a
important features that distinguish treaties from executive agreements and translate them into terms of art in the very different instrument, such that EDCA has not re-introduced the military bases so contemplated under Article
domestic setting. XVIII Section 25 of the Constitution.40

First, executive agreements must remain traceable to an express or implied authorization under the On policy matters
Constitution, statutes, or treaties. The absence of these precedents puts the validity and effectivity of
executive agreements under serious question for the main function of the Executive is to enforce the
Constitution and the laws enacted by the Legislature, not to defeat or interfere in the performance of these rules. Petitioners have littered their motion with alleged facts on U.S. practices, ineffective provisions, or even absent
In turn, executive agreements cannot create new international obligations that are not expressly allowed or provisions to bolster their position that EDCA is invalid.41 In this way, petitioners essentially ask this Court to
reasonably implied in the law they purport to implement. replace the prerogative of the political branches and rescind the EDCA because it not a good deal for the
Philippines. Unfortunately, the Court's only concern is the legality of EDCA and not its wisdom or folly. Their
remedy clearly belongs to the executive or legislative branches of government.
Second, treaties are, by their very nature, considered superior to executive agreements. Treaties are products of
the acts of the Executive and the Senate unlike executive agreements, which are solely executive actions.
Because of legislative participation through the Senate, a treaty is regarded as being on the same level as a EPILOGUE
statute. If there is an irreconcilable conflict, a later law or treaty takes precedence over one that is prior. An
executive agreement is treated differently. Executive agreements that are inconsistent with either a law or a
PAGE 132 of 135 CONSTITUTIONAL LAW -EXECUTIVE DEPARTMENT PART 2 –SESSION 12
While this Motion for Reconsideration was pending resolution, the United Nations Permanent Court of Arbitration No less than the 1987 Constitution states that the principal role of the military under the President as
tribunal constituted under the Convention on the Law of the Sea (UNCLOS) in Republic of the Philippines v. commander-in-chief shall be as protector of the people and the State to secure the sovereignty of the State and
People's Republic of China released its monumental decision on the afternoon of 12 July 2016.42 The findings the integrity of the national territory.56
and declarations in this decision contextualizes the security requirements of the Philippines, as they indicate an
alarming degree of international law violations committed against the Philippines' sovereign rights over its To recall, the Philippines and the U.S. entered into the MDT in 195157 with two things in mind, first, it allowed for
exclusive economic zone (EEZ). mutual assistance in maintaining and developing their individual and collective capacities to resist an armed
attack;58 and second, it provided for their mutual self-defense in the event of an armed attack against the ten-
Firstly, the tribunal found China's claimed nine-dash line, which included sovereign claims over most of the West itory of either party.59 The treaty was premised on their recognition that an anned attack on either of them would
Philippine, invalid under the UNCLOS for exceeding the limits of China's maritime zones granted under the equally be a threat to the security of the other.60
convention.43
The EDCA embodies this very purpose. It puts into greater effect a treaty entered into more than 50 years ago in
Secondly, the tribunal found that the maritime features within the West Philippine Sea/South China Sea that order to safeguard the sovereignty of the Philippines, and cement the military friendship of the U.S. and
China had been using as basis to claim sovereign rights within the Philippines' EEZ were not entitled to Philippines that has thrived for decades through multiple presidents and multiple treaties. While it is a fact that
independent maritime zones.44 our country is now independent, and that the 1987 Constitution requires Senate consent for foreign military
bases, troops, and facilities, the EDCA as envisioned by the executive and as formulated falls within the legal
Thirdly, the tribunal found that the actions of China within the EEZ of the Philippines, namely: forcing a Philippine regime of the MDT and the VFA.
vessel to cease-and-desist from survey operations,45 the promulgation of a fishing moratorium in 2012, 46 the
failure to exercise due diligence in preventing Chinese fishing vessels from fishing in the Philippines' EEZ In the context of recent developments, the President is bound to defend the EEZ of the Philippines and ensure
without complying with Philippine regulations, 47 the failure to prevent Chinese fishing vessels from harvesting its vast maritime wealth for the exclusive enjoyment of Filipinos. In this light, he is obligated to equip himself with
endangered species,48 the prevention of Filipino fishermen from fishing in traditional fishing grounds in all resources within his power to command. With the MDT and VFA as a blueprint and guide, EDCA strengthens
Scarborough Shoal,49 and the island-building operations in various reefs, all violate its obligations to respect the the Armed Forces of the Philippines and through them, the President's ability to respond to any potential military
rights of the Philippines over its EEZ.50 crisis with sufficient haste and greater strength.

Fourthly, the tribunal rejected Chinese claims of sovereignty over features within the Philippine's EEZ,51 and The Republic of Indonesia is strengthening its military presence and defences in the South China
found that its construction of installations and structures, and later on the creation of an artificial island, violated Sea.61 Vietnam has lent its voice in support of the settlement of disputes by peaceful means 62 but still strongly
its international obligations.52 asserts its sovereignty over the Paracel islands against China.63 The international community has given its voice
in support of the tribunal's decision in the UNCLOS arbitration. 64
Fifthly, the tribunal found that the behaviour of Chinese law enforcement vessels breached safe navigation
provisions of the UNCLOS in respect of near-collision instances within Scarborough Shoal.53 Despite all this, China has rejected the ruling.65 Its ships have continued to drive off Filipino fishermen from
areas within the Philippines' EEZ. 66 Its military officials have promised to continue its artificial island-building in
Finally, the tribunal found that since the arbitration was initiated in 2013, China has aggravated the dispute by the contested areas despite the ruling against these activities.67
building a large artificial island on a low-tide elevation located in the EEZ of the Philippines aggravated the
Parties' dispute concerning the protection and preservation of the marine environment at Mischief Reef by In this light, the Philippines must continue to ensure its ability to prevent any military aggression that violates its
inflicting permanent, irreparable harm to the coral reef habitat of that feature, extended the dispute concerning sovereign rights. Whether the threat is internal or external is a matter for the proper authorities to decide.
the protection and preservation of the marine environment by commencing large-scale island-building and President Rodrigo Roa Duterte has declared, in his inaugural speech, that the threats pervading society are
construction works at Cuarteron Reef: Fiery Cross Reef, Gaven Reef (North), Johnson Reef, Hughes Reef, and many: corruption, crime, drugs, and the breakdown of law and order.68 He has stated that the Republic of the
Subi Reef, aggravated the dispute concerning the status of maritime features in the Spratly Islands and their Philippines will honor treaties and international obligations.69 He has also openly supported EDCA's
capacity to generate entitlements to maritime zones by permanently destroying evidence of the natural condition continuation.70
of Mischief Reef, Cuarteron Reef, Fiery Cross Reef, Gaven Reef (North), Johnson Reef, Hughes Reef, and Subi
Reef.54 Thus, we find no reason for EDCA to be declared unconstitutional. It fully conforms to the Philippines' legal
regime through the MDT and VFA. It also fully conforms to the government's continued policy to enhance our
Taken as a whole, the arbitral tribunal has painted a harrowing picture of a major world power unlawfully military capability in the face of various military and humanitarian issues that may arise. This Motion for
imposing its might against the Philippines, There are clear indications that these violations of the Philippines' Reconsideration has not raised any additional legal arguments that warrant revisiting the Decision.
sovereign rights over its EEZ are continuing. The Philippine state is constitutionally-bound to defend its
sovereignty, and must thus prepare militarily. WHEREFORE, we hereby DENY the Motion for Reconsideration.

No less than the 1987 Constitution demands that the "State shall protect the nation's marine wealth in its SO ORDERED.
archipelagic waters, territorial sea, and exclusive economic zone, and reserve its use and enjoyment exclusively
to Filipino citizens. "55
PAGE 133 of 135 CONSTITUTIONAL LAW -EXECUTIVE DEPARTMENT PART 2 –SESSION 12
EN BANC It is the theory of the petitioners that ratification of a treaty, under both domestic law and international law, is a
function of the Senate. Hence, it is the duty of the executive department to transmit the signed copy of the Rome
Statute to the Senate to allow it to exercise its discretion with respect to ratification of treaties. Moreover, petitioners
SENATOR AQUILINO PIMENTEL, JR., G.R. No. 158088 submit that the Philippines has a ministerial duty to ratify the Rome Statute under treaty law and customary
REP. ETTA ROSALES, PHILIPPINE international law. Petitioners invoke the Vienna Convention on the Law of Treaties enjoining the states to refrain
COALITION FOR THE ESTABLISHMENT from acts which would defeat the object and purpose of a treaty when they have signed the treaty prior to
OF THE INTERNATIONAL Present: ratification unless they have made their intention clear not to become parties to the treaty. [5]
CRIMINAL COURT, TASK FORCE
DETAINEES OF THE PHILIPPINES, Davide, Jr., C.J., The Office of the Solicitor General, commenting for the respondents, questioned the standing of the petitioners to
FAMILIES OF VICTIMS OF Puno, file the instant suit. It also contended that the petition at bar violates the rule on hierarchy of courts. On the
INVOLUNTARY DISAPPEARANCES, Panganiban, substantive issue raised by petitioners, respondents argue that the executive department has no duty to transmit
BIANCA HACINTHA R. ROQUE, Quisumbing, the Rome Statute to the Senate for concurrence.
HARRISON JACOB R. ROQUE, Ynares-Santiago,
AHMED PAGLINAWAN, RON P. SALO, *Sandoval-Gutierrez, A petition for mandamus may be filed when any tribunal, corporation, board, officer or person unlawfully neglects
LEAVIDES G. DOMINGO, EDGARDO *Carpio, the performance of an act which the law specifically enjoins as a duty resulting from an office, trust, or station. [6] We
CARLO VISTAN, NOEL VILLAROMAN, Austria-Martinez, have held that to be given due course, a petition for mandamus must have been instituted by a party aggrieved
CELESTE CEMBRANO, LIZA ABIERA, *Corona, by the alleged inaction of any tribunal, corporation, board or person which unlawfully excludes said party from the
JAIME ARROYO, MARWIL LLASOS, Carpio Morales, enjoyment of a legal right. The petitioner in every case must therefore be an aggrieved party in the sense that he
CRISTINA ATENDIDO, ISRAFEL Callejo, Sr., possesses a clear legal right to be enforced and a direct interest in the duty or act to be performed. [7] The Court
FAGELA, and ROMEL BAGARES, Azcuna, will exercise its power of judicial review only if the case is brought before it by a party who has the legal standing
Petitioners, Tinga, to raise the constitutional or legal question. Legal standing means a personal and substantial interest in the case
Chico-Nazario, and such that the party has sustained or will sustain direct injury as a result of the government act that is being
- versus - Garcia, JJ. challenged. The term interest is material interest, an interest in issue and to be affected by the decree, as
distinguished from mere interest in the question involved, or a mere incidental interest.[8]
OFFICE OF THE EXECUTIVE
SECRETARY, represented by Promulgated:
The petition at bar was filed by Senator Aquilino Pimentel, Jr. who asserts his legal standing to file the suit as
HON. ALBERTO ROMULO, and the
member of the Senate; Congresswoman Loretta Ann Rosales, a member of the House of Representatives and
DEPARTMENT OF FOREIGN
Chairperson of its Committee on Human Rights; the Philippine Coalition for the Establishment of the International
AFFAIRS, represented by HON. BLAS OPLE, July 6, 2005
Criminal Court which is composed of individuals and corporate entities dedicated to the Philippine ratification of
Respondents.
the Rome Statute; the Task Force Detainees of the Philippines, a juridical entity with the avowed purpose of
x- - - - - - - - - - - - - - - - - - - - - - - - - - - - - - - - - - - - - - - - - - - - - - - - - - -x
promoting the cause of human rights and human rights victims in the country; the Families of Victims of Involuntary
Disappearances, a juridical entity duly organized and existing pursuant to Philippine Laws with the avowed
DECISION
purpose of promoting the cause of families and victims of human rights violations in the country; Bianca Hacintha
Roque and Harrison Jacob Roque, aged two (2) and one (1), respectively, at the time of filing of the instant petition,
PUNO J.:
and suing under the doctrine of inter-generational rights enunciated in the case of Oposa vs. Factoran, Jr.;[9] and
a group of fifth year working law students from the University of the Philippines College of Law who are suing as
taxpayers.
This is a petition for mandamus filed by petitioners to compel the
Office of the Executive Secretary and the Department of Foreign Affairs to transmit the signed copy of the Rome
Statute of the International Criminal Court to the Senate of the Philippines for its concurrence in accordance with The question in standing is whether a party has alleged such a personal stake in the outcome of the controversy
Section 21, Article VII of the 1987 Constitution. as to assure that concrete adverseness which sharpens the presentation of issues upon which the court so largely
depends for illumination of difficult constitutional questions. [10]
The Rome Statute established the International Criminal Court which shall have the power to exercise its
jurisdiction over persons for the most serious crimes of international concern xxx and shall be complementary to We find that among the petitioners, only Senator Pimentel has the legal standing to file the instant suit. The other
the national criminal jurisdictions.[1] Its jurisdiction covers the crime of genocide, crimes against humanity, war petitioners maintain their standing as advocates and defenders of human rights, and as citizens of the country.
crimes and the crime of aggression as defined in the Statute.[2] The Statute was opened for signature by all states They have not shown, however, that they have sustained or will sustain a direct injury from the non-transmittal of
in Rome on July 17, 1998 and had remained open for signature until December 31, 2000 at the United Nations the signed text of the Rome Statute to the Senate. Their contention that they will be deprived of their remedies for
Headquarters in New York. The Philippines signed the Statute on December 28, 2000 through Charge d the protection and enforcement of their rights does not persuade. The Rome Statute is intended to complement
Affairs Enrique A. Manalo of the Philippine Mission to the United Nations. [3] Its provisions, however, require that it national criminal laws and courts. Sufficient remedies are available under our national laws to protect our citizens
be subject to ratification, acceptance or approval of the signatory states. [4] against human rights violations and petitioners can always seek redress for any abuse in our domestic courts.

Petitioners filed the instant petition to compel the respondents the Office of the Executive Secretary and the As regards Senator Pimentel, it has been held that to the extent the powers of Congress are impaired,
Department of Foreign Affairs to transmit the signed text of the treaty to the Senate of the Philippines for ratification. so is the power of each member thereof, since his office confers a right to participate in the exercise of the powers
of that institution.[11] Thus, legislators have the standing to maintain inviolate the prerogatives, powers and
privileges vested by the Constitution in their office and are allowed to sue to question the validity of any official
action which they claim infringes their prerogatives as legislators. The petition at bar invokes the power of the
PAGE 134 of 135 CONSTITUTIONAL LAW -EXECUTIVE DEPARTMENT PART 2 –SESSION 12
Senate to grant or withhold its concurrence to a treaty entered into by the executive branch, in this case, the Rome The negotiations may be brief or protracted, depending on the issues involved, and may even
Statute. The petition seeks to order the executive branch to transmit the copy of the treaty to the Senate to allow collapse in case the parties are unable to come to an agreement on the points under
it to exercise such authority. Senator Pimentel, as member of the institution, certainly has the legal standing to consideration.
assert such authority of the Senate.
If and when the negotiators finally decide on the terms of the treaty, the same is opened
We now go to the substantive issue. for signature. This step is primarily intended as a means of authenticating the instrument and
for the purpose of symbolizing the good faith of the parties; but, significantly, it does not
The core issue in this petition for mandamus is whether the Executive Secretary and the Department of Foreign indicate the final consent of the state in cases where ratification of the treaty is required.
Affairs have a ministerial duty to transmit to the Senate the copy of the Rome Statute signed by a member of the The document is ordinarily signed in accordance with the alternat, that is, each of the several
Philippine Mission to the United Nations even without the signature of the President. negotiators is allowed to sign first on the copy which he will bring home to his own state.

Ratification, which is the next step, is the formal act by which a state confirms and accepts the
We rule in the negative.
provisions of a treaty concluded by its representatives. The purpose of ratification is to enable
the contracting states to examine the treaty more closely and to give them an opportunity
In our system of government, the President, being the head of state, is regarded as the sole organ and authority to refuse to be bound by it should they find it inimical to their interests. It is for this
in external relations and is the countrys sole representative with foreign nations. [12] As the chief architect of foreign reason that most treaties are made subject to the scrutiny and consent of a department
policy, the President acts as the countrys mouthpiece with respect to international affairs. Hence, the President is of the government other than that which negotiated them.
vested with the authority to deal with foreign states and governments, extend or withhold recognition, maintain
diplomatic relations, enter into treaties, and otherwise transact the business of foreign relations. [13] In the realm of xxx
treaty-making, the President has the sole authority to negotiate with other states.
The last step in the treaty-making process is the exchange of the instruments of
Nonetheless, while the President has the sole authority to negotiate and enter into treaties, the ratification, which usually also signifies the effectivity of the treaty unless a different date has
Constitution provides a limitation to his power by requiring the concurrence of 2/3 of all the members of the Senate been agreed upon by the parties. Where ratification is dispensed with and no effectivity clause
for the validity of the treaty entered into by him. Section 21, Article VII of the 1987 Constitution provides that no is embodied in the treaty, the instrument is deemed effective upon its signature. [16][emphasis
treaty or international agreement shall be valid and effective unless concurred in by at least two-thirds of all the supplied]
Members of the Senate. The 1935 and the 1973 Constitution also required the concurrence by the legislature to
the treaties entered into by the executive. Section 10 (7), Article VII of the 1935 Constitution provided: Petitioners arguments equate the signing of the treaty by the Philippine representative with ratification. It
Sec. 10. (7) The President shall have the power, with the concurrence of two-thirds of all the should be underscored that the signing of the treaty and the ratification are two separate and distinct steps in the
Members of the Senate, to make treaties xxx. treaty-making process. As earlier discussed, the signature is primarily intended as a means of authenticating the
instrument and as a symbol of the good faith of the parties. It is usually performed by the states authorized
Section 14 (1) Article VIII of the 1973 Constitution stated: representative in the diplomatic mission. Ratification, on the other hand, is the formal act by which a state confirms
Sec. 14. (1) Except as otherwise provided in this Constitution, no treaty shall be valid and and accepts the provisions of a treaty concluded by its representative. It is generally held to be an executive act,
effective unless concurred in by a majority of all the Members of the Batasang Pambansa. undertaken by the head of the state or of the government. [17] Thus, Executive Order No. 459 issued by President
Fidel V. Ramos on November 25, 1997 provides the guidelines in the negotiation of international agreements and
The participation of the legislative branch in the treaty-making process was deemed essential to provide a check its ratification. It mandates that after the treaty has been signed by the Philippine representative, the same shall
on the executive in the field of foreign relations.[14] By requiring the concurrence of the legislature in the treaties be transmitted to the Department of Foreign Affairs. The Department of Foreign Affairs shall then prepare the
entered into by the President, the Constitution ensures a healthy system of checks and balance necessary in the ratification papers and forward the signed copy of the treaty to the President for ratification. After the President
nations pursuit of political maturity and growth. [15] has ratified the treaty, the Department of Foreign Affairs shall submit the same to the Senate for concurrence.
Upon receipt of the concurrence of the Senate, the Department of Foreign Affairs shall comply with the provisions
In filing this petition, the petitioners interpret Section 21, Article VII of the 1987 Constitution to mean that the power of the treaty to render it effective. Section 7 of Executive Order No. 459 reads:
to ratify treaties belongs to the Senate. Sec. 7. Domestic Requirements for the Entry into Force of a Treaty or an
Executive Agreement. The domestic requirements for the entry into force of a treaty or an
We disagree. executive agreement, or any amendment thereto, shall be as follows:

Justice Isagani Cruz, in his book on International Law, describes the treaty-making process in this wise: A. Executive Agreements.
The usual steps in the treaty-making process are: negotiation, signature, ratification,
and exchange of the instruments of ratification. The treaty may then be submitted for registration i. All executive agreements shall be transmitted to the Department
and publication under the U.N. Charter, although this step is not essential to the validity of the of Foreign Affairs after their signing for the preparation of the
agreement as between the parties. ratification papers. The transmittal shall include the highlights of the
agreements and the benefits which will accrue to the Philippines
Negotiation may be undertaken directly by the head of state but he now usually assigns this arising from them.
task to his authorized representatives. These representatives are provided with credentials
known as full powers, which they exhibit to the other negotiators at the start of the formal ii. The Department of Foreign Affairs, pursuant to the endorsement
discussions. It is standard practice for one of the parties to submit a draft of the proposed treaty by the concerned agency, shall transmit the agreements to the
which, together with the counter-proposals, becomes the basis of the subsequent negotiations. President of the Philippines for his ratification. The original signed
PAGE 135 of 135 CONSTITUTIONAL LAW -EXECUTIVE DEPARTMENT PART 2 –SESSION 12
instrument of ratification shall then be returned to the Department of
Foreign Affairs for appropriate action.

B. Treaties.

i. All treaties, regardless of their designation, shall comply with the


requirements provided in sub-paragraph[s] 1 and 2, item A
(Executive Agreements) of this Section. In addition, the Department
of Foreign Affairs shall submit the treaties to the Senate of the
Philippines for concurrence in the ratification by the President. A
certified true copy of the treaties, in such numbers as may be
required by the Senate, together with a certified true copy of the
ratification instrument, shall accompany the submission of the
treaties to the Senate.

ii. Upon receipt of the concurrence by the Senate, the Department


of Foreign Affairs shall comply with the provision of the treaties in
effecting their entry into force.

Petitioners submission that the Philippines is bound under treaty law and international law to ratify the treaty which
it has signed is without basis. The signature does not signify the final consent of the state to the treaty. It is the
ratification that binds the state to the provisions thereof. In fact, the Rome Statute itself requires that the signature
of the representatives of the states be subject to ratification, acceptance or approval of the signatory states.
Ratification is the act by which the provisions of a treaty are formally confirmed and approved by a State. By
ratifying a treaty signed in its behalf, a state expresses its willingness to be bound by the provisions of such treaty.
After the treaty is signed by the states representative, the President, being accountable to the people, is burdened
with the responsibility and the duty to carefully study the contents of the treaty and ensure that they are not inimical
to the interest of the state and its people. Thus, the President has the discretion even after the signing of the treaty
by the Philippine representative whether or not to ratify the same. The Vienna Convention on the Law of Treaties
does not contemplate to defeat or even restrain this power of the head of states. If that were so, the requirement
of ratification of treaties would be pointless and futile. It has been held that a state has no legal or even moral duty
to ratify a treaty which has been signed by its plenipotentiaries.[18] There is no legal obligation to ratify a treaty, but
it goes without saying that the refusal must be based on substantial grounds and not on superficial or whimsical
reasons. Otherwise, the other state would be justified in taking offense. [19]

It should be emphasized that under our Constitution, the power to ratify is vested in the President, subject to the
concurrence of the Senate. The role of the Senate, however, is limited only to giving or withholding its consent, or
concurrence, to the ratification.[20] Hence, it is within the authority of the President to refuse to submit a treaty to
the Senate or, having secured its consent for its ratification, refuse to ratify it. [21] Although the refusal of a state to
ratify a treaty which has been signed in its behalf is a serious step that should not be taken lightly, [22] such decision
is within the competence of the President alone, which cannot be encroached by this Court via a writ of mandamus.
This Court has no jurisdiction over actions seeking to enjoin the President in the performance of his official
duties.[23] The Court, therefore, cannot issue the writ of mandamus prayed for by the petitioners as it is beyond its
jurisdiction to compel the executive branch of the government to transmit the signed text of Rome Statute to the
Senate.

IN VIEW WHEREOF, the petition is DISMISSED.

SO ORDERED.

Vous aimerez peut-être aussi